Tps - Tpa 2020

  • Uploaded by: ghanesa goestoschool
  • 0
  • 0
  • February 2021
  • PDF

This document was uploaded by user and they confirmed that they have the permission to share it. If you are author or own the copyright of this book, please report to us by using this DMCA report form. Report DMCA


Overview

Download & View Tps - Tpa 2020 as PDF for free.

More details

  • Words: 124,355
  • Pages: 546
Loading documents preview...
UTBK

Bacalah tulisan berikut, untuk menjawab soal 1-6, kemudian jawablah soal-soal yang tersedia dengan memilih jawaban yang tepat di antara pilihan jawaban A, B, C, D, atau E. […] 1Selama kurun waktu 10 tahun terakhir, kondisi inflasi Indonesia belum stabil. 2Kehidupan […] tidak lepas dari inflasi yang terlihat dari naiknya harga barang-barang kebutuhan pokok dari tahun ke tahun. 3Definisi konsep nilai waktu uang adalah jumlah […] yang sama hari ini lebih bermakna daripada esok. 4Artinya, inflasi dapat mengurangi daya beli uang yang kita miliki sekarang karena menumbuhkan harga terus-menerus. 5Akibatnya, pendapatan […] menurun sehingga banyak masyarakat semakin susah memenuhi kebutuhan sehari-harinya. 6Oleh karena itu, investasi diperlukan untuk melindungi nilai aset atau […] sehingga dapat digunakan sebagai persiapan kebutuhan di masa depan. (Diadaptasi dari: #Yuk Belajar Saham untuk Pemula, Investor Saham Pemula) 1. Judul yang paling tepat untuk melengkapi tulisan di atas adalah …. a) Daya Beli Uang b) Konsep Nilai Waktu Uang c) Inflasi Menggerogoti Nilai Uang d) Kehidupan Perekonomian Indonesia e) Inflasi Meningkatkan Daya Beli Uang Pembahasan Ide pokok teks tersebut terdapat dalam kalimat pertama yang membahas tentang kondisi inflasi. Berikutnya teks membahas tentang kondisi nilai uang yang semakin berkurang akibat inflasi terus-menerus. Jadi, judul yang paling tepat untuk teks di atas adalah “Inflasi Menggerogoti Nilai Uang”. Jawaban: C

1

2. Kata yang paling tepat untuk melengkapi titik-titik pada kalimat 2 adalah …. a) berekonomi b) keekonomian c) perekonomian d) pengekonomian e) mengekonomikan Pembahasan Menurut KBBI, perekonomian adalah tindakan (aturan atau cara berekonomi), sedangkan berekonomi adalah menjalankan suatu usaha berdasarkan teori-teori ekonomi. Frase kehidupan perekonomian dapat diartikan sebagai hal-hal terkait tindakan berekonomi. Frase tersebut sesuai dengan penjelasan keadaan inflasi yang memengaruhinya. Jadi, kata yang tepat melengkapi kalimat 2 adalah perekonomian. Jawaban: C 3. Kata yang paling tepat untuk melengkapi titik-titik pada kalimat 3 adalah …. a) kas b) prive c) utang d) modal e) piutang Pembahasan Konsep nilai waktu uang yang dibahas dalam kalimat 3 terkait dengan jumlah keluar masuknya uang yang ada dalam sehari. Istilah ekonomi untuk keadaan keluar masuknya uang adalah kas. Jadi, kata yang paling tepat untuk melengkapi titik-titik pada kalimat 3 adalah kas. Kata lainnya tidak sesuai karena prive berarti pengambilan dana oleh pemilik perusahaan perorangan untuk keperluan pribadi, sedangkan piutang adalah tagihan perusahaan. Jawaban: A

2

4. Kata yang paling tepat untuk melengkapi titik-titik pada kalimat 5 adalah …. a) riil b) real c) mikro d) makro e) hari-hari Pembahasan Sesuai KBBI, kata riil berarti nyata. Pendapatan riil yang menurun tepat menjelaskan nilai pendapatan yang sama tidak dapat mencukupi kebutuhan di kemudian hari karena harganya terus meningkat. Jadi, kata yang paling tepat untuk melengkapi titik-titik pada kalimat 5 adalah riil. Jawaban: A

5. Kata yang paling tepat untuk melengkapi titik-titik pada kalimat 6 adalah …. a) dana b) entiti c) derma d) entitas e) ekuitas Pembahasan Dana merupakan uang yang disediakan untuk suatu keperluan atau biaya. Investasi berkaitan dengan melindungi dana dan aset karena merupakan bentuk penanaman uang atau modal untuk memperoleh keuntungan. Jadi, kata yang paling tepat untuk melengkapi titik-titik pada kalimat 6 adalah dana. Kata-kata lainnya tidak tepat karena entitas berarti satuan yang berwujud, ekuitas berarti kepemilikan nilai uang, dan derma berarti bantuan uang.

Jawaban: A

3

6. Kata berimbuhan yang salah digunakan pada paragraf di atas adalah …. a) bermakna b) memenuhi c) melindungi d) mengurangi e) menumbuhkan Pembahasan Pada kalimat 4, kata berimbuhan yang tepat mengikuti kata harga adalah kenaikan atau menaikkan bukan menumbuhkan. Jadi, kata berimbuhan yang salah digunakan pada paragraf di atas adalah menumbuhkan. Jawaban: E

Bacalah tulisan berikut, untuk menjawab soal 7-12, kemudian jawablah soal-soal yang tersedia dengan memilih jawaban yang tepat di antara pilihan jawaban A, B, C, D, atau E. (1) Tren jumlah penduduk miskin di Indonesia dari waktu ke waktu lebih banyak tinggal di wilayah pedesaan. Tahun 1970 sampai dengan 1978, penduduk miskin lebih banyak tinggal di pedesaan, antara tahun 1980 sampai dengan 1990 lebih banyak tinggal di kawasan perkotaan, dan 1993 hingga 2000 jumlah penduduk miskin kembali lebih banyak di pedesaan. Bahkan, tahun 1998, selisih jumlah penduduk miskin yang tinggal di pedesaan cukup mencolok lebih besar […] mereka yang tinggal di kota. (2) Beberapa karakteristik penduduk miskin dapat dilihat dari jumlah anggota rumah tangga, pendidikan kepala rumah tangga, dan sumber pendapatan. Menurut BPS, tahun 1993 rata-rata jumlah anggota keluarga penduduk miskin sebesar 5,9 orang, […] jumlah keluarga rumah tangga yang hidup di atas garis kemiskinan lebih kecil dari itu. Hal ini menunjukkan bahwa kepala rumah tangga miskin mempunyai beban yang lebih berat daripada yang tidak miskin.

4

(3) Penduduk miskin di pedesaan 72,00 persen dipimpin oleh kepala rumah tangga yang tidak tamat SD dan 24,30 persen dipimpin oleh kepala rumah tangga berpendidikan tamat SD. Penduduk miskin di perkotaan 57,00 persen dipimpin oleh kepala rumah tangga tidak tamat SD dan 31,80 persen dikepalai oleh seorang tamatan SD. (4) Karakteristik lain yang berkaitan dengan pendidikan dan distribusi penduduk adalah sumber pendapatan. Sumber pendapatan utama penduduk miskin dari sektor pertanian sebesar 62,00 persen, sektor perdagangan 10,40 persen, sektor industri sebesar 7,40 persen, sektor jasa 6,50 persen dan selebihnya dari selain sektor yang disebutkan tersebut. Dengan demikian, sektor pertanian merupakan kantong kemiskinan penduduk. (sumber: Gunawan Sumodiningrat, 2001) 7. Bacaan di atas lebih tepat jika diberi judul …. a) Tren jumlah penduduk miskin b) Karakteristik penduduk miskin c) Distribusi penduduk miskin d) Tren dan karakteristik penduduk miskin e) Tren dan distribusi penduduk miskin Pembahasan Pada teks disebutkan tentang ’trend jumlah penduduk miskin di Indonesia dari waktu ke waktu’ dan ’beberapa karakteristik penduduk miskin dapat dilihat dari…’. Jadi judul yang tepat untuk bacaan ini adalah Tren dan karakteristik penduduk miskin. Jawaban: D

8. Bacaan di atas merupakan jenis karangan …. a) eksposisi b) argumentasi c) narasi d) deskripsi e) persuasi

5

Pembahasan Eksposisi merupakan jenis wacana yang memaparkan objek yang disertai data-data dan bukti dengan maksud menginformasikan kepada pembaca. Jawaban: A

9. Jika bacaan tersebut dilanjutkan paragraf kelima, gagasan utama yang tepat adalah …. a) Sektor pertanian b) Karakteristik dan ditribusi sektor perdagangan c) Karakteristik dan distribusi sektor pertanian d) Karakteristik sektor industri dan jasa e) Sektor jasa Pembahasan Karakteristik dan distribusi sektor pertanian merupakan paragraf yang tepat untuk melanjutkan paragraf kelima karena paragraf keempat membahas tentang pertanian. Jawaban: C

10. Kata ‘tren’ pada paragraf pertama dapat diganti kata …. a) pada umumnya b) dominan c) faktor determinan d) terkonsentrasi e) kecenderungan Pembahasan Kata ‘tren’ bisa bermakna ‘kecenderungan’. Jawaban: E

6

11. Penulisan Kata hubung yang paling tepat untuk melengkapi titik-titik pada paragraph ke-2 adalah …. a) sedangkan b) karena c) sehingga d) walaupun e) meskipun

Pembahasan Tanda koma (,) di antara kedua kalimat merupakan bentuk kata hubung yang menghubungkan dua buah kalimat, kata, ataupun klausa yang sederajat namun mempertentangkan kedua bagian tersebut.Kata hubung yang biasa dipakai pada konjungsi ini adalah tetapi, melainkan dan sedangkan. Kalimat yang tepat adalah “Menurut BPS, tahun 1993 rata-rata jumlah anggota keluarga penduduk miskin sebesar 5,9 orang, sedangkan jumlah keluarga rumah tangga yang hidup di atas garis kemiskinan lebih kecil dari itu.”. Jadi, kata hubung yang tepat melengkapi titik-titik pada paragraph ke-2 adalah sedangkan. Jawaban: A

12. Penulisan kata depan yang paling tepat untuk melengkapi titik-titik pada paragraph ke-1 adalah …. a) di b) karena c) ke d) daripada e) oleh

7

Pembahasan Kata depan adalah kata yang secara sintaksis terdapat didepan nomina, adjektiva, atau adverbia dan secara semantis menandai berbagai hubungan makna antara konstituen di depan dan di belakang preposisi tersebut. Pada kalimat, merupakan kata depan perbandingan. Jadi jawaban yang tepat adalah daripada. Jawaban: D

Bacalah tulisan berikut, untuk menjawab soal 13-20, kemudian jawablah soal-soal yang tersedia dengan memilih jawaban yang tepat di antara pilihan jawaban A, B, C, D, atau E. Nelayan yang berdampak reklamasi Terminal Petikemas pelabuhan Belawan Medan meminta ganti rugi kepada PT Pelabuhan Indonesia I. Nelayan rugi karena reklamasi menutup alur kapal nelayan sehingga mereka harus menambah 1 mil perjalanan setiap hari dengan tambahan 1 liter solar. Perwakilan nelayan terdampak menyampaikan hal tersebut pada rapat dengar pendapat di Dewan Perwakilan Rakyat Daerah Sumatera Utara, Medan. Reklamasi Pelabuhan Belawan sudah berlangsung tiga tahun. Namun, sampai sekarang, nelayan belum mendapat ganti rugi. Rapat yang dipimpin Ketua Komisi B DPRD Sumut itu dihadiri Kepala Bidang Perikanan Tangkap Dinas Pertanian dan Perikanan Kota Medan, Friska Irnawati Purba, serta Kepala Bidang Perencanaan dan Pembangunan Kantor Otoritas Pelabuhan Utama Belawan, Thomas Octavianus Sipa helut. PT Pelabuhan Indonesia I diundang, tetapi tidak hadir. Selama tiga tahun mereka belum mendapat ganti rugi. Baru pada 10 April, ada 3.228 nelayan dinyatakan sudah diverifikasi dan akan menerima tali kasih Rp 3.097.000 pernelayan. Delapan orang menerima dana secara simbolis. Sebanyak 300 nelayan tidak masuk dalam daftar nelayan yang terdampak reklamasi yang diverifikasi. Para nelayan hanya minta ganti rugi untuk tambahan bahan bakar selama empat tahun, sekitar 6 juta per nelayan. Petugas Pertanian dan Perikanan Kota Medan kesulitan melakukan verifikasi karena ada penolakan nelayan di lapangan. Akibatnya, sejumlah nelayan yang tidak masuk dalam daftar nelayan yang terdampak. Data hasil verifikasi terhadap 3.228 nelayan sudah final dan tidak ada tambahan lagi.

8

13. Kalimat inti kalimat pertama paragraf kedua teks tersebut adalah …. a) Nelayan terdampak. b) Nelayan menyampaikan hal tersebut. c) Perwakilan nelayan mendengarkan pendapat. d) Perwakilan nelayan menyampaikan hal tersebut. e) Nelayan dengar pendapat. Pembahasan Yang harus dilakukan untuk menemukan kalimat inti atau inti kalimat adalah menemukan subjek dan predikat. Caranya: Hilangkan semua perluasan (yang, mengenai, yakni, yaitu, seperti, dll) Hilangkan semua keterangan. Jika kalimat aslinya berupa kalimat majemuk, kalimat intinya berupa klausa utama atau klausa pokok kalimat tersebut. Perwakilan nelayan terdampak menyampaikan hal tersebut pada rapat dengar pendapat di Dewan Perwakilan Rakyat Daerah sumatera Utara, Medan. Jawaban: D

14. Topik bacaan tersebut adalah …. a) Keadaan nelayan reklamasi. b) Realisasi ganti rugi nelayan reklamasi. c) Dampak reklamasi. d) Solusi bagi nelayan reklamasi. e) Hak nelayan reklamasi. Pembahasan Topik bacaan sama seperti menemukan kalimat utama. Karena yang ditanyakan adalah topik keseluruhan isi bacaan, kita harus membaca seluruhnya dan menemukan hal yang diulang

9

atau yang dibahas terus-menerus. Topik bacaan ini adalah realisasi ganti rugi nelayan reklamasi. Jawaban: B 15. Pernyataan berikut yang paling benar dari teks tersebut adalah …. a) Realisasi ganti rugi yang diberikan sudah sesuai dengan tuntutan nelayan sekalipun belum terealisasi dengan baik. b) Seluruh nelayan yang telah masuk verifikasi akan segera mendapat ganti rugi reklamasi. c) Reklamasi Pelabuhan Belawan dinilai tidak efektif dan menimbulkan keresahan di masyarakat. d) Nelayan meminta ganti rugi bahan bakar dan biaya melaut selama empat tahun akibat dampak reklamasi. e) Sejumlah nelayan tidak masuk daftar nelayan terdampak karena adanya penolakan di lapangan. Pembahasan Pernyataan yang paling benar berarti yang paling sesuai dengan isi teks tersebut. Pernyataan yang lain kemungkinan juga benar tapi pembaca diminta menemukan satu jawaban yang paling tepat. Pernyataan tersebut terdapat di kalimat terakhir. Jawaban: E

16. Simpulan paragraf kedua teks tersebut adalah …. a) Nelayan menyambut positif hasil rapat tersebut. b) Dengan adanya rapat tersebut, nelayan optimis masalah mereka akan segera teratasi. c) Jadi, rapat tersebut adalah salah satu solusi yang dinanti-nantikan nelayan. d) Oleh karena itu, harus ada realisasi nyata dari rapat yang telah dilangsungkan tersebut. e) Hasil rapat tersebut dinilai menguntungkan nelayan dan merupakan solusi yang terbaik dari semua pihak.

10

Simpulan paragraf berarti hanya mengacu pada paragraf tersebut itu saja dan harus mencakup keseluruhan isi paragraf. Simpulan paragraf kedua teks tersebut ada di pilihan jawaban D Jawaban: D

17. Kesalahan EYD dalam kalimat tersebut ditemukan dalam kalimat …. a) Kalimat kedua paragraf ketiga. b) Kalimat ketiga paragraf kedua. c) Kalimat kelima paragraf keempat. d) Kalimat kesatu paragraf kedua. e) Kalimat keempat paragraf kedua. Pembahasan Kesalahan EYD meliputi pemakaian tanda baca yang salah, penulisan huruf, serta penulisan kata dan angka. Kesalahan penulisan EYD di kalimat tersebut ditemukan di kalimat (2) paragraf (3) tulisan pernelayan harus dipisah menjadi per nelayan. Penulisan per yang digabung jika menyatakan tiap (contoh: per minggu), demi (contoh: satu per satu), dan mulai (contoh 1 Desember 2017). Jawaban: A

18. Salah satu kalimat dalam paragraf tersebut tidak memenuhi unsur sebuah kalimat. a) Kalimat yang dimaksud adalah …. b) Kalimat kedua paragraf ketiga. c) Kalimat ketiga paragraf kedua. d) Kalimat kelima paragraf kesatu. e) Kalimat kesatu paragraf kedua. f) Kalimat kedelapan paragraf ketiga.

Pembahasan

11

Unsur mutlak sebuah kalimat harus terdiri atas subjek dan predikat. Kaliamt (8) paragraf (3) bacaan tersebut tidak mempunyai predikat karena diperluas dengan konjungsi yang. Jawaban: C

19. Nelayan terdampak reklamasi menyampaikan keluhannya. Reklamasi membuat nelayan rugi. Hasil penggabungan dua kalimat tersebut yang tepat adalah …. a) Nelayan terdampak menyampaikan keluhannya kalau reklamasi membuat nelayan rugi. b) Nelayan terdampak menyampaikan keluhannya jika reklamasi membuat nelayan rugi. c) Nelayan terdampak menyampaikan keluhannya bahwa reklamasi membuat nelayan rugi. d) Nelayan terdampak menyampaikan keluhannya tentang reklamasi membuat nelayan rugi. e) Nelayan terdampak menyampaikan keluhannya agar reklamasi tidak membuat nelayan rugi. Pembahasan Hasil penggabungan dua kalimat tersebut yang tepat adalah yang C. Pernyataan di kalimat tersebut sudah terjadi maka konjungsi yang tepat adalah konjungsi bahwa. Jawaban: C

20. Kata reklamasi dalam kalimat tersebut bermakna …. a) Pemanfaatan daerah yang tidak berguna. b) Pertanggungjawaban terhadap sesuatu. c) Penyelesaian suatu hal d) Pelaksanaan suatu kegiatan. e) Perwujudan sesuatu menjadi nyata.

12

Kata reklamasi di kalimat tersebut bermakna pemanfaatan daerah yang tidak berguna (lihat KBBIV) atau bisa dengan cara menyesuaikan dengan konteks kalimat. Jawaban: A

13

1. Pernyataan yang paling sesuai dengan isi tabel tersebut adalah …. a) pertumbuhan upah lebih sejajar dengan pembangunan infrastruktur daripada konsumsi swasta. b) arah pertumbuhan seluruh aspek perekonomian meningkat tajam, kecuali aspek konsumsi swasta. c) pertumbuhan aspek pembangunan infrastruktur dalam tiga tahun membentuk garis lurus yang meningkat. d) angka pertumbuhan pembangunan infrastruktur lebih tinggi daripada akumulasi dari ketiga aspek tersebut. e) konsumsi swasta merupakan aspek perekonomian yang paling rendah peningkatan di tahun terakhir. Pembahasan: Pernyataan yang sesuai dengan isi tabel adalah pertumbuhan aspek pembangunan infrastruktur dalam 3 tahun membentuk garis lurus yang meningkat. Jawaban: C

2. Kalimat yang tidak efektif terdapat pada …. a) b) c) d) e)

1 dan 7 2 dan 8 3 dan 11 5 dan 9 6 dan 10

Pembahasan: Kalimat efektif adalah kalimat yang menggunakan tata bahasa dan EYD yang tepat. Ciri kalimat efektif adalah subjek dan predikat (S-P), kata baku, hemat kata, sesuai EYD dan logis. Kalimat yang tidak efektif dalam paragraf tersebut adalah kalimat 6 dan 10. Jawaban: E

3. Simpulan yang paling tepat untuk teks tersebut adalah …. a) peningkatan produksi dan pendisverivikasian ekspor berdampak positif bagi pertumbuhan ekonomi Indonesia sehingga tercapai peningkatan kesejahteraan.

14

b) sinergi antara stabilitas ekonomi makro dan perbaikan sisi penawaran produksi Indonesia diprediksi akan berkontribusi pada peningkatan kesejahteraan masyarakat. c) kuatnya permintaan domestik Indonesia menyebabkan neraca transaksi akan tetap berada dalam kontribusi surplus namun dengan nilai surplus yang terus mengecil. d) harga komoditas yang tetap tinggi dan negara tujuan ekspor yang lebih terdiverifikasi aka menghasilkan dampak positif bagi produk ekspor Indonesia. e) pertumbuhan ekonomi yang tetap tinggi akan terjadi apabila peningkatan produksi dan perbaikan distribusi dilakukan sehingga tekanan terhadap inflasi akan berkurang. Pembahasan: Simpulan adalah sesuatu yang disimpulkan, hasil menyimpulkan. Simpulan juga berarti kesudahan pendapat (pendapat terakhir yang berdasarkan uraian sebelumnya) atau keputusan yang diperoleh berdasarkan metode berpikir induktif dan deduktif. Simpulan teks tersebut adalah Sinergi antara stabilitas ekonomi makro dan perbaikan sisi penawaran produksi Indonesia diprediksi akan berkontribusi pada peningkatan kesejahteraan masyarakat. Jawaban: B

4. Penulis teks tersebut bertujuan agar pembaca …. a) Menyimpulkan bahwa pertumbuhan ekonomi Indonesia akan berdampak langsung terhadap peningkatan kesejahteraan masyarakat. b) Memperoleh gambaran bahwa ekonomi Indonesia akan tetap tumbuh positif pada 2013 meskipun terjadi pelambatan pertumbuhan ekonomi dunia. c) Menyadari adanya hubungan yang erat antara stabilitas ekonomi, penawaran, kesejahteraan, masyarakat, penyerapan tenaga kerja, ekspor dan inflasi. d) Menyadari bahwa perbaikan infrastruktur, perizinan, bea cukai, dan perpajakan akan memberikan dukungan dalam meninkatkan daya saing ekspor. e) Mengetahui faktor-faktor yang dapat menunjang peningkatan perekonomian di Indonesia, baik ditinjau secara makro maupu mikro. Pembahasan: Penulis teks bertujuan agar pembaca mengetahui faktor-faktor yang dapat menunjang peningkatan perekonomian di Indonesia baik ditinjau secara makro maupun mikro. Jawaban: E

5. Apa kelemahan isi teks tersebut? a) Tingginya nilai impor Indonesia tahun 2013 yang diprediksi mencapai 14,5%- 15,5% tidak diikuti dengan penjelasan faktor penyebabnya.

15

b) Penjelasan tentang pertumbuhan ekonomi dari tahun ke tahun pada aspek konsumsi swasta tidak sejalan dengan isi tabel yang dirujuk. c) Pernyataan tentang meningkatnya daya beli riil masyarakat tidak setara dengan penjelasan tentang faktor penyebab keadaan tersebut. d) Positifnya pertumbuhan ekspor Indonesia akibat harga dan diverifikasi tidak diikuti dengan penjelasan besaran indeks ekspor tersebut. e) Perkiraan tentang meningkatnya data saing produk ekspor dengan Indonesia tidak disertai dengan paparan tentang faktor-faktor pendukung perkiraan itu. Pembahasan: Kelemahan isi teks tersebut adalah Positifnya pertumbuhan ekspor Indonesia akibat harga dan diversifikasi tidak diikuti dengan penjelasan tentang besaran indeks ekspor tersebut. Jawaban: D

6. Dalam majalah Bunda yang diterbitkan tahun 2010, pada halaman 37, terdapat kutipan pendapat Sugiyanto dalam artikel karya Agra Mashudi sebagai berikut: Orang gemuk dan anak-anak usia sekolah jauh lebih rentan terserang sakit hingga meninggal akibat virus flu babi H1N1 (Sugiyarto, 2008: 11). Jika Ikang mengutip pendapat Sugiyarto tanpa membaca bukunya, kutipan yang benar adalah ….

a) Menurut Sugiyarto (2008:11) orang gemuk dan anak-anak usia sekolah jauh lebih rentan terserang sakit hingga meninggal akibat virus flu babi H1N1 (dalam Mashudi, 2010). b) Sugiyarto (2008) dalam (Mashudi, 2010) menyatakan bahwa orang gemuk dan anakanak usia sekolah jauh lebih rentan terserang sakit hingga meninggal akibat virus flu babi H1N1. c) Sugiyarto (dalam Masyudi, 2010) menyatakan bahwa orang gemuk dan anak-anak usia sekolah jauh lebih rentan terserang sakit hingga meninggal akibat virus flu babi H1N1. d) Orang gemuk dan anak-anak usia sekolah jauh lebih rentan terserang sakit hingga meninggal akibat virus flu babi H1N1 (Sugiyarto, 2008 dalam Masyudi, 2010). e) Menurut Sugiyarto (2008:11 dalam Mashudi 2010;37), menyatakan bahwa ”bahwa orang gemuk dan anak-anak usia sekolah jauh lebih rentan terserang sakit hingga meninggal akibat virus flu babi H1N1.” Pembahasan: Kutipan adalah pinjaman pendapat dari seorang pengarang atau ucapan seseorang (ahli atau narasumber). Penulisan kutipan yang benar jika mengutip tanpa membaca buku adalah Menurut Sugiyarto dalam kurung (2008:11 dalam Mashudi, 2010: 37), menyatakan bahwa

16

“orang gemuk dan anak-anak usia sekolah jauh lebih rentan terserang sakit hingga meninggal akibat virus flu babi H1N1”. Jawaban: E

7. Sejumlah pengusaha sepatu kecil menengah di Kabupaten Mojokerto mengalami kesulitan menghadapi aturan perdagangan bebas atau free trade agreement (FTA) ASEAN-Chin. Mereka mengaku merugi sejak isu itu digulirkan pada awal Januari lalu. Para pedagang mengalami dampak negatif aturan itu. Pada sektor usada kecil sepatu, misalnya, pengusaha merugi sampai 50% “Orderan berkurang 50%. Artinya, produksi kami juga berkurang sebesar itu,” kata Budi Utomo, anggota Gabungan Pengusaha Sepatu (GPS) Kabupaten Mojokerto. Sepinya order terjadi karena sebagian besar pelanggan tidak melakukan pemesanan karena menunggu masuknya produk impor lebih murah dari pada harga produk lokal. Pernyataan berikut yang TIDAK SESUAI dengan isi paragraf diatas adalah … a) FTA ASEAN-China membuat panik sejumlah pengusaha sepatu kecil menengah di Kabupaten Mojokerto. b) Para pelanggan pengusaha sepatu di Kabupaten Mojokertolebih memilih produk sepatu impor dari pada membeli sepatu lokal. c) Sejak Januari lalu, kerugian akibat FTA ASEAN-China dirasakan para pengusaha sepatu kecil menengah di Kabupaten Mojokerto. d) Akibat FTA ASEAN-China kerugian para pengusah kecil di Kabupaten Mojokerto mencapai sekitar 50%. e) Banyak pelanggan pengusaha sepatu di Kabupaten Mojokerto menghentikan permintaan pesanan karena menunggu pelaksanaan FTA-ASEAN China. Pembahasan: Pernyataan yang tidak sesuai dengan teks adalah banyaknya pelanggan pengusaha sepatu di Kabupaten Mojokerto menghentikan permintaan pesanan karena menunggu pelaksanaan FTA ASEAN-China. Jawaban: E

Teks berikut ini untuk menjawab soal nomor 8-14. (1) Lembaga KPPOD bekerja sama dengan USAID dan The Asian Foundation (TAF) mengadakan survei daya saing daerah berdasarkan level economic governance. (2) Survei tersebut dilakukan pada 243 kabupaten/kota yang tersebar pada 15 provinsi di Indonesia. (3) Indikator kinerja yang digunakan dalam survei ini adalah manajemen infrastruktur, aksesbilitas, kepastian lahan, dan program pengembangan sektor swasta. (4) di samping itu terdapat empat indikator kinerja lainnya, yakni hubungan antara pemerintah daerah dan bisnis, biaya transaksi, integretas, dan resolusi konflik, serta kualitias dari peraturan daerah.

17

(5) berdasarkan hal tersebut disusunlah indeks tata kella ekonomi/ economic government index (EGI). (6) pembobotan untuk masing-masing indeks disusun menurut presentase perusahaan di mana memandang indeks tersebut sebagai masalah utama yang memengaruhi lingkungan bisnis. (7) hasilnya adalah empat permasalahan utama yang terdapat dalam pengelolahan ekonomi lokal di Indonesia terdiri atas permasalahan manajemen infrastruktur, pembangunan sektor swasta aksesbilitas dan kepastian lahan, serta hubungan pemda dn sektor bisnis. (9) masing- masing permaslahan ini diberi bobot secara berurutan, yakni 35,5%, 14,8%, dan 14%. (10) berdasarkan hasil survei terdapat empat provinsi yang konsisten memporoleh nilai EGI. (11) keempat provinsi itu dengan indeks tata kelolanya dapat diamati pada tabel berikut.

Provinsi Jawa Timur Sumatera Selatan Bali Jawa Barat

Kabupaten/Kota Kota Blitar Kota Madiun Kota Pribumilih Kab. Musi Banyuasin

Nilai EGI 76,0 72,0 74,7 74,3

Kab. Jemprana Kab. Gianyar

73,7 71,3

Kab. Ciamis Kab. Bekasi

67,9 54,8

8. Berdasarkan paragraf tersebut pernyataan yang benar adalah …. a) Indikator kinerja yang digunakan dalam survei daya siang daerah berdasarkan level economic governance ada enam. b) Pembobotan masing-masing indeks disusun menurut laba perusahaan yang terdapat dalam pengelolahan ekonomi nasional. c) Terdapat lebih dari empat provinsi yang konsisten memperoleh nilai EGI. d) Salah satu indikator utama yang terdapat dalam survei daya saing daerah di Indonesia adalah manajemen infrastruktur. e) Survei Lembaga KPPOD dilakukan di 243 kabupaten/kota yang tersebar di 10 provinsi di Indonesia. Pembahasan:

18

Pernyataan yang sesuai dengan isi paragraf tersebut dapat diketahui dengan membaca paragraf tersebut secara keseluruhan dan mencocokkan jawaban yang ada di pilihan jawaban satu per satu. Pilihan jawaban D ada di kalimat (3) paragraf (1) bacaan tersebut. Jawaban: D

9. Berdasarkan tabel Daya Saing Daerah Menurut EGI empat provinsi yang konsisten memperoleh nilai EGI adalah …. a) b) c) d) e)

Bali, Jawa Timur, Sumatera Selatan, Aceh Jawa Timur, Jawa Tengah, Jawa Barat, Bali Sumatera Selatan, Bali, Jawa Barat, Jawa Tengah Sumatera Selatan, Manado, Bali, Lampung Jawa Barat, Sumatera Selatan, Bali, Jawa Timur

Pembahasan: Jawaban yang benar diperoleh dengan cara melihat gambar tabel yang tersedia. Untuk membaca tabel atau grafik dibutuhkan kecermatan dan ketelitian. Jawaban: E

10. Berdasarkan tabel daya saing daerah Menurut EGI kabupaten/kota yang paling bermasalah adalah …. a) b) c) d) e)

Blitar Gianyar Bekasi Ciamis Madiun

Pembahasan: Berdasarkan tabel kabupaten atau kota yang paling bermasalah adalah Blitar. Jawaban: E

11. Berdasarkan tabel Daya Saing Daerah Menurut EGI, kesimpulan terhadap nilai EGI keempat provinsi di Indonesia adalah …. a) b) c) d)

Fluktuatif Variatif Cenderung sama Menurun

19

e) Stabil Pembahasan: Simpulan tabel adalah jawaban dari keseluruhan isi teks tersebut. Secara keseluruhan nilai EGI seluruh kabupaten atau kota cenderung sama.

Jawaban: C

12. Berdasarkan tabel Daya Saing Daerah Menurut EGI Jika Blitar tidak masuk kabupaten/kota yang dinilai maka indeks EGI tertinggi ada di provinsi …. a) b) c) d) e)

Jawa Timur Sumatera Selatan Bali Jawa Barat Jawa Tengah

Pembahasan: Jika Blitar tidak termasuk dalam Kabupaten atau Kota yang ada di tabel tersebut maka nilai EGI terbesar ada di Provinsi Sumatera Selatan . Jawaban: B

13. Di dalam paragraf tersebut terdapat kalimat yang tidak sesuai dengan kaidah Bahasa Indonesia yang baku. Kalimat yang dimaksud adalah …. a) b) c) d) e)

(1) (2) (3) (4) (6)

Pembahasan: INGAT! Kalimat baku atau efektif memiliki enam syarat: – Terdiri atas subjek dan predikat – Hemat kata – Logis atau masuk akal – Sesuai dengan EYD (tanda baca, penulisan huruf, penulisan kata, dan penulisan angka)

20

– Tidak ambigu – Paralel atau sejajar Di dalam paragraf tersebut terdapat kalimat yang tidak sesuai dengan kaidah Bahasa Indonesia yang baku. Kalimat yang tidak sesuai dengan kaidah Bahasa Indonesia adalah kalimat (6). Kalimat tersebut tidak baku karena menggunakan kata dimana. Kata dimana tidak boleh digunakan di dalam kalimat yang bukan kalimat tanya. Kata dimana seharusnya diubah menjadi kata yang. Jawaban: D

14. Kata memengaruhi dalam kalimat tersebut …. a) b) c) d) e)

Tidak perlu diperbaiki Mempengaruhi Pengaruhi Dipengaruhi Terpengaruhi

Pembahasan: Kata mempengaruhi dalam kalimat tersebut tidak perlu diperbaiki karena kata tersebut terkena peluruhan kata. INGAT! Kata dasar yang huruf I K, T, S, P + Huruf II vokal + imbuhan me, me-i, me-kan = huruf 1 luluh. Jawaban: A

15. Jika p adalah bilangan habis dibagi 7 dan nilainya lebih besar daripada 14 dan kurang daripada 28, sedangkan q adalah bilangan ganjil yang nilainya di antara 19 dan 23, maka pernyataan yang paling tepat adalah …. a) b) c) d) e)

p=q p>q p
Pembahasan:

14
21

Jawaban: A

16. Jika a ≥ 3, maka nilai 5a + 3 adalah …. a) b) c) d) e)

≥8 >18 ≥ 18 >23 ≥ 23

Pembahasan: a≥3 5a ≥ 3 . 5 5a + 3 ≥ 15 + 3 5a + 3 ≥ 18 Jawaban: C

17. Jika p = 2a + 3 dan q = a + 6, maka 2p + 4q = …. a) b) c) d) e)

6a + 18 6a + 30 6a + 27 8a + 30 8a + 27

Pembahasan: = 2(2a + 3) + 4(a + 6) = 4a + 6 + 4a + 24

= 8a + 30 Jawaban: D

18. Jika p = q + 2, q = r – 1 dan r = 5, maka yang benar adalah …. a) p < q < r

22

b) c) d) e)

p
Pembahasan: q=4 p=6 maka 4, 5, 6 q< r < p Jawaban: C

19. Jika a = b, maka 4a + 3b = …. a) b) c) d) e)

7 7b 7ab 2ab 2a

Pembahasan: 4a + 3b = 4b + 3b = 7b

Jawaban: B

20. Jika a × b = 12 dengan a dan b adalah bilangan bulat positif, maka nilai maksimum a + b – 1 adalah …. a) b) c) d) e)

6 7 8 12 13

Pembahasan: ab = 12

23

kemungkinan (1, 12)(3, 4)(2, 6) maks a + b – 1 maka, 1 + 12 – 1 = 12

Jawaban: D

24

25

26

27

28

29

30

31

32

33

34

35

36

37

38

39

40

41

42

43

44

45

46

47

48

49

50

51

52

53

54

55

56

57

58

59

60

61

62

63

64

65

66

67

68

69

70

71

72

73

74

75

76

77

78

79

80

81

82

83

84

85

86

87

88

89

90

91

92

93

94

SBMPTN

Informasi Bentuk Tes UTBK 2020 Kelompok Saintek Tes UTBK terdiri atas 2 bagian yaitu (1) Tes Potensi Skolastik (TPS) dan (2) Tes Kemampuan Akademik (TKA). (1) Tes Potensi Skolastik (TPS) TPS mengukur kemampuan berpikir yang diperlukan untuk keberhasilan proses pendidikan di sekolah formal, khususnya di perguruan tinggi. Materi TPS adalah kemampuan penalaran umum, kemampuan kuantitatif (termasuk matematika), pengetahuan dan pemahaman umum, serta kemampuan memahami bacaan dan menulis. Masing-masing komponen ini dirinci pada Tabel 1. Tabel 1 Komposisi Materi TPS Alokasi Waktu No Komponen Jumlah soal (menit) 1 Penalaran umum 2 bacaan dikuti 14 soal, dan 35 6 soal penalaran kuantitatif 2 Pemahaman bacaan dan 3 bacaan diikuti 20 soal 25 menulis 3 Pengetahuan dan 3 bacaan Indonesia dengan 25 pemahaman umum 12 soal, dan 2 bacaan Bahasa Inggris dengan 8 soal. 4 Pengetahuan kuantitatif 20 soal pengetahuan 35 kuantitatif Catatan: Anda hanya dapat mengerjakan soal-soal dalam tiap komponen tes dalam rentang waktu yang telah dialokasikan. Berikut ini diberikan beberapa contoh soal untuk memberikan gambaran berbagai tipe soal yang diujikan pada TPS agar semua calon peserta UTBK mendapatkan informasi yang sama tentang materi UTBK. Soal-soal yang digunakan sebagai contoh ini bukan soal sebenarnya dan bukan berasal dari kisi-kisi soal UTBK 2019.



Contoh 1 Penalaran Umum Teks berikut ini digunakan untuk menjawab soal 1 sampai dengan 6. Gunung berapi merupakan salah satu obyek alam yang mengagumkan namun menyimpan kekuatan berapi-api. Di dalamnya, gunung berapi menyimpan berbagai material seperti batuan cair, puing-puing dan gas yang dapat dipancarkan kapan pun. Apabila meletus, gunung berapi mengeluarkan lava, batu, dan abu ke udara yang dapat membahayakan lingkungan sekitarnya. Gunung yang akan meletus biasanya mengalami kenaikan suhu di sekitarnya. Selain itu, terdapat ciri-ciri lain seperti keringnya mata air, suara gemuruh, dan migrasi hewan di sekitar gunung. Beberapa gunung berapi yang tercatat berbahaya adalah Gunung Agung, Gunung Papandayan, Gunung Kelud, Gunung Sinabung, Gunung Krakatau, dan Gunung Merapi. Indonesia memiliki 127 gunung berapi yang masih aktif. Dari 127 gunung tersebut, sebanyak 69 gunung menunjukkan aktivitas magma di dalamnya sehingga perlu mendapatkan pantauan intensif. Besarnya letusan gunung berapi diukur melalui suatu indeks letusan yang disebut Volcanic Explosivity Index (VEI). Di antara berbagai gunung berapi di Indonesia, Gunung Merapi merupakan salah satu gunung yang memiliki indeks letusan yang termasuk tinggi sehingga mendapatkan perhatian intensif. Gunung Merapi terletak di sebelah utara kota Yogyakarta. Gunung tersebut memiliki tinggi 1700 meter dan tercatat mengalami erupsi berkali-kali. Beberapa catatan menunjukkan indeks letusan yang tergolong tinggi. Berdasarkan 15 data yang tercatat sejak tahun 1780, setiap kali Gunung Merapi mengalami erupsi akan diikuti masa istirahat sekitar 3-5 tahun. Data letusan Gunung Merapi disajikan pada gambar 1. (Diadaptasi dari https://www.bbc.com dan Kiswiranti, D. (2015). Analisis Statistik Temporal Erupsi Gunungapi di Indonesia. Berkala Ilmiah MIPA, 23(3).)

1. Berdasarkan paragraf 1, manakah di bawah ini pernyataan yang BENAR? A. Gunung Agung memiliki mata air yang kering. B. Gunung Agung mengeluarkan lava yang berbahaya. C. Gunung Agung berbahaya bagi lingkungan sekitarnya. D. Gunung Agung memiliki suhu yang lebih tinggi dari gunung lainnya. E. Gunung Agung menyimpan material yang dapat dipancarkan kapanpun.

2. Berdasarkan paragraf 1, jika gunung berapi tidak mengeluarkan lava, batu, dan abu ke udara, manakah di bawah ini simpulan yang BENAR? A. Gunung berapi tidak aktif. B. Gunung berapi tidak meletus. C. Gunung berapi tidak berbahaya. D. Gunung berapi tidak mengalami kenaikan suhu. E. Gunung berapi tidak mengalami kekeringan. 3. Berdasarkan paragraf 1, apabila di suatu gunung berapi terjadi kekeringan mata air, suara gemuruh dan migrasi hewan di sekitar gunung, manakah di bawah ini simpulan yang PALING MUNGKIN benar? A. Gunung tersebut pernah meletus. B. Gunung tersebut sedang meletus. C. Gunung tersebut telah meletus. D. Gunung tersebut akan meletus. E. Gunung tersebut baru saja meletus. 4. Berdasarkan paragraf 3, manakah pernyataan di bawah ini yang PALING MUNGKIN benar mengenai Gunung Merapi? A. Akan erupsi secara rutin dan diselingi dengan masa istirahat. B. Akan erupsi setiap tahun dengan letusan yang tinggi. C. Erupsi dengan indeks letusan yang lebih kecil daripada sebelumnya. D. Sebelum tahun 1780 masa istirahatnya kurang dari 3 tahun. E. Setelah tahun 1780 masa istirahatnya lebih dari 5 tahun. 5. Berdasarkan Gambar 1, pada tahun berapakah Gunung Merapi menunjukkan VEI kedua tertinggi? A. 1920 dan 1961 B. 1920 dan 1956 C. 1930 dan 1961 D. 1930 dan 2010 E. 1961 dan 2010 6. Berdasarkan Gambar 1, apa yang PALING MUNGKIN terjadi jika setelah tahun 2010 Gunung Merapi meletus? A. VEI-nya akan sebesar 2. B. VEI-nya akan sebesar 4. C. VEI-nya akan lebih rendah dari 2. D. VEI-nya akan lebih rendah dari 4. E. VEI-nya akan lebih tinggi dari 4. Jawaban: 1 E

2 B

3 D

4 A

5 C

6 D

Contoh 2 Pemahaman Bacaan dan Menulis Bacalah tulisan berikut, kemudian jawablah soal-soal yang tersedia dengan memilih jawaban yang tepat di antara pilihan jawaban A, B, C, D, atau E. [...] 1Penelitian terbaru menyatakan bahwa larva nyamuk yang menelan mikroplastik dapat menyebarkan mikroplastik ke rantai makanan manusia. 2Diketahui mikroplastik, potongan kecil plastik yang terpecah dari produk buatan manusia, seperti pakaian sintetis, ban mobil, dan lensa kontak, memenuhi lautan dunia saat ini. 3Karena sulit dideteksi, mikroplastik dapat menimbulkan masalah serius bagi hewan laut. 4Baru-baru ini para peneliti dari Universitas Reading menemukan bukti yang menunjukan bagaimana mikroplastik dapat memasuki ekosistem manusia melalui nyamuk dan serangga terbang lainnya. 5Tim peneliti tersebut mengamati larva nyamuk yang menelan microbeads–jenis mikroplastik yang biasa ditemukan pada produk kosmetik—sebelum mengamati siklus kehidupannya. 6Mereka menemukan fakta bahwa banyak sekali partikel plastik yang [...] ke tubuh nyamuk dewasa. 1. Judul yang paling tepat untuk melengkapi tulisan di atas adalah …. A. Serangga Pemakan Mikroplastik B. Bahaya Mikroplastik bagi Tubuh Manusia C. Penyebaran Mikroplastik oleh Serangga D. Mikroplastik, Serangga, Makhluk Hidup E. Cara Baru Penyebaran Mikroplastik 2. Kata berimbuhan yang salah digunakan pada paragraf 2 adalah …. A. menemukan B. menunjukan C. memasuki D. menelan E. mengamati

3. Kata yang paling tepat untuk melengkapi titik-titik pada kalimat 6 adalah …. A. tergantikan B. tertransfer C. berpindah D. teralihkan E. berubah

Tulisan berikut diikuti oleh dua butir pertanyaan. Pertimbangkan apakah kata atau kalimat pada setiap nomor bercetak tebal TIDAK PERLU DIPERBAIKI (A) atau diganti dengan pilihan lain yang tersedia (B, C, D, atau E). Setiap suku memiliki ritual tersendiri, mulai dari ritual yang biasa hingga 4ekstrem, untuk menunjukkan rasa duka akibat kehilangan anggota keluarganya. Bagi suku Dani yang bermukim di 5papua, kebersamaan sangat penting. Oleh karena itu, saat kehilangan anggota keluarga, mereka akan segera memotong ruas jarinya. Tradisi itu dikenal sebagai Iki Palek.

4. A. B. C. D. E.

TIDAK PERLU DIPERBAIKI aneh lazim ganjil luar biasa

A. B. C. D. E.

TIDAK PERLU DIPERBAIKI Papua Papua “Papua” “Papua”

5.

Jawaban: 1 C

2 B

3 C

4 A

5 B

Contoh 3 Pengetahuan dan Pemahaman Umum Bacalah tulisan berikut. Kemudian jawablah soal-soal yang tersedia dengan memilih jawaban yang tepat di antara pilihan jawaban A, B, C, D, atau E. Sebelum mencapai masa dewasa, anak-anak melewati masa remaja. Masa remaja dimulai dari usia 11 tahun sampai dengan 20 tahun. Setelah masa itu, remaja akan beranjak ke masa dewasa dan mencari jati diri mereka. Masa remaja merupakan fase rawan karena pengaruh baik dan buruk dapat masuk dengan mudah. Oleh karena itu, fase inilah yang nantinya akan menentukan karakter remaja setelah dewasa. Tidak hanya mengalami perkembangan dari segi fisik, remaja juga mengalami perkembangan psikis yang mengakibatkan emosi mereka bergejolak. Oleh karena itu, remaja harus mampu mengendalikan emosinya, baik di rumah maupun di sekolah, serta di lingkungan masyarakat. Pada masa rawan ini peran orang tua sangat diperlukan agar remaja tidak terjerumus ke dalam hal-hal negatif. Pendidikan karakter di sekolah dapat membantu para remaja melakukan hal yang positif. Pendidikan karakter mengajarkan para remaja untuk hidup dengan berpegang teguh pada agama dan norma-norma yang berlaku. Dengan demikian dapat disimpulkan bahwa pendidikan karakter sangat penting bagi remaja untuk membantu mereka melakukan kegiatan yang positif. Kegiatan remaja yang positif akan memicu perubahan yang positif pula pada dirinya. Jika remaja sebagai generasi penerus berkualitas baik, negara akan maju. 1. Apa lawan makna kata ‘membantu’ di paragraf 3 bacaan di atas? A. merantai B. mengikat C. memasung D. menghambat E. membendung

2. Apa gagasan utama paragraf 3 pada bacaan di atas? A. Masa remaja adalah fase rawan. B. Remaja perlu melakukan hal-hal yang positif. C. Pendidikan karakter sangat penting bagi remaja. D. Orang tua memiliki peran penting bagi remaja di masa pubertas. E. Remaja mengalami perkembangan fisik dan psikis dengan pesat. Jawaban: 1 D

2 C

Contoh 4 Pengetahuan dan Pemahaman Umum dalam Bahasa Inggris Questions 1—4 are based on the following passage. Forests have the ability to regulate water systems, prevent erosion and flood, and maintain soil fertility. The ability of forests is inseparable from the existence of millions and even billions of trees in a forest area. Trees only store water for their own needs, and the land saves water. Billions of trees with their wide canopies will hold back rain. In this position, rain water does not directly hit the soil surface, but will fall slowly through the leaves and flow through the tree trunk. Then, the soil surface filled with tree trunks will produce quite a lot of litter originating from organic materials in the forms of leaf and dry twigs. Dramatically, littering is heading towards the decay process. The organic materials are collected on the soil surface. Litter blocks the water falling from the canopy so it does not directly hit the soil. Litter also functions as a place to live for millions of organisms (e.g. worms). This organism punctures the land as a home and place of life. This organism’s behavior causes the soil surface to become loose and porous. When rain drops from the canopy fall onto the litter, the water slowly flows to the soil surface. The loose and porous top layer of soil will absorb the water and then the water will be stored in the aquifer, the underground river. The soil surface in the forest has high capacity to absorb rainwater. As a result, most of the rainwater seeps into the soil; only a little becomes running water. Running water is the water that cannot be absorbed by the soil surface. This water will go down to a lower area. If running water exceeds the carrying capacity of the river, it can certainly cause flood. Most of the rainwater that falls in the forest area will be absorbed by the soil and stored in the aquifer. Furthermore, the water stored in the aquifer will come out regularly through springs. From these springs, water flows through rivers that are mostly found in the forest area. In addition, there is also underground water that comes out as a spring in the resident wells. 1. The word ’dramatically’ in paragraph 2 is best replaced by .... A. gradually B. naturally C. amazingly D. surprisingly E. simultaneously



2. What is the main idea of the passage? A. Water guarantees millions of organisms to survive in forest areas. B. Land stores and releases water through various processes. C. Litter gradually flows water to the soil surface. D. Trees store water to fulfill human daily needs. E. Forests have various benefits for human life. 3. With the sentence ‘This organism's behavior causes the soil surface to become loose and porous’ in paragraph 3, the writer intends to .... A. describe the organism and its behavior against loose soil B. confirm the behavior of organisms in loose soil C. compare the loose soil with the porous soil D. uncover natural processes at soil erosion E. explain the causes of soil erosion 4. What most likely motivates the writer in writing the passage? A. There is an unresolved flood problem. B. People’s awareness towards reforestation is low. C. Not many people understand the benefits of water for forests. D. There is the fallacious concept that trees store large amount of water. E. The information about the importance of trees in storing water is not available. Jawaban: 1 A

2 B

3 E

4 D

Contoh 5 Soal Pengetahuan Kuantitatif 1. Bilangan berikut yang merupakan bilangan kuadrat dan sekaligus bilangan pangkat tiga adalah …. A. 8 B. 27 C. 64 D. 81 E. 100 2. Manakah di antara bangun berikut yang merupakan bangun datar? 1. Persegi 2. Balok 3. Trapesium 4. Kerucut A. (1), (2), dan (3) SAJA yang benar. B. (1) dan (3) SAJA yang benar. C. (2) dan (4) SAJA yang benar. D. HANYA (4) yang benar. E. SEMUA pilihan benar.

3. Misalkan x, y, dan z menyatakan bilangan real yang memenuhi persamaan x + 2y + 3z = 10. Berapakah nilai x? Putuskan apakah pernyataan (1) dan (2) berikut cukup untuk menjawab pertanyaan tersebut. 1. z = 1 2. x + y = 5 A. Pernyataan (1) SAJA cukup untuk menjawab pertanyaan, tetapi pernyataan (2) SAJA tidak cukup. B. Pernyataan (2) SAJA cukup untuk menjawab pertanyaan, tetapi pernyataan (1) SAJA tidak cukup. C. DUA pernyataan BERSAMA-SAMA cukup untuk menjawab pertanyaan, tetapi SATU pernyataan SAJA tidak cukup. D. Pernyataan (1) SAJA cukup untuk menjawab pertanyaan dan pernyataan (2) SAJA cukup. E. Pernyataan (1) dan pernyataan (2) tidak cukup untuk menjawab pertanyaan. 4. Tersedia lima kursi yang disusun berjajar dengan setiap kursi ditempati paling banyak satu orang. Manakah hubungan yang benar antara kuantitas P dan Q berikut berdasarkan informasi yang diberikan? P Q Banyak susunan empat orang duduk pada kursi 24 yang disediakan A. P > Q. B. Q > P. C. P = Q. D. Informasi yang diberikan tidak cukup untuk memutuskan salah satu dari tiga pilihan di atas. Jawaban: 1 C

2 B

3 C

4 A

(2) Tes Kemampuan Akademik (TKA) Saintek Tes Kemampuan Akademik (TKA) mengukur pengetahuan dan pemahaman keilmuan yang diajarkan di sekolah dan diperlukan untuk berhasil dalam menempuh pendidikan tinggi. Tes ditekankan pada pengukuran penalaran tingkat tinggi (higher order thinking skills). TKA Saitek terdiri atas Matematika, Fisika, Kimia dan Biologi. Tabel 2 berisi rincian jumlah soal alokasi waktu untuk masing-masing komponen yang diujikan. Tabel 2 Komponen Materi TKA Saintek No Komponen Jumlah soal Waktu (menit) 1 Matematika 20 22,5 2 Fisika 20 22,5 3 Kimia 20 22,5 4 Biologi 20 22,5 Catatan: Anda hanya dapat mengerjakan soal-soal dalam tiap komponen tes dalam rentang waktu yang telah dialokasikan. Berikut adalah beberapa contoh soal TKA Saintek. Contoh yang diberikan juga bukan soal sebenarnya atau bukan berasal dari kisi-kisi soal UTBK 2019, tetapi hanya untuk memberikan gambaran bentuk-bentuk soal yang akan diujikan. Contoh Soal TKA Saintek Contoh 1 Matematika 1. Dua garis yang sejajar disajikan dalam bentuk matriks sebagai 𝑥 2 𝑎 5 ! ' ! ' = ! '. Nilai ab adalah …. 𝑏 −6 𝑦 7 A. – 12 B. – 3 C. 0 D. 12 E. 21 2. Diketahui gradien garis singgung grafik fungsi f di titik P adalah empat kali negatif absis P. Grafik tersebut melalui titik Q(2,42). Nilai maksimum f sebesar b tercapai di x = a. Nilai 3a+b adalah .... A. 0 B. 34 C. 48 D. 50 E. 60 3.

4 Jika 𝑔(𝑥) = ∫5 𝑓(𝑡)𝑑𝑡, untuk 0 ≤ 𝑥 ≤ 7, maka ... A. 𝑔(𝑥) mencapai nilai minimum di 𝑥 = 1

B. C. D. E.

𝑔(𝑥) mencapai nilai minimum di 𝑥 = 7 𝑔(𝑥) mencapai nilai maksimum di 𝑥 = 2 𝑔(𝑥) mencapai nilai maksimum di 𝑥 = 4 𝑔(𝑥) mencapai nilai maksimum di 𝑥 = 6

Jawaban: 1 A

2 D

3 E

Contoh 2 Fisika 1. Sebuah benda bergerak sepanjang sumbu x dengan percepatan a(t) = -4t + 3. Mulamula benda bergerak ke arah sumbu x positif dari keadaan diam. Manakah pernyataan berikut yang benar? A. Pada t = 0,75 detik benda berhenti. B. Sebelum t = 0,75 detik benda mengalami perlambatan sampai berhenti sesaat. C. Setelah t = 0,75 detik benda mengalami percepatan kemudian bergerak konstan. D. Setelah t = 0,75 detik benda mengalami perlambatan kemudian bergerak konstan. E. Setelah t = 0,75 detik benda diperlambat sampai berhenti sesaat kemudian dipercepat. 2. Dua buah sumber tegangan, sebuah lampu, dan sebuah amperemeter ideal dirangkai seperti ditunjukkan gambar.

Nilai sumber tegangan 𝜀< dapat diubah-ubah, sedangkan nilai sumber tegangan 𝜀= tetap. Ketika 𝜀< diatur sama dengan 2 V arus yang terbaca pada amperemeter 40 mA. Ketika 𝜀< diatur sama dengan 5 V arus yang terbaca pada amperemeter 10 mA. Nilai hambatan listrik lampu sama dengan …. A. 100 ohm B. 150 ohm C. 200 ohm D. 250 ohm E. 300 ohm Jawaban: 1 E

2 A

Contoh 3 Kimia Informasi berikut digunakan untuk menjawab soal nomor 1 dan 2. Kadar ion besi dalam darah dapat ditentukan dengan metode destruksi menggunakan H= O= dalam suasana asam. Reaksi yang terjadi adalah sebagai berikut 2Fe=B (𝑎𝑞) + H= O= (𝑎𝑞) + 2HB (𝑎𝑞) → 2FeGB (𝑎𝑞) + 2H= O(𝑙) Satu molekul hemoglobin mengandung 4 ion besi. 1. Pernyataan yang benar untuk reaksi di atas adalah …. A. H= O= berfungsi sebagai oksidator B. destruksi merupakan reaksi reduksi besi(III) dalam darah C. asam berfungsi sebagai elektrolit D. oksidasi 1 mol Fe=B setara dengan oksidasi 1 mol hemoglobin E. reaksi dapat berlangsung tanpa katalis 2. Bilangan oksidasi oksigen dalam senyawa peroksida yang digunakan dalam reaksi destruksi di atas adalah …. A. –2 B. –1 C. 0 D. +1 E. +2 3. Sebanyak 1 mL HCl 1,0 M diencerkan dengan H= O hingga volumenya menjadi 100 mL. Kemudian 50 mL larutan ini direaksikan dengan 50 mL NHG 0,02 M. Jika 𝐾K NHG = 10LM maka pH larutan yang terbentuk adalah …. A. 5 B. 7 C. 9 D. 11 E. 12 4. Senyawa yang jika dioksidasi menggunakan kalium bikromat menghasilkan asam adalah …. A. 2,3-butandiol B. isobutanol C. isopropanol D. sec-butanol E. t-butanol Jawaban: 1 A

2 B

3 C

4 B

Contoh 4 Biologi 1. Faktor lingkungan yang TIDAK menyebabkan tingginya produktivitas hutan hujan tropis adalah …. A. suhu udara yang hangat

B. C. D. E.

tingginya curah huja tanah lamanya intensitas cahaya matahari tingginya keanekaragaman tumbuhan

Partenokarpi adalah pembentukan buah tanpa fertilisasi sedangkan apomiksis adalah pembentukan biji tanpa fertilisasi. Tumbuhan yang mengalami partenokarpi buahnya tidak memiliki biji misalnya pisang Musa paradisiaca), sedangkan tumbuhan yang mengalami apomiksis buahnya memiliki biji misalnya manggis (Garcinia mangostana). Akibatnya, secara genetik tumbuhan-tumbuhan baru yang muncul identik dengan induknya (klon). Walaupun demikian di alam kita menemukan adanya variasi pada manggis dan pisang. Hal ini diduga terjadi karena proses mutasi. 2. Tumbuhan yang mengalami apomiksis selain manggis adalah …. A. salak B. mangga C. duku D. nangka E. durian 3. Tumbuhan yang TIDAK mengalami partenokarpi selain pisang adalah …. A. sirsak B. nanas C. terong D. timun E. sukun 4. Partenokarpi buatan umum dilakukan pada tanaman …. A. cabai B. tomat C. semangka D. jeruk E. buncis

Jawaban:

1 C

2 C

3 A

4 C

Informasi Bentuk Tes UTBK 2020 Kode 611 - Kelompok Soshum

Tes UTBK terdiri atas 2 bagian yaitu (1) Tes Potensi Skolastik (TPS) dan (2) Tes Kemampuan Akademik (TKA). (1) Tes Potensi Skolastik (TPS) TPS mengukur kemampuan berpikir yang diperlukan untuk keberhasilan proses pendidikan di sekolah formal, khususnya di perguruan tinggi. Materi TPS adalah kemampuan penalaran umum, kemampuan kuantitatif (termasuk matematika), pengetahuan dan pemahaman umum, serta kemampuan memahami bacaan dan menulis. Masing-masing komponen ini dirinci pada Tabel 1. Tabel 1 Komposisi Materi TPS Alokasi Waktu No Komponen Jumlah soal (menit) 1 Penalaran umum 2 bacaan dikuti 14 soal, dan 35 6 soal penalaran kuantitatif 2 Pemahaman bacaan dan 3 bacaan diikuti 20 soal 25 menulis 3 Pengetahuan dan 3 bacaan Indonesia dengan 25 pemahaman umum 12 soal, dan 2 bacaan Bahasa Inggris dengan 8 soal. 4 Pengetahuan kuantitatif 20 soal pengetahuan 35 kuantitatif Catatan: Anda hanya dapat mengerjakan soal-soal dalam tiap komponen tes dalam rentang waktu yang telah dialokasikan. Berikut ini diberikan beberapa contoh soal untuk memberikan gambaran berbagai tipe soal yang diujikan pada TPS agar semua calon peserta UTBK mendapatkan informasi yang sama tentang materi UTBK. Soal-soal yang digunakan sebagai contoh ini bukan soal sebenarnya dan bukan berasal dari kisi-kisi soal UTBK 2019.



Contoh 1 Penalaran Umum Teks berikut ini digunakan untuk menjawab soal 1 sampai dengan 6. Gunung berapi merupakan salah satu obyek alam yang mengagumkan namun menyimpan kekuatan berapi-api. Di dalamnya, gunung berapi menyimpan berbagai material seperti batuan cair, puing-puing dan gas yang dapat dipancarkan kapan pun. Apabila meletus, gunung berapi mengeluarkan lava, batu, dan abu ke udara yang dapat membahayakan lingkungan sekitarnya. Gunung yang akan meletus biasanya mengalami kenaikan suhu di sekitarnya. Selain itu, terdapat ciri-ciri lain seperti keringnya mata air, suara gemuruh, dan migrasi hewan di sekitar gunung. Beberapa gunung berapi yang tercatat berbahaya adalah Gunung Agung, Gunung Papandayan, Gunung Kelud, Gunung Sinabung, Gunung Krakatau, dan Gunung Merapi. Indonesia memiliki 127 gunung berapi yang masih aktif. Dari 127 gunung tersebut, sebanyak 69 gunung menunjukkan aktivitas magma di dalamnya sehingga perlu mendapatkan pantauan intensif. Besarnya letusan gunung berapi diukur melalui suatu indeks letusan yang disebut Volcanic Explosivity Index (VEI). Di antara berbagai gunung berapi di Indonesia, Gunung Merapi merupakan salah satu gunung yang memiliki indeks letusan yang termasuk tinggi sehingga mendapatkan perhatian intensif. Gunung Merapi terletak di sebelah utara kota Yogyakarta. Gunung tersebut memiliki tinggi 1700 meter dan tercatat mengalami erupsi berkali-kali. Beberapa catatan menunjukkan indeks letusan yang tergolong tinggi. Berdasarkan 15 data yang tercatat sejak tahun 1780, setiap kali Gunung Merapi mengalami erupsi akan diikuti masa istirahat sekitar 3-5 tahun. Data letusan Gunung Merapi disajikan pada gambar 1. (Diadaptasi dari https://www.bbc.com dan Kiswiranti, D. (2015). Analisis Statistik Temporal Erupsi Gunungapi di Indonesia. Berkala Ilmiah MIPA, 23(3).)

1. Berdasarkan paragraf 1, manakah di bawah ini pernyataan yang BENAR? A. Gunung Agung memiliki mata air yang kering. B. Gunung Agung mengeluarkan lava yang berbahaya. C. Gunung Agung berbahaya bagi lingkungan sekitarnya. D. Gunung Agung memiliki suhu yang lebih tinggi dari gunung lainnya. E. Gunung Agung menyimpan material yang dapat dipancarkan kapanpun.

2. Berdasarkan paragraf 1, jika gunung berapi tidak mengeluarkan lava, batu, dan abu ke udara, manakah di bawah ini simpulan yang BENAR? A. Gunung berapi tidak aktif. B. Gunung berapi tidak meletus. C. Gunung berapi tidak berbahaya. D. Gunung berapi tidak mengalami kenaikan suhu. E. Gunung berapi tidak mengalami kekeringan. 3. Berdasarkan paragraf 1, apabila di suatu gunung berapi terjadi kekeringan mata air, suara gemuruh dan migrasi hewan di sekitar gunung, manakah di bawah ini simpulan yang PALING MUNGKIN benar? A. Gunung tersebut pernah meletus. B. Gunung tersebut sedang meletus. C. Gunung tersebut telah meletus. D. Gunung tersebut akan meletus. E. Gunung tersebut baru saja meletus. 4. Berdasarkan paragraf 3, manakah pernyataan di bawah ini yang PALING MUNGKIN benar mengenai Gunung Merapi? A. Akan erupsi secara rutin dan diselingi dengan masa istirahat. B. Akan erupsi setiap tahun dengan letusan yang tinggi. C. Erupsi dengan indeks letusan yang lebih kecil daripada sebelumnya. D. Sebelum tahun 1780 masa istirahatnya kurang dari 3 tahun. E. Setelah tahun 1780 masa istirahatnya lebih dari 5 tahun. 5. Berdasarkan Gambar 1, pada tahun berapakah Gunung Merapi menunjukkan VEI kedua tertinggi? A. 1920 dan 1961 B. 1920 dan 1956 C. 1930 dan 1961 D. 1930 dan 2010 E. 1961 dan 2010 6. Berdasarkan Gambar 1, apa yang PALING MUNGKIN terjadi jika setelah tahun 2010 Gunung Merapi meletus? A. VEI-nya akan sebesar 2. B. VEI-nya akan sebesar 4. C. VEI-nya akan lebih rendah dari 2. D. VEI-nya akan lebih rendah dari 4. E. VEI-nya akan lebih tinggi dari 4. Jawaban: 1 E

2 B

3 D

4 A

5 C

6 D

Contoh 2 Pemahaman Bacaan dan Menulis Bacalah tulisan berikut, kemudian jawablah soal-soal yang tersedia dengan memilih jawaban yang tepat di antara pilihan jawaban A, B, C, D, atau E. [...] 1Penelitian terbaru menyatakan bahwa larva nyamuk yang menelan mikroplastik dapat menyebarkan mikroplastik ke rantai makanan manusia. 2Diketahui mikroplastik, potongan kecil plastik yang terpecah dari produk buatan manusia, seperti pakaian sintetis, ban mobil, dan lensa kontak, memenuhi lautan dunia saat ini. 3Karena sulit dideteksi, mikroplastik dapat menimbulkan masalah serius bagi hewan laut. 4Baru-baru ini para peneliti dari Universitas Reading menemukan bukti yang menunjukan bagaimana mikroplastik dapat memasuki ekosistem manusia melalui nyamuk dan serangga terbang lainnya. 5Tim peneliti tersebut mengamati larva nyamuk yang menelan microbeads–jenis mikroplastik yang biasa ditemukan pada produk kosmetik—sebelum mengamati siklus kehidupannya. 6Mereka menemukan fakta bahwa banyak sekali partikel plastik yang [...] ke tubuh nyamuk dewasa. 1. Judul yang paling tepat untuk melengkapi tulisan di atas adalah …. A. Serangga Pemakan Mikroplastik B. Bahaya Mikroplastik bagi Tubuh Manusia C. Penyebaran Mikroplastik oleh Serangga D. Mikroplastik, Serangga, Makhluk Hidup E. Cara Baru Penyebaran Mikroplastik 2. Kata berimbuhan yang salah digunakan pada paragraf 2 adalah …. A. menemukan B. menunjukan C. memasuki D. menelan E. mengamati

3. Kata yang paling tepat untuk melengkapi titik-titik pada kalimat 6 adalah …. A. tergantikan B. tertransfer C. berpindah D. teralihkan E. berubah

Tulisan berikut diikuti oleh dua butir pertanyaan. Pertimbangkan apakah kata atau kalimat pada setiap nomor bercetak tebal TIDAK PERLU DIPERBAIKI (A) atau diganti dengan pilihan lain yang tersedia (B, C, D, atau E). Setiap suku memiliki ritual tersendiri, mulai dari ritual yang biasa hingga 4ekstrem, untuk menunjukkan rasa duka akibat kehilangan anggota keluarganya. Bagi suku Dani yang bermukim di 5papua, kebersamaan sangat penting. Oleh karena itu, saat kehilangan anggota keluarga, mereka akan segera memotong ruas jarinya. Tradisi itu dikenal sebagai Iki Palek.

4. A. B. C. D. E.

TIDAK PERLU DIPERBAIKI aneh lazim ganjil luar biasa

A. B. C. D. E.

TIDAK PERLU DIPERBAIKI Papua Papua “Papua” “Papua”

5.

Jawaban: 1 C

2 B

3 C

4 A

5 B

Contoh 3 Pengetahuan dan Pemahaman Umum Bacalah tulisan berikut. Kemudian jawablah soal-soal yang tersedia dengan memilih jawaban yang tepat di antara pilihan jawaban A, B, C, D, atau E. Sebelum mencapai masa dewasa, anak-anak melewati masa remaja. Masa remaja dimulai dari usia 11 tahun sampai dengan 20 tahun. Setelah masa itu, remaja akan beranjak ke masa dewasa dan mencari jati diri mereka. Masa remaja merupakan fase rawan karena pengaruh baik dan buruk dapat masuk dengan mudah. Oleh karena itu, fase inilah yang nantinya akan menentukan karakter remaja setelah dewasa. Tidak hanya mengalami perkembangan dari segi fisik, remaja juga mengalami perkembangan psikis yang mengakibatkan emosi mereka bergejolak. Oleh karena itu, remaja harus mampu mengendalikan emosinya, baik di rumah maupun di sekolah, serta di lingkungan masyarakat. Pada masa rawan ini peran orang tua sangat diperlukan agar remaja tidak terjerumus ke dalam hal-hal negatif. Pendidikan karakter di sekolah dapat membantu para remaja melakukan hal yang positif. Pendidikan karakter mengajarkan para remaja untuk hidup dengan berpegang teguh pada agama dan norma-norma yang berlaku. Dengan demikian dapat disimpulkan bahwa pendidikan karakter sangat penting bagi remaja untuk membantu mereka melakukan kegiatan yang positif. Kegiatan remaja yang positif akan memicu perubahan yang positif pula pada dirinya. Jika remaja sebagai generasi penerus berkualitas baik, negara akan maju. 1. Apa lawan makna kata ‘membantu’ di paragraf 3 bacaan di atas? A. merantai B. mengikat C. memasung D. menghambat E. membendung

2. Apa gagasan utama paragraf 3 pada bacaan di atas? A. Masa remaja adalah fase rawan. B. Remaja perlu melakukan hal-hal yang positif. C. Pendidikan karakter sangat penting bagi remaja. D. Orang tua memiliki peran penting bagi remaja di masa pubertas. E. Remaja mengalami perkembangan fisik dan psikis dengan pesat. Jawaban: 1 D

2 C

Contoh 4 Pengetahuan dan Pemahaman Umum dalam Bahasa Inggris Questions 1—4 are based on the following passage. Forests have the ability to regulate water systems, prevent erosion and flood, and maintain soil fertility. The ability of forests is inseparable from the existence of millions and even billions of trees in a forest area. Trees only store water for their own needs, and the land saves water. Billions of trees with their wide canopies will hold back rain. In this position, rain water does not directly hit the soil surface, but will fall slowly through the leaves and flow through the tree trunk. Then, the soil surface filled with tree trunks will produce quite a lot of litter originating from organic materials in the forms of leaf and dry twigs. Dramatically, littering is heading towards the decay process. The organic materials are collected on the soil surface. Litter blocks the water falling from the canopy so it does not directly hit the soil. Litter also functions as a place to live for millions of organisms (e.g. worms). This organism punctures the land as a home and place of life. This organism’s behavior causes the soil surface to become loose and porous. When rain drops from the canopy fall onto the litter, the water slowly flows to the soil surface. The loose and porous top layer of soil will absorb the water and then the water will be stored in the aquifer, the underground river. The soil surface in the forest has high capacity to absorb rainwater. As a result, most of the rainwater seeps into the soil; only a little becomes running water. Running water is the water that cannot be absorbed by the soil surface. This water will go down to a lower area. If running water exceeds the carrying capacity of the river, it can certainly cause flood. Most of the rainwater that falls in the forest area will be absorbed by the soil and stored in the aquifer. Furthermore, the water stored in the aquifer will come out regularly through springs. From these springs, water flows through rivers that are mostly found in the forest area. In addition, there is also underground water that comes out as a spring in the resident wells. 1. The word ’dramatically’ in paragraph 2 is best replaced by .... A. gradually B. naturally C. amazingly D. surprisingly E. simultaneously



2. What is the main idea of the passage? A. Water guarantees millions of organisms to survive in forest areas. B. Land stores and releases water through various processes. C. Litter gradually flows water to the soil surface. D. Trees store water to fulfill human daily needs. E. Forests have various benefits for human life. 3. With the sentence ‘This organism's behavior causes the soil surface to become loose and porous’ in paragraph 3, the writer intends to .... A. describe the organism and its behavior against loose soil B. confirm the behavior of organisms in loose soil C. compare the loose soil with the porous soil D. uncover natural processes at soil erosion E. explain the causes of soil erosion 4. What most likely motivates the writer in writing the passage? A. There is an unresolved flood problem. B. People’s awareness towards reforestation is low. C. Not many people understand the benefits of water for forests. D. There is the fallacious concept that trees store large amount of water. E. The information about the importance of trees in storing water is not available. Jawaban: 1 A

2 B

3 E

4 D

Contoh 5 Soal Pengetahuan Kuantitatif 1. Bilangan berikut yang merupakan bilangan kuadrat dan sekaligus bilangan pangkat tiga adalah …. A. 8 B. 27 C. 64 D. 81 E. 100 2. Manakah di antara bangun berikut yang merupakan bangun datar? 1. Persegi 2. Balok 3. Trapesium 4. Kerucut A. (1), (2), dan (3) SAJA yang benar. B. (1) dan (3) SAJA yang benar. C. (2) dan (4) SAJA yang benar. D. HANYA (4) yang benar. E. SEMUA pilihan benar.

3. Misalkan x, y, dan z menyatakan bilangan real yang memenuhi persamaan x + 2y + 3z = 10. Berapakah nilai x? Putuskan apakah pernyataan (1) dan (2) berikut cukup untuk menjawab pertanyaan tersebut. 1. z = 1 2. x + y = 5 A. Pernyataan (1) SAJA cukup untuk menjawab pertanyaan, tetapi pernyataan (2) SAJA tidak cukup. B. Pernyataan (2) SAJA cukup untuk menjawab pertanyaan, tetapi pernyataan (1) SAJA tidak cukup. C. DUA pernyataan BERSAMA-SAMA cukup untuk menjawab pertanyaan, tetapi SATU pernyataan SAJA tidak cukup. D. Pernyataan (1) SAJA cukup untuk menjawab pertanyaan dan pernyataan (2) SAJA cukup. E. Pernyataan (1) dan pernyataan (2) tidak cukup untuk menjawab pertanyaan. 4. Tersedia lima kursi yang disusun berjajar dengan setiap kursi ditempati paling banyak satu orang. Manakah hubungan yang benar antara kuantitas P dan Q berikut berdasarkan informasi yang diberikan? P Q Banyak susunan empat orang duduk pada kursi 24 yang disediakan A. P > Q. B. Q > P. C. P = Q. D. Informasi yang diberikan tidak cukup untuk memutuskan salah satu dari tiga pilihan di atas. Jawaban: 1 C

2 B

3 C

4 A

(2) Tes Kemampuan Akademik (TKA) Soshum Tes Kemampuan Akademik (TKA) mengukur pengetahuan dan pemahaman keilmuan yang diajarkan di sekolah dan diperlukan untuk berhasil dalam menempuh pendidikan tinggi. Tes ditekankan pada pengukuran penalaran tingkat tinggi (higher order thinking skills). TKA Soshum terdiri atas Matematika, Sejarah, Geografi, Sosiologi dan Ekonomi. Tabel 2 berisi rincian jumlah soal alokasi waktu untuk masing-masing komponen yang diujikan. Tabel 2 Komponen Materi TKA Soshum No Komponen Jumlah soal Waktu (menit) 1 Matematika 20 18 2 Sejarah 20 18 3 Geografi 20 18 4 Sosiologi 20 18 5 Ekonomi 20 18 Catatan: Anda hanya dapat mengerjakan soal-soal dalam tiap komponen tes dalam rentang waktu yang telah dialokasikan. Berikut adalah beberapa contoh soal TKA Soshum. Contoh yang diberikan juga bukan soal sebenarnya atau bukan berasal dari kisi-kisi soal UTBK 2019, tetapi hanya untuk memberikan gambaran bentuk-bentuk soal yang akan diujikan. Contoh Soal TKA Soshum Contoh 1 Matematika



1. Jika a dan b adalah akar-akar persamaan kuadrat 𝑥 " − 𝑝𝑥 + 𝑞 = 0 maka nilai (𝛼 + 2)(𝛽 + 2) adalah …. A. 𝑝 + 𝑞 + 2 B. 2𝑝 + 𝑞 + 2 C. 𝑝 + 2𝑞 + 2 D. 𝑝 + 𝑞 + 4 E. 2𝑝 + 𝑞 + 4 2. Nilai maksimum 2𝑥 + 𝑦 + 3 yang memenuhi 1 ≤ 𝑥 + 𝑦 ≤ 2, 𝑥 ≥ 0, dan 𝑦 ≥ 0 adalah .... A. 5 B. 6 C. 7 D. 8 E. 9

Jawaban: 1 E

2 C

Contoh 2 Sejarah 1. Pembuatan periodisasi pada masa praaksara seperti masa berburu dan meramu, masa bercocok tanam dan menetap, masa perundagian, dan seterusnya, didasarkan pada perkembangan aspek .... A. sosial-politik B. sosial-ekonomi C. kebudayaan D. ekonomi-politik E. sosial-budaya 2. Salah satu bukti yang memperkuat kebenaran/kekuatan Teori Arus Balik adalah prasasti .... A. Ligor B. Lahore C. Tamil Brahmi D. Nalanda E. Tanjore

3. Wilayah yang berwarna pada peta di atas merupakan wilayah Republik Indonesia pasca …. A. perjanjian Renville B. proklamasi kemerdekaan C. perjanjian Linggarjati D. konferensi Meja Bundar E. kesepakatan Roem Royen 4. Dewa yang banyak disembah pada masa peradaban Mesir Kuno adalah Dewa Matahari yang dikenal dengan nama Dewa .... A. Osiris B. Isis C. Marduk D. Horus E. Ra Jawaban: 1 B

2 D

3 B

4 E

Contoh 3 Geografi Konflik pemanfaatan sumberdaya lahan gambut terjadi antara perusahaan yang memproduksi kayu dan masyarakat yang memanfaatkan lahan secara tradisional sebagai sumber penghidupan. Pengelolaan lahan oleh perusahaan dilakukan dengan membuat kanal-kanal untuk mengatur ketinggian muka air. Pengaturan ketinggian muka air menyebabkan keringnya beberapa rawa dan danau yang menjadi sumber penghidupan masyarakat. Berbagai sumber penghidupan masyarakat lainnya menjadi terganggu dengan berubahnya hutan alam menjadi tanaman kayu budidaya oleh perusahaan. 1. Bentuk awal perubahan lingkungan fisik lahan gambut oleh hadirnya perusahaan yang menyebabkan kehidupan masyarakat terganggu adalah …. A. pembuatan kanal-kanal air B. menipisnya lapisan gambut C. peningkatan risiko kebakaran D. berkurangnya tutupan hutan alam E. mengeringnya rawa dan danau 2. Karakteristik masyarakat di Kepulauan Hawai berbeda dengan Kepulauan Banda yang sama-sama merupakan pulau-pulau gunung api karena .... A. tingkat pendidikan masyarakat B. akar budaya masyarakat C. tingkat pendapatan masyarakat D. jaminan sosial negara E. tingkat kerentanan bencana Jawaban: 1 A

2 B

Contoh 4 Sosiologi 1. Rendi seorang remaja yang tidak suka keluar rumah. Ia selalu melakukan berbagai kegiatan melalui internet. Mulai dari belajar, bermain sampai memesan keperluan hidupnya ia selalu membeli secara online. Kondisi ini menjelaskan bahwa Rendi melakukan interaksi sosial jenis …. A. komunikatif B. interaktif C. langsung media D. tidak langsung E. face to face

2. Kelompok yang keanggotaannya berjumlah kecil, terjadi kontak mendalam dan berulang-ulang, berinteraksi secara face to face, oleh Charles Horton Cooley disebut kelompok .... A. primer B. sekunder C. patembayan D. paguyuban E. minoritas

3. Reza anak seorang direktur yang bergaya hidup mewah ketika bersekolah. Sebelum menyelesaikan sekolahnya, perusahaan orangtuanya mengalami kebangkrutan, sehingga Reza putus sekolah dan mengalami kesulitan dalam kehidupannya. Bentuk mobilitas sosial yang dialami Reza termasuk mobilitas sosial .... A. vertikal naik B. vertikal turun C. intragenerasi D. antargenerasi E. horizontal Jawaban: 1 D

2 A

3 B

Contoh 5 Ekonomi 1. Kebutuhan manusia yang tidak terbatas berhadapan dengan alat pemuas kebutuhan yang jumlahnya terbatas disebut .... A. ketimpangan B. defisit C. inefisiensi D. kelangkaan E. eksternalitas

2. Perusahaan jasa cleaning service menerima pesanan untuk membersihkan kantor. Apabila di awal bulan menerima uang muka pembayaran senilai Rp1.000.000,00, maka perusahaan akan mencatatnya dengan jurnal .... A. debit kas, kredit pendapatan B. debit kas, kredit piutang C. debit kas, kredit utang D. debit kas, kredit ekuitas E. debit kas, kredit laba

Jawaban: 1 D

2 C

TES POTENSI SKOLASTIK PREDIKSI WAKTU JUMLAH SOAL BIDANG STUDI

Bacalah teks berikut untuk menjawab soal nomar 1 sampai dengan nomor 7.

: UTBK SBMPTN 2020 : 35 MENIT : 20 : PENALARAN UMUM

2.

Paragraf manakah yang paling tepat melengkapi teks di atas? (A) Tabel di atas menunjukkan bahwa minat siswa dalam memilih ekstrakurikuler cenderung tidak stabil. Hal itu ditunjukkan dengan jumlah siswa pada setiap jenjang kelas yang tidak tetap, misalnya menyanyi dan seni peran (B) Tabel di atas menunjukkan bahwa minat siswa dalam memilih ekstrakurikuler cenderung stabil. Hal itu ditunjukkan dengan jumlah siswa pada setiap jenjang kelas yang cenderung tetap, misalnya menyanyi dan melukis (C) Tabel di atas menunjukkan bahwa minat siswa dalam memilih ekstrakurikuler cenderung fluktuatif. Hal itu ditunjukkan dengan jumlah siswa pada setiap jenjang kelas yang tidak selalu tetap, misalnya bola basket dan seni peran. (D) Dari tabel di atas diketahui bahwa siswa cenderung memilih seni peran. Hal itu ditunjukkan dengan jumlah siswa yang mengikutinya di kelas XII menduduki jumlah terbanyak di antara semua jenis kegemaran. (E) Dari tabel di atas diketahui bahwa siswa cenderung memilih bola basket. Meskipun peminatnya tidak memiliki kecenderungan naik, basket masih mendominasi kegemaran siswa.

3.

Pernyataan manakah yang paling sesuai dengan isi tabel di atas? (A) Peminat bola basket semakin meningkat setiap tahunnya (B) Seni tari memiliki prospek paling baik karena peminatnya selalu meningkat (C) Seni lukis memiliki prospek paling baik karena peminatnya selalu meningkat

Untuk menyalurkan bakat dan minat siswa SMA Bina Bersaudara Medan, kepada siswa dibagikan formulir identitas guna mengetahui kegemaran tiap siswa. Hal itu dilakukan agar sekolah dapat membina bakat dan minat siswa secara benar. Pembinaan tersebut dimasukkan sebagai kegiatan ekstrakurikuler. Ektrakurikuler merupakan kegiatan-kegiatan yang dilakukan di luar kurikulum inti. Dari formulir yang dibagikan itu dapat dikenali berbagai bakat dan minat siswa. Meskipun tidak semua bakat dan minat siswa itu terpenuhi di sekolah, pihak sekolah mengetahui kegemaran siswa. Dengan cara itu, sekolah dapat mencarikan wadah untuk menyalurkan bakat dan minat siswa. Berdasarkan isian tersebut, diketahui ada lima jenis kegemaran utama siswa SMA Bina Bersaudara Medan, yakni bola basket, seni tari, melukis, menyanyi, dan seni peran sebagaimana disajikan dalam tabel berikut. Tabel Jenis Kegemaran Siswa SMA Bina Bersaudara Medan Jenis Kelas Kelas Kelas XII kegemaran X XI 100 150 120 bola basket 75 78 80 seni tari 68 54 44 melukis 57 46 44 menyanyi 40 7 180 seni peran

1.

Kata itu pada kalimat ke-3 paragraf ke-2 di atas merujuk pada (A) menyalurkan bakat (B) mengisi formulir (C) mengetahui kegemaran (D) mengetahui minat (E) membina ekstrakurikuler

(D) Di setiap jenjang kelas, menyanyi menjadi kegemaran yang paling sedikit peminatnya. (E) Seni peran paling tidak diminati siswa karena pesertanya selalu paling sedikit pada setiap jenjangnya.

4. Dari data jenis kegemaran paling diminati siswa SMA Bina Bersaudara Medan adalah …. (A) bola basket (B) Seni tari (C) melukis (D) menyanyi (E) Seni peran 5. Berdasarkan minat bakat paling diminati siswa SMA Bina Bersaudara Medan untuk kelas XII adalah …. (A) bola basket (B) Seni tari (C) melukis (D) menyanyi (E) Seni peran 6. Berdasarkan jenis kegemaran melukis siswa SMA Bina Bersaudara Medan untuk kelas XII adalah …. (A) 120 (B) 100 (C) 80 (D) 68 (E) 44 7. Pernyataan berikut yang tidak sesuai dengan tabel kegemaran Siswa SMA Bina Bersaudara Medan adalah …. (A) Kegemaran basket adalah paling banyak diminati (B) Jumlah siswa gemar seni peran adalah 227 siswa (C) Jumlah siswa kelas XII sesuai kegemaran adalah 468 (D) Kegemaran seni tari yang paling sedikit ada di kelas XI (E) Jumlah siswa gemar melukis adalah 160

Teks berikut untuk menjawab soal nomor 8 dan 14! Sepanjang empat tahun terakhir ini, produksi udang Indonesia masih menguasai pasar Jepang. Volume ekspor udang Indonesia memang masih paling tinggi ke negeri Matahari ini. Hanya saja, perlu diperhatikan, ternyata volumenya dari tahun ke tahun mengalami penurunan, seiring lesunya sektor udang nasional. Hal yang perlu diwaspadai adalah produksi udang Vietnam yang makin melejit. Sebagai perbandingan pada tahun 2016, ekspor udang Indonesia ke Jepang sebanyak 55.617 ton, sedangkan Vietnam mengekspor 35.664 ton. Untuk lebih jelasnya lihat tabel di bawah ini. Jumlah Udang yang Diimpor Jepang (2016 – 2019) Negara 2016 2017 2018 2019 Asal Indonesia 55.617 53.607 52.367 16.669 Vietnam 35.664 41.516 47.626 13.568 42.991 34.795 28.191 9.707 India 14.926 19.598 20.494 5.629 Cina Thailand 20.574 18.987 16.806 4.771 8. Hal yang digambarkan dalam tabel tersebut adalah … . (A) Jumlah udang yang diimpor Jepang dari tahun ke tahun mengalami penurunan yang cukup drastis, dan ekspor udang dari Indonesia, Cina, dan India mengalami turun naik dari tahun 2016 – 2019. (B) Jumlah ekspor udang Indonesia tahun 2016 dan 2017 hampir sama dengan jumlah ekspor udang dari negara lain pada tahun 2018 sedang jumlah ekspor udang pada tahun 2019 hampir sama dengan jumlah ekspor India tahun 2018 (C) Jumlah ekspor udang Indonesia, India, dan Thailand ke Jepang dari tahun ke tahun menurun, sedangkan Vietnam dari tahun ke tahun makin naik, kecuali tahun 2019 mengalami penurunan yang drastis. (D) Ekspor udang dari negara India ke Jepang mengalami peningkatan pada tahun 2017 dan 2019, sedangkan jumlah ekspor dari negara lain mengalami penurunan pada tahun 2016 dan meningkat drastis pada tahun 2019

9.

(E) Udang yang diimpor Jepang dari Indonesia mengalami kenaikan dari tahun 2016 sampai tahun 2019, sedangkan dari negara lain mengalami penurunan yang cukup drastis pada tahun 2016 dan kenaikan pada tahun 2017

13. Jumlah impor udang Jepang tahun 2019 sebesar …. (A) 16.669 (B) 30.237 (C) 39.944 (D) 50.344 (E) 60.344

Kesimpulan isi tabel tersebut yang tepat adalah … . (A) Ekspor udang dari beberapa negara ke Jepang mengalami kenaikan yang tajam (B) Udang yang diimpor Jepang dari berbagai negara pada tahun 2019 mengalami penurunan (C) Ekspor udang dari beberapa negara selalu stabil dari tahun 2016 sampai tahun 2019 (D) Ekspor udang dari negara lain mengalami kenaikan tajam pada tahun 2018 dan 2019 (E) Jumlah impor udang Jepang dari beberapa negara dari tahun ke tahun mengalami kenaikan

14. Jumlah impor udang Jepang tertinggi dari negara …. (A) Indonesia (B) Vietnam (C) India (D) China (E) Thailand

10. Berdasarkan tabel udang yang diimpor Jepang dari berbagai negara pada tahun 2019 paling tinggi di negara …. (A) Indonesia (B) Vietnam (C) India (D) China (E) Thailand 11. Dari tahun 2016-2019 udang yang diimpor Jepang dari berbagai negara dengan jumlah paling rendah ada di negara …. (A) Indonesia (B) Vietnam (C) India (D) China (E) Thailand 12. Udang yang diimpor Jepang dari tahun 2017-2018 yang mengalami kenaikan di negara …. (A) Indonesia & Vietnam (B) Vietnam & India (C) India & China (D) Vietnam & China (E) Thailand & India

15. Berapa nilai Y ? 3 4 5 (A) (B) (C) (D) (E)

3 4 5

2 3 4

144 8.000 Y

81 810 8.100 81.000 810.000

16. Tentukan nilai Y pada pola berikut!

9

16 12

18 32 X

(F) 28 (G) 24 (H) 20 (I) 18 (J) 12 17. 5,65 − 5,55 + 4,55 − 4,45 + 3,45 − 3,35 + 2,35 − 2,25 = …. 2 (A) 5 4

(B) 5 (C) 1 5 (D) 4 5

(E) 2

18. Di antara bilangan di bawah ini, yang habis dibagi 8 adalah …. (A) 123.456 (B) 234.567 (C) 345.678 (D) 456.789 (E) 567.890 19. Jika 𝑥 adalah bilangan genap dan 𝑦 adalah bilangan ganjil maka yang merupakan bilangan ganjil adalah …. (A) (𝑥 + 𝑦)(𝑥 − 𝑦) (B) 62 (3𝑥 − 𝑦) (C) (𝑥 + 𝑦 2 )2 − 1 (D) 𝑥 2 + 2𝑥𝑦 + 𝑦 + 3 (E) (𝑥 − 3)2 + (𝑦 + 4)2

20. Jika 𝑥 = 0,888, 𝑦 = √0,888, dan 𝑧 = (0,888)2 maka pernyataan yang benar adalah …. (A) 𝑥 < 𝑦 < 𝑧 (B) 𝑥 < 𝑧 < 𝑦 (C) 𝑦 < 𝑥 < 𝑧 (D) 𝑦 < 𝑧 < 𝑥 (E) 𝑧 < 𝑥 < 𝑦

TES POTENSI SKOLASTIK PREDIKSI WAKTU JUMLAH SOAL BIDANG STUDI

Bacaan untuk soal nomor 1 dan 2 Ruang Penguatan Rupiah Menipis. Mata uang Asia melemah pada perdagangan diawal minggu, namun rupiah masih menguat. Ruang penguatan itu hari ini terlihat mengecil seiring kenaikan indeks dollar AS. Setelah data ekspor China diumumkan memburuk, hampir seluruh mata uang di Asia melemah terhadap dollar AS sampai sore kemarin. Akan tetapi rupiah yang sempat melemah berganti arah menjadi menguat. Lelang SUN valas yang dilaksanakan kemarin menarik minat investor melihat penawaran yang mencapai dua kali dari target awal. Riset Samuel Sekuritas Indonesia melihat optimisme dari perekonomian domestik masih ada, namun ruang penguatan semakin terbatas. Dolar indeks mulai merangkak naik sampai dini hari tadi walaupun hanya tipis. Rupiah masih melemah 17 persen secara tahunan tetapi membaik dari posisi Desember yang melemah 26 persen secara tahunan akibat perbaikan fundamental. Antara lain inflasi yang turun, defisit transaksi berjalan yang menipis dan pertumbuhan di atas 5,7 persen secara tahunan. Riset Trust Securities menyatakan laju rupiah masih di bawah level resisten di Rp 11.374 per dolar AS. Hari ini diproyeksikan rupiah ada di rentang Rp 11.464-11.374 per dolar AS (kurs tengah BI) 1. Berdasarkan isi bacaan di atas, pernyataan berikut ini benar adalah (A) Mata uang Asia turut menguat seiring naiknya minat investor (B) Penguatan rupiah mulai terlihat seiring kenaikan dollar AS (C) Rupiah mengalami penguatan yang cukup signifikan terhadap dollar AS (D) Ruang penguatan rupiah mengecil seiring kenaikan dollar AS (E) Ruang penguatan mata uang asia melampaui target awal

: UTBK SBMPTN 2020 : 25 MENIT : 20 : PENGETAHUAN & PEMAHAMAN UMUM

2. Gagasan utama teks tersebut adalah (A) Rupiah malemah di perdagangan Asia (B) Rupiah melemah terhadap dollar AS (C) Rupiah mengalami penguatan walaupun tipis (D) Ruang penguatan rupiah melampaui target awal (E) Ruang penguatan rupiah menipis Bacaan untuk soal nomor 3 – 6 Lebih dari 90% anggota DPR RI periode 2009-2014 yang saat ini duduk di kursi parlemen maju lagi di Pileg 2014. Peneliti senior Forum Masyarakat Peduli Parlemen Indonesia (Formappi) Lucius Karus menegaskan, jika dilihat dari kinerja, 90% dari mereka yang mencalonkan kembali itu tidak layak dipilih rakyat. Lucius menggambarkan rekam jejak mereka sebagai para pelaku perbuatan tercela seperti korupsi, menonton video porno, kerap membolos, dan tidak produktif. Jika dilihat dari fungsi pokok DPR RI pun, yakni legislasi, anggaran, dan pengawasan, menurut Lucius, target mereka meleset jauh. “Kalaupun mau mencalonkan lagi, semua motivasinya hanya mencari kerja, kekuasaan, dan keuntungan. Mereka tidak peduli rekam jejak kegagalan mereka sepanjang periode jabatan. Ini bukan cuma karena faktor kepentingan, melainkan memang karena mereka tidak punya kapasitas,” ungkap Lucius, kemarin. Menurut dia, masyarakat harus disadarkan akan rendahnya kompetensi yang dimiliki anggota DPR saat ini. Ia berharap, masyarakat tidak lagi terpancing oleh rayuan politik uang di Pemilu 2014. Peneliti Indonesia Budget Center (IBC) Roy Salam mengatakan alokasi anggaran selama lima tahun masa jabatan DPR menimbulkan harapan mereka mampu menjalankan fungsi pengawasan, penganggaran, dan legislasi dengan baik.

”Namun, nyatanya, DPR tidak berprestasi. Fungsi pengawasan, penganggaran, apalagi legislasi gagal dijalankan. ”Direktur Advokasi Pusat Studi Hukum dan Kebijakan Ronald Rofi Andri menambahkan legislasi yang dibahas dan disepakati di DPR kerap bermasalah sehingga banyak UU yang diuji materi dan dikabulkan Mahkamah Konstitusi. Ketua Badan Kehormatan DPR Trimedya Panjaitan mengakui seperti itulah kualitas DPRD hasil Pemilu 2009. Ketua Badan Legislasi DPR Ignatius Mulyono juga mengakui target legislasi DPR tidak bisa dicapai. Jika dapat diringkas berikut rapor merah kinerja DPR selama 5 tahun : 1) Dalam kurun waktu lima tahun, DPR telah menghabiskan anggaran hingga Rp. 11,8 triliun, Rp. 2 triliun di antaranya untuk legislasi 2) Dari 2009 hingga 2014 DPR baru menuntaskan 48 RUU, yaitu: Targe Persenta RUU Tahun t se Selesai RUU kinerja 2009/2 8 70 11,4 % 010 2011 18 93 19,3 % 2012 10 64 15,6 % 2013 7 70 10 % 2014 5 60 8,3 % Total 48 357 13,4 % 3) Fungsi pengawasan, penganggaran, dan legislasi tidak berjalan dengan baik 4) Banyak anggota dewan sering membolos dan mangkir dalam rapat. 5) Banyak anggota dewan tertidur pulas saat rapat paripurna 6) Banyak anggota dewan terlibat kasus korupsi atau menerima suap 3. Hubungan isi antarparagraf yang paling tepat adalah (A) Paragraf ke-2 merupakan rincian paragraf ke-1 (B) Paragraf ke-2 merupakan solusi paragraf ke-1 (C) Paragraf ke-2 merupakan akibat paragraf ke-1

(D) Paragraf ke-2 merupakan pembanding paragraf ke-1 (E) Paragraf ke-2 merupakan memaparkan contoh paragraf ke-1 4. Pernyataan manakah yang paling sesuai dengan tabel di atas? (A) Kinerja DPR paling bergairah terjadi pada tahun 2011 dan 2010 (B) Kinerja DPR cenderung rendah di tahun 2009 dan 2014 (C) Kinerja DPR cenderung jauh dari target setiap tahunnya (D) Standar penyelesaian target RUU cenderung naik pada setiap tahunnya (E) Standar penyelesaian target RUU cenderung menurun pada setiap tahunnya 5. Apakah simpulan dari teks di atas? (A) Rapor merah kinerja DPR menggambarkan kegagalan fungsinya sebagai lembaga legislatif negara selama 5 tahun jabatan. (B) Rapor merah kinerja DPR menggambarkan rekam jejak mereka sebagai lembaga legislatif negara selama 5 tahun jabatan. (C) DPR sebagai lembaga legislatif negara gagal menjalankan fungsinya selama 5 tahun jabatan (D) DPR sebagai lembaga legislatif Negara gagal mencapai target kinerja selama 5 tahun jabatan (E) Selama 5 tahun jabatan DPR tidak menunjukkan mampu menjalankan fungsi pengawasan, penganggaran, dan legislasi dengan baik. 6. Kelemahan Paragraf ke-2 teks di atas adalah (A) Gagasan utamanya terletak di tengah (B) Gagasan utamanya ada diseluruh paragraf (C) Tidak ada gagasan utama (D) Mengandung kalimat sumbang (E) Tidak ada gagasan penjelas

Bacaan untuk soal nomor 7 – 10 (1) Pemerintah tengah merumuskan sistem hukuman berlapis untuk para pembakar lahan dan hutan yang menyebabkan bencana kabut asap di Indonesia. (2) Presiden Susilo Bambang Yudhoyono menyebut pelaku pembakaran hutan telah melakukan kejahatan kemanusiaan. (3) Kabut asap pembakaran ini ialah kejahatan kemanusiaan. (4) Orang sesak, tak bisa sekolah, dan tidak bisa beraktivitas selama berminggu-minggu lamanya. (5) Menurut Kepala Negara, sebagian besar kasus kebakaran hutan di Riau terjadi dengan sengaja dibakar. (6) Presiden mengancam tidak akan segan-segan menindak tegas pelaku yang melanggarnya, siapa pun orangnya. (7) “Tidak ada yang setengah hati. Pihak-pihak yang melakukan pelanggaran harus diberi sanksi yang tegas,” tukas Presiden. (8) Saat ini Satgas Penanggulangan Bencana Asap merumuskan sistem hukuman berlapis bagi pelaku kejahatan di sektor kehutanan. (9) Deputi Kepala Unit Kerja Presiden Bidang Pengawasan dan Pengendalian Pembangunan (UKP4) Bidang Penegakan Hukum, Mas Ahmad Santosa, menegaskan nantinya perusahaan yang membakar lahan akan dijerat pasal berlapis dari berbagai undang-undang terkait, seperti UU Perkebunan, atau bahkan UU Nomor 32 Tahun 2009 tentang Perlindungan dan Pengelolaan Lingkungan Hidup. Menurutnya, UU 32/2009 memberi dasar hukum bagi pengenaan tindak pidana korporasi. (10) Mas Ahmad berpendapat pengenaan tindak pidana korporasi ini penting agar pimpinan perusahaan lebih ekstra hati-hati. (11) Bahkan jika ditemui dugaan korupsi sektor kehutanan, pemerintah akan menggandeng Komisi Pemberantasan Korupsi. (12) “Selain pidana, negara juga akan menuntut secara perdata terkait gugatan ganti kerugian ekosistem dan biaya pemulihan lingkungan.” (13) Saat ini sudah 63 orang ditetapkan sebagai tersangka dan satu tersangka korporasi, yaitu PT National Sagu Prima. 7. Manakah rangkuman yang tepat untuk teks di atas? (A) Pemerintah tengah merumuskan sistem hukuman berlapis untuk para pembakar lahan dan hutan yang menyebabkan bencana kabut asap di Indonesia. Kabut

(B)

(C)

(D)

(E)

asap pembakaran ini ialah kejahatan kemanusiaan. Masyarakat sangat terganggu dalam menjalankan aktivitas sehari-hari. Pemerintah tengah merumuskan sistem hukuman berlapis untuk para pembakar lahan dan hutan yang menyebabkan bencana kabut asap di Indonesia. Masyarakat sangat terganggu dalam menjalankan aktivitas sehari-hari. Presiden mengancam tidak akan segansegan menindak tegas pelaku yang melanggarnya. Pemerintah tengah merumuskan sistem hukuman berlapis untuk para pembakar lahan dan hutan yang menyebabkan bencana kabut asap di Indonesia. Kabut asap pembakaran ini ialah kejahatan kemanusiaan. Negara juga akan menuntut secara pidana dan perdata terkait gugatan ganti perusakan ekosistem dan biaya pemulihan lingkungan. Pemerintah tengah merumuskan sistem hukuman berlapis untuk para pembakar lahan dan hutan yang menyebabkan bencana kabut asap di Indonesia. Presiden mengancam tidak akan segansegan menindak tegas pelaku yang melanggarnya. Selain pidana, negara juga akan menuntut secara perdata terkait gugatan ganti kerugian ekosistem dan biaya pemulihan lingkungan. Pemerintah tengah merumuskan sistem hukuman berlapis untuk para pembakar lahan dan hutan yang menyebabkan bencana kabut asap di Indonesia. Kabut asap pembakaran ini ialah kejahatan kemanusiaan. Presiden mengancam tidak akan segan-segan menindak tegas pelaku yang melanggarnya.

8. Kalimat yang tidak efektif ditemukan pada (A) 1 dan 13 (B) 2 dan 7 (C) 3 dan 8 (D) 4 dan 6 (E) 9 dan 10 9. Penulis teks di atas berpihak pada (A) Pemerintah (B) Pelaku pembakaran

(C) Pembaca (D) Masyarakat (E) Tidak berpihak 10. Pernyataan berikut yang sesuai dengan paragraf tersebut adalah (A) Pemerintah sudah merumuskan hukuman berlapis bagi pelaku pembakaran hutan. (B) Pelaku pembakaran dihukum dengan pasal berlapis. (C) Pasal berlapis tengah dirumuskan untuk menjerat pelaku pembakaran hutan. (D) Pihak pelanggar peraturan akan diberi sangsi yang tegas (E) Pemerintah tengah menghukum pelaku pembakaran hutan secara tegas

Teks berikut digunakan untuk menjawab soal nomor 11-12. (1) salah satu proses pembelajaran yang harus dikembangkan oleh guru-guru dalam Kurikulum 2006 atau yang juga dikenal sebagai Kurikulum tingkat satuan Pendidikan (KtsP) adalah mengembangkan kreativitas siswa secara optimal. (2) Pengembangan kreativitas sangat penting terlihat dan bergesernya peran guru. (3) dahulu, guru sering mendominasi kelas, tetapi kini guru harus lebih banyak memberikan kesempatan kepada siswa untuk berperan lebih aktif dan kreatif. (4) membangun pemahaman yang baik kepada siswa akan sulit jika fsik dan psikis mereka dalam keadaan tertekan. (5) Kreativitas siswa dapat tumbuh dan berkembang dengan baik apabila lingkungan keluarga, masyarakat, dan lingkungan sekolah turut menunjang. 11. Kalimat utama paragraf tersebut adalah .... (A) (1) (D) (4) (B) (2) (E) (5) (C) (3) 12. Kalimat yang tidak padu pada paragraf Tersebut adalah kalimat nomor .... (A) (1) (D) (4) (B) (2) (E) (5) (C) (3)

TEXT 1 TEXT A Berlin (Reuters)–No more Coca-Cola or Budweiser, no Marlboro, no American whiskey or even American Express cards  a growing number of restaurants in Germany are taking everything American off their menus to protest the war in Iraq. Although the protests are mainly symbolic, waiters in dozens of bars and restaurants in Hamburg, Berlin, Munich, Bonn and other German cities are telling patrons, "Sorry, CocaCola is not available any more due to the current political situation." The boycotts appear to be part of a nascent worldwide movement. One Web site, www.consumers-against-war.de, calls for boycotts of 27 top American firms from Microsoft to Kodak while another, www.adbusters.org, urges the "millions of people against the war" to "Boycott Brand America." Consumer fury seems to be on the rise. Demonstrators in Paris smashed the windows of a McDonald's restaurant last week, forcing police in riot gear to move in to protect staff and customers of the American fast-food outlet. The attackers sprayed obscenities and "boycott" on the windows. TEXT B This economic advantage, in turn, is used to sponsor terror and killing in Islamic countries such as Afghanistan and Iraq. When product boycott was carried out by consumers in the Middle East and some in the European countries, sales of these companies is reported to be decreased by 10% and this amounted to big numbers for giant companies. Thus, it is a rational for Muslim especially in Malaysia to take similar action. Furthermore, Of late, many other products are available as an alternative for the boycotted products. For example, we have Mukmin toothpaste instead of Colgate etc. and Fab or Breeze could be replaced with Daiya, Puteri Emas etc. There are a few questions arise in regard to the boycott such as how effective is the approach and why not boycotting all the products altogether. In answering these questions, Sabasun has reiterated to look at collapse of the apartheid regime in South Africa as the best example.

To answer the question in regard to why not boycotting all the products altogether, Sabasun has taken an approach to do what can be done when one cannot do all. Hence, a few products that are really needed by consumers are offered on a limited floor space and no promotion is done for the product. While products that are boycott completely will not even get space on the shelves let alone floor space. During the early phase after the campaign was in progress, the effect of the boycott is very obvious when Sabasun suffer a loss of nearly RM 150,000. 13. Both passages are similar in terms of (A) The problem underlying in both passages (B) The writer’s point of view (C) The effect of the problem discussed (D) The area the problem taking place (E) The reason leading to the problem 14. The difference between the first and the second passage is that the latter (A) Asserts the economic advantage resulted from boycotting while the former claims the cause of boycotting (B) Exposes the disserve impact of boycotting for Islamic countries while the former shows the situation in Germany (C) Affirms the need for boycotting American products while the former depicts the activity of boycotting in Europe (D) Repudiates the boycott toward American products while the former presents the reason of boycotting (E) Points out the limitation of the activity while the former clarifies the need to do the activity 15. From the second passage, it can be inferred that the topic discussed before the text B is (A) The basic reason for repelling American products (B) The terror for Islamic countries (C) The motivation for not boycotting all the products altogether (D) The effectiveness in carrying out the program (E) The economic advantage of the American firms

16. The most suitable word to least change the word “nascent” in “The boycotts appear to be part of a nascent worldwide movement.” in the first passage is (A) Newly born (B) Rising (C) Perilous (D) Vital (E) Fretting TEXT 2 Among the environmental specters confronting humanity in the 21st century – global warming, the destruction of rain forests, overfishing of the oceans – a shortage of fresh water is at the top of the list, particularly in the developing world. Hardly a month passes without a new study making another alarming prediction, further deepening concern over what a World Bank expert calls the “grim arithmetic of water.” Recently the United Nations said that 2.7 billion people would face severe water shortages by 2025 if consumption continuous at current rates. Fears about a parched future arise from a projected growth of world population from more than six billion today to an estimated nine billion in 2050. Yet the amount of fresh water on Earth is not increasing. Nearly 97 percent of the planet’s water is salt water in seas and oceans. Close to 2 percents of Earth’s water is frozen in polar ice sheets and glaciers, and a fraction of one percent is available for drinking, irrigation, and industrial use. Gloomy water news, however, is not just a thing of the future: Today an estimated 1.2 billion people drink unclean water, and about 2.5 billion lack proper toilets or sewerage systems. More than five million people die each year from water-related diseases such as cholera and dysentery. All over the globe farmers and municipalities are pumping water out of the ground faster than it can be replenished. Still, as I discovered on a two-month trip to Africa, India, and Spain, a host of individuals, organizations, and businesses are working to solve water’s dismal arithmetic.

610 17. The subject matter discussed in the passage is ___ (A) The effect of fresh water shortage (B) Project to provide fresh and clean water (C) Nature disturbance leading to availability of water (D) Alarming condition of the world water shortage (E) The amount of fresh water on earth 18. The part following the passage will likely discuss about (A) The spread of greater water shortage in Africa, India, and Spain (B) The writer’s discoveries of gloomy water news (C) Techniques taken to obtain maximum efficiency from every drop of water (D) Another environmental specters confronting humanity (E) The solving ideas of individuals, organizations, and businesses problems

19. What inference can undermine the information taken from the passage? (A) Poor sanitation leads to water-related illness (B) The growth of world population is imbalance with the clean water availability (C) There is always new study making another alarming prediction about fresh water (D) It is estimated that 1.2 billion people drink unclean water, and about 2.5 billion lack proper toilets or sewerage systems (E) Only one percent of water is available for drinking, irrigation, and industrial use 20. The followings are the least meaning of the word “parched” (A) Dried (B) Alight (C) Arid (D) Crispy (E) Droughty

SBMPTN Bahasa Indonesia Soal SBMPTN - Bahasa Indonesia

Teks berikut digunakan untuk menjawab soal 1-5. Teks 1 (1)Dalam beberapa tahun terakhir, mulai banyak pasien anak dan remaja yang memiliki keluhan pada tulang belakang. (2) Padahal, rasa sakit seperti ini biasanya dialami orang yang berusia 40 tahun ke atas. (3) Anak dan remaja ini ternyata gemar menggunakan gadget, seperti telepon pintar, komputer, dan tablet. (4) Banyak dari mereka yang bermain gadget sambil tengkurap, membungkuk, atau lehernya ke bawah untuk menatap layar monitor, sehingga kepala membebani leher. (5) Ada pula yang badannya bersandar di kursi dengan posisi layar lebih tinggi dari pada mata. (6) Di samping itu, juga ada yang memiringkan kepala ke satu sisi untuk menjepit gadget di antara telinga dan pundak ketika menelepon. (7) Apabila ini dilakukan terus menerus, tulang belakang akan “protes” dengan mengirimkan sinyal nyeri. (8) Ada berbagai macam sensasi nyeri pada tulang belakang, seperti ditusuk-tusuk, kesemutan, tersetrum, dan nyeri cenat cenut seperti sakit gigi. (9) Pada nyeri yang diakibatkan gadget, biasanya sensasinya seperti tersetrum. (10) Nyeri ini memang tak berbahaya. (11) Tetapi, jika dibiarkan terus menerus, nyeri ini bisa merusak postur tulang. (12) Ada tiga tahapan nyeri pada tulang belakang, yaitu rasa nyeri yang dialami otot, kemudian menjalar ke sendi, dan terakhir mengenai tulang. 01. Apa gagasan utama paragraf pertama? (A) penyakit tulang belakang meningkat beberapa tahun terakhir (B) keluhan pada tulang belakang biasanya dialami orang tua (C) jumlah anak dan remaja yang mengeluhkan sakit tulang belakang meningkat (D) anak dan remaja gemar menggunakan gadget dengan cara yang tidak benar (E) posisi menggunakan gadget menyebabkan nyeri tulang belakang

02. Kesalahan penggunaan tanda baca ditemukan pada kalimat .... (A) 2 (B) 3 (C) 4 (D) 6 (E) 8 03. Manakah pertanyaan yang jawabannya tidak terdapat dalam teks? (A) bagaimana rasa nyeri yang disebabkan oleh penggunaan gadget? (B) apa akibat dari rasa nyeri yang tidak diobati? (C) mengapa banyak anak dan remaja menderita nyeri pada tulang belakang? (D) bagaimana kebiasaan remaja memainkan gadget? (E) kapan para anak dan remaja biasanya menggunakan gadget? 04. Kata 'ini' pada kalimat (7) di atas merujuk pada .... (A) posisi menjepit gadget (B) posisi tengkurap dan membungkuk (C) posisi bersandar di kursi (D) posisi menggunakan gadget (E) posisi menatap layar gadget 05. Kata sambung yang salah adalah .... (A) sambil pada kalimat (4) (B) sehingga pada kalimat (5) (C) apabila pada kalimat (7) (D) seperti pada kalimat (8) (E) tetapi pada kalimat (11)

1

SBMPTN 2018 Bahasa Indonesia, Soal SBMPTN 2018 - Bahasa Indonesia

Teks berikut digunakan untuk menjawab soal 6-8. Teks 2 (1) Penyakit stroke adalah gangguan fungsi otak akibat aliran darah ke otak mengalami gangguan. (2) Akibatnya, nutrisi dan oksigen yang dibutuhkan otak tidak terpenuhi dengan baik. (3) Penyebab stroke ada dua macam, yaitu adanya sumbatan di pembuluh darah dan adanya pembuluh darah yang pecah. (4) Umumnya stroke diderita oleh orang tua karena proses penuaan yang menyebabkan pembuluh darah mengeras dan menyempit serta lemak yang menyumbat pembuluh darah. (5) Pada beberapa kasus terakhir menunjukkan peningkatan kasus stroke yang terjadi pada usia remaja dan produktif (15-40 tahun). (6)Pada golongan ini, penyebab utama stroke adalah stress, faktor keturunan, dan gaya hidup yang tidak sehat, seperti penyalahgunaan narkoba dan alkohol. (7) Pada kasus stroke usia remaja, faktor keturunan merupakan penyebab utama terjadinya stroke. (8) Sering ditemukan kasus stroke yang disebabkan oleh pembuluh darah yang mudah pecah atau kelainan sistem darah, seperti penyakit hemofilia dan talasemia yang diturunkan oleh orang tua penderita. (9) Jika ayah atau ibu menderita diabetes, hipertensi, atau penyakit jantung, kemungkinan anak terkena stroke menjadi lebih besar. 06. Kalimat yang tidak efektif terdapat pada kalimat .... (A) (2) (B) (4) (C) (5) (D) (6) (E) (8) 07. Apa simpulan teks tersebut? (A) penyakit stroke kebanyakan dialami oleh orang usia lanjut (B) penyakit stroke disebabkan gangguan aliran darah ke otak (C) sumbatan atau pecahnya pembuluh darah mengakibatkan stroke (D) stroke dapat terjadi di usia remaja, produktif, dan usia lanjut (E) penyebab stroke adalah stress, penuaan, gaya hidup, dan keturunan 2

08. Apa gagasan utama yang tidak tepat untuk melanjutkan teks tersebut? (A) penanganan stroke (B) risiko stroke (C) ancaman stroke (D) gejala stroke (E) penelitian stroke Teks berikut digunakan untuk menjawab soal 9-11. Teks 3A (1) Perhelatan Forum SS kembali digelar di Kuala Lumpur Convention Centre, Malaysia. (2) Acara ini bertujuan memperkenalkan produk terbaru SS yang dipasarkan di Asia Tenggara. (3) SS telah menjadi brand dengan pertumbuhan yang cepat di kategori aplikasi rumah tangga dan ponsel pintar. (4) SS memantau keinginan konsumen untuk menghadirkan produk-produk yang memudahkan kehidupan manusia. (5) Produk SS yang hadir di tahun ini semuanya terkoneksi. (6) Produk-produk tersebut membangun ekosistem yang mampu menghubungkan produk melalui jaringan internet. Teks 3B Peserta forum diajak ke ruang pameran tempat seluruh amunisi SS menampilkan produk unggulan. (2) Salah satu produk yang mendapat sorotan adalah kulkas canggih. (3) Kulkas yang diotaki oleh sistem operasi Android ini mampu mengakomodasi kebutuhan keluarga dengan cara modern. (4) Selain itu, ada pula televisi terbaru yang dilengkapi dengan teknologi Quantum Dot. (5) Teknologi ini diklaim menawarkan kualitas gambar realistis dengan desain yang semakin menawan. (6) Perangkat lainnya adalah mesin cuci generasi terbaru yang dapat dikontrol lewat perangkat ponsel.

SBMPTN 2018 Bahasa Indonesia, Soal SBMPTN 2018 - Bahasa Indonesia

09. Kalimat manakah yang maknanya sejajar dengan kata mengakomodasi pada kalimat (3) teks 3B? (A) panitia menyediakan fasilitas penginapan secara gratis bagi peserta luar kota (B) sudah menjadi kewajiban orang tua untuk memenuhi kebutuhan anak (C) peserta kemah remaja telah memanfaatkan sarana yang tersedia di lokasi (D) moderator secara maksimal telah mengatur jalannya sidang dengan baik (E) pimpinan sidang mengambil keputusan berdasarkan saran-saran anggota 10. Apa simpulan kedua teks tersebut? (A) SS merupakan perusahaan teknologi terdepan di Asia (B) semua produk SS yang dipamerkan terhubung satu sama lain (C) produk SS canggih dan memudahkan kehidupan manusia (D) kulkas produk SS merupakan yang tercanggih di kelasnya (E) produk SS tidak hanya canggih, tetapi juga terjangkau harganya 11. Informasi manakah yang terdapat pada Teks 3A, tetapi tidak terdapat pada Teks 3B? (A) forum SS di Kuala Lumpur Convention Center (B) kulkas canggih dengan sistem operasi Android (C) mesin cuci yang dikontrol ponsel canggih (D) kecanggihan produk rumah tangga SS (E) kecanggihan produk ponsel pintar SS 12. Kalimat manakah yang salah satu katanya ditulis secara tidak tepat? (A) uji cobakan gagasanmu di depan dewan juri paling lama 30 menit! (B) Propinsi Sulawesi Selatan mendapatkan beberapa penghargaan nasional (C) warga nonmuslim di daerahku juga menghadiri acara halalbihalal (D) dalam bersepak bola kerja sama antarpemain sangat dibutuhkan (E) orang tua itu kesana kemari mencari kebutuhan hidup keluarganya

13. Kalimat manakah yang di dalamnya terdapat pilihan kata yang tidak tepat? (A) pada kesempatan berbahagia ini saya mengucapkan terima kasih kepada hadirin (B) kepribadian siswa yang satu tidak boleh dibandingkan dengan kepribadian siswa lain (C) kita harus berusaha secara maksimal untuk mencapai hasil gemilang (D) akan tetapi, akhirnya pemerintah tetap memperoleh hak kepemilikan lahan tersebut (E) tenggat pengumpulan tugas kelompok belajar kelas XI belum diumumkan guru kami 14. Kalimat manakah yang mengandung bentukan kata yang tidak tepat? (A) komponis itu merubah irama lagu asli ketika mengaransemen ulang lagu itu (B) Christiano Ronaldo memosisikan dirinya sebagai pemain sepak bola profesional (C) peserta standup comedy memarodikan kejadian-kejadian aktual (D) mahasiswa tingkat akhir mempraktikkan salah satu teori untuk menganalisis data (E) pemerintah batal menerapkan peraturan baru di bidang energi kelistrikan 15. Manakah kalimat yang tidak efektif ? (A) masyarakat adat mengkhawatirkan hak mereka atas tanah, wilayah, dan sumber daya alam (B) dengan demikian, terdapat 5.000 kelompok masyarakat adat yang berada di 90 negara di dunia (C) dalam musyawarah masyarakat adat tahun lalu telah dibicarakan hak dan kewajiban mereka (D) perubahan iklim menjadi persoalan bukan hanya bagi manusia, melainkan juga bagi alam (E) bentuk kegiatan ini adalah menyusun desain, mengembangkannya, dan pemasaran produknya

3

SBMPTN 2017 Bahasa Indonesia Soal SBMPTN 2017 - Bahasa Indonesia

(1) Pemerintah sebaiknya meninjau kebijakan Ujian Nasional. (2) Ujian Nasional yang memiliki standar soal dan penilaian sama untuk semua siswa di mana pun memiliki lebih banyak mudarat daripada manfaatnya sehingga timbul hal-hal yang justru tidak sesuai dengan tujuan pendidikan, seperti menanamkan nilai-nilai kejujuran pada siswa. (3) Kemendikbud menyebutkan sejumlah alasan mengapa Ujian Nasional perlu dimoratorium. (4) Salah satu alasannya adalah Ujian Nasional kurang mendorong berkembangnya kemampuan siswa secara utuh. (5) Alasan lain adalah sulitnya memperoleh Ujian Nasional yang kredibel dan bebas kecurangan karena cakupan ujian yang terlalu luas. (6) Selain itu, untuk menggelar Ujian Nasional, dibutuhkan sumber daya yang sangat besar. (Diadaptasi dari http://koran.tempo.co/konten/2016/11/29/ tak-perlu-ujian-nasional)

01. Apakah inti kalimat (2)? (A) Ujian Nasional memiliki lebih banyak mudarat daripada manfaatnya. (B) Ujian Nasional seharusnya menanamkan nilai-nilai kejujuran pada siswa. (C) Ujian Nasional memiliki standar soal dan penilaian yang sama untuk semua siswa. (D) Ujian Nasional menimbulkan hal-hal yang tidak sesuai dengan tujuan pendidikan. (E) Ujian Nasional memiliki standar soal yang sama untuk semua siswa.

02. Pernyataan mana yang tidak sesuai dengan isi teks tersebut? (A) Ujian Nasional telah melenceng dari tujuan pelaksanaan yang sebenarnya. (B) Pemerintah berencana me-moratorium karena banyaknya hal negatif dalam Ujian Nasional. (C) Kemendikbud merencanakan moratorium Ujian Nasional karena besarnya sumber daya. (D) Pelaksanaan Ujian Nasional selama ini belum bebas dari praktik ketidakjujuran. (E) Ujian Nasional tidak jadi di-moratorium karena tidak disetujui oleh berbagai pihak. 03. Apa pesan tersirat di balik teks yang ingin disampaikan penulis? (A) Ujian Nasional sebaiknya mendorong perkembangan kemampuan siswa. (B) Ujian Nasional harus dilakukan dengan prinsip kejujuran dan efisiensi. (C) Pemerintah sebaiknya melaksanakan moratorium Ujian Nasional. (D) Pemerintah harus menghapus Ujian Nasional. (E) Kredibilitas Ujian Nasional perlu diperhatikan.

4

SBMPTN 2017 Bahasa Indonesia, Soal SBMPTN 2017 - Bahasa Indonesia

(1) Salah satu alasan para orang tua tidak memberikan pendidikan prasekolah bagi anaknya adalah karena tidak ingin anaknya kehilangan masa kecil. (2) Telah umum dikatakan bahwa masa kecil adalah masa permainan. (3) Padahal, pendidikan prasekolah anak bukanlah pendidikan yang "menyeramkan" seperti halnya sekolah yang menuntut anak untuk ini dan itu. (4) Pendidikan anak usia dini tetap memberikan kebebasan kepada anak untuk bermain dalam konteks bermain sambil belajar. (5) Lebih daripada itu, pendidikan anak usia dini penting karena dapat membentuk kesiapan diri anak dalam menghadapi masa sekolah. (6) Kecerdasan kognitif, afektif, dan psikomotor anak akan terbentuk dengan baik lewat pendidikan ini. (Diadaptasi dari http://rubrikita.com/2014/12/ pentingnya-pendidikan-anak-usia-dini.html)

04. Kalimat yang menggunakan kata nonformal dalam paragraf di atas adalah kalimat .... (A) (1) (B) (3) (C) (4) (D) (5) (E) (6) 05. Kata yang tidak tepat dalam teks di atas adalah .... (A) memberikan dalam kalimat (1) (B) permainan dalam kalimat (2) (C) menyeramkan dalam kalimat (3) (D) kesiapan dalam kalimat (5) (E) terbentuk dalam kalimat (6) 06. Kalimat manakah yang seharusnya mengakhiri paragraf di atas? (A) Oleh karena itu, orang tua harus memperhatikan kompetensi anak usia dini. (B) Jadi, pendidikan anak usia dini telah memberikan kompetensi yang lengkap dalam mempersiapkan masa sekolah. (C) Untuk itu, seorang anak dapat mengembangkan potensi secara mandiri di sekolah dengan bantuan guru. (D) Dengan demikian, anak perlu didaftarkan pada program pendidikan usia dini yang sesuai. (E) Berdasarkan hal itu, sebelum memasuki masa usia sekolah dasar, seorang anak perlu memperoleh pendidikan usia dini.

(1) Flu singapura pada dasarnya adalah flu yang menyerang kekebalan tubuh, terutama anak-anak, yang dilakukan oleh virus RNA dari jenis Reterovirus. (2) Virus ini pertama kali ditemukan di Singapura dan itu pula yang kemudian menyebabkan flu ini dinamakan flu singapura. (3) Dalam bahasa medis internasional flu ini dinamakan HMFD atau Hand Mouth and Foot Disease. (4) Penamaan ini berdasar pusat serangan yang terfokus pada luka di kaki, tangan, dan mulut. (5) Bila kebanyakan virus flu menyerang area pernafasan dan area lendir, maka serangan virus ini cenderung lebih muncul pada kulit. (6) Kadang gejala pada kulit akan muncul pertama kali dari gejala demam dan tanda-tanda flu lain, seperti munculnya sariawan berat dan peradangan di area tenggorokan depan. (7) Proses penularannya cukup mudah, tetapi kebanyakan penularan berawal dan interaksi langsung kulit dengan media yang sudah terpapar virus RNA. (8) Penularan itu bisa melalui proses sentuhan kulit dengan pengidap flu singapura atau tersentuh benda yang sudah terkontaminasi flu singapura. (9) Aktivitas makan dan minum bercampur dengan pengidap flu singapura juga bisa menyebabkan penularan. (Diadaptasi dari http://deherba.com/apa-obat-herbal-flusingapura-terbaik.html/)

07. Kata yang penulisannya tidak mengikuti kaidah ejaan terdapat pada kalimat .... (A) (1) (B) (2) (C) (4) (D) (5) (E) (9) 08. Penggunaan konjungsi yang tidak tepat terdapat dalam kalimat .... (A) (1) (B) (2) (C) (5) (D) (6) (E) (7)

5

SBMPTN 2017 Bahasa Indonesia, Soal SBMPTN 2017 - Bahasa Indonesia

09. Kata terkontaminasi pada kalimat (8) bermakna .... (A) tersentuh (B) terkotori (C) terkena (D) terpengaruh (E) tercampuri 10. Ide pokok paragraf kedua adalah .... (A) penularan flu singapura (B) media penularan flu singapura (C) gejala flu singapura (D) penyebab flu singapura (E) akibat flu singapura 11. Pernyataan manakah yang sesuai dengan isi teks di atas? (A) Orang dewasa cenderung tidak terserang flu singapura. (B) Penularan virus flu singapura melalui kontak langsung. (C) Singapura menjadi tempat penyebaran flu. (D) Flu singapura menyerang kulit dan tenggorokan. (E) Flu singapura bisa ditularkan melalui udara.

(1) Globalisasi menjadi tantangan untuk semua aspek kehidupan termasuk kebudayaan. (2) Era global menuntut kesiapan kita untuk siap berubah menyesuaikan perubahan zaman dan mampu mengambil setiap kesempatan. (3) Budaya tradisional di Indonesia sebenarnya lebih kreatif dan tidak bersifat meniru, namun yang menjadi masalah adalah bagaimana mempertahankan jati diri bangsa. (4) Sebagai contoh sederhana, budaya gotong royong di Indonesia saat ini hampir terkikis habis, yang digantikan oleh sikap individual dan tidak peduli kepada orang lain. (5) Perlu dipikirkan agar kebudayaan kita tetap dapat mencerminkan kepribadian bangsa. (6) Dalam era globalisasi, kebudayaan tradisional mulai mengalami erosi. (7) Semua orang, terutama anak muda, lebih senang menghabiskan waktunya mengakses internet daripada mempelajari tarian dari kebudayaan sendiri. (8) Orang akan merasa bangga ketika dapat meniru gaya berpakaian orang Barat dan menganggap budayanya ketinggalan zaman. (9) Globalisasi akan selalu memberikan perubahan. (10) Oleh karena itu, harus meneliti apakah berbagai budaya yang masuk tersebut bersifat positif atau negatif. (Diadaptasi dari http:// www.budayatradisionalindonesia.blogspot.co.id/)

12. Kalimat yang tidak efektif dalam teks di atas adalah kalimat .... (A) (1) (B) (2) (C) (5) (D) (8) (E) (10) 13. Kalimat manakah yang merupakan simpulan dan isi teks di atas? (A) Budaya tradisional Indonesia orisinal dan kreatif. (B) Kebudayaan tradisional perlu dipertahankan. (C) Globalisasi menggerus kebudayaan tradisional. (D) Budaya gotong royong di Indonesia terkikis habis. (E) Generasi muda mengidolakan budaya Barat. 6

SBMPTN 2017 Bahasa Indonesia, Soal SBMPTN 2017 - Bahasa Indonesia

14. Gagasan pokok sebelum kedua paragraf di atas adalah .... (A) unsur kebudayaan (B) konsep globalisasi (C) keragaman budaya (D) kebudayaan tradisional (E) dampak globalisasi 15. Mengapa kebudayaan tradisional mengalami erosi di era globalisasi? (A) Globalisasi mengakibatkan perubahan kebudayaan. (B) Kebudayaan asing lebih baik daripada kebudayaan tradisional. (C) Kebudayaan tradisional dinilai ketinggalan zaman. (D) Kebudayaan tradisional tidak disukai generasi muda. (E) Globalisasi dianggap mampu menjawab tantangan zaman.

7

SBMPTN 2016 Bahasa Indonesia Soal - Bahasa Indonesia SBMPTN

Teks 1 digunakan untuk menjawab soal nomor 61 sampai dengan 65! Teks 1 (1) Permainan tradisional dilakukan dengan banyak gerakan oleh anak-anak, misalnya permainan kasti, gasing, dan kelereng. (2) Dengan demikian, dia akan terhindar dan obesitas, (3) Sosialisasi dan komunikasi mereka tercapai, karena dalam permainan tradisional paling sedikit dimainkan oleh dua anak. (4) Permainan tradisional juga dapat menentukan strategi dalam bermain. (5) Mereka juga akan bekerja sama dengan anggota tim. (6) Memang, permainan tradisional bagi anak sangat baik karena banyak nilai positifnya. (7) Permainan tradisional saat ini tidak mudah dilakukan, terutama di kota-kota besar. (8) Permainan tradisional pada umumnya memerlukan arena luas, umpamanya bermain kelereng, bermain gasing, petak umpet, dan lain-lain. (9) Selain itu, banyak orang tua melarang anaknya bermain permainan tradisional karena takut kotor atau takut kulit anaknya terbakar panas matahari. (10) Orang tua banyak memberikan kepada anaknya permainan elektronik, misalnya video game dan mobil-mobilan. (11) Permainanpermainan tersebut dimainkan di dalam rumah saja. (12) Akibatnya, anak kurang bersosialisasi dengan temannya dan kurang bergerak. 61. Apa gagasan utama paragraf ke2? (A) Permainan tradisional yang penting bagi anak (B) Tempat memainkan permainan tradisional (C) Penyebab punahnya permainan tradisional (D) Kendala memainkan permainan tradisional (E) Jenis-jenis permainan tradisional di kota

8

62. Pada kalimat nomor berapa terdapat kesalahan penggunaan tanda baca koma? (A) 1 (B) 3 (C) 6 (D) 7 (E) 9 63. Pernyataan mana yang TIDAK sesuai dengan isi teks? (A) Permainan tradisional sangat baik untuk kesehatan anak. (B) Permainan tradisional perlu mendapat perhatian dari orang tua. (C) Permainan tradisional tidak mudah dimainkan di kota-kota besar. (D) Permainan tradisional sangat bermanfaat untuk kegiatan sosial anak. (E) Permainan tradisional tidak dipahami dengan baik oleh orang tua. 64. Apa yang dirujuk oleh kata itu pada kalimat (9)? (A) Kendala orang tua (B) Kendala budaya (C) Kendala arena (D) Kendala alat (E) Kendala anak 65. Apa kelemahan yang ada di dalam paragraf pertama? (A) Penggunaan kata dia pada kalimat (2) (B) Penggunaan kata mereka pada kalimat (3) (C) Penggunaan kata juga pada kalimat (4) (D) Penggunaan kata akan pada kalimat (5) (E) Penggunaan kata bagi pada kalimat (6)

SBMPTN 2016 Bahasa Indonesia, Soal - Bahasa Indonesia SBMPTN

Teks digunakan untuk menjawab soal nomor 66 sampai dengan 70. Teks 2 (1) Sebuah studi menunjukkan bahwa anak yang dibiasakan mendengarkan cerita sejak dini dan dikenalkan dengan kebiasaan membaca memiliki perkembangan jaringan otak yang lebih awal. (2) Sebaliknya, anak yang tidak dikenalkan dengan kebiasaan membaca memiliki perkembangan yang kurang pada jaringan tersebut. (3) Anak-anak balita dengan orang tua yang rutin membacakan buku untuk mereka mengalami perbedaan perilaku dan prestasi akademik dengan anak-anak dengan orang tua yang cenderung pasif dalam membacakan buku. (4) Menurut sebuah studi baru yang diterbitkan dalam jurnal Pediatrics menemukan perbedaan yang juga terjadi pada aktivitas otak anak. (5) Peneliti mengamati perubahan aktivitas otak anak-anak usia 3 sampai dengan 5 tahun yang mendengarkan orang tua mereka membacakan buku melalui scanner otak yang disebut functional magnetic resonance imaging (FMRI). (6) Orang tua menjawab pertanyaan tentang berapa banyak mereka membacakan cerita untuk anak-anak serta seberapa sering melakukan komunikasi. (7) Para peneliti melihat bahwa ketika anak-anak sedang mendengarkan orang tua bercerita, sejumlah daerah di bagian kiri otak menjadi lebih aktif. (8) Ini adalah daerah yang terlibat dalam memahami arti kata, konsep, dan memori. (9) Wilayah otak ini juga menjadi aktif ketika anak-anak bercerita atau membaca. (10) Pada studi ini menunjukkan bahwa perkembangan daerah ini dimulai pada usia yang sangat muda. (11) Yang lebih menarik adalah bagaimana aktivitas otak di wilayah ini lebih sibuk pada anak-anak yang orang tuanya gemar membaca. (12) Membacakan buku untuk anak membantu pertumbuhan neuron di daerah ini yang akan menguntungkan anak di masa depan dalam hal kebiasaan membaca. (Diadaptasi dari http:health.kompas.com/ read/2016/02/07/135500623/ membacakan.Buku.Meningkatkan.Kinerja.Otak.Balita)

66. Apa judul yang tepat untuk Teks 2 tersebut? (A) Pengenalan Kebiasaan Membaca sejak Dini (B) Balita dan Kebiasaan Mendengarkan Cerita (C) Pembiasaan Anak dalam Mendengarkan Cerita (D) Peningkatan Kinerja Otak melalui Membaca (E) Peran Orang Tua dalam Membacakan Cerita 67. Kalimat manakah yang TIDAK efektif dalam Teks 2? (A) 1 dan 7 (B) 2 dan 8 (C) 3 dan 9 (D) 4 dan 10 (E) 5 dan 12 68. Bagaimana hubungan isi antarparagraf dalam Teks 2? (A) Paragraf ke-2 memaparkan simpulan penelitian yang dibahas pada paragraf ke -1. (B) Paragraf ke-2 memerinci temuan penelitian yang dipaparkan pada paragraf ke-1. (C) Paragraf ke-2 memaparkan perbedaan perilaku yang dibahas pada paragraf ke1. (D) Paragraf ke-1 memaparkan penelitian terhadap balita yang diuraikan pada paragraf ke-2. (E) Paragraf ke-1 memaparkan hasil penelitian yang diuraikan pada paragraf ke-2.

9

SBMPTN 2016 Bahasa Indonesia, Soal - Bahasa Indonesia SBMPTN

69. Apa simpulan teks tersebut? (A) Anak-anak yang belajar membaca pada usia 3-5 tahun akan mempercepat perkembangan otaknya. (B) Semakin awal kebiasaan membaca buku diperkenalkan, semakin aktif otak anak bekerja. (C) Anak-anak yang mulai belajar membaca sejak balita akan menjadi anak-anak yang otak kirinya lebih aktif. (D) Kebiasaan mendengarkan cerita dan membaca sejak usia balita berdampak positif untuk perkembangan otak. (E) Otak kiri bertanggung jawab terhadap proses pemahaman seperti kata dan konsep. 70. Apa gagasan utama yang tepat untuk paragraf selanjutnya dari teks tersebut? (A) Tujuan membiasakan anak membaca cerita sejak usia dini (B) Strategi penerapan hasil penelitian untuk pendidikan anak (C) Faktor-faktor yang dapat mendorong anak gemar membaca (D) Manfaat kebiasaan membaca dan mendengarkan cerita pada anak (E) Kebiasaan membaca dan mendengarkan cerita pada anak Teks 3A dan 3B digunakan untuk menjawab soal nomor 71 sampai dengan 75. Teks 3A (1) Bioteknologi merupakan teknologi dengan pemanfaatan mikroorganisme, tanaman, atau hewan melalui modifikasi proses seluler untuk menghasilkan produk yang bermanfaat. (2) Banyak negara, khususnya negara-negara maju, menjadikan bioteknologi sebagai penahaaan terdepan ketahanan pangan. (3) Penelitian bioteknologi mencakup berbagai bidang, yaitu pertanian, peternakan, farmakoseutika, kimia, pemrosesan makanan, dan fermentasi. (4) Di Indonesia sumber daya manusia yang berkompetensi di bidang bioteknologi masih sedikit dan terbatas. (5) Padahal, perkembangan bioteknologi global dan bisnis yang terkait sangat menjanjikan. (6) Tenaga terampil dan ahli yang kompeten 10

di bidang bioteknologi sangat diperlukan. (7) Pendidikan dan pengembangan SDM di bidang bioteknologi harus mendapat prioritas dan dukungan, baik dari pemerintah, universitas, lembaga penelitian, maupun perusahaan swasta terkait. (8) Semua pihak harus mendukung agar SDM bidang bioteknologi semakin banyak. Teks 3B (9) Dewasa ini perkembangan bioteknologi tidak hanya didasari oleh biologi semata, tetapi juga oleh ilmu-ilmu terapan dan murni lain, seperti biokima, computer, biologi molecular, mikrobiologi, genetika, kimia, matematika, dan lain sebagainya. (10) Dengan kata lain, bioteknologi adalah ilmu terapan yang mengabungkan berbagai cabang ilmu dalam proses produksi barang dan jasa. (11) Banyak negara menjadikan bioteknologi sebagai pertahanan terdepan ketahanan panganannya, khususnya di negara-negara maju. (12) Akan telapi, tidak semua pihak dapat menerima bioteknologi karena dianggap bertentangan dengan kodrat alam. (13) Bioteknologi memunculkan kontroversi, misalnya bayi tabung, pengklonan manusia, dan transplantasi organ. (14) Kemajuan di bidang bioteknologi tidak terlepas dari berbagai kontroversi yang melingkupi perkembangan teknologinya. (Diadapatasi dari beberapa sumber) 71. Apa makna kata kompeten pada kalimat (6) Teks 3A? (A) Ahli (B) Pintar (C) Pakar (D) Hebat (E) Cakap 72. Berdasarkan isi Teks 3A, kepada siapa penulis berpihak? (A) Tenaga terampil bidang bioteknologi (B) Peneliti bidang bioteknologi (C) Lembaga penelitian bioteknologi (D) Pangusaha bidang bioteknologi (E) Sumber daya manusia bioteknologi

SBMPTN 2016 Bahasa Indonesia, Soal - Bahasa Indonesia SBMPTN

73. Apa perbedaan tujuan penulisan Teks 3A dengan Teks 3B? (A) Teks 3A menjelaskan definisi bioteknologi; Teks 3B memaparkan ilmu -ilmu yang mendasari bioteknologi. (B) Teks 3A menjelaskan pentingnya pengembangan SDM bioteknologi; Teks 3B menjelaskan penolakan pemanfaatan bioteknologi. (C) Teks 3A menguraikan peran bioteknologi di bidang pangan; Teks 3B memaparkan peran bioteknologi di bidang kesehatan manusia. (D) Teks 3A memaparkan keunggulan bioteknologi: Teks 3B memaparkan kelemahan bioteknologi. (E) Teks 3A menjelaskan kendala pengembangan SDM bioteknologi; Teks 3B menjelaskan kemajuan bidang bioteknologi. 74. Informasi apa yang ada di dalam Teks 3B, tetapi TIDAK dimuat dalam Teks 3A? (A) Definisi bioteknologi (B) Pengembangan SDM bioteknologi (C) Penolakan terhadap bioteknologi (D) Cakupan bidang bioteknologi (E) Komitmen pemerintah di bidang bioteknologi 75. Apa kelemahan isi teks? (A) Teks 3A tidak memuat secara terperinci contoh bidang bioteknologi. (B) Teks 3B tidak memuat manfaat bioteknologi bagi kehidupan. (C) Teks 3B tidak menjelaskan latar tentang produksi barang dan jasa. (D) Teks 3A tidak memuat alasan pentingnya pengembangan SDM bioteknologi. (E) Teks 3B tidak memuat komitmen negara maju mendukung bioteknologi.

11

SBMPTN 2015 - Bahasa Indonesia

Teks berikut digunakan untuk menjawab soal nomor 60-05. (1) Generasi hari ini berbeda dengan generasi sebelumnya karena generasi hari ini lahir di tengah kecanggihan teknologi digital sehingga mereka dimanjakan game online dan media sosial. (2) Sejatinya, smart phone mendukung proses belajar-mengajar sehingga proses transfer of knowledge dan pembinaan karakter dan keterampilan berjalan lancar. (3) Namun, kita juga sering menjumpai remaja yang berada dalam sebuah forum tanpa berkomunikasi satu dengan yang lain, karena asyik dengan dunianya sendiri. (4) Meminjam bahasa Don Tapscott (2013), generasi ini adalah generasi acuh tak-acuh. (5) minat mereka hanya mengenai budaya populer, para pesohor, dan teman-teman mereka. (6) Hal itu menunjukkan bahwa teknologi digital membawa sejumlah dampak positif dan negatif. (7) Menurut Felder dan Soloman (1993), “Pembelajar di zaman informasi ini memunyai kecenderungan gaya belajar aktif, sequential, sensing, dan visual.” (8) Fokus pembelajaran adalah pembelajaran seumur hidup, bukan demi ujian semata. (9) Guru tidak perlu khawaiir jika siswa lupa tanggal peristiwa penting dalam sejarah, karena mereka dapat mencarinya melalui buku dan web. (10) Guru perlu mengajari mereka cara belajar yang baik dan mendorong mereka untuk gemar membaca dan menulis. (11) Jadi, yang terpenting bukan hanya tentang apa yang diketahui ketika mereka lulus, melainkan juga untuk mencintai pembelajaran seumur hidup. (Di adaptasi dari http: //koran.tempo.co/konten)

01. Kalimat topik paragraf pertama adalah kalimat .... (A) (B) (C) (D) (E) 12

(1) (2) (3) (4) (6)

02. Kesalahan penggunaan tanda baca koma terdapat pada kalimat .... (A) (3) (B) (5) (C) (7) (D) (8) (E) (11) 03. Pertanyaan manakah yang jawabannya tidak ditemukan dalam teks tersebut? (A) Apa keuntungan teknologi digital bagi generasi sekarang? (B) Mengapa generasi sekarang berbeda dengan generasi sebelumnya? (C) Di manakah generasi sekarang biasa menggunakan teknologi digital? (D) Bagaimana pendapat pakar mengenai model pembelajaran yang ideal? (E) Apa tujuan jangka panjang pembelajaran bagi siswa? 04. Kata ganti mereka pada kalimat (5) merujuk kepada .... (A) generasi acuh tak-acuh (B) generasi ini (C) generasi (D) remaja (E) siswa 05. Kelemahan paragraf kedua teks tersebut adalah .... (A) data tidak dipaparkan dengan jelas dan menyeluruh (B) hal yang harus dilakukan guru dalam mengajar tidak dikemukakan (C) pentingnya pembelajaran seumur hidup tidak di jelaskan (D) gaya belajar sequential, sensing, dan visual tidak dijabarkan (E) manfaat web dalam pembelajaran tidak dijelaskan

SBMPTN 2015 - Bahasa Indonesia

Teks berikut digunakan untuk menjawab soal nomor 06—15. (1) Puncak kejayaan maritim nusantara terjadi pada masa Kerajaan Majapahit. (2) Majapahit berhasil menguasai dan mempersatukan nusantara. (3) Mempunyai pengaruh sampai ke negara-negara asing, seperti Thailand, Kamboja, India, Filipina, dan China. (4) Kejayaan ini disebabkan karena kemampuan membaca potensi yang dimiliki. (5) Ketajaman visi dan kesadaran terhadap posisi strategis nusantara telah membawa negara ini disegani oleh negara-negara lain. (6) Namun sayang kini kejayaan itu tidak lagi banyak dikenang. (7) Kejayaan tersebut seakan tertutup oleh potret kemiskinan yang melanda rakyat Indonesia. (8) Kecintaan kita pada laut juga makin dangkal. (9) Rasa keterpihakan negara terhadap dunia maritim pun masih lemah. (10) Meskipun Kementerian Kelautan dan Perikanan sudah dibentuk, namun fokus pembangunan negara ini masih berfokus di sektor darat. (11) Masalah utamanya adalah masalah paradigma. (12) Paradigma darat/agraris masih kuat melekat pada kebanyakan masyarakat Indonesia. (13) Bangsa Indonesia masih mengidap kerancuan identitas. (14) Di satu pihak bangsa Indonesia memunyai persepsi kewilayahan tanah air, tetapi di pihak lain memosisikan diri secara kultural sebagai bangsa agraris dengan puluhan juta petani miskin yang tidak sanggup kita sejahterakan, sedangkan kegiatan industri modern sulit berkompetisi dengan bangsa lain. (15) Akibat dari hal ini adalah pembangunan perekonomian maritim dan pembangunan sumber daya manusia tidak pernah dijadikan arus utama pembangunan nasional, yang didominasi oleh persepsi dan kepentingan daratan semata. (16) Dari paparan tersebut, kita mendapati kenyataan bahwa Indonesia sebagai negara kepulauan terbesar di dunia belum mampu memberdayakan potensi ekonomi kelautan. (17) Negeri ini belum mampu mentransformasikan sumber kekayaan laut menjadi sumber kemajuan dan kemakmuran bangsa. (18) Hal ini ditunjukkan pada tabel berikut.

Tabel Produk Perikanan Indonesia

(19) Agar dapat menjadi bangsa yang kuat dan disegani di mata internasional, maka Indonesia harus kembali brawasan maritim. (20) Permasalahannya adalah apakah masih bisa membangkitkan kembali kejayaan masa lalu tersebut di tengah krisis multidimensi yang belum usai menerpa bangsa kita. (21) Mengembalikan visi kemaritiman bukan sesuatu hal yang mudah. (22) Selain dibutuhkan kemauan yang tinggi untuk merombak sistem yang ada, masalah penyediaan infrastruktur menjadi masalah tersendiri.

(Diadaptasi dari http: portalmaritimindonesia.blogspot.com)

06. Gagasan utama paragraf kedua teks tersebut adalah .... (A) kini kejayaan itu tak lagi dikenang (B) Indonesia saat ini tidak sejaya Majapahit (C) kecintaan rakyat Indonesia pada laut makin rendah (D) negara tidak berpihak kepada kemaritiman (E) fokus pembangunan saat ini masih berada di darat 07. Diksi yang tidak tepat terdapat pada kalimat .... (A) (15) (B) (17) (C) (19) (D) (21) (E) (22) 08. Kesalahan penggunaan tanda baca ditemukan pada kalimat .... (A) (1) (B) (3) (C) (4) (D) (6) (E) (10)

13

SBMPTN 2015 - Bahasa Indonesia

09. Mengapa paradigma menjadi masalah utama pembangunan Indonesia? (A) Indonesia belum dapat mengembalikan kejayaan seperti zaman Majapahit. (B) Paradigma agraris masih kuat melekat pada kebanyakan masyarakat Indonesia. (C) Indonesia merupakan negara maritim, tetapi masyarakatnya berpikir agraris. (D) Kejayaan Indonesia masih tertutup oleh potret kemiskinan rakyat Indonesia. (E) Kecintaan pada laut dan keberpihakan pada maritim masih lemah. 10. Kalimat yang tidak efektif adalah kalimat .... (A) (3) dan (4) (B) (5) dan (6) (C) (7) dan (8) (D) (9) dan (10) (E) (13) dan (14) 11. Apa simpulan isi teks tersebut? (A) Indonesia sebenarnya memiliki potensi maritim yang besar. (B) Keuntungan sebagai negara maritim tidak di manfaatkan. (C) Indonesia merupakan negara maritim yang salah urus. (D) Indonesia belum mampu memberdayakan potensi kelautan. (E) bangsa Indonesia mengidap kerancuan identitas. 12. Tujuan penulisan teks tersebut adalah .... (A) memaparkan bahwa Indonesia pernah disegani bangsa lain melalui kejayaan Majapahit (B) menggambarkan perkembangan maritim Indonesia sejak Majapahit hingga kini (C) menunjukkan bahwa Indonesia dahulu merupakan negara maritim yang kuat (D) membuktikan adanya potensi kekayaan laut Indonesia yang melimpah (E) menyadarkan bangsa indonesia untuk kembali berwawasan maritim

14

13. Apa judul yang tepat untuk teks tersebut? (A) Negara Maritim Versus Negara Agraris (B) Keracunan Identitas Bangsa Indonesia (C) Pemberdayaan Potensi Kelautan Indonesia (D) Pengembalian Kejayaan Indonesia Melalui Maritim (E) Pembangunan Maritim Untuk mengatasi Kemiskinan 14. Apa simpulan keseluruhan isi tabel dalam teks tersebut? (A) Seluruh jenis produksi ikan mengalami kenaik an yang seimbang. (B) Ada kecenderungan produksi ikan meningkat pada tahun berikutnya. (C) Produksi perikanan jenis budi daya Iebih baik daripada perikanan tangkap. (D) Kenaikan produksi ikan tangkap Iebih tajam dibandingkan budi daya tawar. (E) Kenaikan produksi ikan tangkap Iebih tajam daripada perikanan budi daya. 15. Apa kelemahan teks tersebut? (A) Tidak ada hubungan isi paragraf pertama dan kedua. (B) Data dalam table tidak mendukung keseluruhan isi teks. (C) Tabel seharusnya diletakkan setelah paragraf terakhir. (D) Tidak ada uraian mengenai definisi kerancuan paradigma. (E) Tidak dijelaskan program kerja kementerian kelautan.

SNMPTN Bahasa Indonesia

01. Kemunculan pola-pola lingkaran geometris di ladang atau crop circles selalu …. dengan kehadiran alien di tempat itu dengan mengendarai UFO. Dugaan tersebut didasarkan keyakinan bahwa manusia tidak mampu …. pola serumit itu dalam waktu singkat dengan hasil yang hampir sempurna. Sesuai namanya, UFO atau Unidentified Flying Object adalah objek terbang yang tidak …. Bentuknya bermacam-macam, ada yang seperti titik cahaya yang diam sejenak dan menghilang cepat, ada yang seperti piring terbang. UFO selalu dikaitkan dengan alien, sang makhluk luar angkasa yang dalam filmfilm fiksi ilmiah …. sebagai sosok makhluk kecil, berkepala gundul, telinga lebar dan berwarna hijau. Ia digambarkan memiliki kekuatan super, termasuk …. dirinya sama persis dengan makhluk bumi. Urutan kata yang paling tepat untuk melengkapi teks di atas adalah …. (A) Dihubungkan, membentuk, dikenal, ditampilkan, membuat. (B) Dihubungkan, membentuk, dikenal, digambar, mencipta. (C) Dikaitkan, menghasilkan, dikenal, dimunculkan, membentuk. (D) Dikaitkan, membuat, dikenali, digambarkan, mengubah. (E) Disertai, membuat, dikenali, ditampilkan, memunculkan.

02. Walaupun wajib belajar pendidikan dasar 9 tahun sudah dicanangkan dalam kurun waktu yang relatif lama, penuntasannya masih belum tercapai. Banyak masalah yang timbul dalam pelaksanaan wajib belajar 9 tahun, terutama di daerah pedesaan dan daerah pegunungan atau terpencil. Penyebab ketidaktuntasan wajib belajar dapat diidentifikasi sesuai dengan kondisi wilayah dan masyarakatnya. Dari sejumlah hasil penelitian, ditemukan bahwa penyebabnya adalah (1) masyarakat memiliki kondisi ekonomi yang lemah, (2) sosial budaya masyarakat yang kurang mendukung, (3) kurangnya sarana pendidikan, (4) rendahnya kualitas dan dedikasi guru, (5) letak geografis yang sulit dijangkau, (6) keterbatasan informasi, dan (7) persepsi masyarakat yang menganggap kurang pentingnya pendidikan bagi dirinya sendiri. Kenyataan itu diperkuat oleh hasil penelitian pada awal dicanangkannya wajib belajar 6 tahun. Kata itu yang bercetak miring pada teks di atas merujuk pada …. (A) Banyaknya masalah di daerah terpencil (B) Penuntasan wajib belajar belum mampu tercapai (C) Banyaknya masalah pelaksanaan wajib belajar (D) Ketidaktuntasan program wajib belajar (E) Penyebab ketidaktuntasan wajib belajar

19

SNMPTN Bahasa Indonesia

03. Teks 1 Sungguh pun sudah ada program yang namanya Bantuan Operasional Sekolah (BOS), masih banyak anak Indonesia yang kesulitan mengakses pendidikan di sekolah dasar dan menengah. Berdasarkan angka resmi yang dikeluarkan pemerintah, angka putus sekolah untuk tingkat SD dan SMP sekitar 768.960 orang yang terdiri atas 527.850 siswa SD dan 241.110 siswa SMP. Belum lagi, masih ada sedikitnya 8,3 juta orang Indonesia yang masih buta huruf. Teks 2 Salah satu penyebab lancarnya penuntasan wajib belajar 9 tahun di Kabupaten Bandung adalah adanya dukungan BOS yang dikucurkan sejak tahun 2005. Memasuki tahun 2009, program BOS mengalami perubahan tujuan, pendekatan, dan orientasi dari yang semula untuk perluasan akses menjadi peningkatan kualitas. Jumlah siswa yang menerima dana BOS di Kabupaten Bandung, tercatat 511.228 siswa. Masing-masing SD negeri sederajat 379.701 siswa, SD swasta sederajat 9.122 siswa, SMP negeri sederajat 69.289 siswa, SMP swasta sederajat 53.116 siswa. Pernyataan yang paling sesuai dengan kedua teks tersebut adalah …. (A) Kedua teks tersebut mengungkap kegagalan BOS. (B) Kedua teks tersebut mengungkap keberhasilan BOS. (C) Teks 1 mengungkap kegagalan BOS, sedangkan teks 2 mengungkapkan keberhasilan BOS. (D) Teks 1 mengungkap keberhasilan BOS, sedangkan teks 2 mengungkapkan kegagalan BOS. (E) Teks 1 mengungkap akibat kegagalan BOS, sedangkan teks 2 mengungkap penyebab keberhasilan BOS.

20

04. Seseorang akan mengembangkan karangan ilmiah yang bertema pembiasaan berperilaku jujur. Berdasarkan tema tersebut, kerangka isi karangan yang paling runtut berdasarkan pola umum-khusus adalah …. (A) Pentingnya pembiasaan berperilaku jujur, manfaat pembiasaan berperilaku jujur, bentuk pembiasaan berperilaku jujur, langkah pembiasaan berperilaku jujur, langkah berperilaku jujur. (B) Manfaat pembiasaan berperilaku jujur, pentingnya pembiasaan berperilaku jujur, sasaran pembiasaan berperilaku jujur, langkah pembiasaan berperilaku jujur, bentuk pembiasaan berperilaku jujur (C) Pentingnya pembiasaan berperilaku jujur, sasaran pembiasaan berperilaku jujur, manfaat pembiasaan berperilaku jujur, bentuk pembiasaan berperilaku jujur, langkah pembiasaan berperilaku jujur (D) Sasaran pembiasaan berperilaku jujur, manfaat pembiasaan berperilaku jujur, pentingnya pembiasaan berperilaku jujur, bentuk pembiasaan berperilaku jujur, langkah pembiasaan berperilaku jujur (E) Bentuk pembiasaan berperilaku jujur, pentingnya pembiasaan berperilaku jujur, langkah pembiasaan berperilaku jujur, sasaran pembiasaan berperilaku jujur, manfaat pembiasaan berperilaku jujur

SNMPTN Bahasa Indonesia

Bacalah soal dengan seksama kemudian kerjakan soal nomor 05 s.d. 08! (1) Dalam ilmu gizi diyakini bahwa pati dicerna dengan sempurna di dalam usus halus manusia. (2) Akan tetapi, teori tersebut sekarang dikoreksi setelah banyak penelitian baik in vitro maupun in vivo menemukan bahwa tidak semua pati yang dikonsumsi dapat dicerna dengan sempurna. (3) Kebanyakan makanan berkarbohidrat tinggi seperti: sereal, padi-padian, dan umbi-umbian diolah dengan pemanasan (dengan atau tanpa adanya air) sebelum dikonsumsi. (4) Pemanasan pati akan mengakibatkan pati mengalami gelatinisasi, suatu proses yang meliputi hidrasi dan pelarutan granula pati (Wursch, 1989:234). (5) Pemanasan kembali serta pendinginan pati yang telah mengalami gelatinisasi tersebut dapat mengubah struktur pati yang mengarah pada terbentuknya kristal baru yang tidak larut, yaitu berupa pati teretrogradasi yang sering terjadi selama pengolahan bahan berpati dapat mempengaruhi ketercernaan pati di dalam usus halus. (7) Sebagai contoh sejumlah pati pada kentang, pisang, dan kacang-kacangan serta berbagai produk olahan, misalnya roti tawar dan corn flakes ditemukan tidak dicerna dengan sempurna di dalam usus halus manusia dan hewan yang ditandai dengan adanya pati dalam digesta (isi usus) di usus besar. (8) Pati ini sebut pati tahan cerna atau resistant strach (RS) (9) secara fisiologis, RS didefinisikan sebagai jumlah dari pati dan hasil pencernaan pati yang tidak diserap di dalam usus halus individu yang sehat. (10) Secara analitis, RS didefinisikan sebagai pati yang tahan terhadap disperse di dalam air mendidih dan hidrolisis amilase pankreas.

06. Jika Anda mengutip pendapat Wursch dari teks di atas, tanpa membaca sumber aslinya, penulisan kutipan yang paling benar adalah …. (A) Menurut Wursch (dalam Suharsono, 2010 : 57), gelatinisasi terjadi akibat pemanasan pati disertai air berlebihan. (B) “Pemanasan pati disertai air berlebihan akan mengakibatkan pati mengalami gelatinisasi, suatu proses yang meliputi hidrasi dan pelarutan granula pati” (Wursch,1989) (C) Wursch (dalam Toni Suharsono, 2010) menyatakan “Pemanasan pati disertai air berlebihan akan mengakibatkan pati mengalami gelatinisasi, suatu proses yang meliputi hidrasi dan pelarutan granula pati”. (D) Dalam buku Suharsono, Wursch (1989:234) menyatakan bahwa pemanasan pati disertai air berlebihan akan mengakibatkan pati mengalami gelatinisasi, suatu proses yang meliputi hidrasi dan pelarutan granula pati. (E) Wursch (1989) menyatakan “Pemanasan pati disertai air berlebihan akan mengakibatkan pati mengalami gelatinisasi, suatu proses yang meliputi hidrasi dan pelarutan granula pati”. (Suharsono,2010).

05. Pada kalimat manakah kesalahan penggunaan ejaan (tanda baca, penulisan huruf, dan penulisan kata) ditemukan ? (A) 1 dan 3 (B) 3 dan 7 (C) 4 dan 7 (D) 4 dan 8 (E) 1 dan 8

21

SNMPTN Bahasa Indonesia

07. Rangkuman teks di atas yang paling tepat adalah …. (A) Pati yang terdapat dalam makanan yang berkarbohidrat tinggi dengan cara pengolahan pemanasan ataupun pendinginan berdasarkan penelitian in vitro dan in vivo tidak dapat dicerna sempurna dalam usus sehingga disebut patiresistan. (B) Kebanyakan makanan yang berkarbohidrat tinggi, seperti sereal, padi-padian, dan umbi-umbian mengandung pati resistan yang tidak dapat dicerna sempurna dalam usus karena diolah dengan pemanasan ataupun pendinginan. (C) Hasil penelitian terbaru sebagai koreksi sebelumnya menunjukkan bahwa pemanasan ataupun pendinginan pada makanan berkarbohidrat tinggi, seperti sereal, padi-padian, dan umbi-umbian, menghasilkan pati resistan yang tidak sempurna dicerna. (D) Penelitian terbaru menunjukkan bahwa pati yang terdapat dalam makanan yang berkarbohidrat tinggi dengan cara pengolahan pemanasan ataupun pendinginan tidak dapat dicerna sempurna dalam usus sehingga disebut pati resistan. (E) Pandangan bahwa pati dapat dicerna dengan sempurna dibantah melalui penelitian yang menunjukkan bahwa pemanasan pati dengan air berlebihan mengakibatkan gelatinisasi yang mengubah struktur pati menjadi Kristal baru yang tidak larut. 08. Pola paragraf pertama teks di atas adalah …. (A) Kalimat utama - kalimat penjelas - kalimat penjelas - kalimat penjelas (B) Kalimat penjelas - kalimat utama - kalimat penjelas - kalimat penjelas (C) Kalimat penjelas - kalimat penjelas - kalimat penjelas - kalimat utama (D) Kalimat utama - kalimat penjelas - kalimat penjelas - kalimat utama (E) Kalimat penjelas - kalimat penjelas - kalimat utama - kalimat penjelas

22

Bacalah soal dengan saksama kemudian kerjakan soal nomor 09 s.d. 12! Pelajar SMP dan SMA memiliki rentan usia 1417 tahun dan pada umumnya mereka sedang mengalami masa pertumbuhan baik fisik dan sosial, maupun emosional. Usia mereka berada pada awal usia remaja. Perkembangan fisik mereka sangat menonjol, namun perkembangan emosi sangat labil akibat dari perubahan fungsi hormon seksualnya yang pesat. Usia pelajar merupakan masa mencari identitas dan fase kebingungan dalam mencari peran sehingga ada juga yang menyebutnya masa topan dan badai. Dalam proses pencarian identitas jati diri, pelajar memerlukan tempat penyaluran kreativitas yang tepat. Mereka cenderung mencari berbagai cara penyaluran yang selaras dengan usia dan pemahaman mereka. Dampak negatif adalah mereka masih belum dapat mengidentifikasi secara benar peran dalam kehidupannya. Bagi orang dewasa, umumnya mereka cenderung dianggap kontradiksi dan suka mengacau. Permasalahan yang muncul pada masa remaja di antaranya adalah sering tidak masuk sekolah, cenderung berperilaku merusak dan melawan, waktu tidur tidak teratur, dan senang berhalusinasi, selain itu, mereka mudah frustasi dan depresi. Remaja lebih senang membentuk kelompok-kelompok kecil serta lebih percaya pada kelompoknya dari pada kepada orang tuanya ….

SNMPTN Bahasa Indonesia

09. Simpulan yang paling tepat untuk teks di atas adalah …. (A) Usia SMP dan SMA sedang kebingungan dan frustasi mencari figur panutan yang dianggap pantas untuk diteladani sehingga senang berhalusinasi dan mudah depresi. (B) Pelajar usia SMP dan SMA sedang dalam proses pencarian identitas sehingga memerlukan tempat penyaluran yang tepat dan selaras dengan pemahaman mereka (C) Usia SMP dan SMA senang membentuk kelompok kecil sehingga lebih percaya pada kelompoknya sedangkan guru dan orang tua dianggap penghalang (D) Pelajar usia SMP dan SMA sedang dalam taraf senang berhalusinasi membayangkan yang indah-indah sehingga mereka sering melamun dan mudah frustasi (E) Usia SMP dan SMA merupakan masa untuk mencari teman sejati sehingga memerlukan tempat penyaluran yang tepat selaras dengan minatnya 10. Kalimat suntingan yang paling tepat untuk kalimat pertama teks di atas adalah …. (A) Pada usia pelajar SMP dan SMA yang memiliki rentang 14-17 tahun, umumnya mereka semua sedang mengalami masa pertumbuhan secara fisik, sosial, dan emosional (B) Pelajar SMP-SMA memiliki rentang usia 14-17 tahun dan umumnya mereka sedang mengalami masa pertumbuhan, baik fisik, sosial, maupun emosional (C) Pelajar SMP dan SMA yang memiliki rentangan usia 14-17 tahun dan umumnya sedang mengalami masa pertumbuhan secara fisik, sosial, dan emosional (D) Pelajar SMP dan SMA memiliki rentang usia 14-17 tahun umumnya sedang mengalami masa pertumbuhan secara fisik, sosial, dan emosi (E) Pelajar SMP dan SMA dengan rentang usia 14-17 tahun umumnya sedang mengalami masa pertumbuhan secara fisik, sosial, dan emosional

11. Agar menjadi paragraph yang baik, kalimat penutup paragraph terakhir yang paling sesuai adalah …. (A) Oleh karena itu, gur perlu memiliki strategi khusus untuk membina pelajar usia SMP dan SMA agar mereka dapat mengembangkan diri secara positif sesuai dengan tingkat perkembangan intelektual dan emosional mereka. (B) Di samping itu, guru perlu memperhatikan perubahan yang ada pada anak-anak agar dapat mengembangkan mereka secara positif sesuai dengan perkembangan jiwa dan kebutuhan hidupnya. (C) Sehingga, guru perlu memperhatikan perubahan yang ada pada anak-anak agar dapat mengembangkan mereka secara positif sesuai dengan perkembangan jiwa dan kebutuhan hidupnya. (D) Jadi perkelahian antar pelajar usia SMP dan SMA memang wajar karena mereka mengalami perubahan dari masa-masa sebelumnya sesuai fase perkembangan jiwa mereka yang cenderung berperilaku melawan. (E) Karena itulah pelajar usia SMP dan SMA harus menyadari bahwa dirinya berbeda dari masa-masa sebelumnya, mereka berubah karena perkembangan intelektual dan emosional mereka. 12. Gagasan utama paragraf ke-2 teks di atas adalah …. (A) Pelajar SMP dan SMA cenderung bersifat kontradiksi dan suka mengacau (B) Pelajar usia SMP dan SMA sedang dalam proses pencarian identitas (C) Orang dewasa perlu mengetahui perkembangan (D) Pelajar SMP dan SMA memerlukan tempat penyaluran kreativitas (E) Remaja lebih percaya pada kelompoknya dari pada kepada orang tuanya.

23

SNMPTN Bahasa Indonesia

Bacalah soal dengan seksama kemudian kerjakan soal nomor 13 s.d. 15! (1) Tahun ini ujian nasional (UN) tetap dilaksanakan dengan beberapa perubahan. (2) Salah satu perubahan adalah kriteria kelulusan yang mengikutsertakan nilai rapor. (3) Nilai rapor yang sebelumnya tidak menjadi pertimbangan, mulai tahun ini dipertimbangkan pencapaiannya untuk membantu kelulusan siswa. (4) Tentu saja, kejujuran sekolah dituntut untuk itu. (5) Kejujuran sering tidak sesuai dengan harapan, akhirnya sulit memprediksi kemampuan seseorang. (6) Nilai rapor yang bagus belum tentu mencerminkan kemampuan yang sebenarnya jika kejujuran diragukan. (7) Dengan demikian, kualitas seseorang juga tidak dapat diketahui dengan tepat. (8) Sebagai gambaran, berikut disajikan tingkat pencapaian nilai UN mata pelajaran tahun 2010. Tabel 1 Tingkat Pencapaian Kelulusan Mata PeLajaran Tahun 2010 Mata Pela- SMA X SMA Y SMA Z Rerata jaran Matematika

45%

55%

65%

55%

Bahasa. Indonesia

35%

48%

46%

43%

Bhs. Inggris

65%

40%

63%

56%

(9) Dari Tabel 1 dapat diketahui tingkat pencapaian kelulusan di SMA X paling banyak pada mata pelajaran Bahasa Inggris, sedangkan SMA Y dan SMA Z pada Matematika. (10) Selain itu, Bahasa Indonesia mempunyai rerata terkecil dibandingkan dengan Matematika dan Bahasa Inggris. 13. Kalimat yang tidak efektif terdapat pada kalimat bernomor …. (1) 4 (2) 5 (3) 8 (4) 9

24

14. Pernyataan berikut ini yang paling sesuai dengan isi tabel di atas adalah …. (1) Urutan tingkat pencapaian kelulusan mata pelajaran dari yang terbesar ke yang terkecil pada SMA Y sama dengan pada rerata. (2) Urutan tingkat pencapaian kelulusan mata pelajaran dari yang terbesar ke yang terkecil pada SMA X sama dengan pada rerata (3) Urutan tingkat pencapaian mata pelajaran dari yang terbesar ke yang terkecil pada SMA Z sama dengan pada rerata. (4) Urutan tingkat pencapaian kelulusan mata pelajaran dari yang terkecil ke yang terbesar pada ketiga sekolah tidak ada yang sama. 15. Komentar yang paling tepat dengan isi teks di atas adalah …. (1) Nilai mata pelajaran Bahasa Inggris lebih baik daripada nilai Bahasa Indonesia; padahal, Bahasa Indonesia telah dikenal anak sejak kecil dan menjadi bahasa percakapan. (2) Sangat ironis bahwa kelulusan mata pelajaran Bahasa Indonesia terendah pada semua sekolah; padahal Bahasa Indonesia menjadi kunci keberhasilan semua mata pelajaran. (3) Kelulusan mata pelajaran Bahasa Indonesia hampir mengalahkan pencapaian kelulusan mata pelajaran Matematika; padahal, Bahasa Indonesia sudah dipelajari oleh peserta didik sejak dini. (4) Rerata kelulusan mata pelajaran Bahasa Inggris mengalahkan pencapaian kelulusan Bahasa Indonesia; padahal, Bahasa Indonesia dipelajari oleh peserta didik sejak dini.

Xpedia Bahasa Indonesia Kapita Selekta - Set 01

01. Penulisan unsur serapan yang bercetak miring pada kalimat-kalimat berikut benar, KECUALI .... (A) Salinan STTB Anda harus dilegalirsir agar sah. (B) Hak asasi manusia harus dijunjung tinggi. (C) Pasien yang datang itu berasal dari keluarga kurang mampu. (D) Reumatik neneknya akhir-akhir ini sering kambuh. (E) Hipotesis itu telah dapat dibuktikan kebenarannya 02. “ Seorang pemimpin Taliban, mullah dudullah, ditangkap pasukan internasional Provinsi kandahar selatan,” demikian diungkapkan pejabat Afganistan kepada lembaga perberitaan BBC jum’at, 19/5. Dalam kalimat di atas, terdapat kesalahan ejaan, yakni .... (A) pemakaian tanda baca koma (,) (B) pemakaian tanda baca garis miring (/) (C) penulisan kata Provinsi (D) penulisan kata lembaga pemberitaan (E) penulisan kata Jum’at 03. (1) Tiga perusahaan yang berkecimpung di dunia teknologi informasi, bergandengan tangan untuk memberi solusi jaringan teknologi bagi pengusaha kecil dan menengah. (2) Ketiga perusahaan tersebut adalah Microsoft, Cisco, dan Sun. (3) Paket solusi tiga kekuatan berharga 37.000 dolar AS tersebut diperuntukkan bagi usaha manufaktur yang tersebar di berbagai kota. (4) Paket yang ditawarkan meliputi aplikasi Microsoft business solution untuk finansial. (5) Paket tersebut dilengkapi dengan gigabit switch cisco dan dijalankan di atas Server Sun x86. Dalam kutipan di atas, terdapat kata bentukan yang tidak baku. kata bentukan tersebut terdapat dalam .... (A) kalimat 1 (D) kalimat 4 (B) kalimat 2 (E) kalimat 5 (C) kalimat 3

04. Kalimat tentang pengembangan Yogyakarta sebagai kota pendidikan terkemuka mengandung makna mendorong pertumbuhan dan perubahan kota Yogyakarta sebagai pusat pendidikan dan sebagai masyarakat belajar tidak baku. Kalimat tersebut akan menjadi baku apabila .... (A) ditambah kata yang pada kata pendidikan dan terkemuka. (B) kata mengandung diubah menjadi terkandung. (C) kata tentang diubah menjadi kata dalam. (D) kata Yogyakarta yang kedua dihilangkan. (E) kata tentang dihilangkan 05. 1. Pertumbuhan industri bergantung pada tenaga listrik dan produk kimia. 2. Produk kimia cenderung menjadi sumber pencemaran lingkungan. 3. Kita harus menjaga kelestarian lingkungan. Rangkaian yang tepat dari ketiga kalimat tersebut adalah .... (A) Pertumbuhan industri bergantung pada tenaga listrik dan produk kimia yang cenderung menjadi sumber pencemaran lingkungan dan kita harus menjaga kelestariannya. (B) Kita harus menjaga kelestarian lingkungan karena pertumbuhan industri bergantung pada tenaga listrik dan produk kimia yang cenderung menjadi sumber pencemaran lingkungan. (C) Produk kimia cenderung menjadi sumber pencemaran lingkungan, sehingga kita harus menjaga kelestarian lingkungan karena pertumbuhan industri tergantung pada tenaga listrik dan produk kimia. (D) Pertumbuhan industri tidak hanya tergantung pada tenaga listrik dan produk kimia yang cenderung menjadi sumber pencemaran lingkungan sehingga kita harus menjaga kelestariannya. (E) Kita harus menjaga kelestarian lingkungan yang diakibatkan oleh pertumbuhan industri yang bukan saja tergantung pada tenaga listrik dan produk kimia, melainkan juga produk kimia cenderung menajadi sumber.

25

Xpedia Bahasa Indonesia , Kapita Selekta - Set 01

06. Kalimat berikut strukturnya benar adalah .... (A) Meningkatkan kualitas pendidikan melalui penyusunan standar kompetensi nasional berdasarkan bidang-bidang keahlian. (B) Membentuk jaringan komunikasi antar guru sebagai wahana untuk meningkatkan proses pembelajaran. (C) Peningkatan kualitas guru dengan melaksanakan pendidikan dan pelatihan merupakan langkah penting untuk perbaikan mutu pendidikan. (D) Melalui peran serta masyarakat, menjadikan ciri konsep pendidikan pada era otonomi. (E) Dengan mengembalikan pendidikan kepada masyarakat, mengharapkan akan memberi peluang kepada lembaga tersebut semakin ditingkatkan perannya. 07. Masyarakat yang memiliki bahasa dan menggunakan bahasa disebut masyarakat bahasa. Masyarakat desa adalah masyarakat bahasa, sekelompok manusia yang ditandai oleh interaksi teratur dengan menggunakan isyarat -isyarat verbal dan terpisahkan dari kelompok kelompok lain di luar masyarakat itu. Dalam berbahasa secara tunggal ataupun majemuk, masyarakat desa dapat dikaji dari aturan-aturan pemakai bahasanya. Mereka pada umumnya terikat oleh kesamaan kaidah -kaidah berbahasa untuk menghasilkan dan menafsirkan tuturan dan kaidah-kaidah untuk menafsirkan ragam-ragam bahasa. Kalimat kedua paragraf di atas termasuk kalimat yang strukturnya tidak jelas. Agar menjadi kalimat yang jelas maknanya, kalimat tersebut harus ditambah dengan kata .... (A) merupakan (B) adalah (C) di mana (D) yang mana (E) yakni

26

08. Mobil kakak ipar saya yang baru dibeli teman saya. Kalimat di atas akan menjadi kalimat yang jelas maksudnya jika direvisi sebagai berikut .... (A) Mobil kakak ipar saya yang baru, dibeli teman saya. (B) Mobil baru kakak ipar saya dibeli teman saya. (C) Mobil kakak ipar saya yang dibeli teman saya. (D) Mobil kakak ipar saya, yang baru, dibeli teman saya. (E) Mobil kakak ipar yang baru dibeli teman saya. 09. Setelah terjadi kerusuhan di Poso, Sulawesi Tengah, polisi segera membentuk sebuah tim untuk mengetahui penyebab kerusuhan yang diperkirakan akibat provokasi pihak kettiga. Kata yang tepat untuk melambangkan kegiatan yang dilakukan oleh kepolisian adalah .... (A) pengamanan (B) penelitian (C) penyelidikan (D) pengamatan (E) pemeriksaan 10. Pernyataan “Makmur dalam keadilan dan adil dalam kemakmuran” hanyalah sebagai slogan yang dalam kenyataannya sulit untuk direalisasikan. Kata slogan dalam kalimat di atas memiliki padanan dengan kata .... (A) reklame (B) propaganda (C) wawasan (D) provokasi (E) semboyan

Xpedia Bahasa Indonesia , Kapita Selekta - Set 01

11. Pekerjaan analisis ini dimaksudkan untuk mengenali retorika penulis Indonesia, yang terwujud dalam artikel (khususnya yang diterbitkan di harian Kompas). Retorika yang dimaksud dalam tulisan ini adalah pola pikir dalam mengungkapkan gagasan untuk menghasilkan karya tulis dalam bentuk artikel. Dengan melakukan analisis artikel Harian Kompas, diharapkan dapat dikenali pola retorika Indonesia. Berdasarkan kutipan diatas merupakan penggalan dari karya ilmiah, khususnya dari penggalan dari bagian .... (A) kata pengantar (B) pendahuluan (C) kerangka teori (D) pembahasan (E) penutup 12. Menteri perdagangan mengatakan bahwa masalah penting yang terkait dengan perdagangan, khususnya perdagangan luar negeri, adalah bagaimana meningkatkan daya saing produk ekspor. Kunci daya saing tersebut adalah meningkatkan produktivitas produk. Secara sektoral, persoalan penigkatan produktivitas itu memeng berbeda-beda. Namun, secara umum masalah yang timbul dari dunia usaha cukup banyak. Birokrasi yang berbelit-belit dapat menimbulkan ekonomi biaya tinggi. Keserasian hubungan antarkalimat dalam paragraf di atas kurang lancar. Paragraf di atas akan menjadi padu jika direvisi sebagai berikut .... (A) namun (kalimat keempat) dihilangkan. (B) bagaimana (kalimat pertama) diganti dengan upaya. (C) pada awal kalimt kelima ditambahkan di antaranya adalah. (D) kunci daya saing (kalimat kedua) diubah menjadi kunci meningkatkan daya saing. (E) secara sektoral (kalimat kelima) diposisikan sebelum kata memang

13. Novelis Pramudya Ananta Toer mengimbau semua penulis sastra mendokumentasikan karya-karya sastra di tanah air. Dokumentasi sastra dapat menumbuhkan karakter dan kebanggaan sebagai bangsa. Selain itu, dokumentasi sastra yang rapi juga memacu lahirnya tulisan sastra yang kreatif dan inovatif. Menulis sastra adalah dialog dengan publik di seluruh dunia yang membacanya. Meskipun pentingnya dokumentasi sudah disadari berbagai pihak, sayang sekali dokumentasi sastra belum menjadi bagian dari tradisi bangsa kita. Kalimat sumbang pada paragraf di atas terdapat pada .... (A) Kalimat 1 (B) Kalimat 2 (C) Kalimat 3 (D) Kalimat 4 (E) Kalimat 5 14. Perhatikan paragraf berikut. Kanker perut kini masih menjadi penyebab kematian tertinggi kedua ...., dalam dekade terakhir ini jumlah kasusnya cenderung menurun .... mungkin terjadi karena tingginya konsumsi buah-buahan dan sayuran segar, serta berkurangnya kandungan garam dalam makanan akibat proses pendinginan ...., di negeri ini kasus kandungan akibat kanker perut masih menjadi masalah utama yang harus diatasi. Agar paragraf itu menjadi susunan yang padu, maka tempat-tempat yang kosong itu harus diisi dengan kata-kata perangkai yang berfungsi sebagai penghubung antarkalimat. Kata-kata perangkai itu adalah .... (A) namun, hal itu, akan tetapi (B) meskipun demikian, oleh sebab itu, dengan demikian (C) sedangkan, akibatnya, jadi (D) namun, akibatnya, jadi (E) adapun, namun, bahkan

27

Xpedia Bahasa Indonesia , Kapita Selekta - Set 01

27. Menguak purnama Caya menyayat mulut dan mata Menjengking kereta, menjengking jiwa Sayata terus ke dada Chairil Anwar Gaya bahasa puisi di atas adalah .... (A) perumpamaan (B) personifikasi (C) litotes (D) simile (E) simbolik 28. Setelah terjadi kerusuhan di Poso, Sulawesi Tengah, polisi segera membentuk sebuah tim untuk mengetahui penyebab kerusuhan yang diperkirakan akibat provokasi pihak ketiga. Kata yang tepat untuk melambangkan kegiatan yang dilakukan oleh kepolisian adalah .... (A) pengamanan (B) penelitian (C) penyelidikan (D) pengamatan (E) pemeriksaan 29. Setiap melakukan transaksi jual beli melalui kredit yang melibatkan bank selalu dikenakan provisi sebesar 1,5 % dari total kredit, dan wajib menyerahkan agunan ke bank sampai total kredit terlunasi. Kata-kata yang tepat untuk menggantikan kata yang bercetrak tebal dalam kalimat di atas adalah .... (A) perjanjian, biaya, sertifikasi (B) perjanjian, bunga, surat tanah (C) persetujuan, bunga, jaminan (D) persetujuan, bunga, biaya (E) perjanjian, biaya, jaminan

28

30. Kata serapan yang berasal dari bahasa asing digunakan secara tepat dalam kalimat .... (A) Para wakil rakyat yang duduk di DPR harus dapat menyampaikan inspirasi masyarakat kepada pemerintah (B) Tampaknya, isi pasal-pasal dalam perundangan itu tidak efektif untuk menanggulangi kejahatan. (C) Kami tidak suka membeli barang-barang yang kuantitasnya kurang baik. (D) Kita dapat menjangkau desa terpencil itu dengan menggunakan sarana komunikasi sepeda motor. (E) Orang itu dapat memanfaatkan waktu secara selektif sehingga sukses dalam usahanya.

Xpedia Bahasa Indonesia , Kapita Selekta - Set 01

15. Kesadaran masyarakat mengenai masalah lingkungan sudah mulai tumbuh .... tingkat kesadaran yang ada belum cukup tinggi .... mempengaruhi perilaku mereka .... untuk menjadi motivasi yang kuat yang dapat melahirkan tindakan nyata dalam usaha swadaya perbaikan hidup, di daerah perkotaan .... pedesaan masih harus terus dikembangkan. Kata-kata berikut yang paling tepat untuk mengisi bagian paragraf yang runpang ialah .... (A) meskipun, dalam, ataupun, baik (B) meskipun, untuk, dan, maupun (C) meskipun, untuk, ataupun, dan (D) sehingga, untuk, dalam, apalagi (E) tetapi, dalam, dan, maupun 16. Usaha budi daya ikan tuna semakin banyak dilakukan di Australia dan Afrika Selatan. Hasilnya mulai masuk ke pasar dunia. Bahkan, telah mengasai sekitar 20 persen dari total kebutuhan ikan tuna di Jepang. Mereka memprediksikan bahwa permintaan ikan tuna hasil budi daya cenderung meningkat. Pasar ikan tuna dunia kini lebih meminta yang masih segar atau hidup bukan ikan tuna yang dibekukan. Ini berarti, iakn tersebut tidak mungkin diperoleh dari penangkapan , tetapi dari budi daya ikan tuna. Untuk itu negara-negara seperti Australia serta Afrika Selatan telah menangkap gelagat pasar dunia tersebut. Kedua negara tersebut membudidayakan ikan tuna secara besar-besaran. Hal itu, menjadi ancaman serius bagi negaranegara produsen ikan tuna dari hasil penagkapan, seperti Indonesia. Isi pokok paragraf diatas adalah .... (A) Australia dan Afrika Selatan telah menangkap gelagat pasar dunia (B) Australia dan Afrika Selatan telah membudidayakan ikan tuna karena menangkap gelagat pasar dunia (C) Kedua negara membudidayakan ikan tuna secara besar-besaran karena pasar dunia meminta yang masih segar atau hidup, bukan ikan tuna yang dibekukan (D) Australia dan Afrika Selatan merupakan ancaman negara-negara produsen ikan tuna hasil penangkapan (E) Australia dan Afrika Selatan merupakan ancaman produksi ikan tuna Indonesia

17. Pengembangan industri sangat penting karena industri dapat memberikan efek ganda yang lebih besar. Jika industri otomotif, elektronik, dan motor berkembang industri komponen pun akan meningkat. Kenaikan itu dengan sendirinya akan menciptakan ribuan tenaga kerja baru di sektor jasa seperti bengkel, distributor, dan pedagang produk komponen. Pada akhirnya, itu merupakan peluang ekonomi baru bagi masyarakat. Pola pengembangan paragraf di atas adalah .... (A) sebab - akibat - akibat (B) sebab - akibat - sebab (C) akibat - sebab - akibat (D) akibat - akibat - sebab (E) sebab - sebab - akibat 18. Aksi penjarahan di kota-kota besar makin menggila. Setelah beberapa daerah di Pulau Jawa teratasi kini giliran di luar jawa mulai marak. Penjarahan sering diikuti dengan aksi kekerasan yang berdapak pada kekacauan masyarakat. Penggalan berita di atas menggunakan urutan dengan menggunakn pertanyaan .... (A) apa, mengapa, di mana (B) siapa, di mana, kapan (C) mengapa, siapa, apa (D) mengapa, di mana, bagaimana (E) apa, di mana, bagaimana

29

Xpedia Bahasa Indonesia , Kapita Selekta - Set 01

19. Inovasi sekolah dengan menggunakan Manajemen Peningkatan Mutu Berbasis Sekolah (MPMBS) semakin meningkatkan hasil yang menggembirakan. Data yang terhimpun pada Direktorat PLP dengan sampel 1000 sekolah dalam pelaksanaan selama tahun 1999, 2000, dan 2001 menunjukkan bahwa hasilnya : 2% tergolong istimewa, 25% amat baik, 55% baik, 15% cukup, dan 3% kurang. Berangkat dari hasil realistis tersebut, akhiranya MPMBS ditetapkan menjadi salah satu pola meningkatkan mutu dan memajukan sekolah yang berlaku secara nasional. Dalam MPMBS tersebut, terdapat dua komponen yang sangat menonjol, yakni kemandirian sekolah dan partisipasi masyarakat dalam mendukung program-program yang disusun oleh sekolah. Data yang kelihatan jelas dalam pengamatan bersama, kalau sekolah bisa melakukan upaya menstimulasi atau memberi semacam umpan agar masyarakat tergerak bahkan terpancing untuk selanjutnya secara aktif berpartisipasi, bantuan yang diperoleh berupa dana, tenaga, atau sesuatu yang lain akan menjadi berlipat ganda jumlahnya. Masalah berikut yang dibahas dalam kutipan diatas adalah .... (A) persentase keberhasilan pelaksanaan MPMBS dari tahun ke tahun (B) upaya-upaya sekolah dalam meningkatkan mutu melalui MPMBS (C) berbagai usaha yang dilakukan sekolah dalam melaksanakan MPMBS (D) dukungan MPMBS dalam meningkatkan kualitas pembelajaran (E) cara sekolah menstimulasi masyarakat dalam menjalankan MPMBS

30

20. Tren masyarakat dunia saat ini adalah kembali ke alam. Tren itu dapat dijadikan motivasi bagi petani untuk kembali mengelola pertanian yang berbasis faktor alam. harga pupuk yang semakin melambung dapat diganti dengan pupuk kompos atau pupuk kotoran hewan ternak. Pencegahan hama dapat dilakukan melalui rantai makanan sehingga organisme tertentu menjadi pengurai organisme lain. Apabila semua itu dapat dilakukan, niscaya petani tidak lagi direpotkan dengan mahalnya pupuk yang terkadang malah langka dan oleh hama tanaman yang semakin resisten terhadap insektisida. Dengan kembali ke alam, ketergantungan terhadap penunjang pertanian, seperti pupuk selama ini dapat diatasi melalui bahan yang ada di sekitar petani dan bersifat alami. Pernyataan yang sesuai dengan isi paragraf di atas adalah .... (A) Pertanian saat ini tidak lagi menggunakan pupuk kimia buatan pabrik (B) Sampai saat ini, ketergantungan patani pada penunjang pertanian semakin tinggi. (C) Walaupun tanpa obat buatan pabrik, petani dapat mencegah penyakit yang menyerang tanaman (D) Pupuk kompos dapat menunjang produktivitas pertanian lebih tinggi daripada pupuk buatan pabrik. (E) Insektisida membahayakan manusia 21. .... kakak .... adik tidak mau mengalah dalam perselisihan tersebut sehingga ayah dan ibu turut menyelesaikannya. Konjungsi yang paling tepat untuk melengkapi kalimat di atas adalah .... (A) karena, dan (B) akibat, dan (C) baik, maupun (D) tidak, tetapi (E) bukan, melainkan

Xpedia Bahasa Indonesia , Kapita Selekta - Set 01

22. Pemakaian kata daripada berikut ini salah, KECUALI .... (A) Saya mengharapkan bantuan daripada pemerintah. (B) Penggunaan daripada bahasa indonesia harus baik dan benar. (C) Meja ini terbuat daripada pualam. (D) California lebih indah daripada Den Hag. (E) Semua siswa berterima kasih atas perhatian daripada guru-guru mereka. 23. Dari tingkah laku dan ucapannya itulah yang menyebabkan tawuran antara kedua anak itu. Makna yang dinyatakan oleh kata dari adalah .... (A) asal (B) karena (C) sejak (D) tentang (E) bahan 24. Pak Kades mendududki posisi terhormat di desanya. Makna kofiks me-i dalam pernyataan tersebut sama artinya dalam pernyataan .... (A) Mamat mencambuki kerbaunya. (B) Nelayan itu mengarungi samudra dengan perahu layar. (C) Perusahaan itu menanami kebunnya dengan kelapa sawit (D) Tito menugasi adiknya untuk mengambil air. (E) Nenek merestui kepergian kakekku 25. Data yang ada menunjukkan lebih kurang 300 ribu bayi dan separuhnya meninggal dunia pada bulan pertama kehidupannya. Maksud kalimat tersebut adalah .... (A) Separuh jumlah tersebut meninggal pada bulan pertama (B) Setiap bulan ada bayi meninggal sejumlah 300 ribu (C) Sejumlah 150 ribu bayi meninggal pada bulan pertama kehidupannya. (D) Setiap bulan pertama kehidupannya ada bayi meninggal. (E) Setiap bulan ada bayi meninggal sejumlah 150 ribu. 31

Xpedia Bahasa Indonesia Kapita Selekta – Set 02

01. Beberapa analisis asing maupun domestik di pasar modal berpendapat, kesimpangsiuran susunan personalia kabinet merupakan faktor utama yang menjadi penyebab indeks harga saham gabungan terpuruk. Sekalipun demikian, sentimen negatif bursa regional yang berguguran turut memperparah keadaan pasar saham. Akibatnya, harga saham gabunganpun tidak mampu mengatasi keterpurukannya. Dalam paragraf di atas, terdapat beberapa kesalahan ejaan dan pemilihan kata. Paragraf di atas akan menjadi paragraf yang baik jika direvisi dengan cara-cara berikut, KECUALI .... (A) Gabunganpun (kalimat 3) ditulis gabungan pun (B) Maupun (kalimat 1) diganti dan (C) Tanda koma (,) (kalimat 1) dihilangkan dan diganti bahwa (D) Sekalipun demikian (kalimat 2) diganti selain itu (E) Analisis (kalimat 1) diganti dengan penganalis 02. Penulisan kalimat berikut ini mengikuti aturan EYD, KECUALI .... (A) Buku ini disusun untuk membantu mahasiswa yang akan mengambil mata kuliah tentang ekonomi Pembangunan. (B) Kami menulis diktat ini dengan maksud, agar mahasiswa lebih mudah mengikuti kuliah tentang ekonomi Pembangunan. (C) Akuntansi terbagi dalam beberapa bidang sebagai berikut: akuntansi sosial, akuntansi pemerintahan, dan akuntansi organisasi nonprofit (D) Diharapkan diktat yang berjudul Akuntansi sosial ini bermanfaat bagi para peminat akuntansi pada umumnya dan mahasiswa sekolah tinggi ilmu ekonomi pada khususnya. (E) Penjelasan dengan asetilena dilakukan dengan cara membakar bahan bakar gas asetilena dengan O2

32

03. (a) Kondisi ekonomi Indonesia saat ini cukup baik. (b) Hal ini dapat dilihat dari berbagai usaha, baik jasa maupun barang yang berkembang pesat. (c) Salah satunya adalah usaha minimarket yang merupakan bisnis usaha pelayanan. (d) Untuk usaha ini didirikan di daerah yang jauh dari keramaian kota. (e) Fasilitas ini banyak membantu masyarakat dalam mencukupi kebutuhan seharihari, seperti sembako dan peralatan rumah tangga, sehingga proses distribusi barang dari produsen ke konsumen semakin mudah. Dalam alinea tersebut terdapat kalimat yang strukturnya salah. Kalimat yang dimaksud adalah .... (A) kalimat (a) (B) kalimat (b) (C) kalimat (c) (D) kalimat (d) (E) kalimat (e) 04. Kalimat berikut yang seluruhnya ditulis dengan menggunakan ejaan yang benar adalah .... (A) Ketua Jurusan Teknik Informatika STMIK AMIK Bandung, Solikin, M.T., mengungkapkan bahwa di Jawa Barat baru 36 persen yang memiliki sarana komunikasi LAN. (B) Dengan membuat sistem informasi dan komunikasi terpadu antarinstansi pemerintah, pemerintah dan dunia usaha, maupun pemerintah dengan masyarakat dapat dikurangi biaya komunikasi dan koordinasi. (C) Dengan beragamnya kepentingan yang dilahirkan oleh sifat multietnis dan multi-kultural bangsa ini, Indonesia membutuhkan sistem bikameral yang jangan terlalu lemah. (D) Untuk itu, pemerintah Provinsi Jawa Barat menerapkan pendeketaan akselerasi guna mencapai tingkat kesejahteraan yang optimal sesuai dengan potensi atau kemampuan daerah. (E) Pembanguan daerah yang merupakan upaya penggalian, pengondisian, dan pemberdayaan sumber-sumber lokal agar memiliki kualitas hidup perlu mendapat sentuhan yang strategis.

Xpedia Bahasa Indonesia, Kapita Selekta – Set 02

05. Kalimat, Melalui pemahaman perjuangan, dapat menimbulkan kecintaan dan keinginan untuk memberikan loyalitas kepada bangsa dan negara sebagai bentuk penghormatan atas generasi terdahulu merupakan kalimat tidak baku. Yang menyebabkan tidak baku adalah .... (A) Pemakaian kata sebagai (B) Pemakaian bentuk kata memberikan (C) Pemakaian kata kepada (D) Pemakaian kata atas (E) Pemakaian bentukan kata menimbulkan 06. 1. Kami mendengar berita itu 2. Berita itu disiarkan oleh berbagai televisi. 3. Isi berita itu NAD dan Sumatera Utara dilanda gempa bumi dan tsunami. Rangkaian yang tepat dari ketiga kalimat tersebut adalah .... (A) Gempa dan tsunami melanda NAD dan Sumatera. (B) Bahwa NAD dan Sumatera Utara dilanda gempa dan tsunami kami mendengar berita dari berita berbagai televisi. (C) Kami mendengar berita bahwa NAD dan Sumatera Utara dilanda gempa dan tsunami dari berita siaran televisi. (D) Berita yang disiarkan berbagai televisi isinya bahwa NAD dan Sumatera Utara dilanda gempa dan tsunami kami dengar. (E) Berita bahwa gempa dan tsunami melanda NAD dan Sumatera Utara dari siaran televisi.

07. Karena keluarga menjadi korban bencana gempa dan tsunami, maka mereka menjadi sangat menderita. Kalimat tersebut termasuk dalam kalimat tidak baku seperti kalimat-kalimat berikut, KECUALI .... (A) Walaupum sumbangan datang dari berbagai negara, tetapi rakyat Aceh masih tetap menderita. (B) Untuk keperluan biaya pembangunan Aceh memerlukan bantuan berbagai pihak. (C) Dalam rapat tersebut membahas tentang bagaimana membangun Aceh kembali. (D) Kami sangat berduka melihat anak-anak menjadi korban bencana itu, maka jika diizinkan kami ingin menjadi orang tua asuh. (E) Karena sudah banyak orang yang memberikan bantuan sehingga kini mereka tidak perlu gelisah dan merasa takut lagi. 08. Seorang pelukis yang kaya akan ekspresi seni mampu menangkap fenomena alam dan merealisasikannya dalam bentuk lukisan yang menarik. Kenyataan ini menunjukan bahwa goresan kuas pada kanvas merupakan refleksi batin seorang pelukis atas pemahamannya terhadap alam. Kata-kata yang bercetak tebal pada kutipan diatas dapat diganti dengan kata-kata .... (A) jiwa - gejala - menggambarkannya situasi (B) ungkapan - gejala - menggambarkannya cerminan (C) ungkapan - gejala - mewujudkannya cerminan (D) jiwa - suasana - mewujudkan - cerminan (E) jiwa - suasana - melukiskan - situasi

33

Xpedia Bahasa Indonesia, Kapita Selekta – Set 02

09. Setelah berkali-kali mengalami musibah banjir, akhirnya warga masyarakat di wilayah tersebut menggerakkan seluruh potensi yang ada untuk bersama-sama menanggulangi dan mencegah kejadian yang serupa. Di samping itu, mereka juga secara bersama-sama melakukan pemugaran tempat bersejarah yang ada di daerah itu. Dalam menjalankan pekerjaan itu, mereka menerima sejumlah bantuan uang dari pemerintah untuk membeli bahan-bahan yang diperlukan. Kata-kata berikut ini yang merupakan istilah sebagaimana diilustrasikan dalam kutipan di atas adalah .... (A) kooperatif, inovasi, subsidi (B) kinergis, rehabilitasi, subsidi (C) kooperatif, renovasi, subsidi (D) koordinasi, rehabilitasi, donasi (E) sinergis, renovasi, subsidi 10. Apabila kita berperan sebagai moderator yang akan membuka diskusi tentang narkoba, kalimat pembuka yang tepat untuk kita sampaikan adalah .... (A) Peserta diskusi saya harap tenang sejenak (B) Sampailah kita pada diskusi untuk membahas narkoba (C) Narkoba adalah obat terlarang yang membahayakan kehidupan generasi muda. (D) Peserta yang kami hormati, dalam diskusi ini kita akan membahas narkoba. (E) Diskusi akan segera kita mulai, penyaji makalah kami harapkan segera membacakan makalahnya 11. Tidur adalah kebutuan hidup yang tidak bisa diabaikam, .... banyak orang yang menyepelekan aktivitas ini, .... banyak orang memotong waktu tidurnya supaya bisa melakukan pekerjaan atau aktivitas lain dalam jumlah yang lebih banyak ...., hal tersebut justru akan berpengaruh negatif pada tubuh. Kata penghubung yang tepat untuk mengisi titik-titik tersebut adalah .... (A) tetapi, sehingga, sedangkan (B) tapi, hingga, sedang (C) namun, oleh karena itu, padahal (D) tetapi, sampai, meskipun (E) sebaliknya, karena, walaupun 34

12. Melihat fenomena yang terjadi saat ini, maka disinilah peran Bank Perkreditan Rakyat, di mana Bank Perkreditan Rakyat dirancang untuk melayani kebutuhan kredit dan permodalan masyarakat kelas menengah ke bawah. Rangkaian kata tersebut akan menjadi kalimat ragam baku jika diubah menjadi .... (A) Melihat fenomena yang terjadi saat ini, maka disinilah peran Bank Perkreditan Rakyat, Bank Perkreditan Rakyat dirancang untuk melayani kebutuhan kredit dan permodalan masyarakat kelas menengah ke bawah. (B) Setelah melihat fenomena yang terjadi saat ini, maka di sinilah peran Bank Perkresitan rakyat, di mana Bank Perkreditan Rakyat melayani kebutuan kredit dan permodalan kelas menengah ke bawah. (C) Setelah melihat fenomena yang terjadi saat ini, maka di sinilah peran Bank Perkreditan Rakyat, yang dirancang untuk melayani kebutuhan kredit dan permodalan masyarakat kelas menengah ke bawah. (D) Melihat fenomena yang terjadi saat ini, maka di sinilah peran Bank Perkreditan Rakyat, sebagai bank yang dirancang untuk melayani kebutuhan kredit dan permodalan masyarakat kelas menengah ke bawah. (E) Setelah melihat fenomena yang terjadi saat ini, maka di sinilah peran Bamk Perkreditan Rakyat, sebagai bank yang dirancang untuk melayani kebutuhan kredit dan permodalan masyarakat kelas menengah ke atas.

Xpedia Bahasa Indonesia, Kapita Selekta – Set 02 doc. Name: XPIND9902

version : 2012-08 |

13. Daya tarik Yogyakarta sebagai kota pendidikan tidak terbatas karena kamajuan pendidikan sekolah dan pendidikan tingginya. Akan tetapi, juga karena kehidupan sosial dan budaya masyarakatnya yang mengandung pengetahuan, etika, nilai-nilai, dan perilaku yang dapat dijadikan pelajaran bagi siapa pun yang datang. Hal ini justru memberikan konsep yang luas mengenai pendidikan, bahwa pendidikan tidak hanya dapat terjadi secara sistematis disekolah, tetapi juga dalam kehidupan sosial. Antara kedua jenis pendidikan itu tidak dapat dipisahkan dan saling melengkapi, menambah, dan memperkuat bagi pengembangan potensi individu, anak-anak, pemuda, dan orang dewasa. Belajar dari kehidupan sosial tidak dibatasi oleh batas usia, ijazah sekolah, tetapi bersifat terbuka bagi siapa pun untuk mengikuti sesuai dengan kesadaran, minat, dan kebutuhannya. Dalam paragraf tersebut, terdapat kalimat yang belum merupakan kalimat, yaitu .... (A) kalimat pertama (B) kalimat kedua (C) kalimat ketiga (D) kalimat keempat (E) kalimat kelima 14. Secara historis, dapat diketahui bahwa kajian ilmu bahasa atau linguistik dari waktu ke waktu selalu mengalami perkembangan. Perkembangan kajian tersebut disebabkan oleh ketidakpuasan dalam menyusun deskripsi bahasa. Di samping itu, perkembangan ilmu bahasa ini disebabkan juga oleh adanya perbedaan paradigma yang digunakan oleh para analis bahasa dalam mengkaji bahasa. Pernyataan berikut yang secara implisit merupakan simpulan paragraf di atas adalah .... (A) Pengajaran bahasa, termasuk Bahasa Indonesia, selalu mengalami perkembangan dari waktu ke waktu. (B) Setiap ahli bahasa memiliki paradigma yang berbeda-beda dalam mengkaji bahasa. (C) Hasil kajian bahasa selalu tidak memuaskan para pemakai bahasa sehingga terus menerus dilakukan penelitian. (D) Sampai saat ini, belum ada deskripsi bahasa yang lengkap dan utuh yang didasarkan pada hasil kajian bahasa. (E) Pengerjaan bahasa tidak mengalami perkembangan jika tidak ada kajian bahasa.

halaman 4

15. Krisis air bersih dan rentannya udara segar menjadi isu global yang mengancam kota metropolitan. Di tengah perkembangan kota yang kian pesat dari segi pertambahan penduduk dan industri, air sebagai sarana vital kehidupan harus terus selalu diawasi kualitas kebersihan dan kemurniannya. BIla pengawasan ini terabaikan, risiko terbesar akan mengancam kelangsungan hidup manusia. Paragraf di atas akan menjadi paragraf yang utuh jika dilengkapi dengan kalimat penjelasan .... (A) Karena itu, peran pemerintah kota dalam mengendalikan pembuangan limbah sesungguhnya sangat diharapkan. (B) Pada saat ini, pencemaran air di kota metropolitan sudah berada pada ambang batas yang sangat mengkhawatirkan. (C) Demikian pula dengan kebersihan udara di lingkungan kota, perlu diupayakan pengendalian secara serius agar polusi dapat diminimalkan. (D) Karena itu, polutan dari asap kendaraan bermotor dan asap pabrik yang menjadikan lingkungan terpolusi harus diantisipasi. (E) Untuk itu, peran semua pihak diperlukan dalam menjaga agar masalah air bersih dan udara segar dapat diatasi. 16. Perang tarif yang terjadi antaroperator seluler menyebabkan rendahnya loyalitas pelanggan terhadap kartu seluler tertentu. Dengan adanya perang tarif, pelanggan cenderung berganti-ganti kartu hingga masa berlaku kartu telah habis. Hal ini mengakibatkan tingginya tingkat kartu hangus. Berikut ini gambaran rendahnya loyalitas pelanggan kartu selular, KECUALI .... (A) Pelanggan membiarkan kartu selulernya hangus. (B) Pelanggan selalu membeli kartu seluler yang baru. (C) Pelanggan membiarkan kartu selulernya habis masa berlakunya. (D) Pelanggan tidak melakukan isi ulang pulsa kartu selulernya. (E) Pelanggan memilih kartu seluler yang murah harganya.

35

Xpedia Bahasa Indonesia, Kapita Selekta – Set 02

17. LAPORAN HASIL PENGAMATAN Pendahuluan Data kependudukan yang dapat kami peroleh dari Kelurahan Tebet Timur adalah sbb: jumlah penduduk 10.017 jiwa, terdiri atas 4.719 pria dan 5.998 wanita. Dari jumlah tersebut, tercatat penduduk yang berusaha di atas 17 tahun, 7.918 jiwa yang terbesar di 5 RW dan 47 RT. Kalimat yang lengkap untuk melengkapi laporan di atas adalah ... (A) Dari data di atas, dapat disimpulkan bahwa jumlah wanita lebih banyak daripada pria di kelurahan tersebut. (B) Hal yang menarik perhatian kami untuk mengadakan penelitian karena perbandingan penduduk dewasa dan penduduk usia sekolah cukup mencolok. (C) Ini menandakan bahwa program KB di kelurahan tersebut berhasil dengan baik. (D) Dari perbandingan di atas, jelaslah bahwa penduduk kelurahan tersebut telah mengikuti ajakan pemerintah. (E) Hal ini menandakan bahwa perkawinan di atas usia 20 tahun akan menghasilkan keluarga sejahtera.

19. Penentuan warna untuk terapi membutuhkan kehati-hatian. Kesalahan dalam penentuan warna justru bisa menimbulkan keseimbangan energi dalam tubuh. Sebagai contoh, warna merah tidak boleh diterapkan pada penderita tekanan darah tinggi dan jantung karena sifat merah yang penuh energi dan vitalitas akan kenaikan darah. Untuk penderita tekanan darah dan jantung, lebih cocok digunakan warna hijau karena sifatnya menyimbangkan emosi, menciptakan suasana tentram dan tenang. Selain bermanfaat untuk kesehatan, terapi warna dapat digunakan untuk dunia mode, kecantikan, dan desain interior. Informasi inti dalam paragraf tersebut di atas adalah .... (A) kehati-hatian dalam menentukan warna untuk terapi. (B) manfaat lain terapi warna (C) contoh penyakit yang memanfaatkan warna untuk terapi. (D) akibat kesalahan penentuan warna untuk terapi. (E) terapi warna yang tepat untuk penyakit darah tinggi dan jantung.

20. Salah satu langkah yang paling maju dari paket deregulasi 1996 adalah dicantumkannya 18. Produksi beras, impor, dan pengadaan secara jelas tahapan penurunan tarif bea beras masuk hingga tahun 2000 dan 2003 untuk Tahun Produksi Produksi Impor Pengadaan masing-masing kelompok tarif. Padi (juta Beras Beras Beras Informasi yang dinyatakan dalam kalimat ton GKG) (juta ton) (juta ton) (juta ton) tersebut adalah .... 1996 51,10 33,22 2,14 35,36 (A) Penurunan tarif bea masuk dilakukan 1999 50,87 33,06 4,50 37,56 pada tahun 2000 dan 2003. (B) Peluncuran paket 1996 merupakan lang2002 51,49 33,47 1,81 35,28 kah maju. 2004 54,34* 35,32* 0,17** 33,69* (C) Tarif bea masuk barang impor akan tetap dipertahankan. Sumber : BPS GKG (Gabah Kering Giling*) (D) Tahap-tahap paket deregulasi akan dilaRamalan III** Januari - September 2004 kukan secara jelas. Pernyataan yang tidak sesuai dengan isi tabel (E) Pada tahun 1996, pemerintah meluncurdi atas adalah ... kan paket deregulasi. (A) Semakin tinggi produksi padi, semakin rendah impor beras. (B) Impor beras tertinggi terjadi pada kondisi pengadaan beras tertinggi. (C) Tingginya produksi beras seiring dengan tingginya pengadaan beras. (D) Kondisi produksi beras paling tinggi justru pengadaan beras terendah. (E) Impor beras terendah terjadi ketika terjadi pengadaan beras terendah. 36

Xpedia Bahasa Indonesia, Kapita Selekta – Set 02

21. Kalimat yang efektif untuk membuka suatu seminar adalah .... (A) Waktu seminar telah tiba, dengan ini seminar kami buka. (B) Saudara-saudara sangat beruntung dapat menghadiri seminar ini, mari kita mulai. (C) Penyaji dan para peserta, baiklah selaku moderator, saya buka seminar tentang "Reformasi Total" dengan ucapan .... (D) Saudara-saudara, walaupun sudah terlambat, saya buka seminar tentang "Reformasi Total". (E) Ada seorang penyanyi terkenal yang telah hadir ditengah-tengah kita, maka saya buka seminar ini. 22. Kalimat berikut sangat tepat digunakan untuk menyanggah …. (A) Saya tidak setuju dengan pendapatmu! (B) Pendapatmu itu tidak masuk akal, siapa yang mampu untuk melaksanakannya? (C) Pendapatmu itu tidak baik, saya tidak setuju. (D) Saya kurang sependapat dengan Saudara karena memerlukan biaya yang banyak. (E) Saya tidak sepandapat dengan Saudara karena tidak baik untuk dilaksanakan.

24. Pernyataan-pernyataan berikut mengungkapkan informasi faktual, KECUALI .... (A) Kemarin kakak pergi ke Surabaya untuk mengambil buku-buku yang masih tertunda di tempat kost-nya. (B) Hari ini, Farida berbelanja ke pasar untuk keperluan pesta ulang tahun adik yang kesepuluh. (C) Paman, saya disuruh oleh ayah untuk mengambilkan surat ini. (D) Semua siswa kelas tiga berkumpul di aula. (E) Rasanya, cerita "Titanic" begitu mengharukan. 25. 1. Adiknya memang nakal. 2. Ia sudah mengetahui hal itu. Gabungkan kata yang tepat untuk kedua pernyataan tersebut adalah .... (A) Adiknya memang nakal dan ia sudah mengetahuinya. (B) Sudah ia ketahui adiknya memang nakal. (C) Ia sudah mengetahui bahwa adiknya memang nakal. (D) Adiknya memang nakal, ia sudah tahu. (E) Adiknya memang nakal, tetapi ia sudah mengetahuinya.

23. Kalimat yang menggunakan konjungsi korelatif adalah .... (A) Baik Aki maupun isterinya, sangat suka memberi sedekah kepada orang lain. (B) Antonius tidak mau membayar utangnya, padahal dia mempunyai utang. (C) Dicky tahu bahwa masyarakat sekitarnya sudah mengetahui kelicikannya. (D) Sebenarnya dia pandai, tetapi malas belajar. (E) Barang siapa menyimpan atau memiliki uang kertas yang palsu akan dituntut di muka hakim.

37

Xpedia Bahasa Indonesia Kapita Selekta – Set 03

01. Undangan Direktur Utama P.T. Mutiara Sakti DR. Samual Hasan, SH mengejutkan para pemilik saham karena sebelumnya tidak ada rencana pertemuan dalam waktu dekat ini. Kalimat di atas termasuk yang tidak baku karena terdapat kesalahan penulisan ejaan. Agar kalimat tersebut menjadi baku, berikut ini beberapa saran perbaikan, KECUALI .... (A) P.T. seharusnya tanpa titik (B) DR. Samuel Hasan, SH seharusnya ditulis Dr. Samuel Hasan, S.H. (C) Kata Sakti diikuti tanda koma(,) (D) Kata Direktur Utama seharusnya ditulis dengan huruf kecil semua (E) Kata mengejutkan didahului tanda koma (,) 02.

Terdapat penulisan kata yang salah dalam kalimat-kalimat berikut, KECUALI .... (A) Seseorang dipastikan menderita suatu penyakit tertentu setelah melalui hasil analisa pemeriksaan laboratorium. (B) Setiap perusahaan akan selalu berusaha untuk meningkatkan produktivitas para pegawainya agar perusahaan itu dapat menigkatkan layanan kepada konsumen sacara maksimum. (C) Setelah menyelesaikan program strata satu Universitas Gajah Mada, Guntur Saputra pergi ke Tokyo untuk mengambil program pasca sarjana di kota itu. (D) Fotokopi ijazah yang telah dilegalisir oleh kepala sekolah harus disertakan pada formulir pendaftaran untuk ke Universitas. (E) Hanya akibat dari persoalan yang sederhana, pertikaian - bahkan perkelahian - antar wilayah seringkali terjadi pada tahun-tahun terakhir ini.

38

03. Salah satu upaya peningkatan prestasi belajar dapat dilakukan melalui pengelolaan waktu belajar secara profesional. Untaian kata bercetak tebal dalam kalimat di atas terbentuk melalui proses .... (A) salah satu > salah satu upaya > salah satu upaya peningkatan > salah satu upaya peningkatan prestasi > salah satu upaya peningkatan prestasi belajar. (B) prestasi > prestasi belajar > peningkatan prestasi belajar > upaya peningkatan belajar > salah satu upaya peningkatan prestasi belajar. (C) belajar > prestasi belajar > peningkatan prestasi belajar > upaya peningkatan prestasi belajar > salah satu upaya peningkatan prestasi belajar. (D) upaya > upaya peningkatan > upaya peningkatan prestasi belajar > salah satu upaya peningkatan prestasi belajar. (E) peningkatan > upaya peningkatan > upaya peningkatan prestasi > upaya peningkatan prestasi belajar > salah satu upaya peningkatan prestasi belajar. 04. Negara Indonesia dengan wilayah yang begitu luas, mulai dari Sabang sampai Merauke, memiliki begitu banyak objek wisata yang dapat dijadikan modal untuk menarik wisatawan mancanegara maupun wisatawan Nusantara yang dapat menghasilkan devisa untuk membantu perekonomian Indonesia yang sedang mengalami krisis multidimensial. Kalimat panjang tersebut merupakan perluasan dari kalimat inti ... (A) Negara Indonesia memiliki wilayah yang begitu luas. (B) Negara Indonesia dapat dijadikan modal untuk menarik wisatawan. (C) Negara Indonesia memiliki banyak objek wisata. (D) Objek wisata di Indonesia dapat menarik wisatawan yang menghasilkan devisa. (E) Objek wisata di Indonesia dapat menghasilkan devisa.

Xpedia Bahasa Indonesia, Kapita Selekta – Set 03

05. Di antara kalimat-kalimat berikut ini terdapat kalimat yang tidak memiliki kesejajaran bentuk, yakni .... (A) Perjudian dalam segala bentuknya, kejahatan yang merajalela, kemaksiatan yang terbesar di kota-kota dan desadesa, serta penghambur-hamburan kekayaan untuk perbuatan yang mungkar harus kita bersihkan. (B) Program kerja yang telah disusun meliputi konsolidasi organisasi, penertiban administrasi dan keuangan, serta pembentukan kader mubalig. (C) Reorganisasi departemen-departemen, penghentian pemborosan dan penyelewengan, serta mobilisasi potensi-potensi nasional merupakan masalah-masalah yang meminta perhatian pemerintah kita. (D) Tahap terakhir penyelesaian gedung itu adalah pengecatan tembok, pemasangan penerangan, pengujian sistem pembagian air, dan pengaturan tata ruangannya. (E) Koordinasi pelaksanaan tugas satgas kecamatan, merencanakan langkahlangkah yang yang harus dilaksanakan oleh satgas kecamatan sesuai dengan ketentuan yang berlaku, serta supervisi dan evaluasi merupakan tugas yang harus dilaksanakan oleh satgas kabupaten.

06. 1. Manajer baru itu lulusan dalam negeri. 2. Manajer baru itu sopan dan ramah. 3. Menajer baru itu berhasil meningkatkan produksi perusahaannya. Rangkaian yang tepat dari ketiga kalimat tersebut adalah ... (A) Manajer baru lulusan dalam negeri yang sopan dan ramah itu berhasil meningkatkan produksi perusahaannya. (B) Manajer baru yang lulus dalam negeri itu sopan dan ramah berhasil meningkatkan produksi perusahaannya. (C) Manajer baru itu berhasil meningkatkan produksi perusahaannya yang sopan ramah itu lulusan dalam negeri. (D) Manajer baru itu berhasil meningkatkan produksi perusahaannya karena sopan, ramah, dan lulusan dalam negeri. (E) Walaupun tamatan dalam negeri, manajer baru itu berhasil meningkatkan produksi perusahaannya karena sopan dan ramah. 07. 1. Penelitian dilakukan di sejumlah negara Eropa dalam beberapa tahun ini. 2. Jenis musik klasik dapat merangsang fungsi otak manusia. 3. Penelitian ini menunjukkan “sesuatu” (tentang jenis musik klasik). Rangkaian kata yang tepat dari ketiga pernyataan di atas adalah .... (A) Di beberapa negara Eropa yang dilakukan penelitian beberapa tahun ini dapat menunjukkan musik klasik dapat merangsang fungsi otak manusia. (B) Penelitian yang dilakukan di beberapa negara Eropa beberapa tahun ini menunjukkan bahwa jenis musik klasik dapat merangsang fungsi otak manusia. (C) Penelitian jenis musik dilakukan di beberapa negara Eropa beberapa tahun ini yang menunjukkan dapat merangsang fungsi otak manusia. (D) Di beberapa negara Eropa menunjukkan bahwa penelitian jenis musik klasik dalam beberapa tahun ini yang dilakukan dapat merangsang fungsi otak manusia. (E) Musik klasik yang dilakukan penelitian di beberapa negara Eropa menunjukkan bahwa beberapa tahun ini dapat merangsang fungsi otak manusia. 39

Xpedia Bahasa Indonesia, Kapita Selekta – Set 03

08. Hari ini, karyawan yang terlambat masuk kerja dimaafkan oleh pemimpin karena lalu lintas kota macet. Kalimat di atas berasal dari kalimat inti .... (A) Karyawan dimaafkan. (B) Karyawan masuk. (C) Karyawan kerja. (D) Karyawan terlambat. (E) Lalu lintas macet. 09. Semua penduduk kecamatan itu dikerahkan untuk melakukan penghijauan bukit-bukit yang gersang .... pada musim hujan bahaya banjir dapat dicegah .... tidak mudah untuk menyadarkan warga masyarakat untuk ikut dalam kegiatan penghijauan itu .... pemerintah harus mencari upaya yang tepat untuk menggaet massa .... peduli dengan kepentingan umum. Kata penghubung yang tepat untuk melengkapi teks di atas adalah .... (A) Sehingga, tetapi, karena itu, agar (B) Agar, tetapi, untuk itu, sehingga (C) Agar, walaupun demikian, karena itu, sehingga (D) Agar, namun untuk itu, agar (E) Sehingga, namun, karena itu, agar 10. Tindakan anarkis itu sebagai suatu manifestasi kemarahan hatinya. Padanan kata manifestasi yang tidak tepat adalah .... (A) Pengejawantahan (B) Perwujudan (C) Pengungkapan (D) Perlakuan (E) Pernyataan

40

11. Sentimen negatif di antara para pelaku bursa turut mewarnai kondisi pasar modal regional. Sentimen negatif terlihat sejak perdagangan dimulai. Sikap yang tidak menguntungkan pelaku pasar ini sangat memperburuk kondisi perdagangan di pasar modal tersebut. Hal ini berakibat pada turunnya indeks harga saham gabungan, bahkan sampai level penutupan pun tidak bisa mencapai level normal. Paragraf di atas akan menjadi paragraf yang baik jika direvisi dengan cara sebagai berikut .... (A) di antara (kalimat 1) diganti oleh (B) terlihat (kalimat 2) diganti tampak (C) sangat (kalimat 3 ) dihilangkan (D) bahkan (kalimat 4) diganti sehingga (E) penutupan pun (kalimat 4) ditulis penutupanpun 12. Kalimat berikut hanya tepat digunakan dalam ragam surat .... (A) Berdasarkan daftar harga terakhir, pada kesempatan ini, barang-barang berikut dapat dikirim dengan segera. Hal itu berdasarkan permintaan. (B) Setelah harga barang ditetapkan berdasarkan daftar harga terakhir, pada kesempatan ini barang tersebut dikirim dengan segera sesuai dengan permintaan. (C) Atas permintaan, setelah harga barang ditentukan berdasarkan daftar terakhir, pada kasempatan ini barang-barang berikut dikirimkan dengan segera. (D) Barang berikut akan dikirim dengan segera berdasarkan permintaan. Adapun harga barang ditetapkan berdasarkan daftar harga barang terakhir. (E) Berdasarkan daftar harga terakhir dari Saudara, dalam kesempatan kali ini kami harap dapat dikirimkan dengan segera barang-barang berikut.

Xpedia Bahasa Indonesia, Kapita Selekta – Set 03

13. Bab ke-3 UU No. 4 tahun 1974 mengatur kesejahteraan anak secara rinci. Pada bab selanjutanya, diatur hak-hak anak yang harus dipenuhi. Dalam bab IV ini, juga diatur cara pemenuhan hak anak. Semua bab tersebut secara umum mengatur perlindungan anak dan hak asasi anak. Telah banyak perundang-undangan yang ditetapkan pemerintah untuk melindungi hak anak Indonesia. Penulisan yang kurang tepat terdapat pada kalimat ... (A) Kalimat 1 (B) Kalimat 2 (C) Kalimat 3 (D) Kalimat 4 (E) Kalimat 5 14. Selama lima tahun menjadi guru, Nurdin berkalikali mengamati secara cermat nilai-nilai yang diperoleh siswanya yang mengaitkannya dengan kondisi ekonomi keluarga siswa yang bersangkutan. Pada akhirnya, Nurdin berkesimpulan bahwa anak dari keluarga yang tidak mampu unggul dalam bidang matematika, sedangkan anak dari keluarga mampu unggul dalam bidang bahasa. Simpulan yang dibuat Nurdin tersebut merupakan .... (A) Solusi (B) Prediksi (C) Generalisai (D) Interpretasi (E) Inferensi

15. Pemerintah memiliki peran penting dalam menyusun sebuah peraturan yang jelas dan menegakkan peraturan tersebut tanpa pandang bulu. Di sisi lain, pemerintah seharusnya menjadi fasilitator dalam perumusan masalah pendidikan dan sistem pendidikan. Pemerintah yang baik juga harus mampu mengakomodasi dan merespon permasalahan anggota masyarakat. Generalisasi yang tepat untuk melengkapi paragraf di atas adalah ... (A) Pemerintah pernah berperan menyusun dan menetapkan peraturan, menjadi fasilitator, dan mengakomodasi serta merespons permasalahan anggota masyarakat. (B) Pemerintah mempunyai peran strategis di dalam proses pendidikan warga masyarakat. (C) Pemerintah harus bersikap adil dan bijak dalam menangani permasalahan setiap anggota masyarakat. (D) Pemerintah memiliki tanggung jawab yang besar dalam menangani masalah pendidikan terutama permasalahan anggota masyarakat. (E) Pemerinath harus mengutamakan kepentingan anggota masyarakat. Secara umum tanpa membedakan kelompok atau golongan.

41

Xpedia Bahasa Indonesia, Kapita Selekta – Set 03

16. Naiknya harga bahan bakar minyak (BBM) sebagai akibat dari ditariknya subsidi BBM oleh pemerintah beberapa tahun yang lalu memberikan implikasi yang cukup berat bagi masyarakat kita. Beban masyarakat, terutama masyarakat miskin, semakin meningkat manakala kanaikan BBM ini juga dibarengi dengan harga kebutuhan pokok dan biaya penggunaan listrik. Belum lagi, biaya untuk pendidikan. Para orang tua harus berfikir ulang dan bekerja keras untuk memenuhi tuntutan anakanaknya sekolah. Simpulan yang dapat diambil dari kutipan di atas adalah .... (A) Kenaikan harga BBM berimplikasi pada peningkatan beban masyarakat. (B) Subsidi BBM dari pemerintah sangat membantu masyarakat miskin. (C) Penarikan subsidi BBM oleh pemerintah meningkatkan harga kebutuhan pokok. (D) Harga bahan bakar minyak selalu mengalami kenaikan dari tahun ke tahun. (E) Akibat kenaikan harga BBM masyarakat miskin tidak dapat menyekolahkan anaknya. 17. Kalimat yang mengandung kata berimbuhan peristilahan adalah .... (A) Bawahan harus melaksanakan instruksi atasannya. (B) Impor tekstil Indonesia ke AS dikenakan kuota. (C) Kita harus mengerjakan sesuatu secara efektif. (D) Manajemen suatu perusahaan sangat berpengaruh pada kemajuan perusahaan tersebut. (E) Khairil Anwar adalah sastrawan angkatan 45 yang ekspresionitis.

42

18. Usaha-usaha kecil dan menengah di Indonesia mampu melakukan persaingan yang cukup ketat di pasar Internasional. Pernyataan tersebut akan menjadi kalimat utama jika diperjelas dengan kalimat penjelas berikut, KECUALI ... (A) usaha-usaha tersebut masih berorientasi pada produksi. (B) Sementara, ekspansi pemasarannya masih ditandai oleh perusahaan pengekspor dan dibantu oleh pemerintah. (C) Jika transformasi kebijakan ekonomi lebih diarahkan untuk pengembangan ekonomi rakyat. (D) Kemampuan usaha-usaha kecil tersebut merupakan modal besar. (E) Dan dapat dijadikan basis orientasi kebijakan ekonomi pada masa mendatang. 19. 1. Lili mendalami ilmu komputer karena ia ingin menjadi programer. 2. Pada masa mendatang era globalisasi komputerisasi akan terjadi di segala bidang. 3. Tanpa keahlian tenaga kerja, Indonesia akan ketinggalan zaman dan terlindas tenaga kerja asing. 4. Setiap kantor, lembaga, dan instansi nantinya akan membutuhkan tenaga kerja. Pernyataan yang tidak mempunyai kaitan logis dalam paragraf tersebut adalah .... (A) kalimat 1 (B) kalimat 2 (C) kalimat 3 (D) kalimat 4 (E) kalimat 5

Xpedia Bahasa Indonesia, Kapita Selekta – Set 03

20. 1. Sampah dan air yang menggenang harus dihilangkan. 2. Kebersihan dapat dijaga, sampah dibakar atau dipendam dan air dialirkan. 3. Hal itu harus dilaksanakan oleh semua orang. 4. Bukankah keberhasilan itu pangkal kesehatan. 5. Oleh karena itu, kebersihan menjadi tangung jawab kita bersama agar kesehatan yang kita dambakan dapat terwujud. Pernyataan yang berupa simpulan kita temukan dalam .... (A) kalimat 1 (B) kalimat 2 (C) kalimat 3 (D) kalimat 4 (E) kalimat 5 21. Ciri arsitektur Bali, misalnya, banyak mendominasi bangunan komersial dan pemerintahan di pulau dewata itu. Berbeda halnya dengan daerah lain. Di Jawa, misalnya, bangunan khas daerah justru semakin tergusur oleh gaya arsitektur modern. Dalam paragraf tersebut terdapat konjungsi pertentangan .... (A) antarklausa (B) antarkalimat (C) antarwacana (D) antarparagraf (E) antarnalar 22. PLTN merupakan teknologi yang aman dan andal. Masalah utama yang harus diperhatikan oleh negara yang akan membangun ialah soal penelitian dan pengkajian dampak yang perlu dilakukan dengan saksama. Masalah utama yang terungkap dalam paragraf tersebut adalah .... (A) PLTN merupakan teknologi yang andal, namun demikian harus dilakukan pengkajian dampak dengan teliti. (B) PLTN satu-satunya teknologi yang andal. (C) Negara yang ingin membangun PLTN terlebih dahulu harus melakukan penelitian dengan ceramat. (D) Penelitian dan pengkajian dampak yang ditimbulkan oleh PLTN harus dilakukan penelitian. (E) PLTN merupakan teknologi andal maka dari itu perlu kealamiahan.

23. Tema karya tulis : Kirab Budaya sebagai Terapi Mental untuk Memulihkan kepercayaan masyarakat Yogyakarta. Latar belakang yang tepat untuk tema tersebut adalah .... (A) Kirab budaya adalah pawai dengan memberi persembahan ke Gunung Merapi dan Laut Selatan agar terhindar dari bencana yang lebih besar dan lebih mengerikan. (B) Kirab budaya merupakan acara yang dipercaya masyarakat dapat membebaskan mereka dari bencana gunung berapi dan gempa bumi serta tsunami di Indonesia. (C) Masyarakat Yogyakarta ingin memulihkan kepercayaan pemerintah kepada mereka dengan mengadakan kirab budaya sehingga wisatawan akan datang kembali. (D) Aktivitas gunung berapi dan terjadinya gempa bumi akhir-akhir ini telah mengganggu masyarakat Yogyakarta sehingga pemerintah perlu mengadakan kirab budaya untuk memohon keselamatan. (E) Pemerintah memandang perlunya mengadakan kirab budaya di Indonesia untuk memulihkan mental masyarakat yang terkena bencana alam di Indonesia. 24. Berbicara kepada wartawan usai meresmikan pameran kulit dan sepatu internasional di APHJ, Menteri Perindustrian mengatakan tata niaga baru perkulitan akan keluar sekitar Desember 2000. “Sekarang ini, masih dalam penggodokan oleh departement, instansi, mapun pihak yang terkait,” kata Menperin. Tema kutipan tersebut adalah .... (A) pengevaluasian kembali kebijaksanaan tata niaga kulit. (B) peningkatan nilai ekspor untuk masa mendatang. (C) tata niaga baru perkulitan akan keluar sekitar Desember 2000. (D) peresmian pameran kulit dan sepatu. (E) perkembangan perkulitan Indonesia.

43

Xpedia Bahasa Indonesia, Kapita Selekta – Set 03

25. Penggunaan kata ulang yang tepat terdapat dalam pernyataan .... (A) Ia menumpuk selusin kemeja-kemeja di atas meja kerjanya. (B) Peraturan-peraturan yang dikeluarkan oleh atasan wajib ditaati. (C) Para pemilik rumah-rumah mewah harus beradaptasi dengan lingkungan sekitar. (D) Kini, ia baru sadar akan perbuatanperbuatannya. (E) Banyak negara-negara anggota OPEC belum menentuka kuota.

44

Xpedia Bahasa Indonesia Kapita Selekta – Set 04

01. Para mahasiswa termasuk mahasiswa Program Pasca Sarjana merupakan civitas academia sehingga memiliki hak yang sama untuk menggunakan fasilitas perpustakaan. Kalimat di atas akan menjadi kalimat baku jika ejaannya diperbaiki sebagai berikut, KECUALI .... (A) setelah para mahasiswa diberi tanda koma (,) (B) penulisan Pasca Sarjana dirangkaikan (C) setelah kata Sarjana diikuti tanda koma(,) (D) kata Program Pasca Sarjana semuanya ditulis dengan huruf kecil. (E) kata sehingga didahului tanda koma (,) 02. Pada hari pertama, sebelum ujian dimulai, semua peserta tes latihan mengisi lembar jawaban ujian. Setiap perserta ujian mendapatkan satu set soal dan lembar jawaban untuk berlatih. Dalam pengisian lembar tersebut, mereka dipandu oleh pengawas ujian. Perlatihan ini dimaksudkan agar mereka dapat mengerjakan dengan lancar tanpa mengalami banyak kesulitan. Dalam kutipan di atas, terdapat bentukan kata yang salah, yakni .... (A) latihan - mendapatkan - mengerjakan (B) mendapatkan - berlatih - perhatian (C) latihan - berlatih - perlatihan (D) latihan - berlatih - perlatihan (E) berlatih - perlatihan - mengerjakan 03. Kata bercetak miring dalam kalimat berikut ini termasuk kata baku, KECUALI .... (A) Masalah konkret yang sedang kita hadapi sekarang ini adalah bencana alam. (B) Imunisasi haemopilus influenza digunakan untuk menguatkan daya tahan tubuh. (C) Semoga Tuhan Yang Maha Pengasih mengabulkan doa kami. (D) Varietes padi unggul tahan wereng sekarang jarang didapati (E) Jenis daya tahan tubuh yang berupa kolostrum banyak dijual bebas di apotek.

04. Kelas akselerasi adalah untuk memenuhi kebutuhan siswa yang memiliki potensi dan bakat akademis luar biasa. Istilah akselerasi dalam kalimat di atas berarti .... (A) tambahan (B) percepatan (C) unggulan (D) peningkatan (E) khusus 05. Kalimat berikut yang termasuk kalimat bernalar adalah .... (A) Di daerah pendalaman banyak membutuhkan tenaga guru SLTP dan SLTA. (B) Gembong pembrontak itu berhasil diringkus oleh aparat keamanan. (C) Masalah itu minta diselesaikan sampai tuntas agar tidak menjadi beban. (D) Jika melalui gang ini, naik kendaraan bermotor diharp turun. (E) Yang merasa kehilangan kartu ujian, dapat mengambilnya di sekretariat. 06. Kalimat berikut yang baku adalah .... (A) Kebijakan uang ketat perlu ditinjau kembali yang sudah berjalan hampir dua tahun. (B) Kita memerlukan pemikiran-pemikiran untuk memecahkan masalah-masalah yang berkaitan dengan pelaksanaan pengembangan kota. (C) Fluktuasi harga minyak bumi di pasaran internasional memperngaruhi negara pemasukan devisanya yang masih mengandalkan minyak sebagai sumber utama. (D) Masalah ketenagaan (guru) yang mengajar di SD, bukan lagi lulusan SPG, melainkan lulusan Diploma II, yakni lulusan program D II PGSD. (E) Nota pembayaran yang dipersiapkan untuk dilaporkan dalam rapat pimpinan perusahaan, karena untuk pertanggungjawaban. 45

Xpedia Bahasa Indonesia, Kapita Selekta – Set 04

07. Kalimat berikut yang termasuk kalimat baku adalah .... (A) Ia menceritakan tentang peristiwa kepada teman-temannya. (B) Surat itu memberitakan mengenai keadaan neneknya yang sedang sakit. (C) Kita harus saling percaya akan kemampuan oarang lain, tidak boleh saling curiga. (D) Kita harus dapat melawan akan hawa nafsu yang dapat menjerumuskan ke lembah kehinaan. (E) Demi untuk mempertahankan semangat korps, kita tidak boleh saling menghujat. 08. Kalimat berikut ini yang termasuk kalimat baku adalah .... (A) Menurut Yeo bahwa birokrat di Indonesia terlalu rumit sehingga menimbulkan banyak masalah bagi investor yang mau bergabung. (B) Pihak biro perjalanan mengeluhkan tingginya kenaikan harga BBM berpengaruh besar terhadap minat wisatawan untuk melakukan kunjungan. (C) Berdasarkan hasil survei menunjukkan bahwa sampai saat ini wisata pantai masih tetap diminati, terutama oleh kaum muda. (D) Menurut pengamat lingkungan, krisis air bersih dan udara segar mulai merisaukan penduduk di kota-kota besar. (E) Dengan adanya alam yang menelan korban ratusan ribu nyawa manusia mengunggah nurani dan sikap kedermawanan semua orang.

46

09. Bagi sebagian besar orang, minum kopi memang sudah semacam keharusan atau aktivitas yang rutin tiap hari, khususnya pagi hari. Minum kopi diyakini bisa memnamgkitkan semangat seseorang selain rasanya yang memang memberikan kenikmatan tersendiri. Kedua kalimat tersebut dapat digabung menjadi satu kalimat. Gabungan yang tepat adalah .... (A) Bagi sebagian besar orang, minum kopi memang sudah semacam keharusan atau aktivitas yang rutin tiap hari, khususnya pagi hari, mengingat minum kopi diyakini bisa membangkitkan semangat seseorang selain rasanya yang memang memberikan kenikmatan tersendiri. (B) Bagi sebagian besar orang, minum kopi memang sudah semacam keharusan atau aktivitas yang rutin tiap hari, khususnya pagi hari, sehingga minum kopi diyakini bisa membangkitkan semangat seseorang selain rasanya yang memang memberikan kenikmatan tersendiri. (C) Bagi sebagian besar orang, minum kopi memang sudah semacam keharusan atau aktivitas yang rutin tiap hari, khususnya pagi hari, apalagi minum kopi diyakini bisa membangkitkan semangat seseorang selain rasanya yang memang memberikan kenikmatan tersendiri. (D) Bagi sebagian besar orang, minum kopi memang sudah semacam keharusan atau aktivitas yang rutin tiap hari, khususnya pagi hari, sedangkan minum kopi diyakini bisa membangkitkan semangat seseorang selain rasanya yang memang memberikan kenikmatan tersendiri. (E) Bagi sebagian besar orang, minum kopi memang sudah semacam keharusan atau aktivitas yang rutin tiap hari, khususnya pagi hari, karena minum kopi diyakini bisa membangkitkan semangat seseorang selain rasanya yang memang memberikan kenikmatan tersendiri.

Xpedia Bahasa Indonesia, Kapita Selekta – Set 04

10. Pilihan kata yang tepat tedapat dalam kalimat .... (A) Sunaryo, yaitu seorang pensiunan guru yang memiliki dedikasi yang tinggi. (B) Untuk memperoleh kredit, ia harus memiliki jaminan, ialah rumah atau kebun. (C) Ayahnya tidak pergi ke Ambon, melainkan berada di desanya. (D) Dia yang datang kemarin bukan kakaknya, tetapi teman dekatnya. (E) Baik di Jakarta, di Yogyakarta, maupn di Malang, perilakunya selalu menimbulkan masalah. 11. Sejak lahirnya konsep pemikiran baru dalam ilmu kedokteran, yang dicetuskan oleh Profesor Linus Pauling, yakni tentang Ortomolecular medicine yang dasarnya adalah studi biologi molekuler sebagai sains dasar, penelitian medis diarahkan pada molekl-molekul yang sacara normal biologis fisiologis ada dalam tubuh manusia. Inti kalimat yang panjang itu adalah ... (A) Konsep pemikiran baru dicetuskan oleh Profesor Linus Pauling. (B) Ortomoleculer medicine adalah sains dasar (C) Ortomoleculer medicine dasarnya adalah studi boilogi (D) Penelitian medis diarahkan pada molekul (E) Biologi molekuler merupakan sains dasar. 12. Ilmu pengetahuan dan teknologi merupakan tulang punggung pembangunan ekonomi. Dengan berpegangan pada keyakinan itu, sejumlah negara industri baru secara ... mengalokasikan sejumlah besar dana untuk memajukan iptek di negaranya. Hasilnya, mereka berhasil menciptakan invensi dan inovasi secara ... untuk kemudian diterapkan di bidang industri. Kata yang tepat untuk melengkapi kutipan di atas .... (A) aktif, efisien (B) konsisten, signifikan (C) inovatif, efisien (D) aktif, kreatif (E) inovatif, kreatif

13. Segala hal yang dilakukan selama ini sebenarnya tidak konsisten. Kata konsisten dalam kalimat tersenut berarti .... (A) taat asas (B) ada gunanya (C) berbahaya (D) penting (E) tepat 14. Berbicara tentang pendidikan sebenarnya sama halnya dengan berbicara tentang kehidupan. Pendidikan merupakan proses yang dilakukan setiap individu menuju ke arah yang lebih baik sesuai dengan potensi kemanusiaan. Proses ini hanya berhenti ketika nyawa tidak ada dalam raga manusia. Pendidikan pada hakikatnya adalah proses memanusiakan manusia. Profesor Diryakara marumuskan pendidikan sebagai proses memanusiakan manusia muda, yakni suatu pengangkatan manusia muda ke taraf insani sehingga ia dapat menjalankan hidup sebagai manusia utuh dan membudayakan diri. Ikhtisar kutipan di atas adalah .... (A) Pendidikan adalah proses berkesinambungan dalam memanusiakan manusia manjadi manusia utuh dan berbudaya sesuai dengan potensi yang dimiliki. (B) Pendidikan sangat diperlukan setiap individu dalam kehidupan bermasyarakat, terutama dalam menjadikan masyarakat berbudaya. (C) Selama masih hidup, setiap individu memerlukan pendidikan dengan tujuan untuk memanusiakan manusia yang utuh dan menjadikan dirinya berbudaya. (D) Pendidikan dan kehidupan ini tidak dapat dipisahkan karena pendidikan diperlukan oleh setiap individu untuk mencapai taraf insani, yakni sebagai manusia utuh dan membudayakan diri. (E) Melalui pendidikan, manusia dapat mencapai taraf hidup yang lebih baik, yakni sebagai manusia yang utuh dan membudayakan diri.

47

Xpedia Bahasa Indonesia, Kapita Selekta – Set 04

15. Kalimat berikut ini yang termasuk kalimat baku adalah .... (A) Pendidikan adalah proses berkesinambungan dalam memanusiakan manusia manjadi manusia utuh dan berbudaya sesuai dengan potensi yang dimiliki. (B) Pendidikan sangat diperlukan setiap individu dalam kehidupan bermasyarakat, terutama dalam menjadikan masyarakat berbudaya. (C) Selama masih hidup, setiap individu memerlukan pendidikan dengan tujuan untuk memanusiakan manusia yang utuh dan menjadikan dirinya berbudaya. (D) Pendidikan dan kehidupan ini tidak dapat dipisahkan karena pendidikan diperlukan oleh setiap individu untuk mencapai taraf insani, yakni sebagai manusia utuh dan membudayakan diri. (E) Melalui pendidikan, manusia dapat mencapai taraf hidup yang lebih baik, yakni sebagai manusia yang utuh dan membudayakan diri.

18. Karena pentingnya peran kebiasaan membaca, sekolah harusnya mamiliki program yang sistemtis bisa menumbuhkan kebiasaan membaca. Meskipun keluarga berperan besar dalam menumbuhkan kebiasaan membaca, sekolah perlu secara terprogram menumbuhkan kebiasaan siswanya. Hal ini disebabkan oleh kenyataan bahwa banyak sekali waktu yang dihabiskan oleh siswa disekolah dan tidak semua siswa memiliki latar belakang keluarga yang memiliki budaya gemar membaca yang bisa menularkan kebiasaan tersebut. Inti paragraf di atas adalah .... (A) Sekolah perlu menumbuhkan kebiasaan membaca para siswa. (B) Sekolah dan keluarga perlu menumbuhkan kebiasaan membaca para siswa. (C) Keluarga perlu mendukung untuk menumbuhkan kebiasaan membaca para siswa. (D) Siswa perlu ditumbuhkan kebiasaan membacanya. (E) Keluarga bersama sekolah diharapkan menumbuhkan kebiasaan membaca.

16. Kalimat-kalimat berikut menyatakan pendapat, KECUALI .... (A) Bioteknologi sangat bermanfaat bagi kehidupan manusia. (B) Penelitian bidang bioteknologi memberikan harapan baik bagi manusia. (C) Rekayasa genetika dapat dilakukan di laboraturium. (D) Pertanian yang dilakukan dengan cara modern pasti memberikan hasil yang lebih baik daripada pertanian tradisional. (E) Indonesia berusaha mengembangkan ilmu pengetahuan dan teknologi.

19. Klorofil biasanya dianggap sebagai darah bagi tanaman. Keberadaannya dibentuk melalui proses diseluruh alam semesta. Klorofil dibentuk melalui proses fotosintesis di dalam tanaman yang disimpan pada bagian daun utamanya. Klorofil kaya akan sumber mineral alami, vitamin protein, elamen, dan mikronutrien. Semua zat tersebut penting untuk menjaga kesehatan, terutama untuk menyeimbangkan kandungan asam dan basa di dalam tubuh. Paragraf tersebut merupakan pengembangan dari pikiran pokok .... (A) klorofil sebagai darah bagi tanaman (B) keberadaan klorofil di alam semesta (C) manfaat klorofil (D) proses terbentuknya klorofil (E) sumber mineral yang terkandung dalam klorofil

17. Boim akan meneliti kemampuan siswa kelas II SMA dlam menulis karangan ilmiah. Dalam proposal penelitiannya, Boim mencantumkan pijakan dasar yang mengatakan bahwa menurut kurikulum yang berlaku, siswa kelas II SMA telah diajari menulis karangan ilmiah. Pernyataan Boim yang dinyatakan pijakan dasar penelitian disebut.... (A) Hipotesis (B) asumsi (C) prediksi (D) interpretasi (E) solusi 48

Xpedia Bahasa Indonesia, Kapita Selekta – Set 04

20. Sebagaimana telah ditetapkan dalam GBHN, bahwa pembangunan pendidikan dititikberatkan pada peninngkatan setiap jenjang dan jenis pendidikan. Kalimat di atas dapat diperbaiki dengan mengilangkan kata .... (A) sebagaimana (B) telah (C) dalam (D) bahwa (E) pada 21. Tidak diketahui persis awal mula manusia menggunakan warna untuk terapi penyembuhan. Terapi warna disebut-sebut sudah ada sejak zaman Mesir kuno. Dalam motologi Mesir, warna dipercaya mampu menyembuhkan bebagai macam penyakit. Bahkan pada masa itu didirikan bangunan untuk penyembuhan dengan menggunakan cahaya dan warna. Penelitian pada awal abad ke-19, menemukan bahwa tiap warna dalam sinar matahari mengalirkan frekuensi dan gelombang panjang yang berbeda. Inti dari penyembuhan itu adalah penyerapan energi sinar matahari melalui mata dan kulit. Hal-hal berikut dikemukakan dalam paragraf di atas, KECUALI .... (A) Warna dapat dimanfaatkan untuk terapi penyembuhan penyakit. (B) Terapi penyembuhan dengan menggunakan warna sudah berlangsung sejak zaman Mesir kuno. (C) Bangunan merupakan syarat untuk peenyembuhan dengan menggunakan cahaya dan warna. (D) Bukti bahwa warna dapat dipercaya mampu menyembuhkan penyakit ditemukan dalam mitologi Mesir kuno. (E) Pada masa Mesir kuno didirikan bangunan untuk penyembuhan dengan menggunakan cahaya dan warna.

22. Dengan dikembangkannya Manajemen Berbasis Sekolah, para pengelola dan praktisi pendidikan menjadi kunci keberhasilan pendidikan. Alasannya adalah sekarang kepala sekolah, wakil, dan deputideputinya harus siap menangani dan mengelola segudang permasalahan dan beraneka ragamnya sumber daya pendidikan seperti fasilitas, aset, keuangan, dan seluruh stakeholder pendidikan lainnya. Untuk itu, sebuah teknologi informasi yang dapat membantu mereka harus dihadirkan agar dapat membentuk mereka dalam mengambil keputusan sehari-hari. Ide pokok dalam paragraf di atas adalah .... (A) pengembangan Manajemen Berbasis Sekolah semi keberhasilan demi keberhasilan pendidikan. (B) peranan kepala sekolah, wakil dan deputi-deputinya dalam keberhasilan pendidikan. (C) peranan teknologi informasi sebagai alat pengambilan keputusan. (D) teknologi informasi sebagai fasilitas pendidikan. (E) keberhasilan pendidikan ditentukan oleh psokologi. 23. Hutan Indonesia kurang dapat dikelola dengan baik dan tidak dikelola dengan berkesinambungan. Di Finlandia, 62 persen hutan dimiliki kelompokkelompok keluarga, perusahaan kehutanan 9 persen, negara 25 persen, dan yang lain 5 persen. Di Indonesia seluruh kawasan hutan adalah milik negara. Negara mengasai hutan begitu besar, tetapi tidak mampu mengurus, akhirnya keadaan hutan makin rusak. Dalam catatan Persatuan Sarjana Kehutanan (persaki), kerusakan sudah mencapai 3,5 juta hektare per tahun. Urutan kalimat pada paragraf di atas merupakan .... (A) pendapat, fakta, fakta, pendapat, fakta (B) pendapat, fakta, pendapat, fakta, pandapat (C) pendapat, fakta, fakta, pendapat, pendapat (D) pendapat, pendapat fakta, pendapat, fakta (E) pendapat, pendapat, fakta, fakta, pendapat

49

Xpedia Bahasa Indonesia, Kapita Selekta – Set 04

24. Sebagai pemuda yang lahir di desa, Fika sering merenungkan sawah bapaknya yang tidak terlalu luas itu. Udara di desa lembab dan sawahnya memperoleh sinar matahari langsung. Tanah itu ermasuk subur dan tidak pernah kekurangan air walaupun musim kemarau. Dia merenungkan apa yang diperolehnya dari PPL yang bertugas didesanya. Dia banyak bertanya jawab dengan PPL tenteng bercocok tanam jagung manis. Dia yakin bahwa bapaknya cocok untuk bertanam jagung manis. Maka dari itu, dia berbicara dengan bapaknya dan mengemukakan niatnya itu. Bapaknya setuju jika sawahnya dipakai untuk bertanam jagung manis. Dari ilustrasi di atas, Fika memiliki sifat dan perilaku sebagai berikut, KECUALI .... (A) Fika adalah orang yang ulet bekerja dan selalu mencari alternatif yang lebih baik. (B) Fika adalah orang yang suka mencari ilmu dan selalu berusaha keras. (C) Fika senang mencobakan pengetahuannya dalam praktik. (D) Fika berusaha untuk mengembangkan ilmu bercocok tanam jagung manis. (E) Fika memiliki motivasi tinggi untuk meningkatkan produksi lahannya.

50

25. Pada tahun 1948, Indonesia telah berhasil mencapai swasembada beras. Hal ini berarti harga beras relatif terjangkau oleh masyarakat. Meningkatnya ketersediaan beras, pendapatan penduduk dan sistem nilai sosial pada beras telah mengarahkan pola konsumsi pangan pokok beras. Karena itu, di perlukan upaya-upaya penganekaragaman pangan pokok untuk mengurangi ketergantungan pada beras sekaligus mempertahankan swasembada beras, serta meningkatkan mutu gizi pangan penduduk. Pernyataan berikut yang sesuai dengan isi paragraf di atas adalah .... (A) Keberhasilan Indonesia dalam mencapai swasembada beras dapat meningkatkan mutu pangan. (B) Peningkatan kecenderungan penduduk beralih ke pola konsumsi pangan pokok beras perlu diimbangi dengan penganekaragaman pangan pokok. (C) Karena swasembada beras dapat meningkatkan mutu gizi keluarga, pemerintah perlu mengusahakan pola konsumsi pangan pokok yang lain. (D) Penganekaragaman pola konsumsi pangan pokok penduduk merupakan pekerjaan besar yang mendukung program pemerintah. (E) Keberhasilan swasembada beras ditunjang oleh pemerintah dengan upaya perbaikan menu makanan rakyat.

Xpedia Bahasa Indonesia Kapita Selekta – Set 05

Menjelang digulirkannya dana subsidi bahan bakar minyak (BBM) oleh pemerintah khususnya untuk sektor pendidikan, kini sejumlah daerah mulai melakukan publisitas tentang kondisi persoalan pendidikan di daerahnya masing-masing. Namun, disinyalir tak jarang dari data yang diekpos itu sengaja dibesar-besarkan agar mendapatkan dana yang besar pula. Dari kabupaten Purwakarta, dilaporkan sebanyak 2.000 anak usia 7 - 12 tahun hingga saat ini belum manikmati bangku sekolah. Hal itu terjadi akibat ketidakmampuan orang tua dalam membiayai sekolah mereka. Bila kondisi itu dibiarkan, maka dipastikan akan menurunkan tingkat angka partisipasi murni (APM) bagi anak seusia itu dalam memasuki pendidikan dasar (SD). Padahal, APM, SD di Kabupaten Purwakarta tahun 2004 lalu dinilai masih relatif baik, hingga mencapai 92,40%. 01. Pernyataan berikut ini yang tidak sesuai dengan teks di atas adalah .... (A) Sejumlah daerah melakukan publisitas kondisi persoalannya. (B) Agar mendapat dana besar, mereka mengekspos data secara besar-besaran. (C) Lebih dari 2.000 anak usia sekolah di Kabupaten Purwakarta belum menikmati bangku sekolah. (D) Orang tua anak sekolah di Kabupaten Purwakarta tidak mampu membiayai sekolah. (E) APM SD Kabupaten Purwakarta dinilai relatif baik, mencapai 92,40%.

Menjelang digulirkannya dana subsidi bahan bakar minyak (BBM) oleh pemerintah khususnya untuk sektor pendidikan, kini sejumlah daerah mulai melakukan publisitas tentang kondisi persoalan pendidikan di daerahnya masing-masing. Namun, disinyalir tak jarang dari data yang diekpos itu sengaja dibesar-besarkan agar mendapatkan dana yang besar pula. Dari kabupaten Purwakarta, dilaporkan sebanyak 2.000 anak usia 7 - 12 tahun hingga saat ini belum manikmati bangku sekolah. Hal itu terjadi akibat ketidakmampuan orang tua dalam membiayai sekolah mereka. Bila kondisi itu dibiarkan, maka dipastikan akan menurunkan tingkat angka partisipasi murni (APM) bagi anak seusia itu dalam memasuki pendidikan dasar (SD). Padahal, APM, SD di Kabupaten Purwakarta tahun 2004 lalu dinilai masih relatif baik, hingga mencapai 92,40%. 02. Gagasan paragraf dua dikembangkan dengan menggunakan pertanyaan .... (A) Mengapa orang tua anak sekolah tidak mampu membiayai mereka untuk sekolah? (B) Dari manakah kita mengetahui data anak sekolah di Kabupaten Purwakarta. (C) Mengapa anak usia 7 - 12 tahun tidak sekolah? (D) Berapa ribu orangkah anak usia 7 - 12 tahun di Kabupaten Purwakarta belum menikmati bangku sekolah. (E) Apakah tujuan pemerintah memberikan subsidi BBM terhadap anak usia 7 - 12 tahun di Kabupaten Purwakarta.

51

Xpedia Bahasa Indonesia, Kapita Selekta – Set 05

03. Atas dasar pemahaman tersebut, diharapkan dapat menggunakan kaidah-kaidah bahasa Inggris secara baik dan benar sehingga diperoleh keterampilan bahasa, khususnya kemahiran berbahasa inggris. Inti kalimat di atas adalah .... (A) Mereka diharapkan dapat menggunakan kaidah-kaidah bahasa Inggris. (B) Mereka harus memahami kaidah-kaidah bahasa Inggris. (C) Mereka dapat menggunakan kaidahkaidah bahasa Inggris. (D) Mereka terampil berbahasa Inggris. (E) Penguasaan kaidah bahasa Inggris berpengaruh terhadap kemahiran berbahasa Inggris. 04. Pemakaian tanda baca yang benar terdapat dalam kalimat-kalimat di bawah ini, KECUALI .... (A) Dari segi nalar, nalar ilmu atau logika, cara pengobatan es telapak kaki sulit dimengerti. (B) Camat Kota Subang Rumanda, S.H., M.Si. meminta Bupati Subang segera mengganti Lurah Sukamelang Dayat. (C) Lia warga Kamp. Adiarsa Pusaka, RT 01/RW 08, Kel. Adiarsa, Kec. Karawang Timur, tewas tertabrak mobil. (D) Dalam hal tertentu, antara lain oleh rasa takut, resah, cemas, dan atau demam, suhu tubuh manusia dapat berubah. (E) Risiko budi daya udang masih tinggi karena udang sering terkena virus penyakit whitespot „bintik putih‟.

05. Penulisan yang sesuai dengan EYD terdapat pada kalimat .... (A) Timnas Indonesia kembali gagal merebut piala Tiger setelah pada final kedua dikalahkan Singapura 1-2 di Stadion Nasional Singapura. (B) Bila terbukti bersalah, kemungkinan besar ia bakal dijatuhi penjara seumur hidup. (C) “Rangkaian acara Munkernas sendiri sebenarnya sudah dimulai sejak sepekan yang lalu.” Kata Ketua Panitia. (D) Ketika semua ini saya ceritakan kepada Mpu Peniti, ia hanya menggelengkan kepala sambil tersenyum kecut. (E) Penggalan dana dilakukan dengan cara penjualan kupon Rp 5.000,00 per lembar. 06. (1) Vitamin A murni berupa kristal kuning pucat, tahan terhadap panas dan sinar, dan mudah rusak karena oksidasi. (2) Vitamin A merupakan salah satu vitamin yang tidak larut dalam air. (3) Karenanya, vitamin A berkhasiat untuk menjaga keutuhan kulit serta meningkatkan kesehatan mata dan pertahanan tubuh terhadap infeksi kuman, virus, dan parasit. (4) Namun, vitamin ini larut dalam pelarut lemak. (5) Vitamin ini penting untuk keutuhan lapisan epitel uang menyeliputi tubuh, yaitu kulit dan jaringan mukosa yang menutupi mata dan rongga saluran pencernaan, saluran pernapasan, serta saluran kemih. Urutan kalimat yang tepat untuk paragraf di atas adalah .... (A) 1-3-2-5-4 (B) 2-4-5-1-3 (C) 2-5-4-1-3 (D) 5-1-4-2-3 (E) 2-4-1-5-3 07. Kata yang bermakna asosiasi terdapat dalam kalimat, KECUALI .... (A) Orang itu terkenal suka menerima suap. (B) Diterimanya kedatangan orang itu dengan tangan terbuka. (C) Petinju itu mencium kanvas pada ronde ketiga. (D) Rekor nasional lari jarak jauh berhasil ditumbangkan oleh pelari baru. (E) Dia menjadi catut di stasiun kereta api.

52

Xpedia Bahasa Indonesia, Kapita Selekta – Set 05

08. Sejalan dengan perkembangan yang terjadi, jumlah penduduk Kota Bandung terus mengalami peningkatan dengan jumlah urbanisasi sebanyak 2,3%, sedangkan pertumbuhan alami sebesar 1,08%. Kalimat di atas merupakan perluasan kalimat inti .... (A) Jumlah penduduk Kota Bandung berkembang. (B) Jumlah penduduk Kota Bandung mengalami peningkatan. (C) Jumlah urbanisasi ke Kota Bandung sebanyak 2,3% (D) Penduduk Bandung berkembang secara alami sebanyak 1,08%. (E) Jumlah penduduk Bandung dari tahun ke tahun terus meningkat. 09. Dalam dunia pendidikan pada masa kini menuntut guru sebagai fasilitator membuat anak dapat mengaplikasikan ilmu yang diperoleh di sekolah dalam kehidupan sehari-hari. Kalimat di atas tidak baku. Kalimat itu akan menjadi baku jika diperbaiki dengan cara .... (A) mengubah kata menuntut menjadi dituntut (B) keterangannya diletakkan di akhir kalimat (C) menghilangkan kata dalam (D) menghilangkan kata dalam kehidupan sehari-hari (E) mengganti kata sebagai

10. Barang yang dikatakan bekas tidak selamanya harus dibuang atau dimusnahkan. Teknologi canggih kini mendukung pemanfaatan barang bekas menjadi daur ulang yang dapat dijadikan komoditas siap pakai. Sebagai contoh, sampah yang menggunung di tempat pembuangan kini menjadi ajang rezeki. Para pemungut sampah memburu botol bekas, pecahan kaca, karton bekas, plastik, sandal, atau sepatu bekas, kaleng dan lain-lain. Barang bekas yang sejenis dikelompokkan kemudian dijual kepada penadah dengan tarif per kilogram. Oleh penadah, dikirim dengan truk-truk atau kereta api ke pabrik-pabrik ke kota besar lainya sebagai bahan baku. Seorang penadah barang bekas tersebut mencapai omzet penjualan sampai ratusan juta rupiah. Demikian pula dengan barang bekas yang lain seperti ban mobil. Dengan kreativitas, ban mobil bagian luar kini dapat disulap menjadi perangkat meja kursi yang artistik dengan harga jutaan rupiah. Simpulan bacaan di atas adalah .... (A) Barang bekas dapat dijadikan barang siap pakai. (B) Penadah barang bekas mendapat untung berjuta-juta rupiah. (C) Pemulung hidup dari barang bekas. (D) Pemanfaatan barang bekas dapat menjadi sumber penghidupan. (E) Ban mobil bekas dapat dijadikan meja dan kursi yang artistik.

53

Xpedia Bahasa Indonesia, Kapita Selekta – Set 05

11. (1) Wanita Indonesia harus mendapat pendidikan seperti kaum pria. (2) Mereka akan menjadi ibu yang mendidik anakanaknya. (3) Anak-anak akan menjadi manusia yang pandai dan berguna bagi masyarakat. (4) Ibu yang mengasuhnya hanyalah ibu yang bodoh Rangkaian yang tepat dari keempat kalimat tersebut adalah .... (A) Wanita Indonesia harus mendapat pendidikan seperti kaum pria karena mereka akan menjadi ibu yang mendidik anak-anaknya bagaimana anak-anak akan menjadi manusia yang pandai dan berguna bagi masyarakat kalau ibu yang mengasuhnya hanyalah ibu yang bodoh. (B) Wanita Indonesia harus mendapat pendidikan seperti kaum pria sebab mereka akan menjadi manusia yang pandai dan berguna bagi masyarakat jika ibu yang mengasuhnya hanyalah ibu yang bodoh. (C) Wanita Indonesia harus mendapat pendidikan seperti kaum pria karena mereka akan menjadi ibu yang mendidik anak-anaknya bagaimana pun anak-anak akan menjadi manusia yang pandai dan berguna bagi masyarakat andai ibu yang mengasuhnya hanyalah ibu yang bodoh. (D) Wanita Indonesia harus mendapat pendidikan yang mendapat pendidikan seperti kaum pria oleh karena mereka akan menjadi ibu yang mendidik anakanaknya bagaimana anak-anak akan menjadi manusia yang pandai dan berguna bagi masyarakat jika ibu yang mengasuhnya hanyalah ibu yang bodoh. (E) Wanita Indonesia harus mendapat pendidikan seperti kaum pria dan mereka akan menjadi ibu yang mendidik anakanaknya karena anak-anak akan menjadi manusia yang pandai dan berguna bagi masyarakat kalau ibu yang mengasuhnya hanyalah ibu yang bodoh.

54

12. 1. yang akan menuju Nias 2. dari dalam dan luar 3. terlantar di Medan 4. para relawan 5. yang akan menuju 6. karena pesawat 7. Gunung Sitoli, Nias 8. Bandara Binaka 9. masih terbatas Frase-frase di atas dapat disusun menjadi kalimat majemuk bertingkat yang baik dengan urutan .... (A) 6,5,8,9,7,1,3,2,4 (B) 4,2,6,7,8,5,9,1,3 (C) 6,5,7,8,2,4,1,2,9 (D) 4,2,1,3,6,5,8,7,9 (E) 4,2,3,1,6,8,5,7,9 13. Pembanguanan yang menyangkut semua aspek kehidupan manusia harus dilakukan secara merata dan berkesinambungan sehigga dapat meningkatkan kemakmuran dan kesejahteraan masyarakat. Yang harus dilakukan dalam kalimat itu adalah .... (A) pembangunan menyangkut semua aspek (B) pembangunan secara merata dan berkesinambungan (C) pembangunan meningkatkan kemakmuran (D) pembangunan kehidupan manusia meningkatkan kesejahteraan masyarakat (E) pembangunan yang menyangkut semua aspek kehidupan manusia 14. Yang merupakan kalimat baku .... (A) Kepada para peserta SPMB diingatkan agar supaya tidak membawa HP ke ruang ujian. (B) Kami telah selesai membicarakan mengenai masalah strategi SPMB. (C) Cuti besar diberikan kepada yang telah bekerja lebih dari enam tahun. (D) Ahmad selalu tolong-menolong dalam melakukan pekerjaan. (E) Banyak para ibu-ibu menangisi anaknya yang hilang akibat bencana tsunami akhir tahun lalu.

Xpedia Bahasa Indonesia, Kapita Selekta – Set 05

15. Berikut ini adalah kalimat penutup dalam surat yang tepat adalah .... (A) Atas bantuan Saudara, saya ucapkan terima kasih. (B) Kami akhiri surat ini dengan ucapan terima kasih. (C) Kami ucapkan terima kasih atas kerja sama ini. (D) Tugas ini harap diketahui dan dilaksanakan sebaik-baiknya. (E) Atas bantuan Bapak, kami mengucapkan terima kasih. 16. Mengapa kita selalu terlambat mengantisipasi bencana-demi bencana yang terjadi di negeri ini? Banyak jawaban yang dapat diajukan. Namun, ada satu aspek penyebab buruknya dampak peristiwa itu. Masyarakat tidak mendapat informasi yang memadai untuk memprediksi dan mengantisipasi bencana. Mereka tidak cukup mendapat penerangan bagaimana menghindari bencana. Di antara pernyataan di atas ini yang TIDAK dapat disebut kalimat adalah .... (A) pernyataan 1 (B) pernyataan 2 (C) pernyataan 3 (D) pernyataan 4 (E) pernyataan 5 17. Presiden mengharapkan rakyat dapat membangun bangsa dan negara ini dengan kemampuan yang ada pada dirinya. Kalimat di atas mempunyai makna yang sama dengan kalimat .... (A) Harapan Presiden ialah dapat membangun bangsa dan negaranya. (B) Rakyat membangun bangsa dengan kemampuan yang ada pada dirinya. (C) Presiden mengharapkan rakyatlah yang membangun bangsa dan negara ini. (D) Presiden memberi harapan kepada rakyat untuk membangun bangsa dan negara. (E) Presiden mengharap rakyat dapat membangun.

18. Raden Ajeng Kartini lahir di Jepara tanggal 21 April .... adalah salah satu wanita yang telah memperjuangkan nasib kaumnya .... Mau berkorban demi nusa dan bangsa. Oleh karena itu, jasa .... selalu dikenang bangsa Indonesia. Kata ganti orang yang tepat untuk mengisi bagian yang rumpang kalimat di atas adalah .... (A) Ia, dia, nya (B) Beliau, dia, nya (C) Dia, ia, beliau (D) Ia, dia, beliau (E) Beliau, nya, dia 19. Cek adalah merupakan alat pembayaran, tetapi bukan sebagai alat pembayaran yang sah. Rangkaian kata di atas menjadi kalimat baku jika dilakukan hal berikut .... (A) menghilangkan kata merupakan dan sebagai (B) kata pembayaran diubah menjadi bayaran (C) tidak perlu memakai kata adalah (D) frase alat pembayaran tidak perlu diulang. (E) menghilangkan kata pembayaran dan bukan 20. Pemilu nasional untuk memilih presiden dan wakil presiden, anggota DPR dan anggota DPD diselenggarakan secara nasional. Pemilu presiden / wakil presiden diikuti calon perseorangan yang diusulkan oleh partai nasional. Partai lokal tidak dapat mengikuti pemilu nasional. Jika dukungan partai lokal menyebar ke seluruh negeri, ia bisa naik menjadi partai nasional. Partai lokal hanya dapat mengikuti pemilu daerah. Pernyataan berikut yang sesuai dengan isi paragraf di atas adalah .... (A) Yang dapat menyelenggarakan pemilu nasional adalah partai nasional. (B) Partai lokal tidak dapat berperan dalam pemilu. (C) Presiden dan wakil presiden hanya dapat dipilih oleh partai nasional. (D) Pemilihan presiden dan wakil presiden, Anggota DPR, Anggota DPD diselenggarakan secara nasional. (E) Yang dapat memilih presiden / wakil presiden hanya anggota partai nasional.

55

Xpedia Bahasa Indonesia, Kapita Selekta – Set 05

21. Penanganan relokasi pemukiman recovery masyarakat serta pemulihan kondisi lingkungan adalah prioritas utama yang harus segera dilakukan di Kabupaten Ciamis. Hal ini untuk memberikan kepastian kelangsungan nasib korban bencana tsunami. Selain itu, alokasi anggaran untuk menangani bencana tsunami harus segera mendapat kepastian dan bisa secepatnya dicairkan. Paragraf di atas dapat dinyatakan secara ringkas dengan kalimat .... (A) Penanganan masalah masyarakat korban bencana tsunami Kabupaten Ciamis segera dilakukan. (B) Prioritas utama yang harus dilakukan Kab. Ciamis adalah penanganan relokasi pemukiman, recovery perekonomian dan pemulihan kondisi lingkungan. (C) Penanganan pemulihan kondisi korban tsunami di Ciamis harus segera dilakukan dan alokasi anggarannya mendapatkan kepastian dan secepatnya harus dicairkan. (D) Pemulihan Kab. Ciamis pasca Tsunami segera dilakukan dan alokasi anggaran harus secepatnya dicairkan. (E) Penanganan pemulihan korban tsunami di Kab. Ciamis harus segera dilakukan. 22. Kalimat yang merupakan kalimat majemuk bertingkat dengan anak kalimat pengganti keterangan subjek adalah .... (A) Bahwa anaknya tidak masuk sekolah, orang tuanya sudah mengetahuinya. (B) Gunung yang tingginya 6.875 meter itu telah ditaklukkan oleh para pendaki domestik. (C) Seandainya kita berkecukupan, sisihkanlah sebagian rezeki kita untuk fakir miskin. (D) Seluruh bangsa Indonesia mencintai perdamaian yang kini sudah terkoyakkoyak. (E) Kami memerlukan tempat yang layak untuk dihuni manusia.

56

23. Pengembangan kualitas sumber daya manusia, terutama anak dan remaja di bawah usia 19 tahun yang kini berjumlah 85 juta oarng perlu diperhatikan. Kalimat intinya mempunyai pola yang sama dengan .... (A) Adiknya sedang menangis. (B) Ibunya adalah wanita yang soleha. (C) Mereka sangat prihatin akan keadaan ekonomi bangsa Indonesia saat ini. (D) Saya tidak bergairah untuk belajar lagi. (E) Bapak gembira sekali mendengar berita itu. 24. Kalimat permintaan saran yang tepat dalam kata pengantar karya tulis adalah .... (A) Penulis mengucapkan terima kasih atas semua pihak. (B) Puji syukur kami panjatkan kehadirat Allah SWT. (C) Terima kasih atas segala bantuan Bapak/ Ibu (D) Semua kritik dan saran dari Saudara kami tampung. (E) Penulis mohon kritik yang membangun untuk perbaikan karya tulis ini. 25. Pola pembentukan kata pemukiman penduduk dalam kalimat Banjir merupakan satu ciri banyak kota dan tempat lain yang padat permukiman penduduk di hampir sepanjang pantai utara Pulau Jawa sama dengan pola pembentukan kata berikut, KECUALI .... (A) perakitan mobil (B) perdagangan luar negeri (C) perdamaian dunia (D) perlombaan bintang radio (E) perundingan tingkat menlu

Xpedia Bahasa Indonesia Kapita Selekta – Set 06

01. Pada tahun 2009 mendatang, pembangunan proyek monorel di Kota Bandung akam bisa dinikmati masyarakat. Sampai saat ini, proyek sarana transportasi massal yang diharapkan mampu mengurai benang kusur kemacetan di kota ini baru mencapai 80% studi kelayakan (feasibility study). Pada akhir tahun 2006 ini, proses study kelayakan diharapkan dapat diselesaikan. Pada tahun berikutnya, pembangunan sudah dapat dimulai. Dengan demikian, tahun 2009 masyarkat Kota Bandung sudah bisa menikmati sarana transportasi monorel ini. Ide pokok paragraf itu adalah .... (A) Monorel Kota Bandung akan beroperasi 2009. (B) Pembangunan monorel Kota Bandung dilakukan secara bertahap. (C) Sampai saat ini baru 80% studi kelayakan pembangunan projek monorel. (D) Pembangunan monorel Kota Bandung akan berlangsung selama 5 tahun. (E) Pembangunan monorel Kota Bandung diharapkan dapat mengatasi kemacetan lalu lintas. 02. Pada tahun 2009 mendatang, pembangunan proyek monorel di Kota Bandung akam bisa dinikmati masyarakat. Sampai saat ini, proyek sarana transportasi massal yang diharapkan mampu mengurai benang kusur kemacetan di kota ini baru mencapai 80% studi kelayakan (feasibility study). Pada akhir tahun 2006 ini, proses study kelayakan diharapkan dapat diselesaikan. Pada tahun berikutnya, pembangunan sudah dapat dimulai. Dengan demikian, tahun 2009 masyarkat Kota Bandung sudah bisa menikmati sarana transportasi monorel ini. Kalimat-kalimat dalam wacana di atas berupa opini, KECUALI .... (A) kalimat 1 (B) kalimat 2 (C) kalimat 3 (D) kalimat 4 (E) klaimat 5

03. Bentuk kata yang digunakan dalam kalimatkalimat di bawah ini tidak baku, KECUALI .... (A) Kita harus dapat memanfaatkan waktu seselektif mungkin. (B) Kemampuan masing-masing pemain menunjukkan kemampuan yang berarti. (C) Polda Jabar turut mensukseskan WTdJ 2005. (D) Perda itu kurang efektif untuk memberantas perjudian yang ada di Jawa Barat. (E) Pada babak Kwalifikasi pertama, Alonso mencatat waktu 1.29 menit. 04. Penulisan kalimat yang benar menurut EYD terdapat dalam .... (A) Tempat pembuangan akhir sampah pasir Impun hingga hari ini masih belum bisa diaktifkan. (B) Jika tututan ini tidak dipenuhi, warga mengancam akan berunjuk rasa ke kecamatan. (C) Jalan tol Cikampek - Padalarang telah dibuka secara resmi untuk umum tanggal 1 Mei 2005. (D) Direktur Utama PT. Wijaya Karya, A. Sucipto, menyebutkan bahwa jembatan Cikubang merupakan jembatan tertinggi di Jawa Barat. (E) Kemanapun kita pergi, jangan lupa membawa KTP. 05. Susunlah frase-frase berikut menjadi suatu kalimat yang baik ! 1. Numan Abdul Hakim 2. sebab perundangan yang ada 3. Wakil Gubernur Jawa Barat 4. melarang praktik haram tersebut 5. tidak perlu membuat peraturan daerah 6. menegaskan 7. untuk membrantas praktik perjudian Susunan kalimat yang baik adalah .... (A) 1,6,3,5,4,7,2 (B) 1,3,4,2,,5,7,6 (C) 3,1,4,2,7,6,5 (D) 3,1,6,7,5,2,4 57

Xpedia Bahasa Indonesia, Kapita Selekta – Set 06

06. Kata majemuk berkontruksi A dan B terdapat pada kalimat .... (A) “Bila ingin berhasil, patahkan dulu kaki dan tangan musuh!” kata komandan pasukan tempur. (B) Rumah sakit wajib menyiapkan kamar rawat inap bagi pasien yang tidak mampu. (C) Kereta api Bandung - Jakarta itu datang tepat waktu. (D) Orangtua sudah seharusnya memberikan contoh yang baik kepada anak-anaknya. (E) Karena mendung, matahari tidak menampakkan sinarnya. 07. Komersialisasi jabatan sudah membudaya di Indonesia. Kata komersialisasi jabatan dalam kalimat itu mengandung makna .... (A) menjualbelikan jabatan (B) memperdagangkan jabatan (C) menyalahgunakan jabatan (D) mencari keuntungan dengan menyalahgunakan jabatan (E) berbuat jahat dengan menyalahgunakan jabatan 08. Dari hasil wawancara di lapangan, penulis menemukan banyak golongan tua (orang tua, kaum pendidik, pejabat kelurahan, dan para pemuka masyarakat) yang berpendapat bahwa sebenarnya pelajar mempunyai peranan yang besar dalam pembangunan masyarakat terutama pembangunan kelurahan. Kalimat-kalimat di bawah ini mempunyai pola yang sama dengan kalimat di atas, KECUALI .... (A) Setelah bom Bali, Negara Barat melarang warganya pergi ke Bali. (B) Saat ini kami sedang menonton pertunjukan akrobat di Stadion Si Jalak Harupat. (C) Setiap hari, adik saya minum susu sebelum berangkat sekolah. (D) Kemarin, para siswa menunjukkan ketidakpuasannya kepada kepala sekolah. (E) Saat ini, mereka disuruh beristirahat selama satu jam. 58

09. 1. Kalaupun gagal masuk PTN favorit, itu bukan akhir segalanya, masih banyak PTS yang menjanjikan. 2. Tidak bisa di pungkiri, PTN masih merupakan pilihan favorit. 3. Di bursa kerja, beberapa PTS membuktikan lulusannya bahkan bisa mengalahkan saingannya yang lulus dari PTN dalam mencari pekerjaan. 4. Selain karena biayanya relatif murah (jika menggunakan jalur SPMB) dibandingkan biaya kuliah di PTS, menembus PTN di anggap suatu prestasi sekaligus prestise. 5. Karena terbatasnya daya tampung, tentu saja tidak semua anak negeri ini bisa menikmati bangku kuliah PTN. Kalimat-kalimat di atas dapat disusun menjadi sebuah alinea .... (A) 2,4,1,4,5 (B) 2,4,1,3,5 (C) 2,1,3,5,4 (D) 2,4,1,5,3 (E) 2,5,4,3,1 10. Gaji yang diperoleh dari hasil kerja selama empat tahun dibelikan rumah sederhana. Inti kalimat di atas .... (A) Gajinya diperoleh dari hasil kerjanya. (B) Gajinya dibelikan rumah (C) Gajinya cukup untuk membeli rumah (D) Rumah yang dibelinya sederhana (E) Rumah itu dibeli dengan uang gajinya. 11. Sumber … minyak di Blok Ambalat memicu pertentangan dengan negara tetangga Malaysia. Kata yang tepat untuk mengisi bagian yang rumpang dalam kalimat itu adalah .... (A) eksploitasi (B) eksplorasi (C) eksplikasi (D) eksplosi (E) eksaminasi

Xpedia Bahasa Indonesia, Kapita Selekta – Set 06

12. Rencana anggaran merupakan hal yang sangat penting di dalam suatu perusahaan sehingga setiap usahawan harus mampu membuat rencana biaya operasional secara baik sehingga pada suatu saat nanti dapat dipertanggungjawabkan sebagai laporan atau bahan untuk peminjaman modal di bank. Isi kalimat-kalimat berikut sesuai dengan maksud kalimat di atas, KECUALI .... (A) Setiap usahawan harus mampu membuat rencana biaya operasional secara baik. (B) Perusahaan dapat memanfaatkan biaya operasional untuk peminjaman modal di bank. (C) Dalam rencana anggaran perusahaan, terdapat uraian tentang dana dan rencana biaya operasional. (D) Rencana anggaran satu perusahaan dapat dimanfaatkan sebagai laporan pertanggungjawaban. (E) Di dalam membuat rencana anggaran perusahaan, diperlukan pemahaman tentang cara mencari dana. 13. Pengetahuan remaja tentang seks pada umumnya masih sangat minim bahkan masih banyak yang salah tafsir. Mereka beranggapan bahwa seks adalah perbuatan yang nista saja. Pengetahuan tentang muhrim dan bukan muhrim pun termasuk soal ini. Dalam masa pancaroba remaja menyadari adanya perkembangan biologis walaupun tidak diberi penerangan. Namun, tidak sedikit remaja yang buta perkembangan ini dan tidak mau bertanya kepada orang tua atau guru. Hal ini disebabkan tidak adanya keterbukaan antara orang tua dan anak. Simpulan yang dapat ditarik dari bacaan itu adalah .... (A) Pendidikan seks mengalami hambatan karena orangtua tidak menyetujuinya. (B) Pendidikan seks perlu diberikan kepada remaja. (C) Guru hendaknya mengajarkan pendidikan seks. (D) Sangat berbahaya bagi remaja jika pendidikan seks diberikan kepada mereka. (E) Pengetahuan agama harus menyertai pendidikan seks.

14. Pengetahuan remaja tentang seks pada umumnya masih sangat minim bahkan masih banyak yang salah tafsir. Mereka beranggapan bahwa seks adalah perbuatan yang nista saja. Pengetahuan tentang muhrim dan bukan muhrim pun termasuk soal ini. Dalam masa pancaroba, remaja menyadari adanya perkembangan biologis walaupun tidak diberi penerangan. Namun, tidak sedikit remaja yang buta perkembangan ini dan tidak mau bertanya kepada orang tua atau guru. Hal ini disebabkan tidak adanya keterbukaan antara orang tua dan anak. Isi bacaan itu merupakan sebuah .... (A) fakta (B) analisis (C) opini (D) referens (E) pembuktian 15. Penulisan kalimat yang sesuai dengan aturan ejaan yang berlaku terdapat dalam kalimat .... (A) Menanggapi aksi tukang becak, Priatna menegaskan apabila anggotanya yang terbukti melakukan kekerasan, agar dilaporkan kepada petugas berwenang. (B) Tidak sedikit pasien yang berasal dari luar kota yang bertujuan ke poliklinik, tetapi tidak mempunyai ongkos pulang. (C) Di tempat terpisah, walikota Bandung, Dada Rosada, menyampaikan permintaan maaf kepada warga Kota Bandung. (D) Angin Puting Beliung sering terjadi menjelang peralihan dari musim penghujan ke musim kemarau. (E) Pak bupati menandaskan bahwa anggota Satpol PP dilarang melakukan kekerasan.

59

Xpedia Bahasa Indonesia, Kapita Selekta – Set 06

16. Jawa Barat hingga saat ini belum bisa memenuhi kebutuhan daging sapi potong untuk masyarkatnya sebanyak 3.000.000 ekor per tahun. Jabar hanya mampu mensuplai 10% dari kebutuhan atau 30.000 ekor/tahun. Untuk memenuhi kebutuhan sisanya Jabar mendatangkan sapi potong dari propinsi lain. Dalam paragraf di atas, terdapat beberapa kata yang tidak baku. Paragraf di atas akan menjadi baku jika direvisi dengan cara berikut, KECUALI .... (A) pertahun (kalimat 1) di tulis per tahun (B) mensuplai (kalimat 2) diganti menyuplai (C) setelah kata sisanya (kalimat 3) dibubuhi tanda koma(,) (D) ekor/tahun (kalimat 2) di tulis ekor per tahun (E) propinsi (kalimat 3) ditulis provinsi 17. Anggota komisi C, Y. Untung, mengatakan tidak terserapnya anggaran tidak bisa digeneralisasi akibat perencanaan yang tidak matang. Menurut dia, ada dua faktor yang menyebabkan hal itu terjadi. Pertama, dalam realisasi kegiatan ada prinsip efesiensi. Sehingga, ada penghematan saat proses kegiatan. Kedua, volume kebutuhan bisa berkurang karena ada penurunan harga barang saat proses pelaksanaan kegiatan. Dalam paragraf itu, terdapat kalimat yang strukturnya salah. Kalimat yang dimaksud adalah .... (A) kalimat 1 (B) kalimat 2 (C) kalimat 3 (D) kalimat 4 (E) kalimat 5 18. Unsur inti kalimat Agar pada masa yang akan datang tidak ada orang yang menuntut ganti rugi, peristiwa itu perlu mendapat perhatian dari berbagai pihak adalah .... (A) pada masa yang akan datang, tidak ada yang menuntut. (B) tidak ada orang yang menuntut ganti rugi. (C) peristiwa itu mendapat perhatian. (D) berbagai pihak perlu memperhatikan ganti rugi. (E) masa mendatang menuntut ganti rugi.

60

19. BKKBN telah menyiapkan alat kontrasepsi gratis untuk keluarga miskin sebanyak mungkin. Kontrasepsi dalam kalimat di atas mengandung arti .... (A) pencegah kemasukan sel telur. (B) penggugurkan kandungan. (C) pembatasi kandungan. (D) pencegah kesuburan. (E) pencegah kehamilan. 20. Berteriak tidak dijadikan senjata oleh anak, para ahli menyarankan .... orang tua tidak langsung merspon teriakan anak, .... memberitahukannya bahwa berteriak bukan cara berkomunikasi yang tepat. Kata penghubung yang tepat melengkapi kalimat tersebut .... (A) setelah, untuk, sebab (B) jika, agar, tetapi (C) sebelum, andai, sebab (D) sesudah, jika, tetapi (E) biarkan, setelah agar 21. Boleh jadi, sikap feodalistik otoriter itu merupakan bias dari filsafat klasik, tabula rasa. Kata bercetak tebal dalam kalimat itu dapat diganti dengan kata-kata .... (A) ningrat, semaunya, penyimpangan, kertas putih. (B) penjajah, sewenang-sewenang, pembelokan, daftar kosong. (C) bangsawan, sok kuasa, pembelokan, kertas putih. (D) ningrat, berkuasa, penyimpangan, daftar kertas putih. (E) penjajah, absolut, belokan, kertas putih tanpa isi.

Xpedia Bahasa Indonesia, Kapita Selekta – Set 06

22. Tidak banyak guru .... yang mempunyai banyak waktu untuk membaca .... lain, diluar .... untuk memantapkan konsep dasar sains. Istilah yang tepat untuk mengisi bagian yang rumpang dalam kalimat itu adalah .... (A) sains, buku, kebiasaan (B) sains, referensi, kurikulum (C) IPA, buku, sains (D) IPS, buku, pelajaran (E) sains, rujukan, mata pelajaran 23. Polusi udara memang terjadi secara alamiah. sebab fenomena alam seperti letusan gunung merapi, kebakaran hutan, debu, tumbuh-tumbuhan yang membusuk, bangkai hewan, dan garam laut dapat mencemari udara. Akan tetapi, semua ini masih dapat dikatakan dalam batas-batas yang dapat ditoleransi. Artinya proses laju pembersihan polusi dari udara dan laju pencemaran itu masih dalam keseimbangan alam. Oleh karena itu, harus dapat memelihara kebersihan lingkungan. Kalimat yang tidak logis dalam paragraf tersebut adalah .... (A) pertama (B) kedua (C) ketiga (D) keempat (E) kelima

25. Buku ini membahas berbagai cretivity manusia sejak zaman kuno, sampai diketemukannya semua bidang ilmu, sejak dari sejarah lahirnya ilmu pengetahuan sampai penciptaannya. Buku ini akan menambah segudang pemahaman atas lahirnya secara jelas dan detail. Kapan dan di mana ilmu pengetahuan berasal? Kalimat di atas akan menjadi kalimat baku jika diperbaiki dengan cara berikut, KECUALI .... (A) mengganti kata creativity dengan kata kreativitas (B) mengganti kata diketemukannya menjadi ditemukannya (C) mengganti kata detail menjadi mendetail (D) mengganti kata sejak dari menjadi mulai dari (E) menggganti kata dimana menjadi di mana

24. Peribahasa yang mengungkapkan mengerjakan dua atau tiga perbuatan dalam satu waktu ialah .... (A) Hati gajah sama dilapah, hati kuman sama dicecah. (B) Berat sama dipikul, ringan sama dijinjing. (C) Sekali lancung ke ujian, seumur hidup orang tak akan percaya. (D) Sekali merengkuh dayung, dua tiga pulau terlampaui. (E) Adat teluk timbunan kapal.

61

Xpedia Bahasa Indonesia Kapita Selekta – Set 07

Kita harus membedakan antara HIV dan AIDS lebih dahulu. HIV adalah virus yang menyerang sistem kekebalan tubuh manusia. Secara fisik, pengidap HIV (disebut HIV positif) tampak tidak berbeda dengan orang normal lainnya, Namun, setelah 5-10 tahun, seorang pengidap HIV menunjukkan gejala-gejala menderita AIDS. AIDS sendiri adalah gejala penyakit akibat hilang atau menurunnya kekebalan tubuh. Menurutnya, sistem kekebalan tubuh membuat penyakit yang ringan pun dapat menimbulkan gejala yang lebih berat. Misalnya saja flu, pada orang normal akan sembuh dengan sendirinya dalam waktu kurang dari seminggu meski tidak diobati sama sekali asal cukup makan dan istirahat. Namun, pada pengidap HIV atau penderita AIDS, flu akan menetap lama, bahkan semakin parah dari waktu ke waktu. Karena tidak menampakkan gejala apa pun, infeksi HIV menjadi lebih berbahaya. Satu-satunya cara untuk memastikan adanya HIV / AIDS adalah pemeriksaan darah di laboratorium. 01. Dalam bacaan di atas dikemukakan pula yang tercantum berikut ini, KECUALI .... (A) Melalui pemeriksaan darah dapat diketahui apakah seseorang mengidap AIDS. (B) Bila sakit, penderita AIDS sukar diketahui. (C) Setelah beberapa tahun, pengdap HIV baru diketahui menderita AIDS. (D) Penderita AIDS tidak tahan melawan penyakit. (E) Orang yang tubuhnya lemah mudah terjangkit AIDS.

62

Kita harus membedakan antara HIV dan AIDS lebih dahulu. HIV adalah virus yang menyerang sistem kekebalan tubuh manusia. Secara fisik, pengidap HIV (disebut HIV positif) tampak tidak berbeda dengan orang normal lainnya, Namun, setelah 5-10 tahun, seorang pengidap HIV menunjukkan gejala-gejala menderita AIDS. AIDS sendiri adalah gejala penyakit akibat hilang atau menurunnya kekebalan tubuh. Menurutnya, sistem kekebalan tubuh membuat penyakit yang ringan pun dapat menimbulkan gejala yang lebih berat. Misalnya saja flu, pada orang normal akan sembuh dengan sendirinya dalam waktu kurang dari seminggu meski tidak diobati sama sekali asal cukup makan dan istirahat. Namun, pada pengidap HIV atau penderita AIDS, flu akan menetap lama, bahkan semakin parah dari waktu ke waktu. Karena tidak menampakkan gejala apa pun, infeksi HIV menjadi lebih berbahaya. Satu-satunya cara untuk memastikan adanya HIV / AIDS adalah pemeriksaan darah di laboratorium. 02. Bacaan di atas termasuk bentuk karangan yang .... (A) mengajak atau membujuk (B) menguraikan maksud dan tujuan (C) memberikan alasan suatu pendapat (D) menceritakan atau mengisahkan (E) menggambarkan penginderaan

Xpedia Bahasa Indonesia, Kapita Selekta – Set 07

Kita harus membedakan antara HIV dan AIDS lebih dahulu. HIV adalah virus yang menyerang sistem kekebalan tubuh manusia. Secara fisik, pengidap HIV (disebut HIV positif) tampak tidak berbeda dengan orang normal lainnya. Namun, setelah 5-10 tahun, seorang pengidap HIV menunjukkan gejalagejala menderita AIDS. AIDS sendiri adalah gejala penyakit akibat hilang atau menurunnya kekebalan tubuh. Menurutnya sistem kekebalan tubuh membuat penyakit yang ringan pun dapat menimbulkan gejala yang lebih berat. Misalnya saja flu, pada orang normal akan sembuh dengan sendirinya dalam waktu kurang dari seminggu meski tidak diobati sama sekali asal cukup makan dan istirahat. Namun, pada pengidap HIV atau penderita AIDS, flu akan menetap lama, bahkan semakin parah dari waktu ke waktu. Karena tidak menampakkan gejala apa pun, infeksi HIV menjadi lebih berbahaya. Satusatunya cara untuk memastikan adanya HIV / AIDS adalah pemeriksaan darah di laboratorium. 03. Judul yang sesuai dengan bacaan di atas adalah .... (A) Memberantas AIDS (B) Penularan AIDS (C) Pencegahan AIDS (D) Mengenal AIDS (E) Pemeriksaan AIDS

Kita harus membedakan antara HIV dan AIDS lebih dahulu. HIV adalah virus yang menyerang sistem kekebalan tubuh manusia. Secara fisik, pengidap HIV (disebut HIV positif) tampak tidak berbeda dengan orang normal lainnya. Namun, setelah 5-10 tahun, seorang pengidap HIV menunjukkan gejalagejala menderita AIDS. AIDS sendiri adalah gejala penyakit akibat hilang atau menurunnya kekebalan tubuh. Menurutnya sistem kekebalan tubuh membuat penyakit yang ringan pun dapat menimbulkan gejala yang lebih berat. Misalnya saja flu, pada orang normal akan sembuh dengan sendirinya dalam waktu kurang dari seminggu meski tidak diobati sama sekali asal cukup makan dan istirahat. Namun, pada pengidap HIV atau penderita AIDS, flu akan menetap lama, bahkan semakin parah dari waktu ke waktu. Karena tidak menampakkan gejala apa pun, infeksi HIV menjadi lebih berbahaya. Satusatunya cara untuk memastikan adanya HIV / AIDS adalah pemeriksaan darah di laboratorium. 04. Secara fisik pengidap HIV disebut juga HIV positif. kata positif pada teks di atas mengandung .... (A) lawannya negatif (B) menunjukkan perkembangan (C) tidak mengenal (D) menunjukkan adanya penyakit (E) menunjukkan tidak adanya penyakit 05. Berbicara dalam bahasa Inggris bagi sebagian besar orang bukan hal yang gampang. Mereka bukan tidak pernah belajar bahasa tersebut. Sejak SMP, materi kurikulum pendidikan sudah mewajibkan belajar bahasa Inggris. Bahkan, belakangan ini TK dan SD sudah mulai memperkenalkan bahasa tersebut, tetapi tetap saja mereka tidak lancar berbahasa Inggris. Karena itu, pembelajaran bahasa Inggris perlu terus ditingkatkan. Kalimat alasan dalam paragraf tersebut terdapat pada nomor .... (A) 1 (B) 2 (C) 3 (D) 4 (E) 5 63

Xpedia Bahasa Indonesia, Kapita Selekta – Set 07

06. Penulisan unsur serapan yang berikut ini benar, KECUALI .... (A) sistem, apotek, atlet (B) akwarium, kwitansi, sistesis (C) analisis, hipotesis, sintesis (D) formal, tradisional, rasional (E) trotoar, dresoar, repertoar 07. Penarapan tanda baca kalimat ini benar, KECUALI .... (A) Karena krisis ekonomi, banyak buruh pabrik yang di-PHK (B) LKIR diikuti oleh siswa SMU se-JawaTimur (C) Berhati-hatilah terhadap sinar-X (D) Kabinet yang sudah mantap sudah tentu tidak akan di-resufle lagi. (E) Upahnya hanya Rp 15.000,00/hari. 08. Setiap kata ditulis secara benar dalam kalimat .... (A) Di samping itu, ia juga diminta untuk menyelesaikan laporan priyek. (B) Di antara sepuluh orang, hanya tiga orang yang berani masuk kedalam arena pertandingan. (C) Kerjasama di antara mereka sudah diantisipasi oleh kelompok lawannya. (D) Ia tidak pernah lupa bersyukur ke hadirat Tuhan Yang Maha kuasa. (E) Dari dua puluh sembilan korban, lima di antaranya adalah anak pejabat. 09. Setiap orang kaya, yang gagah, yang miskin, yang jahat, ataupun yang baik akan menemui ajalnya. Maksud kalimat di atas, dapat diungkapkan kembali seperti barikut .... (A) Manusia itu tidak ada yang kaya, tidak ada yang miskin, semuanya sama saja. (B) Manusia itu tidak ada yang hidup abadi. (C) Orang yang gagah, yang lemah, yang jahat, yang baik, di antara mereka tidak ada bedanya. (D) Bukankah hidup manusia itu tidak ada yang kekal. (E) Hidup manusia itu relatif.

64

10. Pengairan Indonesia dapat manyumbangkan pangan yang sangat besar jumlahnya. Jika penangkapan ikan kita kerjakan dengan efisien, produksi ikan dapat ditingkatkan menjadi 7,5 ton/bulan. Akan tetapi, produksi ini hanya 1,2 ton sebulan. Apabila diperhatikan letak kalimat utamanya, paragraf tersebut dikembangkan dengan cara .... (A) induktif (B) deduktif (C) definisi (D) analogi (E) sebab-akibat 11. Penggunaan tanda baca yang benar terdapat pada .... (A) Ibu Rully tidak setuju, karena gagasan itu tidak masuk akal. (B) Di Ciwalk terdapat : sepatu, baju, tas, dan perabot rumah tangga. (C) Surat biasa, surat kilat, ataupun surat kilat khusus memerlukan perangko. (D) Oleh karena itu kita harus rajin belajar. (E) Joko, sahabat WIdo mendapat beasiswa. 12. Ruang tempat kami belajar tidak luas hanya 7m x 8m (1). Bangku kami belajar teratur empat baris ke belakang (2). Pada dinding depan kelas tergantung papan tulis putih 1 x 2,5 m (3). Dua lukisan besar mengapitnya (4). Di sebelah kiri gambar Garuda Indonesia dan di sebelah kanan gambar presiden (5). Meja guru terdapat di pojok kiri (6). Jumlah siswa di kelas sebanyak empat puluh orang (7). Setiap siswa selalu meletakkan bunga yang segar dalam jambangan di atas meja itu, karena kami senang melihatnya (8). Di sebelah kiri, delapan jendela besar memasukkan cahaya matahari dan hawa segar ke dalam kelas (9). Dindingnya polos tidak ada hiasan, kecuali kalender dekat meja guru (10). Kalimat sumbang yang terdapat dalam paragraf di atas adalah .... (A) 1 dan 2 (B) 3 dan 4 (C) 5 dan 6 (D) 7 dan 8 (E) 8 dan 10

Xpedia Bahasa Indonesia, Kapita Selekta – Set 07 doc. Name: XPIND9907

version : 2012-08 |

13. Mendiknas mengaku, banjir yang melanda Jakarta dan sekitarnya memberi dampak yang cukup signifikan terhadap sektor pendidikan. Makna signifikan dalam wacana tersebut adalah .... (A) hebat atau luar biasa (B) penting atau berarti (C) tepat dan sesuai (D) pasti dan yakin (E) parah atau merugikan 14. Menurut Wayan Pica, surfer profesional dan ketua Bali surfing Association, atlet surfing Internasional yang pernah mengikuti kejuaraan di Green-land merasa puas karena selalu mendapat ombak yang mereka inginkan disana. Inti kalimat di atas adalah .... (A) Wayan Pica surfer profesional Ketua Bali Surfing Association. (B) Wayan Pica surfing internasional pernah mengikuti kejuaraan. (C) Atlet-atlet surfing internasional pernah mengikuti kejuaraan. (D) Atlet-atlet surfing internasional merasa puas. (E) Atlet-atlet surfing internasional selalu mendapatkan ombak yang diinginkan. 15. Tidak sedikit pengusaha asing berhasrat .... modal di Indonesia. Kata kerja yang tepat untuk melengkapi kalimat tersebut adalah .... (A) menanamkan (B) menanami (C) menanam (D) ditanam (E) ditanami 16. Di antara gabungan kata berikut, mana yang merupakan kalimat tidak baik .... (A) Dalam kehidupan sehari-hari sering, kita jumpai para ibu rumah tangga turut mencari nafkah. (B) Karena kekurangan biaya, kami belum dapat meeralisasikan gagasan itu. (C) Pemerintah menugaskan kami meneliti kebudayaan beberapa suku bangsa terasing di Sulawesi. (D) Sudah berapa lama anda mengajar di sini. (E) Dialah satu-satunya yang akan mewarisi harta kekayaan orang tuanya.

halaman 4

17. Tragedi Santa Cruz yang terjadi di Timor-Timur dan menewaskan puluhan orang itu, setiap tahun diperingati, tetapi pada gilirannya menimbulkan tragedi baru dengan skala yang berbeda. Yang menjadi inti kalimat adalah .... (A) Tragedi Santa Cruz menewaskan puluhan orang. (B) Tragedi Santa Cruz diperingati. (C) Peringatan tragedi Santa Cruz menimbulkan tragedi baru. (D) Tragedi Santa Cruz terjadi Timor-Timur. (E) Tragedi Santa Cruz setiap tahun diperingati. 18. Di antara kalimat-kalimat berikut yang efektif adalah .... (A) Saya, jika ibu menyetujui, akan mengatakan hal itu kepada Intan. (B) Karena anggota belum semuanya hadir, rapat itu diundur setengah jam dari waktu yang telah ditentukan. (C) Berdasarkan keputusan pemerintah, yang melakukan “pungli” dianggap melakukan pelanggaran. (D) UUD itu mengikat pemerintah, setiap lembaga negara dan lembaga masyarkat dan juga mengikat setiap warga negara Indonesia. (E) Saya akan membicarakan hal itu dengan ayah. 19. 1. Pemerintah memperhatikan pedagang

eceran.

2. Waspadalah terhadap penyakit menular. 3. kebiasaan buruk yang sulit diubahnya adalah membual. Inti frase yang tercetak miring dalam sejumlah kalimat di atas adalah .... (A) eceran, pedagang, buruk (B) pedagang, penyakit, kebiasaan (C) pedagang, menular, buruk (D) eceran, penyakit, kebiasaan (E) pedagang, menular, buruk

65

Xpedia Bahasa Indonesia, Kapita Selekta – Set 07

20. Pembangunan yang menyangkut semua aspek kehidupan manusia harus dilakukan secara merata dan berkesinambungan sehingga dapat meningkatkan kemakmuran dan kesejahteraan masyrakat. Kalimat inti pada kalimat di atas adalah .... (A) Pembangunan yang menyangkut aspek kehidupan. (B) Semua aspek kehidupan manusia. (C) Pembangunan harus dilakukan. (D) pembangunan harus merata dan berkesinambungan. (E) Meningkatkan kemakmuran dan kesejahteraan. 21. Pola kalimat Pak Rizal berangkat ke Mekah tanpa pembimbing sama dengan pola kalimat .... (A) Kami melihatnya secara sepintas (B) Kemajuan meskipun sedikit demi sedikit (C) Pasukan itu menyerbu bersama rakyat (D) Calon pegawai itu pasti diterima asal ada yang mem beri rekomendasi (E) Gambar itu dibuat sebaik-baiknya 22. Hubungan makna yang tersebut dalam binatang - burung - kaka tua terdapat juga dalam .... (A) bunga - melati - mawar (B) minum - teh - kopi (C) tanaman - buah - bunga (D) kuning - merah - hijau (E) senang - gembira - sukacita 23. Masyarakat terbiasa menyumbangkan sebagian besar harta benda yang dimilikinya untuk membantu meringankan beban kelompok masyarakat yang kurang beruntung. Kalimat berikut berasal dari kalimat inti .... (A) Masyarakat menyumbangkan hartanya. (B) Masyarakat terbiasa. (C) Masyarakat kurang beruntung. (D) Harta benda meringankan. (E) Masyarakat membantu.

66

24. Dukungan dana untuk mengatasi gejolak moneter yang terjadi di Indonesia terus mengalir. Kali ini, bantuan datang dari Brunei Darussalam. Sabtu pekan lalu Sultan Darussalam Hassanul Bolkiah, bersama sekitar 50 anggota rombongan mendarat di lapangan Halim Perdana Kusuma Jakarta dengan menggunakan pesawat pribadi. Sore harinya selama empat puluh menit Sultan Bolkiah diterima mantan Presiden Soeharto di ruang Jepara, Istana Merdeka. Pikiran utama paragraf di atas adalah .... (A) Dukungan dana terus mengalir. (B) Dana mengatasi gejolak moneter. (C) Bantuan datang dari Brunai Darussalam. (D) Sultan Hassanul mendarat di Halim Perdana Kusuma. (E) Sultan menggunakan pesawat pribadi. 25. Pembangunan pertanian tidak hanya membatasi pada pengembangan tanaman internasional. Kalimat di atas dianggap tidak efektif karena penggunaan kata yang salah. Kata yang salah itu adalah .... (A) pembangunan (B) tanaman (C) tidak (D) tradisional (E) membatasi 26. Kalimat yang lazim untuk mengakhiri surat resmi adalah .... (A) Mengingat pentingnya rapat ini, kehadiran Saudara sangat kami harapkan tepat waktu. (B) Kami mengharapkan hal ini mendapat perhatian Saudara. Untuk itu, kami mengucapkan banyak terima kasih. (C) Atas perhatian Bapak, kami ucapkan banyak-banyak terimakasih. (D) Mengingat pentingnya pertemuan ini, kehadirannya sangat kami harapkan. (E) Atas perhatian Saudara, terima kasih.

Xpedia Bahasa Indonesia, Kapita Selekta – Set 07

27. Kalimat berikut yang memenuhi kepaduan adalah .... (A) Apabila Jaksa Agung ingin menggunakan hak hukum yang tersedia, tetap bisa dilakukan. (B) Kalau kita memperhatikan secara cermat, akan sulit menunjukkan salahnya. (C) Dalam menyusun skripsi, mengumpulkan data merupakan aktivitas yang harus dilakukan. (D) Untuk mengembangkan naskah pidato yang baik, mempersyaratkan seseorang memiliki wawasan yang luas. (E) Karena alasannya tidak masuk akal, pihak jaksa sebagai penuntut umum tidak menerima. 28. Kalimat berikut ini yang merupakan kalimat baku adalah .... (A) Pada setiap pernyataan yang diucapkannya selalu mengandung kritikan yang tajam pada pemerintah. (B) Dalam menghadapi konflik sosial dan krisis ekonomi yang berkepanjangan ini, tidak ada cara lain kecuali berdoa dan berusaha. (C) Jika persoalan ini dirundingkan dan ditanggung bersama, maka akan dapat segera diselesaikan dan terasa lebih ringan. (D) Ketika sedang berada pompa bensin untuk mengisi BBM, untuk tidak menghidupkan HP. (E) Setiap orang memiliki kekuasaan bawaan, entah berkepribadian menarik, entah berkarisma tinggi, untuk memberikan pengaruh.

29. Proses pembentukkan kata pemberlakuan dalam kalimat pemberlakuan undang-undang itu ditunda berpola laku>berlaku>memberlakukan>pemberlakuan. Pernyataan berikut ini benar, KECUALI .... (A) Proses pembentukkan kata pemberontakan dalam kalimat Pemberontakan terhadap pemerintah digagalkan berbeda dengan pola tersebut. (B) Proses pembentukkan kata pemberangkatan dalam Pemberangkatan jemaah haji tertunda sama dengan pola tersebut. (C) Proses pembentukkan kata pemberdayaan dalam kalimat Perusahaan mengadakan progam pemberdayaan karyawan berbeda dengan pola tersebut. (D) Proses pembentukkan kata pemberantasan dalam kalimat Pemberantasan penyakit menular memerlukan partisipasi aktif warga berbeda dengan pola tersebut. (E) Proses pembentukkan kata pemerhentian dalam kalimat kita turun di pemberhentian berikutnya sama dengan pola tersebut. 30. Feri berpenumpang 1.318 orang milik pengusaha Mesir tenggelam saat berlayar dari Dubai menuju Safaga yang berjarak 193 kilometer. Ide terpenting yang perlu diperhatikan saat membaca kalimat tersebut adalah .... (A) Feri berlayar dari Dubai menuju Safaga. (B) Feri berpenumpang 1.318 orang dan milik pengusaha Mesir. (C) Dubai berjarak 193 kilometer. (D) Feri tenggelam. (E) Feri tenggelam saat berlayar.

67

Xpedia Bahasa Indonesia Kapita Selekta – Set 08

Tangkasi merupakan satwa primata keturunan satwa purba. Tangkasi yang termasuk spesies tarsius ini disebut juga binatang hantu yang lucu. Bentuk dan anatomi binatang ini mirip monyet. Mukanya lucu, karena sepasang matanya yang besar mencolok dan besar kupingnya tegak yang tak proporsional dengan mukanya yang kecil. Tubuhnya yang berbulu halus abu-abu kekuningan dan berkilat hanya setinggi 9,5 - 14 sentimeter dengan jumbaian bulu di ujung yang mirip kuas. Tangkasi ini termasuk kelas mamalia atau binatang menyusui. Dilihat dari jenis makanannya, binatang ini termasuk insektivora dan juga karnivora. Oleh sebab itu, selain mengudap serangga seperti semut, kecoa, dan kalajengking, primata ini juga gemar memakan cecak, tikus kecil, bahkan burung-burng kecil. Tangkasi adalah binatang malam. Jika hari mulai gelap, mereka bergentayangan dan meloncat-loncat mencari makanan. Menjelang munculnya surya pagi, mereka cepat-cepat masuk sarang lagi. Tangkasi ini merupakan salah satu satwa endemik khas Sulawesi. Sebetulnya, tangkasi yang juga termasuk spesies tarsius ini juga terbesar di Sumatra, Kalimantan, dan Kepulauan Maluku. Karena spesies ini termasuk satwa purba dan khas Indonesia khusus Sulawesi, wajarlah satwa liar ini menjadi sasaran kaum peneliti internasional. 01. Tema wacana di atas adalah sebagai berikut .... (A) Tangkasi merupakan sejenis binatang yang menjadi sasaran kaum peneliti Internasional. (B) Tangkasi merupakan binatang yang suka bergentayangan dan berlompat-lompat mencari makan. (C) Deskripsi mengenai berbagai jenis primata turunan satwa purba yang terdapat di Indonesia. (D) Deskripsi mengenai berbagai jenis binatang purba yang terdapat di Indonesia. (E) Tangkasi merupakan sejenis primata turunan satwa purba spesies tarsius yang terdapat di Indonesia, khususnya di Sulawesi. 68

Tangkasi merupakan satwa primata keturunan satwa purba. Tangkasi yang termasuk spesies tarsius ini disebut juga binatang hantu yang lucu. Bentuk dan anatomi binatang ini mirip monyet. Mukanya lucu, karena sepasang matanya yang besar mencolok dan besar kupingnya tegak yang tak proporsional dengan mukanya yang kecil. Tubuhnya yang berbulu halus abu-abu kekuningan dan berkilat hanya setinggi 9,5 - 14 sentimeter dengan jumbaian bulu di ujung yang mirip kuas. Tangkasi ini termasuk kelas mamalia atau binatang menyusui. Dilihat dari jenis makanannya, binatang ini termasuk insektivora dan juga karnivora. Oleh sebab itu, selain mengudap serangga seperti semut, kecoa, dan kalajengking, primata ini juga gemar memakan cecak, tikus kecil, bahkan burung-burng kecil. Tangkasi adalah binatang malam. Jika hari mulai gelap, mereka bergentayangan dan meloncat-loncat mencari makanan. Menjelang munculnya surya pagi, mereka cepat-cepat masuk sarang lagi. Tangkasi ini merupakan salah satu satwa endemik khas Sulawesi. Sebetulnya, tangkasi yang juga termasuk spesies tarsius ini juga terbesar di Sumatra, Kalimantan, dan Kepulauan Maluku. Karena spesies ini termasuk satwa purba dan khas Indonesia khusus Sulawesi, wajarlah satwa liar ini menjadi sasaran kaum peneliti internasional. 02. Tangkasi adalah .... (A) sejenis monyet pemakan tumbuhtumbuhan (B) mirip hantu yang lucu (C) binatang peliharaan yang lucu (D) spesies tarsius turunan satwa purba (E) binatang malam pemakan tumbuhtumbuhan

Xpedia Bahasa Indonesia, Kapita Selekta – Set 08

Tangkasi merupakan satwa primata keturunan satwa purba. Tangkasi yang termasuk spesies tarsius ini disebut juga binatang hantu yang lucu. Bentuk dan anatomi binatang ini mirip monyet. Mukanya lucu, karena sepasang matanya yang besar mencolok dan besar kupingnya tegak yang tak proporsional dengan mukanya yang kecil. Tubuhnya yang berbulu halus abu-abu kekuningan dan berkilat hanya setinggi 9,5 - 14 sentimeter dengan jumbaian bulu di ujung yang mirip kuas. Tangkasi ini termasuk kelas mamalia atau binatang menyusui. Dilihat dari jenis makanannya, binatang ini termasuk insektivora dan juga karnivora. Oleh sebab itu, selain mengudap serangga seperti semut, kecoa, dan kalajengking, primata ini juga gemar memakan cecak, tikus kecil, bahkan burung-burng kecil. Tangkasi adalah binatang malam. Jika hari mulai gelap, mereka bergentayangan dan meloncat-loncat mencari makanan. Menjelang munculnya surya pagi, mereka cepat-cepat masuk sarang lagi. Tangkasi ini merupakan salah satu satwa endemik khas Sulawesi. Sebetulnya, tangkasi yang juga termasuk spesies tarsius ini juga terbesar di Sumatra, Kalimantan, dan Kepulauan Maluku. Karena spesies ini termasuk satwa purba dan khas Indonesia khusus Sulawesi, wajarlah satwa liar ini menjadi sasaran kaum peneliti internasional. 03. Tangkasi menjadi sasaran kaum peneliti internasional karena alasan sebagai berikut, KECUALI .... (A) jenis primata ini jarang dijumpai di daerah lain. (B) satwa turunan purba yang langka (C) jenis primata ini merupakan endemik khas Indonesia (D) penyebaran jenis ini hanya di daerah Indonesia (E) tangkasi termasuk insektivora dan karnvora

Tangkasi merupakan satwa primata keturunan satwa purba. Tangkasi yang termasuk spesies tarsius ini disebut juga binatang hantu yang lucu. Bentuk dan anatomi binatang ini mirip monyet. Mukanya lucu, karena sepasang matanya yang besar mencolok dan besar kupingnya tegak yang tak proporsional dengan mukanya yang kecil. Tubuhnya yang berbulu halus abu-abu kekuningan dan berkilat hanya setinggi 9,5 - 14 sentimeter dengan jumbaian bulu di ujung yang mirip kuas. Tangkasi ini termasuk kelas mamalia atau binatang menyusui. Dilihat dari jenis makanannya, binatang ini termasuk insektivora dan juga karnivora. Oleh sebab itu, selain mengudap serangga seperti semut, kecoa, dan kalajengking, primata ini juga gemar memakan cecak, tikus kecil, bahkan burung-burng kecil. Tangkasi adalah binatang malam. Jika hari mulai gelap, mereka bergentayangan dan meloncat-loncat mencari makanan. Menjelang munculnya surya pagi, mereka cepat-cepat masuk sarang lagi. Tangkasi ini merupakan salah satu satwa endemik khas Sulawesi. Sebetulnya, tangkasi yang juga termasuk spesies tarsius ini juga terbesar di Sumatra, Kalimantan, dan Kepulauan Maluku. Karena spesies ini termasuk satwa purba dan khas Indonesia khusus Sulawesi, wajarlah satwa liar ini menjadi sasaran kaum peneliti internasional. 04. Ciri-ciri binatang malam di antaranya .... (A) berbulu halus dan bagus seperti bulu tangkasi (B) mencari makanan dengan melompatlompat (C) aktivitas hidupnya dilakukan pada malam hari (D) binatang ini bersarang di rongga-rongga pohon (E) tangkasi termasuk insektivora dan karnivora yang suka tidur malam

69

Xpedia Bahasa Indonesia, Kapita Selekta – Set 08

05. Penggunaan huruf kapital yang benar terdapat pada kalimat .... (A) Kita harus berusaha menggunakan Bahasa Indonesia yang baik dan benar. (B) Pada bulan Agustus, ia akan berangkat ke Amerika. (C) Di mana banyak terdapat Suku Jawa? (D) Pegunungan yang membentang di daratan Sumatera itu bernama Bukit Barisan. (E) Dengan gembira, masyrakat menyambut Hari Lebaran. 06. Pemakaian huruf miring atau garis bawah dibenarkan, KECUALI untuk .... (A) Satu-satunya majalah sastra di Indonesia ialah Horizon. (B) Jika akan berkemah, kita juga memerlukan kompas. (C) Politik devide et impera pernah merajalela di sini. (D) Siapakah pengarang Layar Terkembang? (E) Huruf pertama kata abad adalah a. 07. Penulisan kata bilangan yang benar terdapat pada kalimat .... (A) 15 orang tewas dalam kecelakaan itu. (B) Kami memerlukan 10 (sepuluh) buah pos pegawai. (C) Untuk keperluan itu, kami harus menyiapkan uang 1.000 an. (D) Dua ratus lima puluh orang diundang pada pertemuan malam itu. (E) Anna menonton drama itu sampai tiga kali. 08. Penggunaan tanda baca yang salah terdapat dalam kalimat .... (A) Membaca buku hendaklah di tempat yang terang. (B) Dia tidak dapat membaca buku, karena lampu padam. (C) Supaya mata tetap sehat, membaca buku di tempat yang terang. (D) Bacalah buku ini di tempat yang terang. (E) Rina, guru adik, sakit.

70

09. Makna reduplikasi pada kata mobil-mobilan dalam kalimat Ketika kecil, kami sering membuat mobil-mobilan dari kulit jeruk bali, terdapat pula pada kalimat .... (A) Mereka memiliki kebun buah-buahan yang sangat luas. (B) Suara tembak-tembakan terus terdengar selama sepuluh menit. (C) Cincin ini kuberikan sebagai kenangkenangan untukmu. (D) Kami terkejut melihat orang-orangan untukmu. (E) kalian harus menguasai tari-tarian dengan sempurna. 10. Fungsi afiksasi yang terdapat pada kata yang menutupi dalam kalimat Asap tebal menutupi berbagai wilayah Indonesia sama dengan fungsi afiksasi pada kata yang bercetak miring dalam kalimat berikut .... (A) Perempuan muda itu telah meracuni suaminya sendiri. (B) Ayahnya menulisi buku agendanya dengan berbagai acara setiap hari. (C) Masyarakat memukuli perampok itu sampai babak belur. (D) Keluarga Sultan akan segera menempati rumah baru mereka. (E) Dosen itu membumbui kuliahnya dengan berbagai cerita. 11. Fungsi ber-an pada kata bertabrakan dalam kalimat Pagi tadi ada mobil bertabrakan sama dengan ber-an pada kalimat .... (A) Orang tua itu terkenal berpandangan luas. (B) Anak-anak sekolah berpakaian seragam. (C) Kalau tidak berhalangan pasti saya akan datang. (D) Anak-anak berlarian keluar. (E) Kami bersalaman sebagai tanda berbaikan

Xpedia Bahasa Indonesia, Kapita Selekta – Set 08

12. Kalimat-kalimat di bawah ini mempunyai pola kalimat sama, KECUALI .... (A) Anton menarik lengan saya seraya menunjuk ke sebuah mobil VW yang sedang diperbaiki. (B) Peristiwa itu terjadi sewaktu keluargaku sedang dalam suasana berkabung. (C) Aku lebih bergembira sejak sikap ibu padaku berubah. (D) Ia baru kembali ke desa setelah biaya untuk melanjutkan sekolahnya tidak ada. (E) Aku melompat dari anak tangga dan kemudian berlari ke halaman. 13. Kata-kata yang bersinonim adalah .... (A) mawas diri - introspeksi (B) egoistis - tinggi hati (C) optimistis - berpandangan ke depan (D) efektif - berhasil cepat (E) umpan balik - balik tanya 14. Peribahasa Laksana apung dipermainkan gelombang mengungkapkan makna .... (A) keadaan yang sangat menyedihkan. (B) mengharapkan sesuatu yang tidak kunjung datang (C) perkara yang tidak dapat diselesaikan (D) mendapatkan situasi yang tidak menentu (E) mengharapkan keuntungan yang besar 15. Kalimat-kalimat berikut berupa fakta adalah .... (A) Sepekan ini, di tengah hiruk-pikuk kampanye pemilu, wabah demam berdarah dengue (DBD) seperti terlupakan. (B) Sebagian di antara mereka masih dirawat di lorong-lorong rumah sakit. (C) Yang masih dirawat di rumah sakit se-DKI Jakarta mencapai 1.830 orang dan 296 orang di antaranya pasien yang masuk hari Rabu. (D) Korban yang meninggal di DKI sampai Selasa lalu mencapai 72 orang. (E) Hingga Minggu lalu korban meninggal di seluruh Indonesia mencapai 389 orang.

16. Wawasan Nusantara tidak hanya bertujuan untuk mewujudkan kesejahteraan bagi bangsa Indonesia saja, tetapi juga ikut serta dalam mewujudkan kebahagiaan bagi seluruh umat manusia. Penggunaan kata pada kalimat ini dapat dihemat dengan menghilangkan .... (A) untuk dan saja (B) untuk dan tetapi (C) untuk, saja, dan tetapi (D) untuk, saja, dan bagi (E) untuk, tetapi dan bagi 17. Berbagai musibah menimpa negeri kita secara berturut-turut. Kosa kata yang tepat untuk menyatakan perasaan tertentu akibat musibah tersebut .... (A) Saya kecewa (B) Saya prihatin (C) Saya menyesal (D) Saya Bingung (E) Saya jengkel 18. Kalimat yang menggunakan ragam bahasa baku adalah .... (A) Semua peserta daripada pertemuan itu sudah pada hadir. (B) Kami mengucapkan terima kasih atas kehadiran Saudara. (C) Bilang dulu dong, sama saya punya istri. (D) Sebelum mengarang, terlebih dahulu tentukanlah tema karangan. (E) Kami menghaturkan terima kasih atas kehadirannya. 19. Akhir-akhir ini, tidak sedikit buruh berbagai perusahaan yang dibeerhentikan. Tidak aneh jika kerusuhan mudah sekali muncul. Konjungsi yang tidak tepat untuk menggabungkan kalimat di atas ialah .... (A) dengan demikian (B) jadi (C) oleh karena (D) sehingga (E) ketika

71

Xpedia Bahasa Indonesia, Kapita Selekta – Set 08

20. Semua penderita jantung tidak boleh bekerja keras. Cicilia penderita jantung. Jadi, Cicilia tidak boleh bekerja keras. Cara penalaran untuk tiba pada simpulan seperti di atas disebut .... (A) analogi (B) generalisasi (C) sebab-akibat (D) akibat-sebab (E) silogisme negatif 21. Kalimat yang terdapat dalam sebuah laporan ilmiah adalah .... (A) Perjalanan bisnis Anda akan menyenangkan bila anda menggunakan jasa kami. (B) Kulit halus, lembut, dan sehat berkat aloevera. (C) Harimau dan gajah sumatera menuju kepunahan. (D) Akhirnya, kami berhasil menemukan anak yang tersesat itu. (E) Habitat terumbu karang di perairan Indonesia Timur semakin menghawatirkan. Rasa sakit pada lambung dan usus halus dapat dicetuskan oleh faktor psikologis seperti perasaan tegang atau stres. Pada beberapa penderita, keluhannya hanya terasa di sekitar lambung dan usus halus. Namun, dalam banyak kasus, ganguannya dapat terasa pada seluruh proses pencernaan. Dalam banyak hal, pola makan serasi dapat untuk memperbaiki proses pencernaan penderita, tetapi tidak untuk gangguan psikologisnya. Jalan keluar yang memungkinkan adalah mengombinasikan makanan serasi dengana terapi relaksasi secara tepat. Misalnya, pijat refleksi untuk mengendurkan ketegangan pada lambung dan usus halus yang secara tidak langsung juga membantu memperbaiki sistem saraf. 22. Simpulan yang dapat ditarik dari paragraf itu adalah .... (A) Rasa sakit pada lambung disebabkan oleh perasaan tegang atau stres. (B) Rasa sakit pada lambung dapat diatasi dengan terapi makanan sehat dan pijat rerfleksi. (C) Rasa sakit pada lambung disebabkan oleh borok lambung atau ketegangan. (D) Kita harus mengubah pola makanan dengan pola makanan serasi. (E) Hindarilah perasaan tegang dan stress. 72

23. Bentuk kalimat-kalimat di bawah ini mengalami kerancuan, KECUALI .... (A) Mereka mengenyampingkan pertimbangan psikologis, tetapi lebih mempertimbangkan hal yang bersifat pragmatis. (B) Berulang kali anak itu dinasihati ibunya, tetapi sampai kini tidak ada perubahannya. (C) Secara teoretis, roti dan keju adalah makanan yang tidak serasi. (D) Polisi menyuruh sopir bus itu memperlambatkan kendaraannya dan diparkir di bahu jalan. (E) Masalah ini pernah dipelajarkan pada semester ganjil tahun yang lalu. 24. “Semakin besar orang memiliki kesempatan untuk bertindak, semakin besar pula godaan untuk berbuat korupsi.” Pernyataan di atas mengandung makna yang sama dengan .... (A) Semakin besar wewenang yang dimiliki oleh seseorang, semakin besar pula kemungkinan untuk menyalahgunakan wewenang tersebut. (B) Semakin besar peluang untuk melakukan korupsi, semakin besar pula kemungkinan korupsi yang dilakukan. (C) Semakin banyak orang bertindak, semakin banyak orang tergoda. (D) Peluang dan kesempatan merupakan suatu godaaan (E) Godaan akan menjadi besar apabila peluangnya diperbesar. 25. Membengkokluruskan nilai, hati nurani, dan kebenaran, merupakan bentuk korupsi nonfinansial. Tindakan berikut yang jelas-jelas merupakan korupsi finansial adalah .... (A) komersialisasi jabatan (B) mengurangi jumlah jam kerja (C) membuat kuitansi palsu (D) meniggalkan tanggung jawab (E) memanfaatkan fasilitas kantor untuk kepentingan pribadi

Xpedia Bahasa Indonesia Kapita Selekta – Set 09

Revolusi di bidang teknologi informasi menghasilkan berbagai produk dan program baru di pasar dunia. Revolusi itu mengubah berbagai segi kehidupan masyarakat. Teknologi internet, CD Rom, dan jaringan televisi internet yang mampu menerobos batas kehidupan pribadi sangat mempengaruhi perkembangan pariwisata. Pemasaran pariwisata berupa penawaran produk dan harga, misalnya, lebih mudah dilakukan melalui internet yang dihubungkan ke jaringan komputer pelanggan di seluruh dunia. Calon konsumen juga lebih mudah memilih produk wisata yang diinginkan dan disesuaikan dengan kemampuan keuangannya. Ravolusi teknologi serupa ini akan mengubah prinsip pariwisata dan membuat pembaruan atas pola yang berlangsung selama ini. Pasar yang semula bersifat massal akan berubah menjadi lebih khusus dan selektif. 01. Pikiran utama dalam paragraf pertama adalah .... (A) revolusi di bidang informatika (B) perubahan dalam segala segi kehidupan (C) masalah dalam kehidupan pribadi (D) masalah jaringan telekomunikasi (E) program baru di pasar dunia

Revolusi di bidang teknologi informasi menghasilkan berbagai produk dan program baru di pasar dunia. Revolusi itu mengubah berbagai segi kehidupan masyarakat. Teknologi internet, CD Rom, dan jaringan televisi internet yang mampu menerobos batas kehidupan pribadi sangat mempengaruhi perkembangan pariwisata. Pemasaran pariwisata berupa penawaran produk dan harga, misalnya, lebih mudah dilakukan melalui internet yang dihubungkan ke jaringan komputer pelanggan di seluruh dunia. Calon konsumen juga lebih mudah memilih produk wisata yang diinginkan dan disesuaikan dengan kemampuan keuangannya. Ravolusi teknologi serupa ini akan mengubah prinsip pariwisata dan membuat pembaruan atas pola yang berlangsung selama ini. Pasar yang semula bersifat massal akan berubah menjadi lebih khusus dan selektif. 02. Semula, hal berikut dibicarakan dalam kutipan di atas, KECUALI .... (A) perubahan seluruh kehidupan pribadi masyarakat (B) revolusi teknologi (C) pengaruh perubahan teknologi informasi (D) perkembangan dalam pasaran pariwisata (E) pasaran pariwisata menjadi selektif 03. Yang dimaksud dengan revolusi adalah .... (A) perubahan tiba-tiba (B) perubahan yang bertahap (C) perubahan radikal (D) perubahan yang mendasar (E) peninjauan kembali

73

Xpedia Bahasa Indonesia, Kapita Selekta – Set 09

04. Pemakaian tanda baca yang betul terdapat pada kalimat .... (A) Mereka ingin membangun masyarakat tetapi perangkat teknologinya kurang lengkap. (B) Perhitungan kita meleset, karena tidak sesuai dengan kenyataan. (C) Tetangga saya menerima hadiah Rp 5.000.000,00. (D) Usahanya maju; anak-anaknya sudah menjadi sarjana; kehidupannya tenang dan tentram. (E) Dari segi ekonomi masyarakat di daerah ini, terbagi atas golongan ekonomi lemah, sedang, dan kuat. 05. Pemakaian ejaan yang tepat terdapat dalam kalimat .... (A) Karena seorang Laksamana, ia pasti hadir dalam pertemuan perwira tinggi di instana negara. (B) Kami baru kedatangan tamu dari Pusat, yaitu Sekretaris Jendral dari Departemen Pertanian. (C) Gubernur Ahmad baru saja dilantik menjadi Gubernur di daerah tempat kelahirannya. (D) Hassanudin, Sultan Makasar, digelari juga Ayam Jantan dari Timur. (E) Ia berkata bahwa, temannya mempunyai mobil baru. 06. Imbuhan ber- yang mengandung makna mengerjakan pekerjaan yang objeknya diri sendiri terdapat pada .... (A) Murid berusaha menyelesaikan tugasnya. (B) Ibu berbelanja di pasar. (C) Gadis itu senang berhias (D) Mereka bertinju sampai tenaganya terkuras habis (E) Saya bermaksud mengunjungi nenek

74

07. Awalan ter- yang menyatakan bahwa suatu perbuatan telah selesai dikerjakan pada kalimat .... (A) Gadis itu menangis tersedu-sedu karena teringat ayahnya yang gugur di medan laga. (B) Kaki dan tanganya terlindas mobil di jalan raya. (C) Mobil itu terperosok di parit. (D) Lampu itu terpasang sampai pagi. (E) Laki-laki dan perempuan yang sedang bertengkar itu sebenarnya masih terikat dengan perkawinan yang sah. 08. Dalam pengembangan ekonomi mikro, seharusnya UKM menjadi prioritas. Kata prioritas dalam kalimat itu bermakna .... (A) yang diutamakan (B) yang harus dipentingkan (C) yang harus didahulukan dan diutamakan (D) yang tidak boleh diabaikan (E) yang dipedulikan 09. Kakak belajar bahasa Inggris selama tiga tahun. Pola kalimat di atas sama dengan pola kalimat .... (A) Paman menjadi guru selama sepuluh tahun. (B) Mereka mempelajari fisika di Jerman. (C) Orang itu minum obat karena sakit. (D) Wati mengharapkan kedatangan kekasihnya bertahun-tahun. (E) Anak kecil itu naik kereta api dari Stasiun Bandung. 10. Adik saya tinggal di Jerman. Transformasi kalimat di atas tertera di bawah ini .... (A) Adik saya sedang belajar di luar negeri dan sekarang tinggal di Jerman. (B) Saya mempunyai adik yang tinggal di Jerman. (C) Mengapa adik saya tinggal di Jerman? (D) Sejak beberapa tahun yang lalu, adik saya yang belajar di Jerman menderita sakit. (E) Karena ia sedang belajar di luar negeri, bapak saya menyarankan supaya adik saya tinggal di Jerman.

Xpedia Bahasa Indonesia, Kapita Selekta – Set 09

11. Ungkapan yang bercetak miring pada kalimat berikut mempunyai makna yang jauh dari makna kata-kata pembentuknya, KECUALI .... (A) Perkara ini harus dibawa ke meja hijau. (B) Ia gugur sebagai kusuma bangsa. (C) Di negara demokrasi, orang bebas berunjuk gigi. (D) Hanya sekadar untuk menghidupi keluarga, ia harus membanting tulang. (E) Piala binaraga sudah menumpuk kamarnya. 12. Dengan terjadinya krisis ekonomi telah menyebabkan pendapatan per kapita turun menjadi $ 1.000.000 per tahun. Kalimat tersebut dapat dijadikan kalimat efektif dengan .... (A) mengubah menyebabkan menjadi disebabkan. (B) meletakkan pendaptan per kapita pada awal kalimt. (C) menghilangkan kata dengan (D) menghilangkan kata telah (E) meletakan dengan terjadinya krisis ekonomi pada akhir kalimat 13. Kalimat berikut yang ejaannya sesuai dengan EYD adalah .... (A) Setiap benda yang dimasukkan ke dalam zat cair akan mendapatkan tekanan ke atas. (B) Wayang Bali berukuran lebih pendek, dengan tatanan tidak terlalu banyak seperti yang biasa dijumpai dengan wayang jawa. (C) Rencana PLN untuk menaikkan tarif dasar listrik banyak ditentang masyarakat. (D) Sebagai catatan warga di kawasan pegunungan Yahukimo memang telah terbiasa memasak beras. (E) Tragisnya, Osteoporosis yang sering di sebut silent disease ini memang muncul tanpa adanya keluhan klinis pasien.

14. Tantangan pendidikan pada masa kini ialah bagaimana membuat anak untuk peka terhadap masalah di sekitarnya. Kalimat di atas dapat diperbaiki dengan menghilangkan kata .... (A) pada (B) ialah (C) untuk (D) bagaimana (E) terhadap 15. Mendengar anak sakit, Ibu Yanti menangis pilu. Bentukan kata yang berpola sama dengan frase menangis pilu adalah .... (A) belajar tekun - berdiam diri (B) membaca buku - memanjat pohon (C) membaca nyaring - bekerja keras (D) bertatap muka - menulis indah (E) bergandengan tangan - berpegang erat 16. Kelompok-kelompok kata berikut menunjukkan benda, hal atau perbuatan benda, hal atau perbuatan sejenis, KECUALI .... (A) bergurau, bercanda, berolok-olok (B) menghisap cerutu, makan nasi, minum air (C) nyeri, ngilu, sakit (D) berhidmat, mengabdi, berkurban (E) menjilat ludah, bergunjing, membual 17. Banyak bekas narapidana berhasil menjadi anggota masyarakat yang baik. sebaliknya, banyak juga yang menjadi penganggur karena mereka hanya menunggu datangnya Dewi Fortuna. Kedua kalimat itu dapat diungkapkan kembali dengan kalimat .... (A) Kita harus membuka lapangan kerja bagi bekas narapidana. (B) Nasib baik tidak selalu menghampiri para bekas penjahat itu. (C) TIdak ada gunanya menolong bekas narapidana. (D) Bekas narapidana yang mau berusaha dapat menjadi warga yang baik, yang tidak akan menjadi tuna karya. (E) Para narapidana harus disiapkan menjadi warga yang baik bila mereka nanti keluar dari penjara. 75

Xpedia Bahasa Indonesia, Kapita Selekta – Set 09

18. Kalimat yang tersusun dengan baik adalah .... (A) Barang siapa kehilangan buku supaya segera diambil di ruang guru. (B) Bagi yang menemukan dompet supaya diserahkan kepada polisi. (C) Sambutan berikutnya dari Bapak Kepala Sekolah. Waktu kami persilahkan. (D) Untuk orang yang membeli ayam supaya diikiat kakinya. (E) Meskipun mendapatkan tekanan dari berbagai pihak, tidak ingin ia mengingkari janjinya kepada almarhum istrinya. 19. Setiap kelompok kata berikut lazim dalam bidang kehidupan yang bersangkutan, KECUALI .... (A) perahu, kapal, biduk, kolek angkutan air (B) properti, saham, valuta, bank keuangan (C) palu, pahat, gergaki, kuda-kuda pertukangan (D) goreng, sangrai, rebus, bakar masakmemasak (E) dukun, dokter, bidan, perawat kesehatan 20. Setiap habis gajian, ia selalu mentraktir temantemanya untuk makan bakso di warung Bu Atika. Kata habis dalam pernyataan berikut ini yang semakna dengan kata habis dalam kalimat tersebut adalah .... (A) Semua dokumen penting yang ada di ruang administrasi habis terbakar. (B) Ia tidak bisa mudik lebaran karena tiket kereta api sudah habis. (C) Uang untuk bayar kuliah habis untuk belanja baju di matahari. (D) Habis riwayatnya terkena panah beracun. (E) Buku karangan R.A Kartini berjudul Habis Gelap Terbitlah Terang.

76

21. Mereka berperangai ekstern, menjunjung tokoh yang disenangi secara berlebihan; dan sebaliknya, memberlakukan secara buruk tokoh yang dimusuhi. Kalimat tersebut menyatakan hubungan .... (A) penjumlahan dan kelanjutan (B) pertentangan dan akibat (C) penjumlahan dan akibat (D) penjumlahan dan pertentangan (E) pertentangan dan sebab 22. Yang merupakan kalimat baku adalah .... (A) Kepada para hadirin diingatkan agar supaya tidak meninggalkan barang bawaannya di sini. (B) Yang kami telah selesai bicarakan adalah tentang masalah penyalahgunaan obat terlarang. (C) Kepada Saudara-saudara sekalian diminta dengan hormat untuk berdiri sejenak. (D) Sudah selayaknya kita selalu saling hormat-menghormati dalam pergaulan sehari-hari. (E) Cuti besar diberikan kepada yang telah bekerja enam tahun berturut-turut. 23. Bilamana pernyataan dasar umum adalah Semua planet di tata surya berputar mengelilingi matahari Dan pertanyaan dasar khusus adalah Bumi termasuk dalam tata surya Makna simpulan dari dua pernyataan itu adalah .... (A) Bumi mengelilingi matahari (B) Bumi perputar mengelilingi matahari (C) Bumi dan semua planet berputar mengelilingi matahari (D) Bumi sebagai planet termasuk dalam tata surya (E) Semua planet termasuk dalam tata surya

Xpedia Bahasa Indonesia, Kapita Selekta – Set 09

24. Bioteknologi pertanian modern dikembangkan melalui dua kegiatan dasar, dua kultur jaringan rekayasa genetika. Teknik kultur jaringan memungkinkan kita mengisolasi jaringan dan sel-sel tumbuhan, kemudian menanamkannya di luar tumbuh-tumbuhan itu sendiri. Intisari bacaan di atas .... (A) Kegiatan dasar kultur dan rekayasa genetika (B) Dua kegiatan dasar modern dikembangkan melalui bioteknologi (C) Pengisolasian jaringan dan sel-sel tumbuhan (D) Bioteknologi pertanian akan dikembangkan (E) Penanaman kultur jaringan di luar tumbuhan 25. Susunlah frase-frase di bawah ini menjadi sebuah kalimat yang baik! 1. setelah kondisi tanggap darurat 2. bagi upaya penanganan pascabencana 3. yang harus dilakukan 4. pada tahap rehabilitasi 5. merupakan langkah pertama 6. penyediaan air bersih 7. dan mendirikan tempat pengungsian Susunan yang baik adalah .... (A) 1,3,2,4,5,6,7 (B) 4,3,6,7,5,2,1 (C) 5,4,1,2,3,6,7 (D) 6,7,5,4,3,2,1 (E) 6,7,3,4,2,1,5

77

UMBUI Bahasa Indonesia Kode Soal

01. Sebagai negara kepulauan dengan 65 persen penduduk yang tinggal di wilayah pesisir, pemanasan global yang berdampak pada naiknya permukaan laut harus dilihat sebagai ancaman yang serius bagi Indonesia. Sementara itu, sebagai negara agraris, produktivitas pertanian negeri ini pun terancam gangguan tidak kalah serius akibat dampak perubahan iklim. Dari kutipan di atas dapat disimpulkan halhal berikut ini, KECUALI …. (A) Pemanasan global mengancam bidang pertanian dan kelautan di Indonesia. (B) Ancaman terbesar dari pemanasan global dihadapi oleh penduduk yang tinggal di wilayah pesisir. (C) Penduduk yang hidup dari pertanian tidak mengalami pemanasan global karena sudah mengalami masalah perubahan iklim. (D) Naiknya permukaan laut dan perubahan iklim merupakan akibat dari pemanasan global. (E) Pemanasan global mengancam Indonesia sebagai Negara kepulauan dan Negara agraris.

02. Selain dapat memberikan gambaran data secara efektif, penggunaan grafik juga dapat mempermudah penyampaian ide yang kompleks. Ciri utama grafik adalah sederhana, tetapi jelas. Grafik memberikan gambaran perbandingan atau gambaran asosiasi antara dua atau beberapa variabel. Selain itu grafik juga menggambarkan dan mengikhtisarkan hubungan antara data statistik dengan bagianbagian lain secara komprehensif, padat, singkat, dan sederhana. Grafik merupakan bentuk penyajian visual yang dipakai untuk membandingkan jumlah data pada saat-saat yang berbeda. Simpulan yang dapat diambil dari paragraf di atas adalah …. (A) Ide yang kompleks dapat disampaikan dengan singkat dan mudah melalui grafik. (B) Ciri utama grafik adalah sederhana, tetapi jelas. (C) Grafik merupakan gambaran perbandingan atau gambaran asosiasi antara dua atau lebih variabel. (D) Grafik dapat memberi gambaran lebih efektif kepada orang lain. (E) Grafik merupakan penyajian visual yang dipakai untuk membandingkan jumlah data. 03. Sebagaimana telah ditetapkan dalam GBHN. Bahwa pembangunan pendidikan dititikberatkan pada peningkatan setiap jenjang dan jenis pendidikan. Kalimat di atas dapat diperbaiki dengan menghilangkan kata …. (A) sebagaimana (B) telah (C) dalam (D) bahwa (E) pada

78

UMBUI Bahas Indonesia, Kode Soal

04. Bila ada seorang individu yang memaksakan kebenarannya pada individu lain seharusnya sistem sosial yang ada mengkritiknya. Kalau ada seorang individu yang memamerkan bahwa dia mampu melakukan apa saja dengan uangnya, seharusnya sistem sosial yang ada menyadarkannya bahwa itu belum dapat diterima. Kalau ada seorang individu yang bersedia membeli sebatang pohon tanaman hias seharga 1,2 miliar, sebenarnya sistem sosial tidak mendukungnya. Dengan begitu, sistem sosial yang ada akan menjadi semakin kuat. Rasa kebersamaan akan benar-benar dapat dirasakan oleh semua anggota masyarakat. Isi ringkas paragraf tersebut adalah …. (A) Perilaku seorang individu seharusnya memperhatikan sistem sosial masyarakatnya. (B) Sistem sosial harus menjadi satu-satunya tolak ukur dalam kehidupan bermasyarakat. (C) Sistem sosial hendaknya dapat berperan sebagai pengontrol dan pengendali perilaku individu. (D) Kepentingan individu merupakan bagian dari sistem sosial. (E) Sistem sosial merupakan pembentuk rasa kebersamaan antaranggota masyarakat. 05. Pendidikan merupakan usaha manusia untuk menumbuhkan dan mengembangkan potensi-potensi pembawaan baik jasmani maupun rohani sesuai dengan nilai-nilai yang ada dalam masyarakat dan kebudayaan. Pendidikan merupakan kebutuhan mutlak yang harus dipenuhi sepanjang hidup. Tanpa pendidikan mustahil manusia dapat hidup berkembang sejalan dengan aspirasi untuk maju, sejahtera, dan bahagia menurut konsep pandangan hidup mereka.

Generalisasi yang tepat untuk melengkapi paragraf di atas adalah …. (A) Pemaknaan pendidikan sebagai salah satu pilar pendidikan bangsa tidak terlepas dari pembatasan pemaknaan pendidikan itu sendiri. (B) Hanya melalui proses pendidikanlah, kita dapat berharap bagi kepentingan kemajuan peradaban bangsa ini. (C) Pendidikan adalah proses yang berisi berbagai macam kegiatan yang cocok bagi individu untuk kehidupan sosialnya dan membantu meneruskan adat dan budaya serta kelembagaan sosial dari generasi ke generasi. (D) Oleh karena itulah, kita harus memperhatikan definisi pendidikan yang dikemukakan para ahli. (E) Pendidikan pada dasarnya adalah usaha sadar untuk mengembangkan kepribadian dan kemampuan di dalam dan di luar sekolah dan berlangsung seumur hidup. 06. Sepanjang 971 kilometer di Pantai Utara Jawa kini dalam keadaan memprihatinkan. Bahkan telah mengalami degradasi fungsi. Khususnya ketika abrasi, reklamasi, dan pencemaran yang mengakumulasi, telah mengakibatkan rusaknya ekosistem pantai. Kata-kata yang tepat untuk menggantikan kata-kata yang bercetak tebal dalam kalimat di atas adalah …. (A) penyimpangan fungsi, pengerukan tanah pantai, lingkungan (B) kegagalan fungsi, pengambilan alih tanah pantai, komponen lingkungan (C) kemerosotan fungsi, pengerukan tanah pantai, keanekaragaman pantai (D) pengalihan fungsi, tukar guling tanah pantai, fauna flora (E) ketidaksamaan fungsi, pembangunan tanah pantai, sumber daya pantai

79

UMBUI Bahas Indonesia, Kode Soal

07. Dalam perkembangannya, perbedaan paham yang sudah mengakar pada diri setiap insan tumbuh menjadi konflik sehingga menodai pertemuan yang sudah dibangun bertahuntahun. Kalimat inti dari kalimat luas tersebut adalah …. (A) Paham menjadi konflik (B) Paham sudah mengakar (C) Perbedaan tumbuh (D) Perbedaan menodai pertemuan (E) Perbedaan sudah mengakar 08. Regulasi yang memihak konsumen dan pengembang serta pembangunan infrastruktur dan utilitas bagi perumahan diharapkan segera diwujudkan. Padanan kata yang tepat untuk kata regulasi dalam kalimat tersebut adalah …. (A) pengaturan (B) peraturan (C) aturan (D) keteraturan (E) keberaturan 09. Walaupun sudah berkali-kali diperingatkan oleh pemerintah, penduduk desa tidak mau meniggalkan daerah yang sudah dinyatakan tidak aman itu. Pernyataan yang tepat untuk menggambarkan tindakan pemerintah dalam kalimat di atas adalah …. (A) menegakkan benang basah (B) berkering air liur (C) menggantang asap (D) berlidah di lidah orang

80

(E) membuang bunga terjerat 10. Meskipun terkecil dan terletak di ujung utara Benua Afrika, Tunisia masih menyimpan banyak kekayaan, mulai dari pesisir pantai Laut Mediterania sepanjang 1.400 kilometerhingga Gurun Sahara yang kerap menjadi lokasi syuting film seperti Star Wars dan The English Patient. Inti kalimat panjang tersebut adalah …. (A) Tunisia terletak di ujung utara Benua Afrika. (B) Letak Tunisia mulai dari pesisir pantai Laut Mediterania hingga Gurun Sahara (C) Tunisia menjadi lokasi syuting film (D) Benua Afrika kecil (E) Tunisia menyimpan kekayaan 11. Kalimat yang merupakan kalimat baku adalah …. (A) Ternyata jamu tidak hanya terkenal di Indonesia saja, tetapi di negara-negara lain pun mengkonsumsi jamu. (B) Untuk negara Asia di antaranya ada Cina dan Thailand, sedangkan untuk benua Eropa, jamu banyak ditemukan di Eropa Barat dan Jerman. (C) Sedangkan omset produksi industri jamu Indonesia di dunia berjumlah 300 juta dolar AS. (D) Penurunan nilai ekspor terjadi karena adanya isu bahwa pada obat-obatan herbal terdapat kandungan bahan kimia berbahaya. (E) Dan masalah kualitas jamu yang mengandung campuran kimia kini dipermasalahkan oleh negara importer karena ada yang meninggal setelah mengkon-

UMBUI Bahas Indonesia, Kode Soal

12. Sejak lahirnya konsep pemikiran baru dalam ilmu kedokteran, yang dicetuskan oleh Profesor Linus Pauling, yakni tentang Ortomolecular medicine yang dasarnya adalah studi biologi molekuler sebagai sains dasar, penelitian medis diarahkan pada molekul-molekul yang secara normal biologis fisiologis ada dalam tubuh manusia. Inti kalimat panjang tersebut adalah …. (A) konsep pemikiran baru dicetuskan oleh Profesor Linus Pauling. (B) Ortomolecular medicine adalah sains dasar. (C) Ortomolecular medicine dasarnya adalah studi biologi. (D) penelitian medis diarahkan pada molekul. (E) biologi molekuler merupakan sains dasar. 13. Di antara kalimat berikut, yang merupakan kalimat baku adalah …. (A) Untuk membangun kampung wisata, maka masyarakat harus menjadi aktor kunci atau pemrakasa. (B) Untuk memaknai kehidupan, peradaban manusia selalu ditandai dengan perubahan. (C) Klein mengajak Talia dan Francesca berlibur ke Panama, dimana konglomerat itu memilik villa. (D) Bersahabat dengan teknologi di era informasi, memang menjadi sebuah tuntutan. (E) Musim penghujan, harga sejumlah komoditi sayuran di beberapa daerah telarang. 14. Ada ribuan TKI yang terpaksa dipulangkan dari Malaysia karena mereka masuk ke negara itu secara illegal. Kalimat di atas berasal dari kalimat inti …. (A) Ribuan TKI dipulangkan (B) Ribuan TKI masuk ke Malaysia (C) Ada ribuan TKI (D) Ribuan TKI terpaksa (E) Ribuan TKI ilegal

15. Ikatan Komoditas Literasi Indonesia (IKLI) membuat sistem jaringan informasi di antara sesama komunitas sehingga kelak akan ada pertukaran informasi seputar metode kegiatan dan jaringan bantuan serta memperkuat posisi tawar komunitas ke berbagai pihak. Kalimat inti dari kalimat luas di atas adalah …. (A) Ikatan Komunitas Literasi Indonesia membuat sistem jaringan informasi. (B) Ikatan Komunitas Literasi Indonesia ada pertukaran informasi. (C) Ikatan Komunitas Literasi Indonesia memperkuat posisi. (D) Ikatan Komunitas Literasi Indonesiamemperkuat komunitas. (E) Pertukaran informasi memperkuat posisi tawar komunitas. 16. Kalimat berikut ini yang termasuk kalimat baku adalah …. (A) Sarana dan prasarana yang disediakan oleh pihak sekolah itu untuk memfasilitasi siswa yang aktif dan kreatif agar dapat mengembangkan minat dan bakat mereka. (B) Pada minggu kedua pertemuan kita akan membicarakan tentang pola-pola sinkretisme dan akulturasi budaya yang terjadi di nusantara. (C) Jika disampaikan secara lebih sistematis dengan ilustrasi yang memadai, maka brosur ini akan memberikan sugesti yang lebih kuat pada pembacanya. (D) Sebelum presiden memberikan kesempatan kepada hadirin untuk bertanya, maka terlebih dahulu meminta seorang stafnya memandu jalannya Temu Wicara. (E) Kalau dibandingkan dengan situasi politik masa lalu, praktik KKN, yakni kecenderungan perilaku korupsi, kolusi, dan nepotisme oleh jajaran birokrasi, saat ini, mungkin sudah sangat jauh berkurang.

81

UMBUI Bahas Indonesia, Kode Soal

17. Tindakan anarkis itu sebagai suatu manifestasi kemarahan hatinya. Padanan kata manifestasi yang TIDAK tepat adalah …. (A) pengejawantahan. (B) perwujudan. (C) pengungkapan. (D) perlakuan. (E) pernyataan. 18. Penulisan bilangan dalam kalimat yang sesuai dengan aturan EYD adalah …. (A) Anda dapat membeli buku ini seharga Rp 50.000 di toko saya. (B) Sebanyak sembilan puluh enam kelurahan di Jakarta rawan demam berdarah dengue (DBD). (C) Pinjaman Asosiasi Pembangunan Internasional biasanya diberikan dengan jangka waktu 35 tahun, masa tenggang sepuluh tahun, dan tingkat suku bunga tetap 0,75% per tahun. (D) Ratusan orang yang berunjuk rasa mengaku perwakilan warga Dayak dari 13 kabupaten dan kota di Kalimantan Timur. (E) Harga kedelai impor naik sejak pekan lalu dari Rp 6.500 per kilogram menjadi Rp 7.000. 19. Kata bercetak miring yang huruf awalnya tidak perlu ditulis dengan huruf kapital terdapat dalam kalimat …. (A) Sampah menumpuk di Pintu Air Manggarai, Jakarta Selatan, setelah mendapat banjir kiriman dari Bogor. (B) Warga Kampung Pulo, Kelurahan Kampung Melayu, Jakarta Timur, menyingkirkan berbagai barang yang terapung di dapur rumah mereka. (C) Antrean truk pengangkut barang terjadi akibat salah satu dermaga di Pelabuhan Bakauheni, Lampung, rusak. (D) Pemerintah Provinsi Daerah Khusus Ibu Kota Jakarta menguji coba layanan perpanjangan KTP melalui telepon genggam. (E) Alat berat dikerahkan untuk mengeruk badan Sungai Krukut di Daerah Penjernihan, Jakarta Selatan. 82

20. Penggunaan tanda koma (,) yang salah dijumpai dalam kalimat …. (A) Sebetulnya, pembangunan rumah sehat sederhana memiliki multi player effect yang lebih luas. (B) Sasaran pelatihan adalah para pimpinan perusahaan, bagian promosi public relations, humas, dan bagian lain yang memiliki kompetensi dengan publik. (C) Hanan mengharapkan, agar banyak pengembang ikut berperan dalam pembangunan rumah sehat sederhana. (D) Meskipun untung yang diperoleh tidak cukup besar, pembangunan rumah sehat sederhana ini memiliki manfaat yang lebih besar. (E) Harga ikan di tempat pelelangan ikan Tanggulangin, Kecamatan Klirong, Kabupaten Kebumen, dikeluhkan nelayan setempat. 21. Kata serapan yang salah penulisannya terdapat dalam kalimat …. (A) Pembangunan fisik tanpa pembangunan mental seumpama bangunan tanpa pondamen. (B) Basket adalah olahraga favoritnya. (C) Perjanjian antara dua pihak disebut perjanjian bilateral. (D) Gadis itu menjadi sangat ideal untuk menjadi peragawati. (E) Aku jarang pergi ke bioskop.

UMBUI Bahas Indonesia, Kode Soal

22. Penulisan kalimat berikut ini mengikuti aturan EYD, KECUALI …. (A) Buku ini disusun untuk membantu mahasiswa yang akan mengambil mata kuliah Akuntansi Perusahaan pada fakultas ekonomi. (B) Kami menulis diktat ini dengan maksud agar mahasiswa lebih mudah mengikuti kuliah tentang ekonomi Pembangunan. (C) Akuntansi terbagi dalam beberapa bidang sebagai berikut: akuntansi sosial, akuntansi pemerintah, akuntansi perusahaan, dan akuntansi organisasi nonprofit. (D) Diharapkan diktat yang berjudul Akuntansi Sosial ini bermanfaat bagi para peminat akuntansi pada umumnya dan mahasiswa sekolah tinggi ilmu ekonomi pada khususnya. (E) Pengelasan dengan asetilena dilakukan dengan cara membakar bahan bakar gas asetilena dengan O2.

25. Ombak tinggi dan angin kencang kembali …. penyeberangan antarpulau, Merak Bakauheni. Cuaca buruk yang kerap …. Selat Sunda satu bulan terakhir membuat perusahaan kapal cepat khusus penumpang …. hingga miliaran rupiah dan …. gulung tikar. Kata-kata yang tepat untuk mengisi rumpang dalam kalimat di atas adalah …. (A) mengganggu, melanda, merugi, terancam (B) diganggu, dilanda, rugi, mengancam (C) terganggu, terlanda, kerugian, diancam (D) gangguan, melanda, kerugian, ancaman (E) mengganggu, terlanda, dirugikan, mengancam

23. Penulisan kata serapan yang sesuai dengan aturan EYD adalah …. (A) Perlu adanya standarisasi dalam mutu produksi sabun. (B) Agaknya telah terjadi marginalisasi peran terhadap kelompok tertentu. (C) Perubahan yang positif sangat diharapkan entah dalam bentuk material ataupun non material (D) Keterampilan managerial yang tinggi sangat diperlukan bagi setiap pemimpin. (E) Respon masyarakat terhadap rencana perbaikan kampung sangat baik. 24. Anak yang haus kasih sayang biasanya akan mencari perhatian gurunya. Proses pembentukan kata yang sama dengan kasih sayang terdapat dalam kalimat …. (A) Cucunya dibelainya dengan penuh kasih mesra. (B) Wanita itu mencintai darah dagingnya sendiri. (C) Dengan gagah berani ia menghadapi musuh-musuhnya. (D) Kamu berdoa untuk keluarga kami di kampung halaman. (E) Ia mengungkapkan perasaannya dengan terus terang. 83

SBMPTN 2018 Bahasa Inggris Soal SBMPTN 2018 - Bahasa Inggris

Education includes both the teaching and learning of knowledge, proper conduct, and technical competency. It thus focuses on the cultivation of skills, trades or professions, as well as mental, moral, and aesthetic development. Formal education consists of systematic instruction, teaching, and training by professional teachers. This consists of the application of pedagogy and the development of curricula. Educational systems are established to provide education and training, often for children and the young. A curriculum defines what students should know, understand, and be able to do as the result of education. A teaching profession delivers teaching which enables learning, and a system of policies, regulations, examinations, structures, and funding enables teachers to teach to the best of their abilities. Sometimes educational systems can be used to promote doctrines or ideals as well as knowledge, which is known as social engineering. This can lead to political abuse of the system, particularly in totalitarian states and governments. Primary (or elementary) education consists of the first years of formal, structured education. In general, primary education consists of six or seven years of schooling starting at the age of 5 or 6, although this varies between, and sometimes within, countries. Globally, around 70% of primary-age children are enrolled in primary education and this proportion is rising. In most contemporary educational systems of the world, secondary education consists of the second years of formal education that occur during adolescence. It is characterized by transition from the typically compulsory, comprehensive primary education for minors, to the optional, selective tertiary, or "higher" education (e.g., university, vocational school) for adults. (Adapted from: http:// www.myenglishpages.com/site_php_files/ reading_education.php)

84

01. What does the passage mainly deal with? (A) formal education (B) varieties of education (C) curriculum development (D) modern education system (E) pedagogy transformation 02. Based on the passage, the word “conduct” in line 1 means …. (A) manner (B) model (C) control (D) principle (E) management 03. One function of education which can be wrongly practiced is referred to as ... (A) social engineering (B) higher education (C) political abuse (D) policy making (E) primary education

SBMPTN 2018 Bahasa Inggris, Soal SBMPTN 2018 - Bahasa Inggris

The integration of technology and media can enhance early childhood practice. Successful integration of technology and media into early childhood programs involves the use of resources such as computers and the internet in daily classroom practices. True integration occurs when the use of technology and media becomes routine and transparent — when the focus of a child or educator is on the activity or exploration itself and not on the technology or media being used. Technology integration has been successful when the use of technology and media supports the goals of educators and programs for children provides children with digital tools for learning and communicating and helps improve child outcomes. As the lives of children, parents, families, and educators are infused with technology and media, early childhood classrooms can benefit from the possibilities of extending children’s learning through judicious use of these tools. As part of the overall classroom plan, technology and interactive media should be used in ways that support existing classroom developmental and educational goals rather than in ways that distort or replace them. For example, drawing on a touch screen can add to children’s graphic representational experiences; manipulating colorful acetate shapes on a light table allows children to explore color and shape. By focusing on technology and interactive media as tools—not as ends in and of themselves—teachers can avoid the passive and potentially harmful use of non-interactive, linear screen media that is inappropriate in early childhood settings. Intentionality is a key to developmentally appropriate use. Exciting new resources in today’s technology-rich world represent the next frontier in digital learning for our youngest citizens, leaving it to talented educators and caring adults to determine how best to leverage each new technology as an opportunity for children’s learning in ways that are developmentally appropriate. (Adapted from: https://issuu.com/naeyc/docs ps_technology_issuu_may2012)

04. How does the first sentence relate to the other sentences in paragraph 1? (A) the other sentences in paragraph 1 explain further the kinds of integration of technology and media that can be of benefits to young learners (B) the other sentences in paragraph 1 explain the impact of the integration of technology and media in early education explained in the first sentence (C) the other sentences in paragraph 1 are examples of the use of technology in education (D) the first sentence contains information that is contrary to the information in the other sentences (E) the other sentences vaguely support part of the statement in the first sentence 05. The author would apparently agree that .... (A) successful integration of technology and media is determined by the quality of the computers and the connection of the internet (B) the integration of technology and media occurs when the teacher focuses only on the technology itself (C) the successful integration of technology and media in early childhood education really depends on good educators. (D) educators should be selective in choosing educational goals to be integrated with the use of technology and media. (E) both passive and interactive uses of technology and media are needed as a key to successful learning in early childhood settings.

85

SBMPTN 2018 Bahasa Inggris, Soal SBMPTN 2018 - Bahasa Inggris

06. The author holds an assumption that …. (A) the interactive media as a learning tool should be used to replace the existing classroom practice (B) the successful integration of new technology in education requires helpful and good teachers (C) the benefits of technology and media childhood classrooms can be achieved by decreasing the use of those tools in teaching and learning (D) manipulating the colorful shapes to explore color and shapes using technology is the best way to develop interactive learning (E) the use of paints, markers, crayons, and other graphic art materials should be replaced by the integration of technology and media 07. Which paragraph(s) explain(s) the importance of using technology and media wisely so that it can support existing classroom practice? (A) 1 (B) 2 and 3 (C) 2 and 4 (D) 3 and 4 (E) 4 The green movement is catching on in many pockets of the world. This is especially true in the construction industry. Today’s buzz Words, which include global warming and zero emission, are causing everyday people to look for ways to reduce their carbon footprint. Purchasing environmentally-friendly property is a good investment for those who are concerned about their own health and the well-being of the earth. Based on this trend, entire districts, known as eco-communities, are being designed with green initiatives in mind. Dockside Green in Victoria, British Columbia, Canada is one of these communities. Its goal is to become the world’s first zero-emissions neighborhood. Builders of Dockside Green have the environment in mind with every choice they make. They ensure proper ventilation, and 86

guarantee residents 100% fresh indoor air. Interior and exterior building materials, such as paints and wood, are natural and non-toxic. Eco-conscious builders use bamboo wherever possible because it is durable and does not require pesticides to grow. Energy efficiency is one of the top priorities in eco-communities, such as Dockside Green. Not only do energy-efficient appliances and light fixtures reduce the environmental impact of heating and hot water, they also save residents and business owners money. Dockside Green claims that home owners will use 55% less energy than average residents in Canada. Though they are sharing space by investing in condo-style living, residents will have individual utility metres. Studies show that people use approximately 20% less energy when they are billed for exactly what they use. In addition, water is treated at Dockside Green and reused on site for flushing toilets. Planners of eco-communities such as Dockside Green must take the future into account. Dockside Green will reuse 90% of its construction waste. They also plan to continue using local suppliers for all of their transport and maintenance needs. This is a great way to reduce emissions. Dockside residents will be encouraged to make use of a mini transit system and buy into the community’s car share program. Finally, plans are in the works for a high-tech heating system that will use renewable biomass instead of fossil fuels. (Adapted from: http://tx.engiish-ch.com/ teacher/lyn/level-c/ecocommunities-docksidegreen/) 08. By saying “Today’s buzz words. which include global warming and zero emissions, are causing everyday people to look for ways to reduce their carbon footprint,” in lines 1-3 the author implies that? (A) environmental issues have made people more cautious of their actions (B) global warming and zero emissions become everyone 's concern (C) people have to take care of their greenhouses carefully (D) eco-friendly environmental programs should be strongly enforced (E) all people around the world are aware of global warming

SBMPTN 2018 Bahasa Inggris, Soal SBMPTN 2018 - Bahasa Inggris

09. Which of the following best restates the sentence “The green movement is catching on in many pockets of the world.” in line 1? (A) many people consider it crucial to have eco-friendly environments (B) people who are aware of environmental issues love safe and healthy surroundings (C) house designers should consider the importance of green materials (D) everyone should understand the effect of global warming and zero emissions well (E) environmentalists are concerned with the green environment 10. The false idea that the author may hold about Dockside Green is that …. (A) it provides all eco-friendly properties (B) it is a good asset to buy a property in it (C) it encourages all houses to utilize efficient energy (D) it reduces global warming and emissions (E) it is where more natural livings can exist 11. The author organizes the ideas in the passage by …. (A) describing a new housing trend and providing a detailed example (B) discussing an environmental issue and providing ways to solve the issue (C) identifying a community problem and discussing possible solutions to the problem (D) explaining an environmentally friendly initiative and discussing the impacts of the initiative (E) establishing a new community initiative that is eco-friendly and explaining the ideas behind the initiative The explanation shows that life condition is becoming more and more uncondusive for good -quality life. Therefore, today more people than ever are looking for ways to improve their health, increase energy, reduce stress, restore or enhance functionality, relieve aches and pains, balance emotions, and sharpen mental focus. Because of the demands of modern life, most people want those ways to be relatively simple and not time consuming. If you still have not found something to fit your needs, you may want to consider

Chinese healing exercises. Chinese healing exercises make up a branch of Chinese medicine. It is almost never taught as a separate system of healing. Rather, it is used in many practices you probably recognize. Some are part of the oldest of Chinese medical practices, including acupressure and tuina, a type of Chinese massage therapy. Others have their origins in ancient self-healing or spiritual practices, such as Chinese forms of yoga and meditation. They are routinely taught together with more detailed and complex practices, such as taiji (t’ai chi) qigong (chi gung), and other advanced medical, spiritual, or martial practices. They are intended to prepare a student for those more demanding disciplines. They are also conducted as adjunctive exercises that can make some aspects of the main practices easier to perform. Some sources are less well known in the west, like medical qigong and paidagong. They are tapping and patting techniques used to break up qi blockages. Qi is life force. and is responsible for all healthy functionality, animation, vitality, emotional balance, and mental clarity. Chinese exercises have a unique positive impact on all those qualities. Chinese healing exercises can amplify the benefits from taiji or qigong if you already have such a practice. In that context, they will open targeted areas of physical or energetic restriction, deepen your sensitivity to qi sensations, and increase qi flow. They root you more securely, thereby improving your overall performance. (Adapted from: http:(/www.llewellyn.com/ journal/article/2408) 12. The author’s attitude towards the topic in the passage is …. (A) critical (B) positive (C) skeptical (D) doubtful (E) realistic

87

SBMPTN 2018 Bahasa Inggris, Soal SBMPTN 2018 - Bahasa Inggris

13. What topic does the paragraph preceding the passage most likely discuss? (A) good-quality life (B) health improvements (C) sharpening mental condition (D) ways to increase body energy (E) adverse life condition 14. What is the best summary of the passage? (A) people are encouraged to do Chinese healing exercises because they provide many benefits for people’s life and cost very low. Moreover, these exercises do not require special equipments, sports gear, and apparel (B) while some types of Chinese healing exercises have been considered useful, some others still need further researchbased justification. These exercises tend to be more detailed and complex, thus they demand much higher skills from people who want to practice them (C) despite many benefits provided by Chinese healing exercises, they are not so popular among people seeking for alternative medical treatments. This is because they are not aware of the advantages those exercises have (D) Chinese healing exercises can be a good option for those who want to improve their quality of life. These exercises include acupressure, tuina, taiji, qigong, and other medical, spiritual, and selfdefense practices, which may promote overall functionality of person’s wellbeing (E) Chinese healing exercises are becoming more and more popular as many benefits are associated with them. Furthermore, the variations in the alternative treatment are quite interesting and complex

88

15. It can be predicted from the passage that …. (A) the more people do Chinese healing exercises, the longer the time they need to master other advanced medical, spiritual, or martial practices (B) the more people do Chinese advanced medical, spiritual, or martial practices; the less they need to do Chinese healing exercise (C) the more do Chinese healing exercises, the worse they will be in other advanced medical, spiritual, or martial practices (D) the more people do Chinese healing exercises, the their overall health will be (E) the more people do Chinese healing exercises, the more simple their life will be

SBMPTN 2018 Bahasa Inggris Soal SBMPTN 2018 - Bahasa Inggris paket 2

Nowadays, our lives are practically dependent on the Information Communications Technolog y (ICT). Data from the Communications and Information Ministry shows that Internet users in Indonesia increased from 74 million people in 2013 to 111 million in 2014. In Indonesia, Internet burst has been mainly used for accessing social media. Indonesia ranked third among countries with the highest number of Facebook users, below the U.S and India. The popularity of the Internet and social media unfortunately has not extended to the education sector. ICT has not become a backbone of improving the country’s competitiveness through education. The Ministry of Education and Culture recorded that only about 50 percent of the 234,919 primary and secondary schools in Indonesia had access to the Internet in mid 2014. Advancement of ICT should ideally be able to revolutionize education. Technology brings new sources of learning beyond teachers. Moreover, the divide between students and subjects is further narrowed by the availability of educational content through ICT. A revolution in education is also possible through ICT as students can learn at the appropriate speed according to their capacity. Interactive digital content allows students to pick particular topics that they want to explore more. In a nutshell, there is a democratization of the learning process. Unfortunately, it is still a long road to revolutionize education in Indonesia through ICT. In addition to building technology infrastructure across the country’s island, ICT literacy for teachers, parents and students is also of importance. Without it, we are constrained from reaping the full benefits of ICT in education.

1. What does the passage mainly deal with? (A) the access to ICT for Indonesian students (B) the positive impact of ICT on modern life (C) the use of ICT in education (D) the benefits of ICT for life (E) ICT and modern life 2. Based on the passage, the word burst in first paragraph means .... (A) penetration (B) connection (C) presence (D) benefit (E) device 3. What word in the passage gives the idea that technology provides students with freedom to use varied learning resources? (A) democratization (B) competitiveness (C) revolution (D) advancement (E) importance

(Adapted from: http://www.thejakartapost.com/ news/2015/08/03/new-old-trend-education . html)

89

SBMPTN 2018 Bahasa Inggris, Soal SBMPTN 2018 - Bahasa Inggris Paket 2

The integration of technology and media can enhance early childhood practice. Successful integration of technology, and media into early childhood programs involves the use of resources such as computers and the Internet in daily classroom practices. True integration occurs when the use of technology and media becomes routine and transparent—when the focus of a child or educator is on the activity or exploration itself and not on the technology or media being used. Technology integration has been successful when the use of technology and media supports the goals of educators and programs for children provides children with digital tools for learning and communicating and helps improve child outcomes. As the life of children, parents, families, and educators are infused with technology and media, early childhood classrooms can benefit from the possibilities of extending children’s learning through judicious use of these tools. As part of the overall classroom plan, technology and interactive media should be used in ways that support existing classroom developmental and educational goals rather than in ways that distort or replace them. For example, drawing on a touch screen can add to children’s graphic representational experiences; manipulating colorful acetate shapes on a light table allows children to explore color and shape. By focusing on technology and interactive media as tools—not as ends in and of themselves—teachers can avoid the passive and potentially harmful use of non-interactive, linear screen media that is inappropriate in early childhood settings. Intentionality is a key to developmentally appropriate use. Exciting new resources in today’s technology-rich world represent the next frontier in digital learning for our youngest citizens, leaving it to talented educators and caring adults to determine how best to leverage each new technology as an opportunity for children’s learning in ways that are developmentally appropriate. (Adapted from: https://issuu.com/naeyc/docs/ ps_technology_issuu_may20l2)

90

4. How does the first sentence relate to the other sentences in paragraph 1? (A) the other sentences in paragraph 1 explain further the kinds of integration of technology and media that can be of benefits to young learners (B) the other sentences in paragraph 1 explain the impact of the integration of technology and media in early education explained in the first sentence (C) the other sentences in paragraph 1 are examples of the use of technology in education (D) the first sentence contains information that is contrary to the information in the other sentences (E) the other sentences vaguely support part of the statement in the first sentence 5. The author would apparently agree that .... (A) successful integration of technology and media is determined by the quality of the computers and the connection of the Internet (B) the integration of technology and media occurs when the teacher focuses only on the technology itself (C) the successful integration of technology and media in early childhood education really depends on good educators (D) educators should be selective in choosing educational goals to be integrated with the use of technology and media (E) both passive and interactive uses of technology and media are needed as a key to successful learning in early childhood settings

SBMPTN 2018 Bahasa Inggris, Soal SBMPTN 2018 - Bahasa Inggris Paket 2

6. The author holds an assumption that .... (A) the interactive media as a learning tool should be used to replace the existing classroom practice (B) the successful integration of new technology in education requires helpful and good teacher (C) the benefits of technology and media in childhood classrooms can be achieved by decreasing the use of those tools in teaching and learning (D) manipulating the colorful shapes to explore color and shapes using technology is the best way to develop interactive learning (E) the use of paints, markers, crayons and other graphic art materials should be replaced by the integration of technology and media 7. Which paragraph(s) explain(s) the importance of using technology and media wisely so that it can support existing classroom practice? (A) 1 (B) 2 and 3 (C) 2 and 4 (D) 3 and 4 (E) 4 The green movement is catching on in many pockets of the world. This is especially true in the construction industry. Today’s buzz words, which include global warming and zero emission, are causing everyday people to look for ways to reduce their carbon footprint. Purchasing environmentally-friendly property is a good investment for those who are concerned about their own health and the well-being of the earth. Based on this trend, entire districts, known as eco-communities, are being designed with green initiatives in mind. Dockside Green in Victoria, British Columbia, Canada is one of these communities. Its goal is to become the world’s first zero-emissions neighborhood. Builders of Dockside Green have the environment in mind with every choice they make. They ensure proper ventilation, and guarantee residents 100% fresh indoor air. Interior and exterior building materials, such as

paints and wood, are natural and non-toxic. Ecoconscious builders use bamboo wherever possible because it is durable and does not require pesticides to grow. Energy efficiency is one of the top priorities in eco-communities, such as Dockside Green. Not only do energy-efficient appliances and light fixtures reduce the environmental impact of heating and hot water, they also save residents and business owners money. Dockside Green claims that home owners will use 55% less energy than average residents in Canada. Though they are sharing space by investing in condo-style living, residents will have individual utility meters. Studies show that people use approximately 20% less energy when they are billed for exactly what they use. In addition, water is treated at Dockside Green and reused on site for flushing toilets. Planners of eco-communities such as Dockside Green must take the future into account. Dockside Green will reuse 90% of its construction waste. They also plan to continue using local suppliers for all of their transport and maintenance needs. This is a great way to reduce emissions. Dockside residents will be encouraged to make use of a mini transit system and buy into the community’s car share program. Finally, plans are in the works for a high-tech heating system that will use renewable biomass instead of fossil fuels. (Adapted from: http://tx.english_ch.com/teacher/lyn/Ievel_c/ ecocommunities_dockside-green/)

8. By saying ‘Today’s buzz words, which include global warming and zero/emissions, are causing everyday people to look for ways to reduce their carbon footprint.’ in paragraph 1, the author implies that .... (A) environmental issues have made people more cautious of their actions (B) global warming and zero emissions become everyone’s concern (C) people have to take care of their greenhouses carefully (D) eco-friendly environmental programs should be strongly enforced (E) all people around the world are aware of global warming

91

SBMPTN 2018 Bahasa Inggris, Soal SBMPTN 2018 - Bahasa Inggris Paket 2

9. Which of the following best restates the sentence ‘The green movement is catching on in many pockets of the world.’ in paragraph 1? (A) many people consider it crucial to have eco-friendly environments (B) people who are aware of environmental issues love safe and healthy surroundings (C) house designers should consider the importance of green materials (D) everyone should understand the effect of global warming and zero emissions well (E) environmentalists are concerned with the green environment 10. The false idea that the author may hold about Dockside Green is that .... (A) it provides all eco-friendly properties (B) it is a good asset to buy a property in it (C) it encourages all houses to utilize efficient energy (D) it reduces global warming and emissions (E) it is where more natural livings can exist 11. The author organizes the ideas in the passage by .... (A) describing a new housing trend and providing a detailed example (B) discussing an environmental issue and providing way to solve the issue (C) identifying a community problem and discussing possible solution to the problem (D) explaining an environmentally friendly initiative and discussing the impacts of the initiative (E) establishing a new community initiative that is eco-friendly and explaining the ideas behind the initiative

Most people would agree with the definition of good health as being a state where you are free from sickness. Despite this, there are many different opinions about how a person can actually have good health. People used to only think of their health when they were sick. But these days more and more people are taking measures to make sure that they do not get sick in the first place. In this article I will describe a few of the most common things that you can do to stay healthy. One of the best things you can do for your body is exercise. But how much is enough? Not everyone agrees on exactly how much people should exercise each day. Some people think that doing simple things like cleaning the house are helpful. Other people do heavy exercise everyday such as running or swimming. One thing experts do agree on is that any kind of exercise is good for you. Along with exercise, having a healthy diet can help promote good health. Foods like vegetables and fruits should be eaten several times each day. It is also important to eat foods high in fiber such as beans, grains, fruit and vegetables. Fiber helps your body to digest the food you eat. It also helps your body in other ways such as decreasing the chance of getting some cancers, heart disease and diabetes. Avoiding foods with a lot of sugar, salt and fat is a good idea. In today’s modem world, we all have some level of stress in our lives. Different things cause stress for different people. Money problems, work and relationships with other people can all cause stress. Instead of trying to remove stress, people need to be aware of what causes them and find ways to reduce the impact that stress has on their lives. There are several ways to fight against stress in your life. Exercise and sports are a great way to reduce stress. Other activities like Tai Chi, yoga or taking a walk also help reduce stress. Changing the way you think can also reduce stress. Try living for now, and do not worry about the future. (Adapted from: http:/www.eslreadinglessons.com/ good_health.htm)

92

SBMPTN 2018 Bahasa Inggris, Soal SBMPTN 2018 - Bahasa Inggris Paket 2

12. The author’s attitude regarding good health in the passage is .... (A) pessimistic (B) assertive (C) objective (D) responsive (E) reactive 13. What topic does the paragraph following the passage most likely discuss? (A) how to educate people about stress (B) kinds of sports to reduce stress (C) eating healthy foods to avoid stress (D) how peaceful mind helps relieve stress (E) different types and levels of stress

15. It can be hypothesized from the passage that .... (A) the more we eat, the little risk we will suffer from sickness (B) the younger we are, the more likely we will get stress (C) the more varied the exercise we do, the healthier we will be (D) the less physical exercise we do, the more likely we will get sick (E) the more problems we face, the farther we are from being healthy

14. What is the best summary of the passage? (A) Doing physical exercises and maintaining healthy diet can effectively lead to a better life. Avoiding too much stress in life is another important measure to take. (B) Eating good food makes us healthier and more balanced. With lots of fibers in food, we can easily fight cancerincluding agents. (C) Critical thinking makes us stressed and uncontrolled. This will eventually erode our well-being and in the long run posing us to serious illness. (D) To stay healthy, one should spend a lot of money on good food and going to the fitness center for well-controlled sports. This is also a way to reduce stress. (E) The way we think about life, food, sports and relationships determine the level of stress we have. We have to control our mind to stay healthy.

93

SBMPTN 2017 B. INGGRIS Soal SBMPTN 2017 Inggris

TEXT A In today's economy vocational jobs are becoming more and more important. This is why vocational education programs are popular. Vocational education training provides career and technical education to interested students. These students are prepared as trainees for jobs; jobs that are based upon manual or practical fields or jobs that are related to specific trades, occupations, and vocations. Instructors teach students the knowledge required for their field. Community colleges have long been offering vocational education. These colleges around the country provide certificates in various vocational fields. They also offer certain degree programs that focus on some popular occupations. The vocational field expands each year to include new fields. The training for vocational jobs requires less education than four year degree programs. They are also much less expensive. Instructors at this level of education use traditional methods of teaching. They use lesson plans, teacher resources, worksheets, an other tools in this process. One difference to other education programs is the on-the-job training component. Many students will have the opportunity to work in their field while being educated. Some will be accepted into valuable apprenticeship programs. Some of the jobs in vocational fields include construction workers, blacksmiths, and steel workers. Today, there are other great choices of vocational jobs. These include retail, tourism, and cosmetology. Also, there are some portions of the information technology field taught. This allows students to decide from a variety of career choices.

94

The retail field is one of those growing career opportunities. Workers can start as trainees and then reach management position. Most retail companies have their own training programs for specific jobs. These make retail even more welcoming to new employees. Tourism is also a great field to consider in the vocational field. This area includes planning trips to being a tour guide.

(Adapted from: http://www.teachnology.com/teachers/ vocational_ed/)

01. What is the topic of the passage? (A) Jobs based upon manual or practical fields. (B) Increased opportunities for vocational career. (C) Vocational education for vocational jobs. (D) Variety of choices of vocational jobs. (E) Training programs for vocational jobs. 02. The word "manual" in paragraph 1 means .... (A) hand-operated (B) customary (C) doable (D) habitual (E) concrete 03. The following statements are true based on the passage, except .... (A) tourism is a potential field in vocational education (B) every year new areas of vocational education are developed (C) on-the-job training is a typical component of vocational education programs (D) retail companies open new opportunities for vocational jobs (E) now a days young people prefer practical jobs

SBMPTN 2017 B. INGGRIS, Soal SBMPTN 2017 Inggris

TEXT B The applications, games, and websites that are promoted as 'educational' are not always the best ones for supporting learning. The fact that they are interactive does not necessarily mean that they are much better than an old-fashioned workbook with its right and wrong answers. Children may enjoy these products for a while but then get a bit bored. Thus, they are not the most appropriate or engaging way to learn. Treat so-called 'free' applications with caution. Some will expose your child to advertisements. Others are designed to wait until your child is engaged in a game or storyline and then demand payment before they can go any further. This can lead to frustration as young children do not understand why they cannot continue. Sometimes it is better to make a small payment in advance if the application promises no further purchases. However, some applications are free to download and completely free of advertisements or in-app purchases. Choosing an app needs the same kind of thought and care you would put into buying anything else for your child. Do not rely only on the star rating. Instead, read the user reviews and check the privacy policy if you are worried about the personal information that the app might be collecting. If you want your child to enjoy learning, develop curiosity, and think about things creatively, provide them with a range of games and apps. Openended games have become progressively more challenging and encourage children to explore and have fun. They are, therefore, more likely to establish a love of learning and to lay the foundation for their future development. Physical activity, reading, and other more 'traditional' activities continue to play a very important part in children's development. But, most parents do rely on screen devices from time to time to engage their child while they are busy with something else. This is not a problem in itself, just as long as children's time is made up of a balanced range of activities.

04. The author reminds readers to be more careful with commercialism in free applications in paragraph(s) .... (A) 1 (B) 2 (C) 3 (D) 1 and 4 (E) 2 and 3 05. How does the idea in sentence 6 relate to the other ideas in paragraph 2? (A) Sentence 6 elaborates the other ideas in paragraph 2. (B) Sentence 6 is the result of the other ideas in paragraph 2. (C) Sentence 6 strongly contradicts the other ideas in paragraph 2. (D) Sentence 6 is the implementation of the theory discussed in paragraph 2. (E) Sentence 6 provides another type of free applications discussed in paragraph 2. 06. The author would apparently agree that .... (A) open-ended games can stimulate children's love for learning (B) some games and applications are boring because they are too easy (C) there is no convincing prediction for the future of the games and apps (D) it is better to pay for games in advance so that children can learn freely (E) apps star rating provides users with reviews on the good things of games and apps

(Adapted from : http://www.bbc.co.uk/guides/z3tsyrd)

95

SBMPTN 2017 B. INGGRIS, Soal SBMPTN 2017 Inggris

07. Regarding busy parents' reliance on screen devices to help keep their children entertained, the author assumes that .... (A) it is the parents' responsibility to make their children less engaged with screen devices (B) children whose parents are busy with their activities must use screen devices less (C) parents need to be aware of their children's need for varied activities (D) such parents' reliance is acceptable as long as the children are given the right proportion of other activities (E) it is not tolerable as their children need to have more physical activities TEXT C The latest round in an ongoing debate over global-warming trends claims that warming has indeed slowed down this century. An obvious slowing in the rise of global temperatures was recorded at the beginning of the twenty-first century. This was referred to as a "hiatus" or a "pause". This hiatus was first observed several years ago. Climatechange skeptics have used this as evidence that global warming has stopped permanently. But in June last year, a study in science claimed that the hiatus was just an artifact which disappears when biases in temperature data are corrected. Now a prominent group of researchers is countering that claim. They argue in Nature Climate Change that even after correcting these biases the slowdown was real. "There is this mismatch between what the climate models are producing and what the observations are showing," says lead author John Fyfe. Fyfe is a climate modeler at the Canadian Centre for Climate Modeling and Analysis in Victoria. "We can't ignore it." Fyfe uses the term "slowdown" rather than "hiatus". He also stresses that it does not in any way weaken global-warming theory.

96

The debate turns in part around statistics on temperature trends. The study that questioned the existence of the slowdown corrected known biases in the surface temperature record maintained by the US National Oceanic and Atmospheric Administration (NOAA). The finding showed differences in temperature readings from ships and buoys. This effectively increased the record about warming. The researchers also extended the record to include 2014. This set a new record high for average temperatures. Thomas Karl, director of National Centers for Environmental Information in Asheville, calculated the rate of global warming between 1950 and 1999 as being 0.113°C per decade. This was similar to the 0.116°C a decade calculated for 2000-14. This, Karl said, meant that an assessment done by the influential Intergovernmental Panel on Climate Change in 2013 showing that warming had slowed was no longer valid. Therefore, it can be concluded that global warming is a fabricated issue. (Adapted flora :http://www.nature.com/news/globalwarming-hiatu-debate-flarets-up-again-1.19414)

08. Which of the following best restates the sentence “Climate-change skeptics have used this as evidence that global warming has stopped” in paragraph 1? (A) Climate-change believers have used global warming as evidence to stop this. (B) This has been used as a proof by climatechange disbelievers to claim that global warming has come to an end. (C) That global warming has come to an end has been used by climate change believers as a proof. (D) This has been exploited by climatechange disbelievers as a proof to stop global warming. (E) This can be used as evidence to stop climate change disbelievers.

SBMPTN 2017 B. INGGRIS, Soal SBMPTN 2017 Inggris

09. How does the author organize paragraph 2? (A) A claim is followed by a description about this claim. (B) A claim is followed by contrasting arguments about it. (C) A claim is followed by quotations from an expert. (D) A problem is followed by some solutions. (E) A cause is followed by several effects. 10. It can be inferred from paragraph 2 that John Fyfe is .... (A) a disbeliever of hiatus theory (B) a proponent of climate change (C) an opponent of climate change (D) a model at the Canadian Centre for Climate Modeling and Analysis (E) a debater at the Canadian Centre for Climate Modeling and Analysis 11. Which of the following obviously shows the author’s false idea? (A) An obvious slowing in the rise of global temperatures was recorded at the beginning of the twenty-first century. (B) Fyfe is a climate modeler at the Canadian Centre for Climate Modeling and Analysis in Victoria. (C) The finding showed differences in temperature readings from ships and buoys. (D) Thomas Karl calculated the rate of global warming between 1950 and 1999 as being 0.113°C per decade. (E) It can be concluded that global warming is a fabricated issue.

TEXT D Young people have put the spotlight on mental health in Mission Australia’s Youth Survey this year, naming it as one of the top three issues facing Australia. The survey found concerns about mental health across the country that have doubled alarmingly since 2011. About 22,000 young people aged 15 to 19 took part in the survey and more than 20 per cent cited mental health as among their top national issues. Alcohol and drugs were cited as their top concern, followed by equity and discrimination. Mission Australia chief executive Catherine Yeomans said concerns about mental health were at their highest level in the survey’s 15-year history. “If young people are telling us that they think this is one of the top three concerns facing the nation, then we should sit up and pay attention and we should think about whether we’ve got the right responses in place,” she said. “Let’s look at the issues and put in programs that are going to support young people.” The results did not surprise 19-year-old Savannah van der Veer, who has managed depression and obsessive compulsive disorder for more than a decade. “People don’t take you seriously, they just assume all children are kind of moody and unusual - they do strange things that don’t make sense,” she said. “But I was really suffering and I didn’t really know how to talk about it and I didn’t really know that what was happening to me wasn’t normal.” Miss van der Veer said she turned to her mother and counselors for support.

97

SBMPTN 2017 B. INGGRIS, Soal SBMPTN 2017 Inggris

Youth mental health group Batyr held more than 150 workshops in Australian high schools last year. The program is facilitated by young people who have experienced mental health issues. “What our programs are designed to do is to make it OK to not be OK - to show young people that there are people out there like them who are suffering and going through tough times but that we can talk about it as a group,” chief executive Sam Refshauge said. The sessions incorporate music and activities to shift negative stigma around mental health issues. (Adapted from : http//www.abc.net.au/news/2016-12-05/ mental-health-top-concern-among-young-australians-survey -finds/8092846)

12. What topic does the paragraph preceding the passage most likely discuss? (A) Alcohol and drugs (B) Mental health of adults (C) Equity and discrimination (D) Mission Australia’s Youth Survey (E) Mental problems faced by Australians 13. What is the author’s attitude toward the topic of the passage? (A) Concerned (B) Ignorant (C) Pessimistic (D) Doubtful (E) Critical 14. Based on the passage, young people will understand their own mental health condition if .... (A) high school programs prioritize social welfare (B) workshops on mental health are effectively conducted (C) their family and school consider mental disorder seriously (D) more research on mental health reveals the roots (E) mental health becomes a school subject

98

15. Which of the following is the best summary of the passage? (A) A survey this year named mental health as the second top issue in Australia. The first top issue was alcohol and drugs, while the third one was equity and discrimination. (B) Unlike what most people believe, mental health is a serious issue for Australians. The government has quickly reacted to a survey result and provided free counseling for everyone who needs professional help. (C) Youth mental health workshops are held in 150 high schools. The workshops are facilitated by teenagers who have recovered from mental health problems so that participants with mental health issues can relate with them well (D) Mental health has become a main issue for Australian teenagers according to a survey this year. This problem receives serious attention, and youth mental health programs are held in Australian high schools to support young people. (E) While many people did not expect it, mental health apparently became a significant problem in Australia. In particular, many teenagers suffer from depression and obsessive compulsive disorder. Therefore, they need help from their family and school.

Text 01-03. There have been a number of differences in the way geography is now to be approached in the National Curriculum. It was decided that there would be a renewed emphasis on spatial knowledge, as well as the human and physical processes. This should cover some technical procedures such as using grid references. There should also be a renewed commitment towards the concept of fieldwork and the use of maps, as well as written communication. Dealing with geography, the National Curriculum includes certain topics, but not necessarily how they should be taught. For example, the focus at key stage 1 is developing knowledge about the United Kingdom and the world. Students should study certain fact such as the world‟s seven continents and their locations. They should be able to name and identify the four countries and capital cities of the UK. Trips to London may include extra-curricular education that can aid students‟ understanding of the United Kingdom. They also should be able to identify seasonal weather patterns, identify hot and cold areas of the world, and use world maps and globes to identify the UK and other countries and oceans. As they progress to key stage 2, students are expected to extend their knowledge to include Europe, North, and South America as well as significant human and physical features. They should be able to identify the position of latitude, longitude, the Equator and other large features of the world such as the Tropics of Cancer and Capricorn. Students at key stage 2 should study more physical geography including the climate zones, biomes and features such as volcanoes and earthquakes. Trips to destinations such as Iceland could encourage further learning about some of the world‟s physical geography. There is a huge emphasis on geographical skills at this stage. Students should be able to use the eight points of a compass, four and six-figure grid references and keys on Ordinance Survey maps in order to develop their knowledge.

01. What is the topic of the passage? (A) The art of teaching geography. (B) The world‟s physical geography. (C) Topical approaches in teaching geography. (D) Different strategies in teaching geography. (E) The geographical topics in the national curriculum. 02. The underlined word ‘seasonal’ in paragraph 2 means …. (A) serial. (B) cyclical. (C) current. (D) regular. (E) situational. 03. According to the passage, key stage 2 focuses more on …. (A) the Tropics of Cancer and Capricorn. (B) broader geographical abilities. (C) patterns of climate change. (D) geographical physics. (E) the trip to Iceland. Text 04-07. Over the last two decades. the use of ICT has been an important topic in education. On the one hand, studies have shown that ICT can enhance teaching and learning outcomes. For example, in science and mathematics education, scholars have documented that the use of ICT can improve students‟ conceptual understanding, problem solving, and team working skills. Consequently, most curriculum documents state the importance of ICT and encourage school teachers to use them. (A) However, teachers need to specifically trained in order to integrate ICT in their teaching.

99

Schools are known to be resistant to innovation and change. However, the spread of ICT is beginning to affect how teachers teach. One of the current issues about the use of ICT is how it is integrated into the curriculum. The curriculum document provide arguments for introducing ICT in the school setting. Therefore, schools expect that graduates from teacher education programs have a reasonable knowledge of how to use ICT. (B) However, this may not be the case because most current teachers‟ pre-service preparation, and subsequent in-service courses were designed by using traditional educational technology and settings. Thus, the participants in these courses are not familiar with the processes, interaction patterns, features, and possibilities of teaching learning processes based on ICT. This issue becomes complicated because the students‟ thinking skills are often weak. Also, they typically lack information literacy skills although they were born in or after 1982. In addition, they belong to the “Net Generation”. (C) Furthermore, they are accustomed to operating in a digital environment for communication, information gathering, and analysis. The problem is that students do not have to understand how their use of technology affects their habits of learning. (D) Effective development of preservice teachers‟ ICT proficiency does not seem to be a direct process, but is the one asking for a careful, complex approach. First, a need assessment is important to find out what ICT skills and knowledge teachers need at schools. Second, designers of teacher education programs should know the pre-service teachers‟ perceptions of ICT and their attitudes toward ICT integration into curriculum. Third, teacher education programs need to consider the two typical arguments that support the ICT use in schools.

100

04. With the statement „One of the current issues about the use of ICT is how it is integrated into the curriculum‟ in paragraph 2, the author intends to …. (A) emphasize the need for teachers with good literacy in technology. (B) explore the reasons for including ICT in the curriculum document. (C) explain the curriculum documents for ICT introduction in education. (D) argue the current teachers already have good knowledge of using ICT. (E) show that teacher education programs have been running expected ICT curriculum. 05. The author‟s idea of the relationship between the use of ICT and learning outcome is analogous with …. (A) vitamin - health. (B) speed - aeroplane. (C) harvest - irrigation. (D) cellphone - crime. (E) books - intelligence. 06. The assumption the author has about teacher education programs is that …. (A) the programs have introduced a reasonable knowledge of how to use ICT. (B) the programs have found out what ICT skills and knowledge the teachers need. (C) the programs have given materials related to the pre-service teachers perceptions of ICT. (D) the programs were still designed in reference to traditional educational technology and settings. (E) the programs have participants who are familiar with the processes of technology-mediated educational transactions. 07. Which tines of the passage illustrate the ideal ICT teacher education programs most effectively? (A) sentence(s) A in the passage (B) sentence(s) B in the passage (C) sentence(s) C in the passage (D) sentence(s) D in the passage (E) all the sentences in the last paragraph.

Text 08-11. Our ancestors destroyed moot of our natural areas before anyone had a chance to study or to try to understand them. As a result, we have lost the opportunity to learn about this areas and the benefits and opportunities they might have yielded. Today our remaining natural areas serve important roles in the study of ecology, botany, zoology, geology, and soil science. They provide controls for comparison against managed or exploited resources. They also provide educational and cultural information important in the study of science, local history, conservation, and nature studies such as bird-watching, insect study, and tree identification. Preservation of natural areas also provides many practical benefits. For example, natural areas contain the biological raw materials necessary for the development of products that could greatly benefit the health and well-being of a man. A new wonder drug or fine industrial product may exist now only in some inconspicuous organism harboured in a nature preserve. Up to approximately half of the drugs currently in use contain derivatives of wild plants, yet only a small percentage of all plants have been investigated for their potential in such uses. The need to protect the remaining 98 percent of natural areas until they can be researched is obvious. From a genealogical standpoint, we have roots linking us to our ancestors. With so many of our natural areas now gone, those that remain are a vital link to the past. They can help us and future generations better understand the landscape and natural resources from which the pioneers molded their lives.

Certainly, one of the most important benefits to be derived from natural areas is the perceptive-recreational benefit they provide. This is evidenced by the increasing numbers of people turning to the nature preserve in order to escape briefly from the hustle-bustle of fast-paced society. The quiet environment of nature calms the monotony in our daily lives whether from skyscrapers and pavement or unbroken horizons of corn and beans. Healthy natural areas can offer substantial economic benefits to our communities as well. Wetlands, for example, help with flood protection and the removal of pollutants from our water supply. 08. Paragraph 4 implies that …. (A) the number of people turning to the nature preserves becomes more and more. (B) we cannot see any skyscrapers and pavement due to natural preservation. (C) preservation of natural areas can help us throw away our stressful days. (D) nature preserves briefly lighten busy life of urban environment. (E) quiet enjoyment of nature creates better daily lives. 09. The sentence „Our ancestors destroyed most of our natural areas before anyone had a chance to study o try to understand them‟ in the first paragraph can best be restated as …. (A) after we had an opportunity to deeply study our natural areas, most of them had been destroyed by our ancestors (B) we had an opportunity to deeply study our natural areas although most of them had been destroyed by our ancestors. (C) most of our natural areas had been destroyed by our ancestors, besides, we had an opportunity to deeply study them. (D) before most of our natural areas had been destroyed by our ancestors, we had an opportunity to deeply study them. (E) we might have had an opportunity to deeply study our natural areas if most of them had not been destroyed by our ancestors. 101

10. Which of the following obviously shows the authors bias about natural areas? (A) As a result, we have lost the opportunity to learn about these areas and the benefits and opportunities they might have yielded. (B) Wetlands, for example, help with flood protection and the removal of pollutants from our water supply. (C) The need to protect the remaining 98 percent until they can be researched is obvious. (D) With so many of our natural areas now gone, those that remain are vital link to the past. (E) Preservation of natural areas also provides many practical benefits. 11. The organizational pattern of passage is .... (A) listing order. (B) time order. (C) classification. (D) cause-effect. (E) comparison-contrast. Text 12-15. According to the latest New York Times poll, most Americans are troubled by performance-enhancing drugs, which they believe are widely used by the nation‟s athletes. The public believes that professional athletes in major American sports leagues use steroids to a greater degree than American Olympic athletes do. But the prospect of achievement through illicit means in the Winter and Summer Olympics is more troubling. Also, younger Americans are much less troubled by drug use in sports and believe it to be more widespread than do Americans age 30 and above. After recent revelations about the use of performance-enhancing drugs in football. baseball, and track-and-field, 43 percent of those polled said they believe that at least half of professional athletes in the United States use steroids. In comparison, 18 percent of those surveyed said they believe that at least half of American Olympic athletes use banned performance-enhancing substances. 102

While 61 percent of Americans said they were bothered by the use of steroids among professional athletes. 75 percent said they were concerned about the use of performance-enhancing drugs among Olympic athletes. This response seem to be based partly on the ideal that American Olympic spoils are purer than professional sports, and on the widely held belief that Olympic athletes are still amateurs. “The Olympics are pure and clean,” Jason Mannino, 32, a tax assessor, said. “We want these kids to be all on the same playing field. A lot of these kids are trying to make a name for themselves purely on their athletic ability, and they are not getting high-dollar contracts to perform. They are performing out of pride for the country and out of pride for getting a gold medal.” In fact, the Olympics have been open to professionals since the late 1980‟s. And most of the drug scandals in recent years have involved sports related to the Olympics. 12. What is the author‟s attitude towards topic of the passage? (A) Tolerant. (B) Unhappy. (C) Worried. (D) Critical (E) Unconcerned. 13. The paragraph following the passage will likely talk about .... (A) drug use by Olympic athletes. (B) the opinions of young people and those above 30. (C) what the surveyor think about drug use. (D) what is being done about the problem. (E) what the problems will lead to.

14. Which of the following is the best summary of the passage? (A) Most Americans are worried about the widespread us of steroids among athletes and they believed that professional athletes used more drugs than American Olympic athletes do, but the fact that the latter make use of drugs made many more concerned. (B) Many Americans are worried about the fact that many professional American athletes were using drugs; however, they did not think that Olympic athletes used steroids because they were not playing for the money, but for their country. (C) Some Americans are bothered about the widespread use of drugs among athletes, both profesional and Olympic athletes and they believed the latter should not be taking the drugs. (D) Although many Americans believed that at least half of the professional athletes used drugs they don‟t believe that Olympic athletes use them because they are still amateurs. (E) Most Americans are worried about the widespread use of drugs among professional as well as national athletes. 15. Based on the passage, many American athletes .... (A) both professional and amateur, will be disqualified if drug tests are done stringently. (B) would not be able to achieve what they have achieved of they had taken drugs. (C) involved in professional sports would do much better if they stayed away from steroid. (D) would be able to break more records if they had not been taking illicit drugs. (E) will not be taking steroids if they are not competing.

103

SBMPTN 2015 Bahasa Inggris Bahasa Inggris Problems 76, 77, and 78 are based on the following passage. Choose one option that best completes the blank spaces in the passage.

The present study sought to document the word reading and comprehension levels attained by children who were implanted by 5 year of age. It was hypothesized that the improved speech perception abilities acquired with cochlear implantation would promote phonological coding skills. (1) ... Three subtests of diagnostic reading assessment batteries standardized on hearing children were administered to 181 children between 8 year 0 month and 9 year 11 month of age who had 4 to 6 years of implant experience. (2) ... It included a lexical decision task, a rhyme task, and the digit span subtest of the Wechsler Intelligence Scale for Children. Over half of the children scored within the average range for their age compared with the normative data for hearing children. (3) ... They were higher nonverbal intelligence, higher family socio-economic status, and later onset of deafness between birth and 36 months. (Adapted from Journal of Ear and Hearing)

76. Which option best completes (1)? (A) The finding showed that the hypothesis was accepted. (B) The implantation was eventually shown to be successful. (C) The following paragraph would discuss the findings of the study. The objective of the study was to see the subjects’ reading skills. (D) It would also facilitate the acquisition of beginning reading skills. 77. Which option best completes (2)? (A) Likewise, the children were obliged to take a reading test. (B) In addition, a battery of processing measures was administered. (C) However, standard measurement was 104

applied to test the subjects. (D) Consequently, the assessment was used to evaluate the reading skills. (E) Instead, an interview consisting of many different tasks was conducted. 78. Which option best completes (3)? (A) The results were categorized based on the subjects’ status. (B) Data were collected on the basis of several different items. (C) Reading competence was associated with three aspects. (D) The subjects consisted of different age children. (E) Findings showed various levels of reading skills.

Questions 79-84 are based on the following passage. Ginger, nuts, fatty fish, and whole grains are just some of the many foods that have been publicized to have antiinflammatory properties. But do they work? It turns out that experts agree that eating a diet rich in such foods may in fact help lower the levels of inflammation in the body. However, they stress that adding or increasing the consumption of any one food is likely not going to have a profound effect on one’s health. In a new, small study, researchers found that men who consumed flaxseed for 42 days experienced a significant decrease in inflammatory markers compared with men who did not consume flaxseed. In another study, the authors found that taking ginger root extract appeared to reduce markers of colon inflammation. In addition, according to the results of a study, curcumin, which is the main compound in the spice turmeric, which is used in curry, could help suppress biological mechanisms that lead to the inflammation in diseases of the tendons. Rheumatoid arthritis, cardiovascular disease, and diabetes are some of the conditions that have been linked with higher levels of inflammation.

SBMPTN 2015 - TPA. 06 - Gamabar

However, Kirkpatrick stressed that adding just one specific food to your diet is unlikely to work wonders for lowering inflammation, or for improving health in general. Kirkpatrick also stressed that people who take dietary supplements do not get the same results as those who consume real foods that have anti-inflammatory properties. Moreover, in order to lower inflammation through diet, it is also important to stay away from foods that can promote inflammation, such as sugar, she said. “So it is not just about adding these things in, it is also about taking pro- inflammatory foods out,” Kirkpatrick said. “Food is medicine, but it is hard to compare medicine that is created in a lab with something that is grown in the ground to determine what is going to be beneficial,” Kirkpatrick said. Depending on a person’s condition, and the reasons for their inflammation, medication may be necessary, while food can be still a nice add-on.

(Adapted from Medical News Today)

79. The conditions required to reduce the inflammation stated in paragraph 2 are found in paragraph (s) .... (A) (B) (C) (D) (E)

1 3 4 1 and 3 2 and 4

80. Based on the passage, which of the following reflects the idea of relationships between consuming gingers, nuts, fatty fish, and whole grains and health? (A) Thirst and drinking (B) Age and physical strength (C) Learning efforts and grades (D) Sleeping hours and body freshness (E) Bodily exercise and physical growth 81. The argument in the last line of paragraph 4 assumes that .... (A) having a balance diet can help reduce inflammation (B) consuming more curry is good to avoid inflammation (C) getting medication is a better way to

solve inflammatory problems (D) taking foods with anti-inflammatory materials can cure inflammation (E) studies on the influence of foods with anti-inflammatory property are consistent 82. The possible role of anti-inflammatory foods is best described in paragraphs .... (A) 1 and 2 (B) 1 and 3 (C) 2 and 3 (D) 2 and 4 (E) 3 arid 4 83. Paragraph 3 implies that .... (A) a well-proportioned diet is good to reduce inflammation (B) carbohydrate is the source of proinflammatory foods (C) pro-inflammatory foods should not be in the diet list (D) various anti-inflammatory supplements work better (E) the nature of anti-inflammatory foods is still secret 84. Which of the following best restates the idea of paragraph 2? (A) The inflammation, a study found, is caused by improper biological mechanism. (B) Some studies showed that antiinflammatory foods could lower inflammation. (C) The studies conclude that consuming the extract shows to be more effective. (D) Anti-inflammatory foods are factually spicy foods we consume every day. (E) Inflammation results in arthritis, cardiovascular disease, and diabetes. Questions 85-90 are based on the following passage. Gardeners and homeowners might sometimes need to add nitrogen fertilizers to their gardens and lawns to provide just the right food for their plants. The amount used typically depends on plant and seed instructions as well as 105

SBMPTN 2015 - TPA. 06 - Gamabar

on the other sources of nitrogen, such as manure, already applied to the soil. While an addition of nitrogen to the lawn and garden may be necessary, homeowners often use more than what is recommended, and over time this excess nitrogen pollutes water and air. Nitrogen at higher levels causes a loss of certain plant species, depletion of soil nutrients, death of fish and aquatic organisms, and contamination of drinking water. Though nitrogen serves to aid plants in their growth, weeds and nonnative plants tend to grow more readily with additional nitrogen supplies. Other plants that have lower nitrogen needs end up dying, causing a decline in native species, according to the Ecological Society of America (ESA). In California, for example, it is reported that too much nitrogen encourages the growth of nonnative grasses and kills off lichens on trees. In the coastal areas of the western United States, soils have higher levels of nitrogen, which feed nonnative grasses. The shift in plant species increases the chances of wildfires because these new grasses are flammable. In the soil, too much nitrogen also creates an imbalance of nutrients that causes a depletion of other important minerals such as calcium, phosphorus, and magnesium. While fertilizer overuse causes this change, nitrogenpolluted air, caused by nitrates from automobiles and industrial plants, also results in this acidification of the soil when acid rain falls. When the nitrogen abundance reduces important minerals, toxic elements such as aluminum can proliferate and harm plants as well as fish in rivers.

When nitrogen levels in rivers and streams increase, they aid in algae overgrowth. As algae dies and decomposes. organic matter in the water increases. This process uses up oxygen, causing levels to drop. Without the oxygen, fish, crabs and other aquatic life die. In the San Francisco Bay Delta, for example, blue-green algae blooms occur in numbers during the warmer months, especially when the conditions such as increased nitrogen occur. Researchers at the Woods Hole Oceanographic Institution say these algae blooms, which can appear in different colors, produce toxins that can be harmful. A soluble substance, nitrogen soaks 106

deeply into the soil after a rainstorm or after irrigation, reaching ground water and nearby wells. When babies under a year old and elderly people ingest water with high nitrogen levels, they can develop symptoms such as gastrointestinal swelling and irritation, diarrhea, and protein digestion problems, according to the University of NebraskaLincoln Extension. Because nitrogen is odorless and colorless, only testing can determine whether contamination has occurred. (Adapted from SFGATE)

85. According to the passage, which of the following best shows the author’s bias? (A) The need of fertilizer to increase agricultural production. (B) Blooms of algae in spring but leaving polluted waters afterwards. (C) Change of plants in the forest and forest fire because of nitrogen. (D) Growing of unwanted plants and plants extinction caused by nitrogen. (E) Nitrogen as a useful supplement for plants, be it in the air, water, or soil. 86. In connecting the idea in paragraph 2 and that in paragraph 3, the author .... (A) shows evidence why change of plants causes forests more vulnerable from fire (B) gives further impacts of nitrogen not only on soil but also in water and air (C) argues that nitrogen harms not only plants and animals, but also humans (D) provides a reason why the change of plants may occur in forests (E) states the characteristics of nitrogen in the nature 87. What is the author’s attitude towards overuse of nitrogen? (A) Pessimistic (B) Concerned (C) Mindful (D) Critical (E) Neutral

SBMPTN 2015 - TPA. 06 - Gamabar

88. The paragraph following the passage will most likely discuss .... (A) a case of polluted water contaminated by nitrogen (B) trainings on how to use fertilizers wisely to possible users (C) results of testing an area possibly contaminated by nitrogen (D) a case of plant change because of nitrogen excess in the soil (E) efforts to reduce acid rains to avoid nitrogen contaminating wells 89. Which of the following best summarizes ideas of the passage? (A) In the long run, fertilizers will bring more harms than benefits to environments. (B) Nitrogen can help plants grow, but it is potential to pollute soil, water, and air. (C) Use of fertilizers is the major cause of nitrogen excess in the soil and water. (D) Acid rain in the environment has the potential to bring revolution of plants. (E) Nitrates together with other elements in the atmosphere form acid rains. 90. Based on the passage, it can be hypothesized that the more .... (A) houses, the more nitrogen excesses (B) polluted soil, the more crop problems (C) fertilizers, the more agricultural products (D) nitrogen, the more acid rains and plant shifts (E) industrial areas, the more polluted air and soil

107

SBMPTN Bahasa Inggris Soal

Text A Part 1 Parents send their children to school with the best of intentions, believing that formal education is what kids need to become productive, happy adult. Many parents do have qualms about how well schools are performing, but the conventional wisdom is that these issues can be resolved with more money, better teacher, more challenging curricula, or more rigorous tests. But what if the real problem is school itself ? The unfortunate fact is that one of our most cherished institutions is, by its very nature, failing our children and our society . Children are required to be in school, where their freedom is greatly restricted, far more than most adults would tolerate in their workspace. In recent decades, we have been compelling them to spend ever more time in this kind of setting, and there is strong evidence that this is causing psychological damage to many of them. And as scientists have investigated how children naturally learn, they have realized that kids do so most deeply and fully, and with greatest enthusiasm, in conditions that are almost opposite to those of school. Text A Part 2 Compulsory education has been a fixture of our culture now for several generations. President Obama and Secretary of Education Arne Duncan are so enamored of it that they want even longer school days and years. Most people assume that the basic design of today's schools emerged from scientific evidence about how children learn. But nothing could be further from the truth. Schools as we know them today are a product of history, not of research. The blueprint for them was developed during the Protestant Reformation, when schools were created to teach children to read the Bible, to believe Scripture without questioning it, and to obey authority figures without questioning 108

them. When schools were taken over by the state, made compulsory, and directed toward secular ends, the basic structure and methods of teaching remained unchanged. Subsequent attempts at reform have failed because they have not altered basic blueprint. The top-down, teach-and-test method in which learning is motivated by a system of rewards and punishments rather than by curiosity or by any real desire to know, is well designed for indoctrination and obedience training but not much else. It is no wonder that many of the world's greatest entrepreneurs and innovators either left school early (like Thomas Edison) or said they hated school and learned despite it, not because of it (like Albert Einstein). 01. What is the topic of the text above? (A) Restrictions on children's freedom at the US schools (B) Parents' expectation on reformation in American school system. (C) Restrictions on children's freedom at the US schools. (D) Doubts on the effectiveness of American school systems. (E) Absence of a research-based school system in the USA. 02. What is the purpose of the text? (A) To discuss if the American school system is truly effective to educate children (B) To remind American parents that the formal school is basically a product of culture (C) To tell the readers that formal schools in the USA have been constantly developed for a long time (D) To review how compulsory education in the USA has met parents’ expectation (E) To describe how American children learn at school and in the real-life settings

SNMPTN Bahasa Inggris, Soal

03. Which of the following is closest in meaning to the word "qualms" (line 4)? (A) Beliefs (B) Requests (C) Remarks (D) Views (E) Doubts Text B Part The benefits of fasting must be preceded by a look at the body's progression when deprived of food. Due to the lack of incoming energy, the body must turn to its own resources, a function called autolysis. Autolysis is the breaking down of fat stores in the body in order to produce energy. The liver is In charge of converting the fats Into a chemical called a ketone body, and then distributing these bodies throughout the body via the blood stream. The less one eats, the more the body turns to these stored fats and creates these ketone bodies, the accumulation of which is referred to as ketosis. Detoxification is the foremost argument presented by advocates of fasting. “Detoxification is a normal body process of eliminating or neutralizing toxins through the colon, liver, kidneys, lungs, lymph glands, and skin.• This process is precipitated by fasting because when food Is no longer entering the body, the body turns to fat reserves for energy. A second prescribed benefit of fasting is the healing process that begins in the body during a fast. During a fast energy is diverted away from the digestive system due to its lack of use and towards the metabolism and Immune system. The healing process during a fast is precipitated by the body’s search for energy sources. Abnormal growths within the body, tumors and the like, do not have the full support of the body's supplies and therefore are more susceptible to autolysis. Text B Part 2 In addition, there is a reduction In core body temperature. This is a direct result of the slower metabolic rate and general bodily functions. Following a drop in blood sugar level and using the reserves of glucose found

in liver glycogen, the basal metabolic rate is reduced in order to conserve as much energy within the body as can be provided. Growth hormones are also released during a fast, due to the greater efficiency in hormone production. Finally, the most scientifically proven advantage to fasting is the feeling of rejuvenation and extended life expectancy. Part of this phenomenon is caused by a number of the benefits mentioned above. A slower metabolic rate, more efficient protein production, an improved immune system, and the increased production of hormones contribute to this long-term benefit of fasting. In addition to the Human Growth Hormone that is released more frequently during a fast, an anti-aging hormone is also produced more efficiently. 04. The text states all the following, EXCEPT ... (A) detoxification is the primary advantage of fasting (B) the less one eats, the more ketone bodies are created (C) one benefit of fasting is a curative process (D) the most scientifically proven benefit of fasting is the feeling of rejuvenation (E) there is a production in core body temperature during a fast 05. The main purpose of the text is to ... (A) discuss the effect of fasting (B) present suggestion for fasting (C) describe the benefits of fasting (D) suggest methods of fasting (E) inform readers about fasting 06. The pronouns “which” (paragraph 1, last sentence) refers to ... (A) strong fast (B) the blood stream (C) the body (D) ketone bodies (E) the liver

109

SNMPTN Bahasa Inggris, Soal

07. The word “deprived of ” (paragraph 1, first sentence) is most similar to which of the following? (A) Left without (B) Provided with (C) Presented with (D) Giver (E) Supplied with 08. which is the topic of the text? (A) The body’s progression (B) The function of autolysis (C) A ketone body (D) The health benefits of fasting (E) Detoxification in fasting 09. It is implied in paragraph 5 that the more frequently you fast, ... (A) the more hungry you feel (B) the longer you will live (C) the less production of hormone occurs (D) the less immune system your body produces (E) the less efficient protein is produced Text C Part 1 What is happening in Egypt is the latest example of the Interplay between democracy, protest and government efficacy. Democracy is a way of deciding the decision -makers, but it is not a substitute for making the decision. I remember an early conversation with some young Egyptians shortly after President Mubarak's downfall. They believed that, with democracy, problems would be solved. When i probed on the right economic policy for Egypt, they simply said that it would all be fine because now they had democracy: chance of working. I am a strong supporter of democracy. But democratic government does not on its own mean effective government. Today, efficacy is the challenge. When governments do not deliver, people protest. In fact, as Turkey and Brazil show, they can protest even when, on any objective basis, countries have made huge progress But as countries move from low to middle income status, the people's expectations rise. They want quality services, better housing, good infrastructure, 110

especially transport. This is a sort of free democratic spirit that operates outside the convention of democracy that elections decide the government. It is enormously fuelled by social media, itself a revolutionary phenomenon. And it moves very fast in precipitating crisis. It is not always consistent or rational. A protest Is not a policy, or a placard a programme for government. But if governments don't have a clear argument with which to rebut the protest, they're in trouble. Text C Part 2 In Egypt, the government's problems were compounded by resentment at the ideology and intolerance of the Muslim Brotherhood. Across the Middle East, for the first time, and this is a positive development, there is open debate about the role of religion in politics. Despite the Muslim Brotherhood's superior organization, there is probably a majority for an intrinsically secular approach to government in the region. Society can be deeply imbued with religious observance, but people are starting to realize that democracy only works as a pluralistic concept where faiths are respected and where religion has a voice, not a veto. For Egypt, a nation with an immense and varied civilization, around 8 million Christians and a young population who need to be connected to the world, there Isn't really a future as an Islamic state that aspires to be part of a regional caliphate. 10. Which of the following is TRUE according to the text? (A) Muslim Brotherhood insists to replace secular government into Islamic one (B) The author perceives negatively the current political trend In Egypt (C) Egypt is the largest country in the Middle East (D) Egyptian society do not like mixing religion and politics (E) Religion's role is Important in democracy to voice the truth

SNMPTN Bahasa Inggris, Soal

11. The purpose of the text is to ... (A) inform readers about democracy in Egypt (B) describe the system of democracy In Egypt (C) tell readers about the government problems in Egypt (D) publicize conditions of Egypt after Mubarak's downfall (E) give a suggestion of how democracy can work in Egypt 12. In which paragraph does the author mention that democracy does not correlate with the government efficacy? (A) 1 (B) 2 (C) 3 (D) 4 (E) 5

15. Which of the following best describes the organization of the text? (A) A criticism of the effect of democracy in Egypt (B) An explanation of problems of democracy In Egypt (C) An example of the interplay between democracy, protest, and government efficacy (D) A comparison between democracy in Egypt and In other countries (E) A description of what democracy is in the middle East

13. Which of the following can best replace the word “rebut” (paragraph 3, last sentence)? (A) Stop (B) Reject (C) Rebuke (D) Counter (E) Calm down 14. What can be inferred from paragraph 2? (A) Government efficacy is more Important than democracy (B) Democracy does not guarantee for wealth to occur (C) There are no people protests in poor countries (D) There is no democracy when there is no protest (E) Democracy means to serve the people

111

SBMPTN Bahasa Inggris Kode Soal

Text 1 Over this decade, employment in jobs requiring education beyond a high school diploma will grow more rapidly than employment in jobs that do not; of the 30 fastest growing occupations, more than half require post-secondary education. With the average earnings of college graduates at a level that is twice as high as that of workers with only a high school diploma, higher education is now the clearest (31) ... into the middle class. In higher education, the U.S. has been outpaced internationally. While the United States ranks ninth in the world in the proportion of young adults enrolled in college, we have fallen to 16th in the world in our share of certificates and degrees awarded to adults ages 25-34 — lagging behind Korea, Canada. Japan and other nations. While more than half of college students graduate within six years, the (32) ... for low-income students is around 25 percent. Acknowledging these factors early in his administration, President Obama challenged every American to commit to at least one year of higher education or post-secondary training. (33) ... that America would once again have the highest proportion of college graduates in the world by 2020. 1. The opinion that best completes (31) is ... (A) Effort (B) Position (C) Beginning (D) Advantage (E) Pathway 2. The opinion that best completes (32) is ... (A) learning achievement (B) Academic capacity (C) Completion rate (D) Logical understanding (E) Intellectual development

112

3. The opinion that best completes (33) is ... (A) Americans will deserve higher education for their future (B) Middle class Americans are invited to provide financial aids (C) American students are suggested to take entrepreneurial skill (D) The President has set up a new educational goal for the country (E) The government recommends Americans for college education Text 2 Did you ride your bike to school when you were a kid, A generation ago most kids rode, walked or caught the bus to school; very few of us were dropped off by our parents at the school gate. These days most of us have experienced the daily traffic jams around schools at drop-off and pick-up times, as parents drive their children to the school gate. While there is no national data on the number of children who walk or ride to school, a recent Victorian survey found nearly half of all children are driven to school every day. Parents choose to drop their kids at school for a number of reasons — mostly to do with safety and convenience. But experts say chauffeuring your kids to school every day could mean they arc missing out on much-needed exercise and other life skills. Research suggests at least a third of Australian children aged 9-16 years are not getting the amount of daily physical activity recommended in national guidelines. But this is not because children's participation in leisure or sporting activities has dropped off, says Dr Jan Garrard. Participation in these activities has not altered much over the years, Garrard says but what has changed is the level of incidental activity children do. "When you look at countries where children are just active as part of everyday life, they do not have to be sporty. All they have to do is to get around the way the Community gets around by walking and cycling, and they get enough physical activity," she says.

SNMPTN Bahasa Inggris, Kode Soal

4. The author develop some ideas in paragraph 2 by ... (A) describing parents’ chauffeuring followed by its effect (B) explaining reason for chauffeuring and their advantages (C) discussing the function of chauffeuring and the impact (D) arguing for chauffeuring practice for children’s safety (E) exposing how parent chauffer and its drawbacks 5. By writing the sentence “… chauffeuring your kids to school everyday could mean they are missing out on much-needed exercise and other life skills.” (paragraph 2, line 2 -3), the author implies that …. (A) taking kids to school makes them deprived individuals when they grow up (B) kids given a lift to school likely lose vital social and physical advantages (C) schooling means not only learning in classes but also socializing with others (D) parents spoil their kids’ future social and physical life by giving them a lift (E) When a child needs physical and social training, parent should facilitate them 6. Dr. Garrard’s statement “… where children are just active as part of everyday life, they do not have to be sporty… “ (paragraph 3line 5) may be best restated that ... (A) children who are active do not automatically mean they will be good at sport (B) being muscular should not be the aim of children who are naturally active (C) children’s physical fitness is not closely relates with their daily activities (D) activeness in children does not mean to make these children physically fit (E) when naturally active, children need no more scheduled sport activities

7. The pat following the passage above most likely contains information on … (A) advice to parents for their children to have enough physical activities (B) the decreasing trend of children to do physical activities at their will (C) parental motives behind chauffeuring their children to school (D) reasons for children not to do fun and incidental activities (E) effects of having children not to be given a lift to school. Text 3 Agroecologists do not always agree about what agroecology is or should be in the long-term. Different definitions of the term agroecology can be distinguished largely by the specificity with which one defines the term "ecology," as well as the term's potential political connotations. Definitions of agroecology, therefore, may be first grouped according to the specific contexts within which they situate agriculture. Agroecology is defined as "the study of the relation of agricultural crops and environment." This definition refers to the "- ecology" part of "agroecology" narrowly as the natural environment. Following this definition, an agroecologist would study agriculture's various relationships with soil health, water quality, air quality, meso- and microfauna, surrounding flora, environmental toxins, and other envimornental contexts. A more common definition of the word can be taken from Dalgaard et al., who refer to agroecology as the study of the interactions between plants, animals, humans and the environment within agricultural systems. Consequently, agroecology is inherently multidisciplinary, including factors from agronomy, ecology, sociology, economics and related disciplines, in this case, the '-ecology" portion of "agroecology” is defined broadly to include social, cultural, and economic contexts as well.

113

SNMPTN Bahasa Inggris, Kode Soal

Agroecology is also defined differently according to geographic location. In the global south, the term often carries overtly political connotations. Such political definitions of the term usually ascribe to it the goals of social and economic justice; special attention, in this case, is often paid to the traditional farming knowledge of indigenous populations. North American and European uses of the term sometimes avoid the inclusion of such overtly political goals. In these cases, agroecology is seen more strictly as a scientific discipline with less specific social goals. 8. The points provided in paragraph 3 of the passage explain that … (A) different places tend to define agroecoly scientifically (B) interpreting agroecology needs to include political sides (C) agroecology is associated geographically and politically (D) people’s aspiration of agoecology differs significantly (E) no body can define agroecology with scientific precision 9. Ideas is paragraph 2 and 3 define agroecology as shown in the consecutive relation as … (A) political and interactive approaches (B) general and cross authoritative sides (C) multidisciplinary and restricted angles (D) ecology and socio economic viewpoints (E) agricultural and socio cultural perspectives 10. Based on the passage above, if someone is a genuine agroecologist, he/she will likely … (A) examine social, cultural, and economic aspects (B) do research on environmental and political loads (C) include scientific methodology in his/ her approach (D) put aside social aspect in his/her ecological studies (E) make use of multidisciplinary 114

11. regarding the definition of agroecology, the author assumes that it … (A) tends to denote scientific loads (B) may be understood contextually (C) has universal underlying values (D) can be interpreted differently (E) should be exclusively situated Text 4 People still collect books as valuable antiques or for a hobby, but you get virtually all the information you need from the view screen of your home computer system. The computer is linked to a library — not a library of books but an electronic library where information on every subject is stored in computer memory banks. Having this service at your fingertips is like having a huge brand-new encyclopedia in your homes at all times. The computer can tell you anything you want to know, and the information is always the very latest available. There need be only one central library to which computers in homes, offices, schools and colleges are connected. At the library experts are constantly busy, feeding in the very latest information as they receive it. In theory one huge electronic library could serve the whole world! (blogs.smithsonianmag.com) E-books have not spelled the demise of the local library in New York. In fact, according to a new report from the Center for an Urban Future, 40.5 million people visited the city's public libraries, more than all of the city's professional sports teams and major cultural institutions combined. The report "Branches of Opportunity" looks at the changing role of the city's libraries in the digital age. It finds that while public libraries are serving more New Yorkers than ever, they are "undervalued by policymakers and face growing threats. "New York City's library system is a unique hybrid. Three organizations — the New York Public Library, along with the Brooklyn and Queens libraries— operate 206 local branches throughout the five boroughs. (www.wnyc.org)

SNMPTN Bahasa Inggris, Kode Soal

12. Which ides in passage 5 is different from that in passage 4? (A) E-libraries require sophisticated IT expertise (B) Access to information in e-libraries is unlimited (C) Collections of e-libraries are regularly updated (D) E-libraries function as a huge information bank (E) In reality most people are still e-library illiterate 13. Which of the following statements is true according to both passages? (A) People would rather go to the library than watch sporting events (B) People still dream of using libraries in spite of time and space (C) Access to information in an e-library requires a good IT system (D) Libraries provide quick access to free ebooks and newspapers (E) The role of library changes fast due to advanced IT Technology 14. Based on the information in both passages, it can be hypothesized that … (A) Conventional libraries will continue despite of the threats (B) Policies should be made to conserve common libraries (C) Threats to book publishers become more serious (D) Electronic libraries will gain mush popularity (E) Unpopularity of ordinary libraries is obvious 15. The topic discussed in both passages is … (A) the electronic library establishment in the digital era (B) the importance of libraries in the computer era (C) the preference to choosing public libraries (D) the advanced technology in managing libraries (E) the number of public library visitors in New York 115

SBMPTN Bahasa Inggris Latihan Soal 01 - SBMPTN

Text 1-3. Electronic cigarettes: Are they safer than tobacco? Or are they a high-tech way to hook a new generation on a bad nicotine habit? Research into the effect of e-cigarettes lags behind their popularity. But ready or not, the era of e-cigarette is here. It‟s a booming, billion-dollar industry - on track to outsell tobacco products within a decade. The number of teens and tweenagers (children between the ages of about 10 and 14) using these products doubled between 2011 and 2012. 01. What does the passage say about the effects of e-cigarettes? The passage … (A) says that the effects are popular. (B) says that the research of the effect of e-cigarettes is still insufficient (C) says that the effects are good for teens and tweenagers. (D) says that the effects are high-tech. (E) provides no information about the effects. 02. From the passage above, which sentence is a prediction? (A) Electronic cigarettes: Are they safer than tobacco? Or are they a high-tech way to hook a new generation on a bad nicotine habit? (B) Research into the effects of e-cigarettes lags behind their popularity. (C) It‟s a booming, billion-dollar industry on track to outsell tobacco products within a decade. (D) The number of teens and tweenagers (children between the ages of about 10 and 14) using these products doubled between 2011 and 2012. (E) Nothing.

116

03. Which one is TRUE according to the text? (A) Electronic cigarettes are safer than tobacco. (B) E-cigarette effects are unpopular. (C) The number of people using e-cigarettes has double between 2011-2012. (D) The number of e-cigarettes sold is greater than that of normal tobacco products. (E) Some teens use e-cigarettes. Text 4-8. Marie Curie was a Polish-born French scientist who, with her husband, Pierre Curie (1859-1906). was an early investigator of radioactivity. From 1896. the Curies worked together. building on the results of Henri Becquerel. who had discovered radioactivity from uranium salts. Marie Curie discovered that thorium also emits radiation and found that the mineral pitchblende was even more radioactive than could be accounted for by any uranium and thorium content. The Curies then carried out an exhaustive search and in July 1898 announced the discovery of polonium, followed in December of that year with the discovery of radium. They shared the 1903 Nobel Prize for physics with Becquerel for the discovery of radioactivity. The Curies did not participate In Becquerel‟s discovery but investigated radioactivity and gave the phenomenon its name. Marie Curie went on to study the chemistry and medical applications of radium, and was awarded the 1911 Nobel Prize for chemistry in recognition of her work in isolating the pure metal. At the outbreak of World War I in 1914. Marie Curie helped to equip ambulances with X-ray equipment and drove the ambulances to the front lines. The International Red Cross made her head of its Radiological Service. She taught medical orderlies and doctors how to use the new technique. By the late 1920s. her health began to deteriorate: continued exposure to high energy radiation had given her leukemia. She entered a sanatorium and died

SBMPTN Bahasa Inggris, Latihan Soal 01 - SBMPTN

on July 4. 1934. Throughout much of her life, Marie Curie was poor. and the painstaking radium extractions were carried out in primitive conditions. The Curies refused to patent any of their discoveries, wanting them to benefit everyone freely. They used the Nobel Prize money and other financial rewards to finance further research. One of the outstanding applications of their work has been the use of radiation to treat cancer, one form of which cost Marie Curie her life. 04. Which one is TRUE according to the passage? (A) Marie Curie and her husband discovered radioactivity from uranium salts. (B) Marie Curie got only one Nobel Prize for physics. (C) The Curies had worked together with Becquerel in a research. (D) Marie Curie financed her researches with their patent money. (E) Marie Curie participated in the World Wars. 05. Why didn‟t the Curies patent their discoveries? (A) They already had some money from Nobel Prize. (B) They were considered wealthy at that time. (C) They wanted their research to be used freely. (D) They were already famous. (E) Becquerel had already patented their discoveries. 06. The phrase “the pure metal” (paragraph 1) refers to … (A) uranium. (B) radiation. (C) thorium. (D) radium. (E) polonium. 07. From the text we know the following, EXCEPT … (A) A research of the Curies was based on a Becquerel‟s research. (B) Marie Curie passed away after Pierre Curie had passed away.

(C) The curies had discovered polonium before radium. (D) The mineral pitchblende was more radioactive than thorium content. (E) Nobel Prize only recognized Marie Curie and Becquerel for the discovery of radioactivity. 08. What is the best title of the passage? (A) How Radioactivity Changed the World. (B) Science and its applications. (C) The Discoveries of Marie Curie. (D) The Competition between Marie Curie and Henri Becquerel. (E) The History of Radioactivity. Text 09-15. Between 13,000 and 12,600 years ago, members of the Clovis culture appeared in North America, where they made and used distinctive stone-tipped spears to hunt mammoth, bison and mastodon. Until recently, all that archeologists knew about the Clovis people came from studying their tools, which have been unearthed at wide-ranging sites across the country. Now, DNA analysis of a single human skeletonthat of a one-year-old boy buried in a rocky field in modern-day Montana - has allowed scientists to link the Clovis culture to Native Americans throughout the Western Hemisphere. Construction crews first discovered the ancient remains of an infant in 1968 on private property owned by the Anzick family in western Montana. Dubbed Anzick-1, the one-year-old boy is the only human skeleton that has been identified as a member of the widespread, sophisticated Ice-Age culture known as Clovis. Now, a team of scientists has succeeded in mapping the infant‟s DNA, in the oldest genome sequence of an American individual ever performed. According to their findings, published in the journal Nature in February 2014, the Clovis people are direct ancestors of many Native Americans now living in North America, and can be linked to many native peoples in Central and South American as well. Up to this point, all scientists studying the Clovis culture had to go on where the 117

SBMPTN Bahasa Inggris, Latihan Soal 01 - SBMPTN

stone and bone tools have been found at sites ranging from Washington State to Florida, along with many states in between. By sequencing the genome of the infant recove re d a t the A nzick site, the international team of researchers gained the most vivid insight yet about who these people might actually have been. They compared the DNA of the Clovis infant to several different genomes, including a 24,000 -year-old sample from a young man buried on the banks of Lake Baikal in Siberia, a 7,000-year-old sample from Spain and a 4,000-year-old sample from Greenland. The Clovis DNA showed the most similarity with that of the Siberian youth, whom scientists genetically linked with today‟s Native Americans late last year. The new study adds to existing archeological evidence that Native American descended from humans who migrated to North America from Asia through Siberia around 15,000 years ago. They are believed to have made the voyage across the Bering land bridge, which connected Asia with North America during the last Ice Age. According to archeologist Michael Waters of Texas A&M University, a member of the team who conducted the new study, the genetic evidence “strongly suggests that there was a single migration of people into the Americas … [T]hese people were probably the people who eventually gave rise to Clovis.” Such evidence casts doubt on other theories arguing that Clovis‟ ancestors came from Europe, rather than Asia. Such hypotheses rely partially on the fact that the “Clovis points” found on their tools and weapons are so similar to the flint tools used by the Solutrean culture, which flourished in Spain and France during the Ice Age. While Anzick-1 showed the most genetic similarities with Native Americans in North America, the study also revealed ties with the indigenous peoples of Central and South America. The team‟s data indicates that sometime between 13,000 and 24,000 years ago, the same ancient people that arrived from Asia split into two lineages: One gave rise to Clovis and today‟s Native Americans of North America, and the other became the 118

ancestors of Central and South American tribes. The scientists studying Anzick-1 have worked closely with Native American tribes in Montana, sharing the results of the study with them and ensuring that the remains were treated appropriately. The infant will be reburied later this year, on the same property from which he was unearthed. For their part, the tribes have shown little surprise at the scientists‟ conclusions. Shane Doyle, a professor of Native American History at Montana State University and co-author on the study, is also a member of the Crow tribe. As he told NBC News, after conversations with more than 100 tribe members, the main reaction was “We have no reason to doubt that we‟ve been here for this long.” 09. What was Clovis? (A) a widespread. sophisticated Ice Age culture in North America. (B) a culture that existed in Greenland about 4.000 years ago. (C) a culture that flourished in Spain and France during the Ice Age. (D) a culture that gave rise to Central and South American tribes. (E) a skeleton found in Montana. 10. The text describes a possible sequence of human descent. At the end of the sequence are Native Americans. What people are at the beginning of the sequence? (A) People who migrated from Asia to North America about 15,000 years ago. (B) Tribes living in Central and South America today. (C) People who were living in Greenland about 4,000 years ago. (D) People who were living in Spain about 7.000 years ago. (E) The Crow Tribe. 11. The Clovis people descended from humans who migrated to North America from Asia through Siberia around 15,000 years ago. What evidence supports this theory?

SBMPTN Bahasa Inggris, Latihan Soal 01 - SBMPTN

(A) the similarity between DNA of a Clovis infant and the DNA of a member of the Crow tribe. (B) the similarity between the DNA of a Clovis infant and the DNA of a person from Iceland who lived 4,000 years ago. (C) the similarity between the DNA of a Clovis infant and the DNA of a person from Spain who lived 7,000 years ago. (D) the similarity between the DNA of a Clovis infant and the DNA of a Siberian youth who lived 24,000 years ago. (E) the similarity between the DNA of a Clovis infant and the DNA of all European people. 12. What is the main idea of the passage? (A) Construction crews discovered the ancient remains of an infant in 1968 in western Montana. (B) Similarities exist between the tools of the Clovis people and the tools used by members of the Solutrean culture. (C) DNA analysis of an ancient infant‟s remains has allowed scientists to link the Clovis culture to Native Americans. (D) Scientists studying the remains of an ancient infant worked closely with Native American tribes in Montana to ensure that the remains were treated appropriately. (E) DNA analysis of an ancient infant‟s remains has been approved by scientists. 13. The Clovis people descended from humans living in Europe. What evidence supports this theory? (A) the similarity between the tools of the Clovis people and the tools of people in Siberia. (B) the similarity between the tools of the Clovis people and the tools of people in Montana. (C) the similarity between the tools of the Clovis people and the tools of people in France and Spain.

(D) the similarity between the tools of the Clovis people and the tools of people in Greenland. (E) the similarity between the tools of the Clovis people and tools of people in the Crow tribe. 14. Read this sentence from the text. „According to their findings, published this week in the journal Nature, the Clovis people are direct ancestors of many Native Americans now living in North America. and can be linked to many native peoples in Central and South America as well.‟ What does the author mean by writing that the Clovis can be linked to many native peoples in Central and South America? (A) The author means that many native peoples in Central and South America could communicate with the Clovis people. (B) The author means that many native peoples in Central and South America are related to the Clovis people. (C) The author means that the Clovis people used the same technology as many native peoples in Central and South America. (D) The author means that the Clovis people got along well with many native peoples in Central and South America. (E) The author means that the Clovis people went to war with many native peoples in Central South America. 15. Read these sentences from the text. „While Anzick-1 showed the most genetic similarities with Native Americans in North America. the study also revealed ties with the indigenous peoples of Central and South America. The team‟s data indicates that sometime between 13.000 and 24,000 years ago, the same ancient people that arrived from Asia split into two lineages: One gave rise to Clovis and today‟s Native Americans of North America, and the other became the ancestors of Central and South American tribes.‟ What word could bet replace While in the first sentence? 119

SBMPTN Bahasa Inggris, Latihan Soal 01 - SBMPTN

(A) (B) (C) (D) (E)

120

Currently. Although. Consequently. Finally. If.

SBMPTN Bahasa Inggris Latihan Soal 2 - SBMPTN

Text 1-3. Ancient Mars harbored long-lasting lakes, boosting the odds that life could have existed on the Red Planet billions of years ago, a new study suggests. A series of freshwater lakes within Mars' 96-mile-wide (154 kilometers) Gale Crater likely persisted for hundreds or thousands of years at a time, and perhaps even longer, according to the new study, which is based on observations made by NASA's 1-ton Curiosity rover. While these individual lakes were apparently transient, drying out and filling up repeatedly over time, the overall lake-andstream system inside Gale Crater existed for a quite a long time, researchers said. "Even if the lake goes away, there's still going to be a groundwater table," study lead author John Grotzinger, of the California Institute of Technology in Pasadena, told Space.com. "If life had evolved on Mars, you now have a habitat which is perpetually wet that would allow microbes to be sustained," added Grotzinger, a Curiosity team member who previously served as project scientist on the $2.5 billion mission. "Those environments would have existed probably for millions, if not tens of millions of years throughout the rocks that we see." [Source: http://www.space.com/30778-ancient-marslakes-curiosity-rover.html] 01. The underlined “transient” in paragraph 3 most likely means … (A) transparent. (B) free from moisture. (C) temporary. (D) arid. (E) perpetual.

02. Which one is WRONG according to the text? (A) It is probable that there was freshwater under ancient Mars’ Gale Crater. (B) Ancient Mars’ Gale Crater might have had lakes underneath. (C) The constantly wet habitat in the Gale Crater might support the life of microbes. (D) Each lake within the Gale Crater was 154 kilometers in width. (E) When the lakes dry out within the Gale Crater, a researcher thinks that there is another source of water. 03. The best title for the passage is … (A) The Gale Crater. (B) Life Could Have Existed in Mars. (C) NASA’s Curiosity Rover. (D) How Old is Mars? (E) Human Quests for Freshwater. Text 4-8. The Business of tennis clothes has grown astoundingly in past few years. Over $250 million is spent annually on the trapping of tennis. Apparently everyone wants to look like a pro, even though 20% of the clientele has never even played the game. Manufacturers pay the stars lucrative fees for wearing their brands of clothes and wielding their racquets on center court. Chris Evert-Lloyd, for example, is rumored to have signed a five-year contract for $5 million with Ellesse, a producer of fancy, expensive tennis wear. John McEnroe received a reported $600,000 for playing with a Dunlop racquet, $330,000 for sporting Tacchini clothes, and $100,000 for typing his Nike tennis shoes. Obviously, in a bad year, these stars would have made more as fashion models than as athletes.

121

SBMPTN Bahasa Inggris, Latihan Soal 2 - SBMPTN

Not only tennis players get free clothing, but also all the people involved in the game-the referees, linespeople, ball boys and girls-are living advertisements for tennis wear producers. Where, traditionally; conservative white clothing was required for the entire tennis coterie, changing times have seen a new vogue in tennis outfits. Flamboyant colors, designers' nameplates, geometric figures, and bold lines distinguish the new tennis togs from their predecessors. 04. A good title for this passage would be … (A) The Stars at Play. (B) Big Business in Tennis Wear. (C) The High Cost of Playing Tennis. (D) Tennis Stars’ Flamboyant Clothes. (E) Designers and Tennis Court. 05. It is stated that John McEnroe … (A) wore flamboyant clothings on the court. (B) must have earned over $1 million for endorsing tennis products. (C) was a fashion model more than he was a tennis player. (D) had had a bad year in tennis competition. (E) wore tennis clothings with geometric figures and bold lines. 06. It can be inferred from the passage that … (A) tennis clothing appeals only to the wealthy (B) conservative tennis clothes don’t tend to have designer brands on it. (C) the price of tennis racquets has remained stable. (D) bright colors entice people to buy tennis wear. (E) the tennis referees may get free clothes. 07. The author’s intention is to … (A) explain why the cost of tennis clothes has risen. (B) defend tennis wear manufacturers from complaints about their high prices. (C) describe the means of advertising expensive tennis clothes. (D) describe the new tennis clothing. (E) explain why tennis has become a more difficult game. 122

08. Obviously. in a bad year. these stars would have made more as fashion models than as athletes. Who are “these stars” mentioned in the text? (A) stars that use Dunlop racquets. (B) stars that get endorsement from certain tennis products. (C) stars that wear white clothing in the tennis court. (D) stars that have photoshoots contracts. (E) stars that have changed the trend in tennis Text 9-15. Today, the Viking are mostly known as violent pirates and raiders. And it is true that Vikings did raid and destroy many towns and villages along coastlines, all the way from what is now northern Russia to Morocco. But the Vikings were also traders and merchants and didn’t simply destroy things. They also built towns and markets of their own, including Hedeby, which in the 10th century had a population of 1,500, making it the largest trading town in Northern Europe. At their height, the Vikings attacked, settled or traded on four continents. They were active all the way from Canada (they became the first Europeans to travel to the Americas) to present day Istanbul.

SBMPTN Bahasa Inggris, Latihan Soal 2 - SBMPTN

All of their travel, trade and warfare were made possible by Viking ships, which were far more advanced than anything else sailing around Europe at the time. The most famous, and most feared, was the drekar, or longship. At sea, these ships could move quickly thanks to their large sails. The hulls or the body - of the ships were shallow and fat, which made them ride high in the water. This meant they could be driven right onto beaches, where the soldiers would jump over the side to attack and plunder villages and cities. The ships were also light enough that they could be carried from one body of water to another over short sections of land called portages. This greatly extended their range. Several such drekar ships were found off the coast of Roskilde, formerly the capital of Denmark, between 1957 and 1962. The longest drekar measured 119 feet long with a crew of 100 men and space for 72 oars. With its gigantic sail, shallow hull and so many oarsmen, the ship must have been incredibly fast and highly maneuverable. But Viking ships weren’t just built for warfare. Another type, called the knarr or ocean-going ship, had cargo holds built into the bow - the front part of the ship - and stern - the back part. One such cargo ship discovered near Roskilde was capable of carrying 24 tons. The knarrs would have looked similar to the drekars except they were longer, fatter and taller, and the space dedicated to cargo left less room for oarsmen. These were the backbones of the Viking empire, which they used to carry everything from gold coins to timber, spices and fine fabrics. Both the drekar and the knarr were built using the same method called the clinker method. Traditionally, ocean-going ships have used a keel the bottom of the ships, shaped like the fin of a fish. The keel sinks into the water below the hull. It helps the ship maintain a straight line through the water and counters the force of the wind against the sail, which otherwise might blow the ship over. Traditional ships are also built with ribs which function just like the ribs on a human being, starting at the spine and growing out in a curve to protect the space inside.

Using the clinker system, Viking ships had no deep keel. Instead they were built fat enough to carry lots of soldiers or pieces of cargo whose weight helped keep the ship planted in the water. The construction process started with a heavy piece of wood at the bottom. From there, oak tree trunks were split into long, thin planks. Two planks were fastened to the bottom piece, and then each plank was fastened to the one before it like overlapping shingles on the roof of a house. A massive beam was laid across the bottom to strengthen it and also to support the mast. Finally, crossbeams were laid inside to create a deck and benches for oarsmen to sit. The result was sturdy, fast and light. Viking ships were so advanced for their time they often were the biggest, tallest and most striking ships many people had ever seen. The Vikings made them even more intimidating using bright colors and intricate designs. A monk at the St. Omer Monastery, in France, wrote this description of a royal Viking ship in 1013: “On one side lions molded in gold were to be seen on the ships, on the other birds on the tops of the masts indicated by their movements the winds as they blew, or dragons of various kinds poured fire from their nostrils...” The description makes clear that Vikings were not simple marauders. They built a wealthy empire through trade as well as plunder and used their wealth to continuously improve their ships. [source: www.readwork.org/get/246321] 09. What is a drekar? (A) a Viking town. (B) a merchant ship. (C) a longship. (D) an ocean-going ship. (E) the Vikings’ neghbor

123

SBMPTN Bahasa Inggris, Latihan Soal 2 - SBMPTN

10. The author tries to persuade the reader of what? (A) Vikings were only violent pirates and raiders. (B) There was no connection between the Vikings’ success and their ships. (C) Viking ships were more advanced than ships today. (D) Vikings were more than pirates and raiders. (E) Vikings’ knowledge of ships surpassed the modern-day knowledge of ships.

13. Read the following sentences: Viking ships were so advanced for their time they often were the biggest, tallest, and most striking ships many people had ever seen. The Vikings made them even more intimidating using bright colors and intricate designs.” What does striking” mean in this sentence? (A) Violent. (B) Marauding. (C) Impressive. (D) Delightful. (E) Offensive.

11. The Vikings considered speed an important quality in a ship. What evidence from the passage supports this conclusion? (A) The hulls of the drekar were shallow and fat so the ships rode high in the water (B) The drekar had very large sails and space for many oarsmen. (C) The drekar were light enough to be carried from one body of water to another. (D) The drekar could be driven right onto beaches to allow soldiers to jump over the side. (E) The drekar and the knar are the fastest ships in the world.

14. Vikings designed and used their ships for multiple purposes. … warfare, trade, and travel. (A) finally (B) although (C) ultimately (D) Including (E) similar

12. Read the following description of the knarr: “The knarr would have looked similar to the drekars except they were longer, fatter, and taller, and the space dedicated to cargo left less room for oarsmen. These were the backbones of the Viking empire, which they used to carry everything from gold coins to timber, spices, and fine fabrics.” What can you infer about the knarrs? (A) They were not designed for warfare. (B) They were faster than the drekars. (C) They were designed to carry soldiers. (D) They were used for the same purpose as drekars. (E) They were owned by the wealthier Vikings.

124

SBMPTN Bahasa Inggris Latihan Soal 3 - SBMPTN

Text No 01-05 L.L Zamenhof would later say that he had dreamed of a world language since he was a child. At first he considered a revival of Latin , but after learning it in school he decided it was too complicated to be a common means of international communication. When he learned English, he realized that verb conjugations were unnecessary, and that grammatical systems could be much simpler than he had expected. He still had the problem of memorizing a large vocabulary, until he noticed two Russian signs labeled Швейцарская – from швейцар, which means porter’s house and porter, respectively – and Кондитерская – from кондитер, which mean a confectioner’s shop and confectioner, respectively. He then realized that a judicious use of affixes could greatly decrease the number of root words needed for communication. He chose to take his vocabulary from Romance and Germanic , the languages that were most widely taught in schools around the world and would therefore be recognizable to the largest number of people. Zamenhof taught an early version of the language to his high-school classmates. Then, for several years, he worked on translations and poetry to refine his creation. In 1895 he wrote, "I worked for six years perfecting and testing the language, even though it had seemed to me in 1878 that it was already completely ready." When he was ready to publish, the Czarist censors would not allow it. Hampered, he spent his time in translating works such as the Bible and Shakespeare . This enforced delay led to continued improvement. In July 1887 he published his first book Unua Libro, a basic introduction to the language. This was essentially the language spoken today. 01. Who is L.L Zamenhof as stated in passage? (A) He was a language teacher. (B) He was the creator of Esperanto. (C) He translated many languages to Esperanto. (D) He was a fan of the Bible and Shakespeare. (E) He worked for a publishing company.

02. Which one id NOT TRUE according to paragraph 2? (A) Zamenhof created the early Esperanto in high school. (B) Zamenhof had ever worked as a translator. (C) Because of a censorship policy. Zamenhof’s publication had been delayed. (D) Esperanto is spoken today. (E) The first Zamenhof book Unua Libro has yet to be published. 03. Why did Zamenhof decide to take his vocabulary from Romance and Germanic languages? Because…. (A) Romance and Germanic languages had difficult grammar. (B) Romance and Germanic languages had greater exposure to people. (C) Romance and Germanic languages had a lot of strong vocabulary. (D) Romance and Germanic languages had many affixes. (E) Romance and Germanic language learning were still limited. 04. What can be inferred from paragraph 1? (A) L.L Zemenhof thought that Latin was too complicated to use globally. (B) Latin language was not quite used by people anymore. (C) English was a global language. (D) Russian language had a larger number of root words. (E) Verb conjugations in English was already much simpler. 05. What is the topic of the passage? (A) the revival of Latin. (B) the early creation of Esperanto (C) L.L Zamenhof as a child (D) Esperanto versus English (E) the vocabulary of Esperanto

125

SBMPTN Bahasa Inggris, Latihan Soal 3 - SBMPTN

Text No 06-09 If you’re a man, how much you eat may have more to do with the gender of your dining companions than your appetite. A new Cornell University study, published in the journal Evolutionary Psychological Science, found that men will eat significantly more food in the company of women than they will with other men. For the study, researchers observed 105 adults lunching at an all-you-can-eat Italian buffet over the course of two weeks. They recorded the number of pizza slices and how many bowls of salad each diner ate. Gender of each diner’s eating partner or partners was also noted. Before leaving the restaurant, the diners were intercepted by a researcher to ask them to complete a short survey indicating their level of fullness after eating, and their feelings of hurriedness and comfort while eating. Men who dined with at least one woman ate 92% more pizza and 86% more salad than men who dined with only other men. The amount that women ate didn’t differ when eating with other women or with men. When they ate with men, many women indicated feeling that they overate and were rushed through their meal. ―These findings suggest that men tend to overeat to show off – you can also see this tendency in eating competitions which almost always have mostly male participants,‖ explains lead author Kevin Kniffin, PhD, of Cornell University. The study was conducted by Kniffin, Ozge Sigirci, a former visiting scholar at the Cornell University Food and Brand Lab, and Brian Wansink, professor and director of the Food and Brand Lab. 06. According to the research, why does the man in the study tend to overeat? (A) To show his habit. (B) To gain admiration from all of their eating companions. (C) To impress female eating companions. (D) To get the full benefit from eating for free. (E) To satisfy their hungry stomach.

07. The following are true according to the text, EXCEPT ... (A) there are 105 participants in the research (B) through observation, the researchers wrote down how much the participants ate (C) women tend to eat the same amaount of food when eating with either gender (D) men tend to eat more when eating with male friends (E) the research involves not only observation but also survey 08. What can replace the phrase ―in the company of‖?(paragraph 1) (A) associated with (B) from the same corporate with (C) with a group of (D) than (E) in a building with 09. What is the main idea of the text above? (A) men will eat more food when eating with women than they will with other man (B) people will eat more food when eating with a different gender (C) many women overeat when eating with men (D) men tend to participate in eating competitions (E) the study was conducted for two weeeks Text No 10-15 When parents have high hopes for their children’s academic achievement, the children tend to do better in school, unless those hopes are unrealistic, in which case the children may not perform well in school, according to research published by the American Psychological Association. ―Our research revealed both positive and negative aspects of parents’ aspiration for their children’s academic performance. Although parental aspiration can help improve children’s academic performance, excessive parental aspiration can be poisonous,‖ said lead author Kou Murayama, PhD, of the University of Reading. The study was published in the Journal of Personality and Social Psychology®. 126

SBMPTN Bahasa Inggris, Latihan Soal 3 - SBMPTN

Murayama and his colleagues analyzed data from a longitudinal study from 2002-07 of 3,530 secondary school students (49.7 percent female) and their parents in Bavaria, Germany. The study assessed student math achievement as well as parental aspiration (how much they want their child to earn a particular grade) and expectation (how much they believe their child can achieve a certain grade) on an annual basis. They found that high parental aspiration led to increased academic achievement, but only when it did not overly exceed realistic expectation. When aspiration exceeded expectation, the children’s achievement decreased proportionately. To reinforce the results, the researchers attempted to replicate the main findings of the study using data from a two-year study of over 12,000 U.S. students and their parents. The results were similar to the German study and provided further evidence that parents’ overly high aspirations are associated with worse academic performance by their kids. Previous psychological research has found the association between aspiration and academic achievement, but this study highlights a caveat, said Murayama. ―Much of the previous literature conveyed a simple, straightforward message to parents — aim high for your children and they will achieve more,‖ said Murayama. In fact, getting parents to have higher hopes for their children has often been a goal of programs designed to improve academic performance in schools. This study suggests that the focus of such educational programs should not be on blindly increasing parental aspiration but on giving parents the information they need to develop realistic expectations. ―Unrealistically high aspiration may hinder academic performance. Simply raising aspiration cannot be an effective solution to improve success in education,‖ he said.

127

10. Which one is TRUE according to the text? (A) parental aspiration may decrease students academic achievement (B) children do better in school whather parental aspiration is high or not (C) the US study has rebutted the german study (D) the academic achievement measured in the german study involves german and math achievements (E) ambitious parents always lead to ambitious and more intelligent students 11. Which data are collected in the German research? (A) Math achievement and parental mathematical performance. (B) Some positive and negative aspects of parental aspiration. (C) Students’ math achievement, parental aspiration, and parental expectation. (D) Students’ math achievement, hope, and ambition. (E) How parental aspiration can influence children’s academic performance. 12. What is the goal of the US study as shown in paragraph 5? (A) to measured parental aspiration (B) to change the results of the german study (C) to support the results of the aforementioned German study (D) to tackle the problems of low academic achievement in students (E) to create an environment where students can achieve higher in learning

SBMPTN Bahasa Inggris, Latihan Soal 3 - SBMPTN

13 What make the German and Us studies different from much of the previous literature? (paragraph 7) The German and the US studies …. (A) analyze academic achievement only from math subject, while the previous literature analyze academic achievement from numerous school subjects. (B) find that aspirations of parents may not always result in good academic performance. (C) provide association between parental aspiration and students’ academic achievement. (D) used data from Germany and the United States, while the previous literature used data from the rest of the world. (E) propose that parents should lower their aspiration toward students’ grades. 14. The passage mostly tells us that ... (A) parents should not involve in children’s academic performance (B) parents should copy the experiment in Germany (C) parents expecting too much may harm a child academic achievement (D) a student must not take parental advice to be a better performer at school (E) parents need to aim higher for their children 15. The underlined word ―hinder‖ (last paragraph) is mostly similiar to ... (A) hamper (B) facilitate (C) affect (D) increase (E) enlarge

128

SNMPTN Bahasa Inggris Kode Soal We all know that mobile phone, cellphones, hand-phone, whatever we want to call them (and shouldn‟t we all be calling them the same thing?) are changing our lives. But it takes a good old fashioned survey to wake us up to the glaring reality they have changed who we are The mobile phone has indeed changed the way we behave But perhaps we don‟t realize how much we have become its slave. Consider other elements of the Siemens Mobile Survey With the exception of Australia, in every country surveyed the majority polled said they would go back for their phone if they left it all home (In Australia it was a respectable 39%). If you‟ve endured the traffic in Indonesia the Philippines and India, you‟ll know what kind of sacrifice some two-thirds of those surveyed are making I can‟t think of anything I would go back for-except my wallet, maybe, or my clothes And even if we remember to bring it, we‟re still not happy. Many out us get anxious if it hasn‟t rung or a text massage hasn‟t appeared for a while being about an hours). Once again of those surveyed Indonesian (65%) and Filipinos (77%) get paticularry jittery. Australian are more laid back about this (20%) but every other user in Asia seems to be glancing at the phone avery view second. This statistic, I have to say, is highly believable, and the instinct highly annoying There‟s nothing worse than chatting to some one who constantly checks his or her handphone Then there‟s the fact that mobile phones are not only enslaving the user. they’re trampling the right of everyone else. Around a third of folk surveyed acknowledge they get so engrossed in mobile conversation that they re often unaware of speaking loudly while discussing their private lives in public At least most of us agree on one thing With the exception of China, Hong Kong and Taiwan, the increasing use of mobile phones has led to a decline in courtesy and considerate behavior

129

SNMPTN Bahasa Inggris, Kode Soal

The bottom line here is that we are more than a little bit out of control. Mobile phones are great but if we allow them to dominate our lives to this extent-interrupting conversations with those around us to take a call, starring at our phones rather than relating to the world and people around as, sending fifty text massages to random numbers-then I can only assume that in another 10 years, society as we know it will no longer exist All we‟ll see is a blur of digital data going out and having all the fun, socializing, falling in love, and taking skeaky pictures of each others . 01. Which of the following statements is NOT TRUE about cellphones? (A) By using cellphones people can change their lives (B) People become upset when they forget to bring their cellphone (C) Cellphones have made users quite enslaved (D) The writer is not one who is enslaved (E) The change in people‟s behavior may becaused by cellphone 02. The main purpose of the writer is to inform the readers about .... (A) the popularity of communication technology (B) the effects of cellphones on the behavior of users (C) the various kind of hand-phone available in the market (D) the adventage of using cellphones in many Asian country (E) the percentage of cellphones users in Asia 03. Cellphones have not only enslaved the users but have also .... (A) Disturbed other people‟s conversation in public (B) Eliminated the existence of social life (C) Made the users behave impolitely (D) Prevented people from socializing with each other (E) Caused the user to become quite considerate 04. What makes Asian users of cellphones different from Australian ones? (A) They mostly use their cellphone for chatting (B) The number od users in Asia is greater than in Australia (C) They become angry when nobody calls or send messages (D) Their callphones ring every hour throught the day (E) They are much more dependent on their cellphones

130

SNMPTN Bahasa Inggris, Kode Soal

05. The phrase trampling the right of everyone else in lines 23—24 means .... (A) Interfering with somebody else affairs (B) Prohibiting somebody else‟s right to talk (C) Rejecting to acknowledge one‟s ownership (D) Ignoring the rights of the other people (E) Making use of other people‟s property 06. Text II Due to the cases of salmonella food poisoning in Europe, the sale of duck eggs reached its lowest point in the 1970‟s. Although it was never conclusively shown that duck were to blame the egg-eating public stopped buying and many egg producers went bankrupt indeed, there is a risk salmonella poisoning when ducks lay their eggs in damp conditions. such as on ground that constantly wet, but the same can be said for the eggs of hens. Moreover, comercial duck production in France and England, where the outbreaks of salmonella poisoning took place followed, the same standars as those used in the hen industry, which experienced no salmonella problem. Storage of egg‟s whethet those to hen or duck , can also be a factor in contamination. Studies have found that bacterial growth reaches polentially dangerous levels at storage temperatures to 500C or greater. __________________________________ __________________________________ ___________ With which of the following sentences should the paragraph end? (A) Yet, it has never been clear what caused Salmonella food poisoning in the 1970s. (B) Because of these many duck egg producers stopped their business in the 1970‟s (C) As a result, eggs which are laid in damp conditions can cause Salmonella food poisoning (D) Consequently, it was also possible that the food poisoning was caused by the bad storage system or eggs (E) Therefore, wet ground and bad storage system of eggs could be the cause of decreased sale of duck eggs the 1970‟s 131

07. Text II Due to the cases of salmonella food poisoning in Europe, the sale of duck eggs reached its lowest point in the 1970‟s Although it was never conclusively shown that duck were to blame the egg-eating public stopped buying and many egg producers went bankrupt indeed, there is a risk salmonella poisoning when ducks lay their eggs in damp condition. such as on ground that constantly wet, but the same can be said of the eggs of hens. Moreover, comercial duck production in France and England, where the outbreaks of salmonella poisoning took place followed, the same standars as those used in the hen industry, which experienced no salmonella problem. Storage of egg‟s whethet those to hen or duck , can also be a factor in contamination. Studies have found that bacterial growth reaches polentially dangerous levels at storage temperatures to 500C or greater. _________________________________ _________________________________ _____________ What is the topic of the paragraph? (A) Salmonella food poisoning as the cause of the colapse of duck business in Europe (B) The increase of commercial duck egg production in Europe in 1970 (C) The effect it Salmonella food poisoning on the sale duck eggs (D) Ways to increase the sale of duck eggs in Europe in 1970‟s (E) Wet ground and bad storage system of eggs as the causes of Salmonella food poisoning

SNMPTN Bahasa Inggris, Kode Soal

08. Text III (1) __________________________________ __________________________________ (2) Biodiesel is free or lead, contains virtually no sulfur and produces loser, quantities of cancer causing emissions than petro diesel. (3) In particular, using biodiesel in school buses make a lot of sense (4) Young children are more susceptible than adults to the toxic and potentially cancer-causing emissions from petro diesel. (5) Many teachers are also suffering from asthma (6) this fact has led more than 50 school boards across nation to require that their buses use biodiesel fuel (7) This cleaner burning fuel is also an attractive option in recreation areas (8) Yellow stone National Park was the first national park to test biodiesel as a fuel, and the project was a succes that the National Park Service has introduced biodiesel to 20 other parks across the country Which sentence does not belong in the paragraph? (A) three (B) four (C) five (D) six (E) seven

09. Text III (1) _________________________________ _________________________________ __ (2) Biodiesel is free or lead, contains virtually no sulfur and produces loser, quantities of cancer causing emissions than petro diesel. (3) In particular, using biodiesel in school buses make a lot of sense (4) Young children are more susceptible than adults to the toxic and potentially cancer-causing emissions from petro diesel. (5) Many teachers are also suffering from asthma (6) this fact has led more than 50 school boards across nation to require that their buses use biodiesel fuel (7) This cleaner burning fuel is also an attractive option in recreation areas (8) Yellow stone National Park was the first national park to test biodiesel as a fuel, and the project was a succes that the National Park Service has introduced biodiesel to 20 other parks across the country With which of the following sentences should the paragraph begin? (A) Biodiesel is now compulsory for public transport across the country because it is cleaner (B) Biodiesel is catching on in many markets, largely because it is a cheaper burning fuel (C) Because is a cheaper burning fuel, biodiesel has become compulsory in all markets. (D) Biodiesel has replaced petro diesel as a fuel for school buses because it is cleaner (E) As it does not produce as many pollutants, biodiesel‟s popularity is decreasing

132

SNMPTN Bahasa Inggris, Kode Soal

Text IV Small genetic difference makes one person different from another. Now medical researchers have a new map to help them find these _______(10)_______This is possible because some time ago more than two hundred scientist from six nations _______(11) ____ the Hap Map. The name comes from the word haplotype . A haplotype is a group of differences that are ______(12)______to come close together, in a block These blocks_____(13)____ to pass from parent to child The Hap Map scientist hope to identify up to six millions DNA differences ______(14)______They finish. The scientist say the finding may lead to ____(15) ___ genes that cause common diseases like diabetes and heart disease_____(10) ____diseases to genes coul lead to new treatment______(17)_____people will be able to know if they in have increased risk of a disease because to their genes. 10. (A) advantage (B) consequences (C) comparisons (D) characteristics (E) differences 11. (A) develop (B) developed (C) have developed (D) had developed (E) were developing 12. (A) similarly (B) comparatively (C) likely (D) timely (E) accordingly 13. (A) emerge (B) create (C) contimate (D) appear (E) establish 14. (A) after (B) while (C) before (D) at the time (E) afterwards 133

15. (A) identify (B) identify (C) identification (D) identifiable (E) identifying 16. (A) linking (B) finding (C) curing (D) diagnosing (E) knowing 17. (A) However (B) As a result (C) In short (D) Moreover (E) In addition 18. You didn‟t like the horror film you saw yesterday, did you? „You‟re right, I really wish _____‟ (A) I did not see it (B) I would not see it (C) I have not seen it (D) I had not seen it (E) I would not have seen it 19. For the past few year the rate of crime has been high due to the increase in the rate of unemployment. To day people wish that the local government ___first priority to the solution of this problem. (A) gives (B) would give (C) will give (D) should give (E) had given

SNMPTN Bahasa Inggris, Kode Soal

20. More government policies related to people‟s welfare ____ to help people who have suffered because of the drastic oil price hike. (A) are to be implemented (B) implement (C) are to implement (D) to be implement (E) are implementing 21. Your car is badly damage, I need more than two days to repair it (It‟s ok, _____ then.) (A) Take your time (B) You can not finish it (C) Finish it on time (D) Do you need two day (E) Do not repair it 22. So far turn term, the student in the writing class have learned how to write these statement, _____ and summarite Their conclusion. (A) The materials organization (B) The organizing of their materials (C) Organize their materials (D) Their organization of their materials (E) Now their materials are organized 23. What government should do to create pilotri Stability 2 (A) Law enforcement should work effectively (B) The effective work of law enforcement (C) Enforcing the low to work effective (D) Working to make law enforcement effective (E) To make law enforcement work effectively 24. I‟am ashamed because now all my family secrets are axposed in court. “You _____ your neighbor for such a trival case.” (A) Had better sue (B) Should not have sue (C) Might not have sued (D) Would rather sue (E) Could not have sued

134

Xpedia Bahasa Inggris DP SNMPTN 01 95A with reading

01. Sarah’s parents could not sleep until she came home. This means that____. (A) although Sarah came home. Her parents could not sleep (B) sarah did not come home. Until her parents had slept (C) before Sarah came home. Her parents could not sleep (D) when Sarah came home. Her parents wer sleeping (E) her parents were were still sleeping when Sarah came home. 02. The increase in the sales of new cars ___ Expected to make traffic jams worse. (A) is (B) are (C) be (D) being (E) to be 03. Little children are usually afraid of ____ By their mother. (A) left (B) to leave (C) being left (D) leaving (E) been left 04. “When did you realize you had lost your purse?” “When I ____ money to pay the conducter.” (A) was needing (B) needed (C) had needed (D) have needed (E) am needing

135

05. “Why didn’t Didi want to go home?” “His mother ____ him for causing the car accident.” (A) would blame (B) has blamed (C) ia blaming (D) had blaming (E) blames 06. “The way professor Mattis teacher English not only keeps the students unterest____. (A) and also increases their motivation. (B) but also increasing their motivation. (C) and he also increases their motivation. (D) but also increases their motivation. (E) and also increase their motivation. 07. The thief ____ into the room through this window because there are footprints near the window. (A) was to get (B) may get (C) would rather get (D) must have got (E) should get 08. He told me a lot about the Philippines He ____ there for a long time. (A) must have lived (B) might be living (C) ought to have lived (D) should be living (E) would have lived 09. “Your son will be operated on tomorrow morning. He _____ have a good rest tonight.” the doctor said (A) might (B) may (C) can (D) must (E) would rather

Xpedia Bahasa Inggris, DP SNMPTN kode 01 95A with reading

10. Having given the prescription to the patient_____. (A) The medicine was taken regularly by the patient (B) The doctor told the patient to take the medicine regurlaly. (C) The patient took the medicine regurlaly (D) The medicine had to be taken regurlaly by the patient (E) The medicine was told to take regurlaly by the patient 11. ____ a few million rupiahs, He went on a tour to Europa. (A) Saved (B) Have saved (C) He has saved (D) Having saved (E) After the saves 12. The fact that he was put into prison for something that he had not done made his wife____. (A) cry (B) to be srying (C) cried (D) to cry (E) crying 13. “Andi, will you close the windows, please. I’m busy right now.” “I’m busy my self. But I _____ anyhow.” (A) Will close then (B) Will have to close them (C) Will have them closed (D) Will ask you to close them (E) Will have closed them 14. We are going on a long trip, so we must______. (A) have checked the car (B) have had the car checked (C) to have the car checked (D) have the car checked (E) had the car checked

15. The company’s new product was ____ advertised on TV. (A) nation (B) national (C) nationally (D) nationalize (E) nationatization 16. The original ____ manuscript of the play is on display in the museum (A) Written by had (B) Writing hand (C) Hand-written (D) That is hand-written (E) Hand-written 17. The thief denied ____ the jewellery although the police forced him to admit it. (A) Steal (B) Stealing (C) To steal (D) In stealing (E) Stolen 18. His shop was burnt down. _____ his car that was parked nearby. (A) and either did (B) also was (C) but neither was (D) so did (E) and so was 19. “Who won the Ford Foundation scholar ship?” “The high school student _____ the City Council had chosen as the best student.” (A) He who (B) Whose (C) Whom (D) Which (E) Of which 20. My brother needs two ____ to help him run the store. (A) Mechanics (B) Carpenter (C) Instructors (D) Porters (E) Assistants 136

Xpedia Bahasa Inggris, DP SNMPTN kode 01 95A with reading

21. _____ is not yet known. (A) Can he be involved in the trade of narcotics (B) He is involved in the trade of narcotics (C) Whether he is involved in the trade of narcotics (D) When he is involved in the trade of narcotics (E) Why is he involved in the trade of narcotics 22. If hadn’t been promoted, he’d have quit his job. The underlined words mean____. (A) He kept his job (B) He would like to keep the job (C) He had to quit his job (D) He lost his job (E) He didn’t like to keep his job

137

Xpedia Bahasa Inggris DP SNMPTN 02 98A with reading

Whales are the largest animals on earth. Bigger than elephants, they may grow 95 feet long, and weigh 150tons. A baby blue whale, just born, can be 23 feet long and weigh 30 tons. Although whales live in the oceans and swim like fish, they are not fish. They are mammals, like cows and elephants. Unlike fish they bear their young alive, not as eggs. Their babies live on their mother’s milk. They breathe through their lungs and hold their breath when they go underwater. If they cannot come to the surface to breathe fresh air, they will drown. Further more, they are warm-blooded. Fish, however, lay eggs, breathe oxygen in the water, and are coldblooded. Whales live in all the oceans. In the winter some of them go to warm waters to feed. There are two kinds of whales, whales with teeth (toothed whales) and whales without teeth (baleen whales). The toothed whales eat fish and squid, which they can catch with their teeth, although they swallow their food without chewing it. The baleen whales eat plankton (small sea animals and plants). When they find plankton, they open their mouths and swim into the plankton. When they close their mouths they squeeze out the water and swallow the plankton. Whales have few enemies. Only human beings and killer whales attack whales. And whales do not seem to fight among themselves. The usually live to be around 20 to 30 years. 01. Which of the following is TRUE about whales? (A) Whales are generally 95 feet long and weight 150 tons (B) Big animals living in the ocean tend to be their enemies (C) Like fish, whales can live in all kinds of waters. (D) Their life span is usually longer than human beings. (E) A new-born baby whales may weight one-fiftieth of is mother

02. A suitable title for the text is _____. (A) large animals on earth (B) difference between whales and elephants (C) whales, the biggest animals on earth (D) similarity between fish and whales (E) the life of whales in the oceans 03. The difference between whales and fish is that whales____. (A) do not have to come to the surface to get fresh air. (B) deliver their young like elephants do. (C) will die if they cannot get oxygen in the water. (D) are mammals that are cold blooded (E) can easily breathe when they are underwater 04. Whales occasionally live in warms water____. (A) to get more food (B) for reproduction (C) to avoid winter (D) for fresh air (E) to feed their babies 05. From the text we may conclude that___. (A) a whale can eat both squid and plankton. (B) toothed whales chew their food before swallowing it (C) the two types of whales live in different waters (D) baleen whales swallow the water containing plankton (E) each type of whale eats a specific kind of food

138

Xpedia Bahasa Inggris, DP SNMPTN kode 02 98A with reading

People manage to count even when they do not have names for numbers. Early men demonstrated numbers to each other by counting their fingers and some primitive tribes still do this. In fact, some tribes ham=ve names for only the number “one” and “two” and can only indicate higher numbers by pointing to a particular finger. In other tribes, the numbers do have names, but these names are directly connected with finger counting. 06. Which of the following statements is TRUE according to the text? (A) It was not necessary to have names for number in counting (B) Only in earlier days did people use their fingers in counting (C) There are tribes who only consider “one” and “two” important (D) For some tribes, it is unusual to use fingers when counting (E) The words “one” and “two” were unpopular among many tribes 07. The text tells us about … (A) early demonstrations of numbers (B) the use of fingers in counting (C) the importance of numbers (D) primitive tribes (E) various names for number

139

The United States and Mexico signed a peace treaty on February 2, 1848. Mexico was to give up much of its land to the U.S. The land included Texas, Arizona, New Mexico, and California. The treaty signers did not know that James Marshall had just discovered gold in California 10 days earlier. Nobody knew this outside of the area near the Sacromento River. One year later, everybody knew. In 1849 the world heard about the gold discovery. Gold fever brought thousands of people to California. We call the people who joined the gold rush “forty-niners” they all though they could become rich quickly. They though they could stay in California a short time and return home as wealthy people. 08. The main information of the text is about ______. (A) a peace treaty between the U.S. and Mexico. (B) james Marshall, the gold miner. (C) the discovery of gold in California. (D) the gold rush in the United State (E) the dream of the forty-niners to get rich quickly 09. From the text we may conclude that____. (A) Mexico regretted that it had to give up California to the U.S. (B) James Marshall did not publicly announce his discovery. (C) A lot of “fortyniners” became rich within a very short time. (D) California became a place where many foreigners (E) Mexico made great efforts to eim California back

Xpedia Bahasa Inggris, DP SNMPTN kode 02 98A with reading

Today, no one would deny the genius of Albert Einstein. __10__ that was not always the case. In his youth, Einstein’s parents and teachers __11__ him both undisciplined and dull. As a child, he was slow to talk, and his parents were __12__ he might be retarded. As an adolescent, he had difficulty adjusting to the strict discipline of the Germain high school, and his teachers were not pleased with his __13__. Not surprisingly he found college a little better than high school, and again he __14__ the appropriate academic discipline. In the end, he was able to graduate, but only with the help of a friend who kept him up-to-date on all his course. 10

11.

(A) (B) (C) (D) (E)

Since But So Because Therefore

(A) (B) (C) (D) (E)

declared determined predicted accused considered

(A) (B) (C) (D) (E)

hopeful amazed expectant worried timid

(A) (B) (C) (D) (E)

success increase advantage program progress

(A) (B) (C) (D) (E)

achieved served lacked wasted disrupted

12.

13.

15. ____ I can drive, I often ask my brother to drive me to the city. (A) Since (B) When (C) Because (D) Whether (E) Although 16. ____ at two prestigious universities , Amanda has a hard time deciding which one to choose. (A) Accepting (B) She was accepted (C) Accepted (D) To be accepted (E) She was accepted 17. Due to the monetary crisis, the number of people without jobs ____ at the moment. (A) increase (B) is increasing (C) increased (D) was creasing (E) will increase 18. ____ a few thousand dollars, he went on a tour to Europe. (A) Saved (B) Have saved (C) He has saved (D) Having saved (E) After he saves 19. Rudi broke his leg in an accident; therefor, ____ drive him to work. (A) he doesn’t have to ask Amir to (B) he won’t have to get anybody (C) amir has asked Rudi to (D) amir doesn’t have to (E) he will have Amir

14.

140

Xpedia Bahasa Inggris, DP SNMPTN kode 02 98A with reading

20. “Harry could not pay his school fee because his father was unemployed.” “His rich uncle _____ him with his school fee” (A) may support (B) should have supported (C) would rather support (D) will have supported (E) might support 21. At the beginning of the semester, each of the students ___ given a new timetable (A) He is (B) They are (C) Are (D) Is (E) Be 22. “These are all government offices.” “_____ is your father’s.” (A) Which (B) When (C) Whose (D) Why (E) What 23. The fact that he was put in prison for something that he had not done made hid wife____. (A) cry (B) to be crying (C) cried (D) to cry (E) crying

141

Xpedia Bahasa Inggris DP SNMPTN 03 99A with reading

01. In almost all developing countries, the lack of adequate supplies of cheap. Con venient and reliable fuel is a major prob lem Rural communities depend largely on kerosene. Wood and dung for their cooking and lighting needs. But forested areas are in short supply. The search for firewood occupies a large part of the working day and has resulted in wide sprespread deforestation. Dung is in constant supply wherever there are farm animal ang. When dried. It is convenient to store and use. But burning dung destroys its values as fertile izer. Thus depriving the soil of a much needed source of human and nitrogen. Rural areas of developing countries are also plagued by a lack of adequate sanitation improper waste disposal spreads disease. Contaminates water sources and provide breeding grounds for disease –carrying insects. The problems of improving environ mental hygiene. Conserving resources, and finding alternative sources of fuel may be unrelated. Their solutions, howe ver. Are not,as many countries experi menting with biogas technology are dis covering. Biogas,a mixture of methane ang carbon dioxide. Is produced by the fermentation of organic matter.the proc ess of anaerobic fermentation is natural one. Occurring whenever living matter decomposes.by keeping the matter ang the process in a digester or biogas plan. The combustible gas can be trapped and used as fuel for household lighting ang cooking.the digested slurry that remains can then be used un the land as soil con ditioner fertilizer.

.

What is the text mainly about? (A) Socio - economic problems in developing countries. (B) The process of producing biogas. (C) The effect of burning dung on the soil. (D) Biogas as a solution to many problems (E) The disadvanteges of using dung as fuel.

02. What are the two problems mentioned in the text? (A) Pollution and the high price of fuel. (B) Widespread deforestation and the use of dung. (C) The lack of adequate fuel and hygienic conditions. (D) The lack of humus and nitrogen in the soil. (E) Contaminated water sources and rural areas. 03. The disadvantage of traditional fuels used in developing countries is that they (A) Reduce people’s energy. (B) Are difficult to extract. (C) Make the soil poor. (D) Do not meet people needs. (E) Cause damage to the environment 04. The main factor causing the lack of adequate sanitation is___ (A) The domestic use of biogas. (B) The great number of farm animals. (C) The pollution of water sources. (D) The badly-managed waste disposal. (E) The increase of disease-carrying insects.

142

Xpedia Bahasa Inggris, DP SNMPTN kode 03 99A with reading

05. why is the digasted slurry of the fermented organic matter considered valuable? (A) It improves the quality of the soil for agriculture. (B) It increases the export from developing countries. (C) It is considered a relatively cheap vertilizer. (D) It is the best fertilizer in the word. (E) It can be used as fuel for cooking. 06.

Hurricanes generally occur in the North Atlantic from May through November with the peak of hurricane season in September. Only rarely will the occur from Desember through April in the part of the ocean. The main reason for the occurrence of hurricanes during this periode is the temperature on the water’s surface is at its warmest and the humidity of the air is at its warmest and the humidity of the air is at its highest. Of the tropical storms that accureach year in the North Atlantic, only about five, on the average,are as a hurricane. A tropical storm must have winds reaching speeds of at least 117 kilometers per hour. But the wind are often much stronger than that: the winds of intense hurricanes can easily surpass 240 kilometeres per hour. The passage mainly discusses…. (A) The number of hurricanes in a year. (B) The strength of hurricanes. (C) The weather in the North Atlantic (D) Hurricanes in a certain part of the world. (E) Hurricanes and disasters of the world.

143

07. Which of the following is TRUE accord ing to the text? (A) There are always hurricanes in the North Atlantic all year round. (B) Extremely warm weather and high humidity can cause hurricanes (C) Very strong winds are the result of hurricanes. (D) Hurricanes occur every month from May to November (E) Tropical countries have a lot of intense hurricanes. 08. There are many different causes of car accidents in the United States. Sometimes accidents are caused by bad weather. Ice or snow can make road very dangerous. Accidents can also result from problems with the car. Even a small problem like a flat tire can be serious .bad road are another cause of accidents are caused by drinking too much alcohol. In fact. This is one of the most important causes of accidents What is paragraph about? (A) Problems with car engines. (B) Results of car accidents. (C) The frequency of car accidents. (D) Weather conditions in US (E) Car accidents and their causes. 09. Car accidents in the US are caused by the following exept____ (A) Drunken drivers (B) Bad roads (C) Unskilled drivers (D) Ice roads (E) A flat tire

Xpedia Bahasa Inggris, DP SNMPTN kode 03 99A with reading

10.

How much living space does a person need? What happens when these spatial__10__are not adequately met? Sociologists and psychologists are conducting experiments on rats to try__11__ the effects of over crowding on humans. Recent studies have shown that the behaviour of rats is greatly affected by space. They cat well. Sleep well. And reproduce well.but if their living conditions become too crowded. Their behavior patterns and even their health change __12__They can not sleep and eat well.and signs of fear and tension become __13__The more crowded they are. The more they tend to bite each other and even kill each other__14__for rats. Population and violence are directly related. Is this a natural law for human society related. Is this a natural law for human society as well? Is adequate space not only desirable but essential for human survival? (A) requirements (B) influences. (C) problems (D) resources (E) possibilities

14. (A) (B) (C) (D) (E)

However Yet Thus Moreover Finally

11. (A) To emerge (B) To propose (C) To imagine (D) To examine (E) To simplify 12.

(A) To perceive (B) Perceptibly (C) perceptive (D) perceptible (E) perception

13. (A) tremendous (B) registered (C) concise (D) obsolete (E) obvious

144

Xpedia Bahasa Inggris DP SNMPTN 04 00A with reading

Read this text to following number 1-5! There are three separate sources of hazard in the process of supplying energy by nuclear power. The radioactive material mint travel from its place or manufacture to the power station. Although the power stations themselves are solidly built, the containers used for the transport of the materials are not. There are normally only two methods of transport available, namely road or rail. Unfortunately, both of these involve close contact with the general pubic, since the routes are sure to pass near, or even through, heavily populated areas. Next, there is the problem of waste. All nuclear power stations produce wastes that in most cases will remain radioactive for thousands of years. It is impassible to make these wastes non-radioactive, and so they must be stored in one of the inconvenient ways that scientists have invented. For example they may be buried under the ground, or dropped into abandoned mines, or sunk in the sea. However, these methods do nor solve the problem, since an earthquake could easily the containers open. Finally, there is the problem of accidental exposure due to a leak or an explosion at the power station. As with the other two hazards, this is not very likely, so it does not provide a serious objection to the nuclear program. Nevertheless, it can happen. Separately, these three types of risks are not a great cause for concern. On the whole though, the probability of disaster is still high. 01. The whole tern tells us about ________. (A) the danger of transporting radioactive materials (B) the way to make radioactive wastes not dangerous for people (C) the potential dangers in supplying energy by nuclear power (D) the accidents caused by nuclear power stations (E) people’s objections to the establishment at nuclear stations 145

02. What makes the transport of radioactive materials dangerous for people’s live? (A) The road (B) The containers (C) The rail (D) The power station (E) The heavily populated areas 03. Which of the following statement is TRUE about radioactive wastes? (A) They will be no more danger when radioactive wastes are buried underground (B) There is no chance of saving people’s live from radioactive wastes. (C) Radioactive wastes become nonradioactive when they are sunk in the sea (D) Only earthquakes can change radioactive waste into non-radioactive waste. (E) Natural disasters of radioactive wastes 04. In spite of the problems which are likely to occur in supplying energy b nuclear power, the writer thinks that _____. (A) people will strongly object to it (B) the probability of disaster is high (C) hazards should be rigidly controlled (D) it’s worth conducting nuclear programs (E) there is great cause for concern 05. We may conclude from the third paragraph that the problem of accidental exposure is caused by ______. (A) human error (B) an earthquake (C) radioactive waste (D) hazards (E) bad design of the nuclear station

Xpedia Bahasa Inggris, DP SNMPTN kode 04 00A with reading

Read this text to following number 6-7! Among the various effects of air pollution, climatic change are slow, the deterioration of health is insidious, and the damage to plant, animal, or material seems remote to those not directly involved. But smoke is visible to all, and the stinks of sulfides are all over the place. As a result, most people only object to something that can be seen or smelled 06. What is the main ideal of paragraph? (A) The effect of air pollution on people’s live can be direct and indirect. (B) People are not directly affected by the damage of plant and animals. (C) People complain about all the pollutants in the air. (D) Climatic chance does not immediately have an effect on people’s live (E) Complaints by individuals have been made to pollution agencies. 07. The main information about the paragraph is ________. (A) the danger of air pollution (B) the slow climatic change (C) the effects air pollution (D) wastes emitted by air pollution (E) complaints about air pollution Read this text to following number 8-9! Both the sun and the moon appear large when they are rising or setting, although there is no real difference in the distance they are from the earth. This perceptional phenomenon, known as the “Moon illusion” has been studied over the years. Various explanations, including the muscle strain for the person looking up and the comparison of the moon with other things on the horizon that make it appear larger, have been disputed by scientists, but thus far there is no widely accepted explanation of the phenomenon.

08. What is the topic of the paragraph? (A) Perceiced sized of the sun and moon (B) Comparison of object on the horizon (C) Perception of distance (D) The rising and the sitting of the sun and the moon (E) The “Moon illusion” 09. Which of the following statements is TRUE about the “Moon illusion”? (A) Through various experiments scientists discovered the causes of “Moon illusion” (B) Although the phenomenon has been studied for years, scientists haven't agrees about its cause (C) The already known distance of the sun and moon from the earth explain the “Moon illusion” (D) All the answer to the question of what cause the phenomenon are the same (E) Scientists do not think that the mystery of the “Moon illusion” can be solved Read this text to following number 10-14! In canning, heat can be to destroy bacteria and halt enzymic activity in food. Bacterial __10__ from external sources is prevented by sealing the food in a can. The can is then heated to a temperature __11__ to sterilize the contents. The length of time and the temperature __12__ to ensure sterilization depends on the product and on the type of bacteria that needs to be destroyed. It is important that the contents of the can are heated __13__. An experimental check can be made on the exact temperature in any part of a sample can __14__ it is being heated by inserting a thermo couple into the can. 10. ... (A) (B) (C) (D) (E)

influence effect contamination excess infectiousness

146

Xpedia Bahasa Inggris, DP SNMPTN kode 04 00A with reading

11. ... (A) (B) (C) (D) (E)

regular convenient operative sufficient complete

12. ... (A) (B) (C) (D) (E)

require requirement requisite requisition required

13. ... (A) (B) (C) (D) (E)

unity uniform uniformity uniformly uniformed

14. ... (A) (B) (C) (D) (E)

while because whereas since so that

147

Xpedia Bahasa Inggris DP SNMPTN 05 01A with reading

Computers are a relatively new invention. The first computers were built fifty years ago and it is only in the last thirty years that its influence has affected our everyday life. Personal computers were introduced as recently as the early eighties. In this short time they have made a tremendous impact on our lives. We are now so dependent on computers that it is hard to imagine what things would be like today without them. You have only got to go into a bank when the main computer is broken to understand the chaos that would occur if computers were suddenly removed world-wide. In the future, computers will be used to create bigger and even more sophisticated computers. The prospects for this is quite alarming. They will be so complex that no individual could hope to understand how they work. They will bring a lot of benefits but they will also increase the potential for unimaginable chaos. They will, for example, be able to fly planes and they will be able to coordinate the movements of several planes around the airport. Providing all the computers are working correctly, nonthing can go wrong; but if one program fails there will be a disaster. There is no doubt that technology will progress and become increasingly complex. We should, however, ensure that we are still in a position where we are able to control technology. It will be all too easy to suddenly discover that technology is controlling us. By then it might be too late. I believe that it is very important to be suspicious of the benefits that computers will bring and make sure that we never become totally dependent on a completely technological world

01. From the text we may conclude that the author ____. (A) Tries to warn us not to be too dependent on computers (B) Emphasizes the importance of developing future computers (C) Believe computers will make peoples lives more comfortable (D) Reminds us about the threat of computers in the past (E) Persuades us to make use of computers in our job. 02. People’s current dependence on computers is exemplified by____. (A) The influence of computers on our lives for the last thirty years (B) The introduction of computers in the nineteen fifteens. (C) The chaos which took place when the computers were removed (D) The disorder at home when the main computers is broken (E) The unimaginable things that may occur on an airport 03. Future computers are predicted to ____. (A) Be so complex that their prospect will be quite promising (B) Bring great benefits as well as unexpected harm to people’s lives (C) Bring great benefits to the transportation industry (D) Make many small programs cause big disasters (E) Be able to coordinate the traffic of planes all over the world

148

Xpedia Bahasa Inggris, DP SNMPTN kode 05 01A with reading

04. Although computers will become more sophisticated, _____. (A) Human beings will be entirely dependent on them (B) It is difficult for individuals to understand them (C) Technology tends to control human beings (D) There is no doubt that they bring a lot of benefits (E) Human being should have control over them 05. A suitable title for the text is____. (A) The historical background of future computers (B) The unimaginable chaos caused y computers (C) The potentials of future computers (D) The control over advanced technology (E) The rapid progress in technology A new hearing device is now available for some hearing impaired people. This device uses a magnet to hold the detachable sound-processing portion in place. Like other aids, it converts sound into vibrations, but is unique in the sense that it can transmit the vibrations directly to the magnet, and then to the inner ear. This produces a cleaner sound. The new device will not help all hearing-impaired people, only those with a hearing loss caused by infection or other problem in the middle ear. This produces a clearer sound. The new Device will not help all hearing-impaired people.only those with a hearing loss caused by infection or other problem in the middel ear. It will probably help no more than 20 percent of all people with hearing problems. Those people, who often have persis tent ear infections, should however find relief and restored hearing with the new device.

149

06. What is the author’s main purpose in writing the next ? (A) To describe new cure for ear infections. (B) To urge doctors to use the new device. (C) To explain the use of the magnet (D) To inform people of a new hearing device. (E) To study a new device for ear infections. 07. According to the passage, what does the device NOT do ? (A) Transmit sound to the inner ear. (B) Help all hearing-impaired people.. (C) Produce clear sound (D) Change sound into vibrations. (E) Relieve hearing loss caused by ear infection

Xpedia Bahasa Inggris, DP SNMPTN kode 05 01A with reading

Wheather a tree is coniferous or deciduous, whether it bears fruit or not, whether it grows in the tropics or in the temperate zone. Every tree has three oarts: the roots, the trunk, and the crown. The roots the part of the tree underground.hold the tree firmly againts windstrorms and provide ther tree with food gathered from the soil. The trunk, which is the most important woody part of the tree, is the body of the tree: it carries the food from the root to the branches. The crown of the tree consists of the branches, the leaves, and the fruit of the tree. The leaves use the food sent from the trunk for many purposes, one of which is a process that is particularly useful to humans. In this process called photosynthesis. The leaves absorb carbon dioxide from the atmosphere and give to the atsmosphere. The process is essential to human life because humans have to breathe oxygen in order to continue existing. . 08. The main information of the text is about____ (A) The difference between coniferous and deciduous trees (B) The climates in which.different types of trees grow (C) The common functions of the parts of any kind of tree. (D) The description of the process of photosynthesis (E) The usefulness of tree leaves to a human being’s life 09. trees are very important for the life of human being because ____. (A) they protect human being against windstroms. (B) their leaves produce carbon dioxide. (C) their trunks distribute food to the roots. (D) photosynthesis absorbs carbon dioxide (E) they contribute oxygen to the atmosphere

For most of us, tortoises and turtles seem quite ordinary as we often keep them as pets. Yet, theseanimals are living fossils, they are __10__ of animals that existed bevore the firs dinosaurus Turtles., terrampis, and tortoises belong to group of over 200 reptiles known __11__ as the Chelonia. These creatures are encased in an armor made of bone and hom, and their lives are controlled by this shell, making it difficult for them even to breathe and mate. Living chelonians have no teeth. Instead they cut their food with sharp.horny beaks.Tortoises mostly __12__ plans while terrapins are fleshheaters. __13__ they have survived for so long.. The large chelonians are all endangered. Mainly because of human activity. The biggest land tortoises now survive in tiny numbers on remote islands., and egg-laying sites of sea turtles are __14__ for many reasons. 10

(A) (B) (C) (D) (E)

To descend descent descending descendants descended

(A) (B) (C) (D) (E)

secretly scientifically knowledgeably artificially accordingly

(A) (B) (C) (D) (E)

Feed on Provide with Carry on Agree with Keep on

(A) (B) (C) (D) (E)

because before although since whenever

11.

12.

13.

150

Xpedia Bahasa Inggris, DP SNMPTN kode 05 01A with reading

14.

(A) (B) (C) (D) (E)

to threaten threatened threat threatening threateningly

15. “Did you thank Andi for the dictionary he bough for you ?” “Yes, I did: I realy appreciate___ it for me.” (A) to buy (B) buys (C) buying it (D) to have bought (E) his buying 16. “ To whom are you going to dedicate your book?” “ To those ____ patience and support have enabled me to write this book.” (A) of which (B) whom (C) to whom (D) whose (E) who their 17. the teacher had students who didn’t bring their identification cards ____ the auditorium where the exams were held. (A) left (B) leave (C) To leave (D) leaving (E) leaves 18. “While living in bogor, we always went to school on foot.” This means that we ____ on foot while we were living in bogor. (A) go to school (B) are going to school (C) used to go to school (D) are used to going to school (E) liked going to school

151

19. “was there any violence during last week’s demonstrasion ?” “No. ____ there were thousands of demontstrators.” (A) since (B) eventhough (C) before (D) when (E) whereas 20. “What if your father asks you about the scratch on his new car ?” “I _____ that it’s my fault.” (A) Simply told him (B) Am simply telling him (C) Will simply tell him (D) Would simply tell him (E) Would simply have told him 21. “I can’t find my handphone in my bag!” “Well,you. ___ in the car. Let’s take a look!” (A) may have to leave it (B) had to leave ij (C) should have to leave it (D) may have to leave it (E) must have levt it 22. The office building,as well as several houses, _____ completely damaged by the bomb explosion. (A) was (B) they were (C) it was (D) were (E) to be 23. “I’d like to book three standard room for next week.please.” “I’m sorry,Sir. Our hotel ____.until the end of the month.” (A) fully booked (B) being fully booked (C) is fully booked (D) be fully booked (E) to be fully booked

Xpedia Bahasa Inggris, DP SNMPTN kode 05 01A with reading

24. Fast-food restaurants have become popularin big cities because many people prefer ____ quickly. (A) served (B) Being served (C) serving (D) They are served (E) Be serving 25. “I finally bought a computer but I’m not satisfied with it.” “That’s too bad. you _____ Darno,our computer technician.for advice.” (A) Should have asked (B) May have asked (C) Ought to ask (D) Must have asked (E) Would ask 26. ASEAN trade ministers. ___ the possibility of developing economic cooperation they attended the summit neeting last month. (A) discuss (B) discussed (C) Have discussed (D) Were discussing (E) Would discuss

29. Living in a remote mountainous area in West Java ____ (A) People can still see how Badul people live in simplicity (B) Natural resourses can be effectively conserved by the Badui people (C) The Badul people are almost completely untouched by modern life (D) How the Nadui people maintain their old way of life amazing (E) Modern technology has hardly ever touched the Badui people 30. “I wonder how Laura will even manager the business after her father died in that car accident.” “She has both the expertise in running the business and _____ .” (A) The strong support from all the members of her family (B) All the member of the family strongly support her too (C) She is also supported by all the members of her family (D) there is strong supported from all the members of her family (E) She has the strong support from all the member of her family

27. Smoking can the cause of many illnesses and respiratory disorders: _____ it may harm non-smokers. (A) consequently (B) however (C) In addition (D) nebertheless (E) otherwise 28. “Can you turn on this electric stove ?” “I’m sorry.I don’t know____.” (A) Operating it (B) The way operating it (C) What is operated (D) How to operate it (E) It is operated

152

SNMPTN Bahasa Inggris

This text is for question number 31-34! The most common of tsunamis are underearth quakes. To understand underwater earthquakes, you must first understand plate tectonics. The theory of plate tectonics suggest that the lithosphere, or top layer of the Earth, is made up of a series of huge plates. There plates make up the continents and seafloor. They rest on an underlying viscious layer called the asthenosphere. Think of a pie cut into eight slices. The piecrust would be the lithosphere and the hot, sticky pie filling underneath would be the asthenosphere. On the Earth, these plates are constantly in motion, moving along each other at a speed of 1 to 2 inches (2,5-5 centimeters) per year. The movement occurs most dramatically along fault lines (where the pie is cut). These motions are capable of producing earthquakes and volcanism, which, when they occur at the bottom of the ocean, are two possible sources of tsunamis. When two plates come into region known as a plate boundary, a heavier plate can slip under a lighter one. This is called subdiction. Underwater subdiction often leaves enormous “handprints” in the form of deep ocean trenches along the seafloor. In some cases of subdiction, part of the seafloor connected to the lighter plate may “snap up” suddenly due to pressure from the sinking plate. This results in an earthquake. The focus of the earthquake is the point within the Earth where the rupture first occurs, rocks break, and the first seismic waves are generated. The epicenter is the point on the seafloor directly above the focus. When this piece of the plate snaps up and sends tons of rock shooting upward with tremendous force, the energy of that force is transferred to the water. The energy pushes the water upward above normal sea level. This is the birth of a tsunami. The earthquake that generated the December 26,2004 tsunami in the Indian Ocean was a 9.0 on the Richter scale - one of the biggestin recorded history.

153

31. Which of the following best states the topic of this text? (A) The birth of a tsunami (B) The magnitude of tsunamis (C) Tsunamis in the Indian Ocean (D) Series of huge plates on earth (E) Lithosphere and asthenosphere 32. The main idea of this text is that …. (A) deep ocean trench is a result of an earthquake (B) the energy of subdiction can lead to earthquakes (C) plate tectonics lead to an earthquake and volcanism (D) tsunamis in the Indian Ocean are the biggest in the history (E) strong movements of undersea fault lines cause tsunamis 33. Implied in the text is that the earthquake will never occur when …. (A) the subdiction is evidenced (B) seismic waves are generated (C) no fault line of plate tectonics happens (D) heavier plates and lighter ones break up (E) ocean trenches lie along the seashore 34. The following information is true about tsunamis mentioned in the text, EXCEPT that tsunamis are …. (A) predictable following any incidence of earthquakes (B) close to the rise of sea levels from the sinking plate (C) connected to deep ocean trenches along sea floors (D) related to strong movements of plates tectonics (E) highly linked to underwater earthquakes

SNMPTN Bahasa Inggris

This text is for question number 36-40! A Spanish researcher and a Paraguayan scientist have presented the most complete and detailed European study into the repertoire of sounds used by bottlenose dolphins (Tursiops truncatus) to communicate. The study reveals the complexity and our lack of understanding about the communication of these marine mammals. Until now, the scientific community had thought that whistles were the main sounds made by these mammals, and were unaware of the importance and use of burst-pulsed sounds. Researchers from the Bottlenose Dolphin Research Institute (BDRI), based in Sardinia (Italy) have now shown that these sounds are vital to the animals’ social life and mirror their behavior. “Burst-pulsed sounds are used in the life of bottlenose dolphins to socialize and maintain their position in the social hierarchy in order to prevent physical conflict, and this also represents a significant energy saving.” Bruno Diaz, lead author of the study and a researcher at the BDRI, which he also manages, said. According to Diaz, bottlenose dolphins make longer burst-pulsed sounds when they are hunting and at times of high aggression: “These are what can be heard best and over the longest period of time,” and make it possible for each individual to maintain its position in the hierarchy. The dolphins emit these strident sounds when in the presence of other individuals moving towards the same prey. The “least dominant” one soon moves away in order to avoid confrontation. “The surprising thing about these sounds is that they have a high level of unidirectionality, unlike human sounds. One dolphin can send a sound to another that it sees as a competitor, and this one clearly knows it is being addressed,” explains the Spanish scientist.

36. It can be inferred from the text that …. (A) producing sounds requires a lot of energy (B) whistles are more complicated than burst-pulsed sounds (C) mother dolphins cannot make burstpulsed sounds (D) bottlenose dolphins avoid physical frictions (E) hierarchy is not vital for bottlenose dolphins 37. The word “these” in “These are what can be heard ….” (line 14) refers to …. (A) Wistles (B) High aggression (C) Hunting dolphins (D) Other individual (E) Burst-pulse sounds 38. The paragraph following the text most probably discusses …. (A) Dolphins most favourite preys (B) Least dominant dolphin’s eating habits (C) Another kind of sounds made by dolphins (D) The characteristics of burst-pulsed sounds (E) Hunting dolphins use of sounds to communicate 39. In organizing ideas in the text the writer …. (A) lists reasons why scientist were unaware of burst-pulsed sounds (B) explains how different sounds are used by bottlenose dolphins (C) compares similarities between burstpulsed sounds with whistles (D) describes the unique characteristics of bottlenose dolphins (E) presents examples of bottlenose dolphins sounds

154

SNMPTN Bahasa Inggris

40. Which of the following mostlikely restates the gist of paragraph 22 …. (A) Bottlenose dolphins are effective communicators (B) Burst-pulsed sounds are used by all kinds of dolphins (C) Burst-pulsed sounds are vital for bottlenose dolphins (D) Whistles and burst-pulsed sounds are interchangeable (E) Whistles are vital for bottlenose dolphins to communicate This text is for questions number 41-45! TEXT A Human spend about a third of their lives asleep: Hence, there must be a point to it. Scientist have found that sleep helps consolidate memories, fixing them in the brain so we can retrieve them later. Now, new research is showing that sleep also seems to reorganize memories, picking out the emotional details and reconfiguring the memories to help you produce new and creative ideas, according to the authors of an article in Current Directions in Psychological Science. “Sleep is making memories stronger,” says Jessica D. Payne of the University of Notre Dame, who co-wrote the review with Elizabeth A. Kensinger of Boston College. “It also seems to be doing something which I think is so much more interesting, and that is reorganizing and restructuring memories.” Payne and Kensinger study what happens to memories during sleep, and they have found that a person tends to hang on to the most emotional part of a memory. For example, if someone is shown a scene with an emotional object, such as a wrecked car, in the foreground, they’re more likely to remember the emotional object than, say, the palm trees in the background - particularly if they’re tested after a night of sleep. They have also measured brain activity during sleep and found that regions of the brain involved with emotion and memory consolidation are active.

155

One of the first to go in fast-paced society is sleep. That is based on a profound misunderstanding that the sleeping brain isn’t doing anything. “In fact, the brain is busy. It’s not just consolidating memories, it’s organizing them and picking out the most salient information. Payne thinks this is what makes it possible for people to come up with creative, new ideas. TEXT B Sleep not only protects memories from outside interferences, but also helps strengthen them, according to research presented at the American Academy of Neurology’s 59th Annual Meeting in Boston. The study looked at memory recall with and without interference (competing information). Forty-eight people between the ages of 18 and 30 took part in the study. All had normal, healthy sleep routines and were not taking any medications. Participants were divided evenly into four groups - a wake group without interference, a wake group with interference, a sleep group without interference and a sleep group with interference. All groups were taught the same 20 pairs of words in the initial training session. The wake groups were taught the word pairings at 9 a.m. and then tested on them at 9 p.m. and tested on them at 9 a.m. after a night of sleep. Just prior to testing, the interference groups were given a second list of word pairs to remember. The first word in each pair was the same on both lists, but the second word was different, testing the brain’s ability to handle competing information, known as interference. The interference groups were then tested on both lists. The study found that people who slept after learning the information performed best, successfully recalling more words. Those in the sleep group without interference were able to recall 12 percent more word pairings from the first list than the wake group without interference. With interference, the recall rate was 44 percent higher for the sleep group.

SNMPTN Bahasa Inggris

41. Which of the following can best sum up both texts …. (A) Sleep patterns considerably affect one’s creativity (B) Sleep leads recall abilities to a better performance (C) The longer people sleep, the better they memorize (D) Healthy people usually sleep without interferences (E) Memories and regular sleep patterns affect each other 42. Based on the information in both texts, sufficient sleep …. (A) is a sole key to memory consolidation (B) Optimally enhances brain functions (C) Will contribute to better learning (D) Implies absence of interferences (E) Helps long-term memory retrieval

45. The focus of research in both texts differs. Text A explains about …. (A) Protection of healthy sleep routines; texts B explains about memory strength and vocabulary (B) Consolidation of emotion and memory; text B explains about protection of healthy sleep routines (C) Sleep patterns with interferences; text B explains about the role of sleep in memory recall (D) The role of sleep in memory and creativity; text B explains about interferences and memory recall (E) On memory strengths on vocabulary; text B explains about the role of sleep in memory recall

43. The best topic for both texts would be …. (A) Research on memory (B) Experiments on sleep (C) Dangers of lack of sleep (D) Sleep and creative thinking (E) Impact of sleep on memories 44. Which of the following statements represents a fact that can be found in either text? (A) People who sleep sufficiently tend to be more creative (B) Sleep helps enhance one’s memories and creativity (C) Sleep is highly needed in a busy and modern society (D) Enough sleep is a cital factor for critical thinking (E) The brain is not active when people are sleeping

156

UMB UI Bahasa Inggris

This text is for number 01-05! Veins are responsible for transporting blood from the body back to the heart where it is loaded with oxygen in the lungs before being distributed to the rest of the body again. Veins on the legs have the toughest job because they must push the blood against the force of gravity and the pressure of body weight in an upstream motion. To assist in this task, veins have one way valves that prevent the blood from flowing backward. When there are problems with the valves, it causes blood to stay in the leg and cause the vein to swell. There are two kinds of enlarged veins: vancose veins and spider veins. Varicose veins are dark blue or purplish in colour and they bulge above the skin‟s surface. Spider veins are much smaller and they look like red or blue spider webs which lie close to the surfaceof the skin. Forty-five-year-old Sarah developed varicose veins on her thighs many years ago, after giving birth to her third child. Initially, she ignored it, thinking that it was nothing more than an aesthetic hindrance. Shortly after, Sarah experienced cramps, especially at night and both legs were constantly aching, tired, and swollen. She also felt conscious of its appearance whenever she wears a skirt to work. Varicose veins are common among the general population-35% of women and 20% of men over 20 years old are diagnosed with this condition. Factors that increase the risk of developing varicose veins include prolonged sitting or standing, multiple pregnancies, family history of varicose veins, obesity, and wearing of high heels or tight garments. To prevent the development of varicose veins, one should exercise regularly to keep the blood flowing. Women should also avoid wearing high heel shoes (not more than 3-4 cm high) and tight clothing that wraps a round the legs. To alleviate the discomfort, people with varicose veins can try raising both legs above their chest to facilitate the blood flow or wearing compression stockings. Clinical treatments for varicose veins are also available to improve the cosmetic appearance and reduce the aching, swelling, and other serious problems such as 157

blood clots and ulcers. In Sarah‟s case, the many doctors she had consulted offered surgery as an optionto remove the varicose veins on her legs. However, this is an invasive procedure which requires an anesthesia and hospitalization. 01. What is the text mainly about? (A) Types of veins (B) Functions of veins (C) Varicose veins (D) Kinds of enlarged veins (E) Spider veins 02. This text is most probably found in a/an …. (A) medical textbook (B) newspaper editorial (C) sports magazine (D) health sport (E) health magazine 03. Which of the following is NOT included in the causes of varicose veins? (A) Delivering a baby (B) Constant aching of both legs (C) Wearing high heel shoes (D) Sitting or standing for hours (E) Hereditary 04. „an aesthetic hindrance‟ (lines 10) means …. (A) a restriction to keep one‟s health (B) an obstruction to become beautiful (C) a difficulty to overcome the disease (D) a constraint to the beautyof the skin (E) an obstacle to move freely 05. According to the text, a surgery is most needed for a patient with varicose veins when …. (A) the patient‟s legs are constantly aching and swollen (B) the patient has problems after delivering a baby (C) the patient‟s exercise keeps the blood flowing (D) the patient wants to improve his/her appearance (E) the patient has already suffered for a long time

UMBUI Bahasa Inggris

This text is for questions number 06 and 07!

This text is for questions number 08 and 09!

(1) ____________________ (2) Psychologist call this “math anxiety”. (3) Teachers used to think that this happened because the students were not very good a math. (4) Anxiety is an uncomfortable feeling of nervousness or worry about something that is happening or might happen in the future. (5) Now, however, researchers think that students who get math anxiety are not necessarily bad at math. (6) there is a very different reason for their poor performance on math tests. (7) New studies show that their feelings of anxiety prevent their brans from working well. (8) One area of the brain that is especially affected is the working memory, which holds new information in your mind. (9) this type of memory is essential for doing math problems. (10) But why do students get math anxiety in the first place? (11) That is another important question for teachers and researchers in education.

Up to now confessions that have been obtained from defendants in a hypnotic state have not been admitted into evidence by courts in the U.S. Experts in the fields of hypnotic have found that such confessions are not completely reliable. Subjects in hypnotic state may confess to crimes they did not commit for one or two reasons. Either they fantasize that they committed the crimes or they believe that others want them to confess. A landmark case concerning a confession obtained under hypnotic went all the way to the U.S. Supreme Court. In the case of Layra v Dinno, a suspect was hypnotized by a psychiatrist for the district attorney; in posthypnotic state the suspect signed three separate confessions to a murder. _____________.

06. With which of the following sentences should the text begin? (A) Whenever students have to do a math problem or take a math test, they experience math anxiety. (B) Some students who are not very good at math experience an uncomfortable feeling of nervousnees. (C) Some students become very anxious whenever they have to do a math problem or take a math test. (D) Research found the reason why some students are good at math while others are not. (E) There is an area in the brain that especially affects a person‟s ability at math. 07. The sentence which is irrelevant to text is sentence number …. (A) 4 (B) 5 (C) 6 (D) 7 (E) 8

08. What is the topic of the text? (A) Valid confession in the US courts (B) The unreliability of hypnotised confessions (C) The case of Layra v Dinno is US Supreme Court (D) The reasons why confessions are hypnotised (E) Psychiatrists role in making defendants confess 09. Which of the following statement should best end the text? (A) The result of hypnotised confessions might mislead the criminal and the court (B) To conclude, the confessions were the only evidence againts the murder (C) To summarize, making hypnotised confession is a phenomenon worth considering (D) Invalid confessions are rejected in the US because they are all the result of hypnotism (E) Thus, hypnotised confessions could accelerate the truth-finding process in court

158

UMBUI Bahasa Inggris

This text is for questions number 10-17! Studies indicate that the average global surface temperature has increased by approximately 0,5oF-1,0oF (0,3oC - 0,6oC) over the last century. Global warming - a (10)…. increased condition in planet-wide temperatures - is now well documented and accepted by scientists as fact. Average global temperatures may increase by 1,4oC 5,8oC (that‟s 2,5oF - 10,4oF) by the end of the 21st century. Although the numbers sound small, they can (11)…. significant changes in climate. (12)…. resulting in more hot days, many scientists believe an increase in temperatures may lead to changes in precipitation and weather patterns. Warmer ocean water may result in more (13)…. and frequent tropical storms and hurricanes. Sea levels are also expected to (14)…. by 0,09 - 0,88 m. In the next century, mainly from melting glaciers and expanding seawater. Global warming may also affect wildlife, and species that cannot survive in warmer environments may become (15)….. Human health is also at (16)…., as global warming may result in the spreading of certain diseases such as malaria. This warming is largely attributed to the increase of greenhouse gases (primarily carbon dioxide and methane) in the Earth‟s upper atmosphere caused by human burning of fossil fuels, as well as (17)…. activities, farming, and deforestation. 10. (A) grading (B) gradual (C) gradually (D) graded (E) grade 11. (A) affect (B) result (C) lead (D) trigger (E) contribute 12. (A) despite (B) besides (C) for example (D) according to (E) due to

159

13. (A) intense (B) intensive (C) intensified (D) intensifying (E) intensity 14. (A) improve (B) expand (C) rise (D) grow (E) climb 15. (A) extended (B) extinguished (C) modified (D) dead (E) vanished 16. (A) danger (B) threat (C) stake (D) warning (E) hazard 17. (A) industry (B) industrial (C) industrially (D) industrialization (E) industrialized 18. “Fred, you should have given the letter from the School Principal to your parents.” This means that Fred …. (A) gave the letter to his parents (B) will give the letter to his parents (C) was giving the letter to his parents (D) is giving the letter to his parents (E) didn‟t give the letters to his parents 19. My friend got the job as a manager not because of his uncle‟s position in the company, …. (A) but because he is very well qualified for the job (B) but because of his good qualifications (C) also because he is very well qualified for the job (D) but also because of his good qualifications (E) but also because he is very well qualified for the job

UMBUI Bahasa Inggris

20. “How about seeing the new film at the nearest cinema?” “……..” (A) Really? (B) Pardon? (C) My pleasure. (D) Sounds great. (E) Are you sure? 21. The ex-president passed away before having the chance …. on trial. (A) put (B) to put (C) be put (D) to be put (E) being put

160

Ujian Masuk UNPAD Bahasa Inggris

With the technological advancements created in the late 20th and early 21st centuries, multi media forms of biography became much popular than literary forms of personality. The popularit of these forms of biography culminated in the creation of such cable and satellite television networks as A&E The Biography Channel, the History Channel and History International. Along with documentary film Channel, the History Channel and History International. Along with documentary film biographies, Hollywood produced numerous commercial films based on the lives of famous people. Also, new web 2.0 application such as annoknipsi.com enalble users all over the world to compile their own biography and illustrate it with other people’s photos. More recently,CD-ROM and online biographies are appearing. Unlike books and films, thay often do not tell a chronological story instead, they are archives of many discrete media elements related to an individual person, including video clips, photographs, and text aricles. Media scholar Lev Manovich says that such archives exemplify the database form allowing users to navigate the materials in many ways. 1. The primary subject of the passage is (A) technological advancements (B) biographies in multimedia forms (C) cable and satellite television networks (D) documentary film biographies (E) online biographies 2. What does the word they in line 10 refer to? (A) commercial films (B) people photos (C) CD-ROM and online biographies (D) books and films (E) archives

161

3. The word exemplify in line 12 could best be replaced with? (A) characterize (B) identify (C) juxtapose (D) illuminate (E) meander 4. How did multimedia biography get popularity? (A) in the late 20th and early 21st centuries (B) with the technological developments (C) in the forms of cable and satellite television networks (D) along with documentary film biographies (E) though commercial films 5a. According to the passage Hollywood (A) has promoted the popularity of multimedia biographies (B) has created television networks (C) has produced commercial films (D) has enabled people to publish their biographies (E) has created application on the web

Ujian Masuk UNPAD Bahasa Inggris

6. Line

The World Digital Library (WDL) is an international digital library operated by the American Library of Congress and UNESCO. The WDL has stated its mission is to promote international and intercultural understanding, expand the volume and variety of cultural content on the Interest, provide resources for educators,scholars, and general audiences, and to build capacity in partner institutions to narrow the digital devide within and between countries. It aims to expand non-English and nonWestern content on the Internet, and contribute to scholarly research. The library intends to make available on the internet, free of charge and in multilingual format, significant primary materials from culture around the world,including manuscripts, maps, rare books,musical scores, recording, films, prints, photographs,architectural drawings, and other significant cultural materials.

(5)

(10)

The main idea of the passage is (A) the history of the World Digital Library (B) the American Library of Congres and UNESCO (C) the purpose of the World Digital Library (D) expansion of non-English and non-Westren content on the Internet (E) a wide variety of materials on the Internet 7. The word primary in line 9 could best be replaced with (A) elementary (B) important (C) initial (D) earliest (E) inaugural 8. The World Digital Library (A) is operated by UNESCO (B) covers a wide variety of materials on the Internet (C) was inspired by uducation scholars, and general audiences (D) costs users some amount of money (E) involves international and intercultural understanding 9. What information can you get from the World Digital Library? (A) how cattle breed (B) why floods happen in sme coutries (C) how pollution affet the planer Earth (D) where a certain places is located (E) when solar eclipse will happen

162

Ujian Masuk UNPAD Bahasa Inggris

10. Line

(5)

(10)

What effect does your personality have on your longevity? Do some kinds of temperaments lead to longer lives? A new study in the Journal of the American Geriatrics Society looked at this question by examining the personality traits of 246 children of people who had lived to be at least 100. The study shows that those who live the longest are more outgoing, more active and less neurotic than other people. Long-living women are also more likely to be empathetic and cooperative than women with a normal life span. These findings comort with what you would expect from evolutionary theory those who are extroverted enough to make friends and help other are those who are goning to be able to gather enough resources to make it through tough times Interestingly, however, other traits that you might consider advantageous had no impact in this study in wheter participants were likely to live longer. Those who were more self-disciplined, for instance, were no more likely to live to be very old (which might explain the long life of the smoking French lady). Also, being open to new ideas had to relationship to long life, which might explain all those cantankerous old people who are fixed in their ways. Whether you can successfully change your personality as an adulr is the subject of a longstanding psychological debate. However, the new paper suggest that if you want long life, you should strive to be as outgoing as possible.

(15)

The passage mainly discusses (A) how personality relaten to longervity (B) characteristics of longevity (C) a study on the Journal of the American Geraiatrics Society (D) empathy and cooperativeness among women 11. According to the passage long-living women (A) have fever activities than other people (B) are likely to be extroverted (C) have always wanted to live longer (D) are more disciplined (E) succeeded in changing their personality 12. In line 5 the word outgoing could best be replaced with (A) leaving (B) retured (C) amiable (D) anxios (E) pious

163

Ujian Masuk UNPAD Bahasa Inggris

13. What would you expect from evolutionary theory? (A) long-living woman are more likely tobe empathetic (B) those who are more outgoing live the longest (C) it supports the study of longevity (D) dynamic people survive during difficult periods (E) being more outgoing more active, and less neurotic are the characteristics needed to live longer 14. The word cantankerous in line 15 could have the same meaning as (A) bad tempered (B) friendly (C) ridiculous (D) concerned 15. The word those in line 12 refer to …. (A) resources (B) times (C) women (D) participants (E) traits 16. The participants _____ working groups to address the special challenges of each of the four project areas. (A) forms (B) formed (C) has formed (D) has been formed (E) will have been formed 17. Dr. James H.Billington, Librarian of Congress, ____ as a Commissioner of the U.S National Commission toUNESCO in 2003. (A) was nominated (B) had been nominated (C) nominated (D) he nominated (E) who was nominated

18. Google Inc. became the first partner of this public-private partnership and in 2005 donated $3 million ____ development of the World Digital Library (A) which has been supported (B) who supports (C) that is supporting (D) to support (E) Supporting 19. ____Tylor’s definition of culture,in the 20th century “culture” emerged as the central and unifying concept of American anthropology. (A) however anthropologists worldwide who refer to (B) although anthropologists worldwide refer to (C) despite anthoropologists worldwide refer to (D) as soon as anthropologists worldwide refer to (E) because of anthropologists worldwide refer to 20. Student Health provides routine examinations ____ (A) and trated minor injuries (B) and trats minor injuries (C) and minor injuries were treated (D) and minor injuries are treated (E) and minor injuries have been treated 21. Other 19th century critics, following Rousseau, have accepted this differentiation between higher and lower culture, but have seen the refinement and ____ of high culture as corrupting and unnatural developments. (A) sophisticated (B) sophisticating (C) sophisticate (D) sophistication (E) sophistry 22. The telescope has _____ which splits light to find wobbles in different wavelengths (A) a special instrument (B) a specifically instrument (C) an instrument specific (D) an instrument special (E) an instrument specifically 164

Ujian Masuk UNPAD Bahasa Inggris

23. Biographical works _____ usually non-fiction, but fiction can also be used to portray a person’s life (A) is (B) has been (C) are (D) is being (E) are being 24. The kind of learning characteristic of human children is Imitative learning, which means reproducing an instrumental act to be understood ______ (A) intend (B) intended (C) intent (D) intentional (E) intentionally 25. Intensive Program students will find ____about on-campus housing at the Sheraton International House, and home stay on the enclosed applicaton form. (A) details information (B) details information (C) detail information (D) detailed information (E) detailed informations

165

SPMB Bahasa Inggris Kode Soal 711

01. Text The 44.5 carat Hope Diamond is the world‟s largest blue diamond. It is also one of the most popular attractions in the Smithsonian National Museum of Natural History in Washington D.C. The Hope Diamond is flawless and priceless. However ownership of this gem has not always been a stroke of great fortune. The Hope Diamond is a remnant of a 112-carat Indian diamond stolen from a statue of the Hindu goddess. Sita. The angry goddess cast a spell of misfortune on anyone who acquired the gem. The spell began to work when the stone arrived in France. It was sold to the French King. Louis XIV. Louis had the gem cut into a 67-carat jewel. After wearing it once. He became sick and died of smallpox. Next the stone was passed down to Louis XVI and his wife. Marie Antoinette. Their bad misfortune was to lose their heads in the French Revolution. The Hope Diamond was originally ___. (A) a jewel exhibited in a museum in Washington. DC (B) a bigger gem stone attached to an Indian statue (C) a 112-carat diamond owned by European kings (D) mean to be auctioned to the royal families in France (E) a sacred object worshipped by Indian people 02. The main topic of the whole text is ___. (A) the Smithsonian National Museum of Natural History (B) the diamond on the statue of the Hindu goddess (C) the cutting of the Hope Diamond into smaller carats (D) the history of the world‟s largest blue diamond (E) the kings who were at one time owners of the Hope Diamond

03. The following were misfortunes happening to the owners of the Hope Diamond, except ___. (A) death from smallpox (B) be headed in a revolution (C) broken marriage (D) a fatal car accident (E) getting married to an exotic dancer 04. Which of the following statements is TRUE about the Hope Diamond? (A) Dame McLean was the most cursed person among the owners of the Diamond (B) Not only people but also an animal had worn the Hope Diamond (C) Having been the owner of the diamond for a decade. Winston donated it to the Smithsonian (D) The hope diamond maintained its original shape after it had been cut (E) As the diamond is flawless and priceless, it has never been sold 05. “To cast a spell of misfortune on someone‟ in line 5 means ___. (A) to wish someone bad luck (B) to do magic by saying special words (C) to put a curse on someone (D) to throw bad words to someone (E) to look at something quickly 06. Which of the following statements is the best conclusion of the paragraph? (A) These factors do not need to exists at the same time for a community to be regarded as urban (B) However, the main characteristic of a city is the construction of public buildings (C) Therefore, it is difficult for archaeologists to determine whether a discovered area was a city (D) To conclude, the definition of a city is based on which of the above characteristics an area has (E) Only by having all those characteristics can an area be called a true city 166

SPMB Bahasa Inggris, Kode Soal 711

07. The main idea of this paragraph is that ___. (A) none of the definitions of a city has been satisfactory (B) archaeologists have agreed as to what exactly (C) a city must include permanence of settlement (D) certain characteristics mark true urbanization (E) an area must reach certain size to become 08. The sentence that best begins the paragraph above is ___. (A) Many people have condemned fats for their bad effects (B) Fats store food more efficiently than carbohydrates (C) Although condemned, fats are in fact essential for us (D) Fats prove to be able to prevent some kinds of diseases (E) Fats are an increasingly important factor 09. The sentence which is irrelevant to the text is sentence number ____. (A) 2 (B) 3 (C) 4 (D) 5 (E) 6

167

10. Text IV Only humans have a spoken symbolic language : scientist have long thought that non human primates had much less sophisticated communication systems. True, but chimpanzees use gestures and many voice sounds in the wild. __10__ other apes use sounds to communicate territorial information. Chimpanzees seem to have a ___11___ talent for learning symbolic language under controlled __12__. A famous chimpanzee named Washoe, __13__ to communicate with humans. Using no less than 175 sign language gestures similar to those of the American sign language. After more than a year Washoe could associate __14__ signs with activities, such as eating and drinking. Another chimpanzee __15__ symbols and __16__ a vocabulary of 130 different words, to the extent that she __17__ sequences of written instructions given with the symbols. (A) (B) (C) (D) (E)

when while if so that although

SPMB Bahasa Inggris, Kode Soal 711

11.

12.

(A) (B) (C) (D) (E)

naturally naturalize natural naturalistic naturalism

(A) (B) (C) (D) (E)

protection reservations performance conditions experience

(A) (B) (C) (D) (E)

It trained being trained it was trained was training was trained

(A) (B) (C) (D) (E)

particulars particular particularly particularity particularize

(A) (B) (C) (D) (E)

naming named It names to name names

13.

14.

15.

16. (A) (B) (C) (D) (E)

applied required acquired produced attempted

(A) (B) (C) (D) (E)

practiced reminded detected obeyed listened

17.

18. I‟m really worried about his health as his next semester starts two weeks from now.‟ „Let‟s just hope that by that time he ___. (A) has recovered (B) is going to recover (C) will be recovering (D) will have recovered 19. „These tourist, ___ are Japanese, were among the crowd participating in the “dangdut” dance.‟ (A) there are many (B) many of them (C) whose many (D) many of whom (E) they who 20. „That woman over there looks confused.‟ „Why don‟t you ask her ___?‟ (A) does she need help (B) if she needed help (C) whether she needs help (D) do you need help (E) she needs help or not 21. „Can you come to my birthday party this Saturday?‟ „___. I will have to work overtime.‟ (A) Of course (B) I‟d like to (C) Sorry I can‟t (D) I‟m afraid so (E) I might not 22. „When will the debate contest take place this year?‟ „I‟ll let you know later as it___.‟ (A) has not scheduled yet (B) has been scheduled (C) has to schedule (D) has been scheduling (E) has not been scheduled yet

168

SPMB Bahasa Inggris, Kode Soal 711

23. The local society cannot continue with the construction of the two school buildings because of the lack of funds and ___. (A) the local political condition is very unstable (B) the locally unstable political condition (C) the political condition is locally unstable (D) the instability of the local political condition (E) the local instability of the political condition 24. „Why were they suddenly ill after having lunch in cafetaria?‟ „There ___ something wrong with the food.‟ (A) must have been (B) ought to be (C) would have been (D) might be (E) should have been 25. „Have you finished writing your final paper?‟ „Not quire.‟ From the above dialogue we may conclude that the paper ____. (A) is ready to be submitted (B) still has to be published (C) is rejected by the examiners (D) still needs revising (E) is to be presented in a seminar

169

SBMPTN 2018 Matematika Dasar Soal SBMPTN 2018 - Matematika Dasar - Kode Soal 517

01. Jika x1 dan x2 memenuhi, maka nilai x1x2 adalah …. 5 (A) 3 4 (B) 3 1 (C) 3

( 27 log

1 2 1 )  x 1 9

2

(D) - 3 4

(E) - 3

a 1 02. Jika A =  b 2  , B =  10  14 

(A) (B) (C) (D) (E)

 a 1  1 0  , dan AB = .  

a b  Maka nilai ab adalah ….

9 10 12 14 16

03. Diketahui persegi panjang ABCD dengan AB = 15 cm dan AD = 5 cm. Jika E merupakan titik potong diagonal persegi panjang tersebut, maka besar ∠BEC adalah …. (A) 30° (B) 45° (C) 60° (D) 75° (E) 90°

170

SBMPTN 2018 Matematika Dasar, Soal SBMPTN 2018 - Matematika Dasar - Kode Soal 517

04. Diketahui 10 bilangan genap berurutan. Jika kuartil pertama bilangan-bilangan tersebut adalah 32, maka mediannya adalah …. (A) 34 (B) 35 (C) 36 (D) 37 (E) 38 05. Himpunan adalah …. (A) (B) (C) (D) (E)

penyelesaian

x

6 x 0

{x | x ≤ -3 atau x ≥ 2} {x | x ≤ -3 atau 2 ≤ x ≤ 6} {x | 0 ≤ x ≤ 6} {x | 2 ≤ x ≤ 6} {x | x ≤ 6}

06. Diketahui a, b, dan c adalah bilangan real positif dengan ab > 1. Jika x + ay = c, bx + y = 2c, dan x < y, maka …. (A) 2a > b - 1 (B) 2a > b - 2 (C) 2a < b - 3 (D) 2a < b - 2 (E) 2a < b - 1

07. Diketahui A = {9, 7, 6, 5, 4, 3, 2, 1}. Lima anggota A diambil secara acak. Peluang terambilnya lima anggota tersebut berjumlah genap adalah …. (A)

1 2

25 56 5 (C) 12

(B)

1 4 5 (E) 56 (D)

171

SBMPTN 2018 Matematika Dasar, Soal SBMPTN 2018 - Matematika Dasar - Kode Soal 517

08. Diketahui suatu barisan aritmetika yang terdiri atas empat suku. Jika hasil kali tiga suku pertamanya adalah 10, hasil kali tiga suku terakhirnya adalah –8, dan hasil penjumlahan dua suku tengahnya adalah –1, maka hasil kali dua suku tengahnya adalah …. (A) -5 (B) -2 (C) 2 (D) 4 (E) 10

09. Titik (a, b) terletak pada grafik y = bx2 + (1-b2)x - 49. Jika ab = 6, maka nilai a - b adalah …. (A) 7 (B) 5 (C) 1 (D) -1 (E) -5

10. Diketahui x2 + a2x + b2 = 0 dengan a > 0, b > 0. Jika jumlah akar persamaan tersebut sama dengan -(b+1) dan hasil perkalian akarakarnya a2 + 5, maka nilai a + b - ab adalah …. (A) -2 (B) -1 (C) 0 (D) 1 (E) 2

172

SBMPTN 2018 Matematika Dasar, Soal SBMPTN 2018 - Matematika Dasar - Kode Soal 517

1 11. Jika fungsi f(x) = , g(x) = x2+ b, xa 1 (f o g)(1) = , dan (g o f)(1) = 2, maka nilai ab adalah ….2

(A) -1 (B) 0 1 (C) 2 (D)

3 2

(E) 2

12. Diketahui fungsi f dan g mempunyai invers. Jika g(2f(x)) = 2x - 1 dan f(x - 2) = x + 3, maka nilai f -1(-1).g-1(-1) adalah …. (A) -60 (B) -50 (C) -40 (D) -30 (E) -20

 16  6 x 4 13.   x2

  dx  .... 

(A) 16  2x 3  C

x 16 (B)  2x3  C x (C)  16  x3  C x 8 (D)   2x3  C x

(E)

8  2x3  C x

173

SBMPTN 2018 Matematika Dasar, Soal SBMPTN 2018 - Matematika Dasar - Kode Soal 517

14. Diketahui f(x) = x2 + ax dan g(x) = x2 - 2x + a.

f ( x) dengan h(1) = -2, maka g ( x) nilai h’(0) adalah ….

Jika h(x) =

(A) -

3 2 1

(B) 6 (C) 0 (D) 1 3 (E) 2

15. x

A

P

B x Q

S x D

R

x

C

Diketahui persegi panjang ABCD dengan ukuran panjang 12 cm dan lebar 8 cm. Pada masing-masing sisi, ditetapkan sebuah titik sejauh x cm dari setiap titik sudut, sehingga terbentuk sebuah segi empat PQRS seperti tampak pada gambar. Luas terkecil yang mungkin dari segiempat PQRS adalah … cm2. (A) 40 (B) 46 (C) 64 (D) 72 (E) 85

174

SBMPTN 2017 Matematika Dasar Soal SBMPTN 2017 - Matematika Dasar - Kode Soal 264

01. Misalkan AT adalah transpos matriks A. Jika 1 x y 2

A =

BAT =

dan B =

2

4

5 10

1 3 4 1

sehingga

maka nilai 2x + 3y

adalah .... (A) 2 (B) 4 (C) 6 (D) 8 (E) 10 02. Jika himpunan penyelesaian |2x - a| < 5 adalah {x | -1 < x < 4} maka nilai a adalah .... (A) -4 (B) -3 (C) -1 (D) 3 (E) 4 A

03.

K L B

M

N

C

Pada segitiga siku-siku samakaki ABC, sisi AB dan BC masing-masing terbagi menjadi tiga bagian yang sama, berturut-turut oleh titik K, L, dan M, N. Jika luas ∆ABC adalah x cm2 maka luas ∆KMN adalah ... cm2. (A) (B) (C) (D) (E)

x 3 2x 9 x 9 x 18 x 36

175

SBMPTN 2017 Matematika Dasar, Soal SBMPTN 2017 - Matematika Dasar - Kode Soal 264

2

04. Sumbu simetri grafik f(x) = ax + bx + c adalah x = 2. Jika f(0) = 1 dan f(1) = -2 maka nilai a - b adalah .... (A) 1 (B) 2 (C) 3 (D) 4 (E) 5

05. Diketahui median dan rata-rata berat badan 5 balita adalah sama. Setelah ditambahkan satu data berat badan balita, rata-ratanya meningkat 1 kg, sedangkan mediannya tetap. Jika 6 data berat badan tersebut diurutkan dari yang paling ringan ke yang paling berat maka selisih berat badan antara balita terakhir yang ditambahkan dan balita di urutan ke-4 adalah ... kg. (A) 4 (B)

9 2

(C) 5 (D) 6 (E)

13 2

06. Akan dikonstruksi suatu barisan aritmetika dengan beda b yang terdiri atas n suku. Selisih suku terbesar dan suku terkecil adalah 27. Jika n = b + 7 maka selisih suku ke-n dan suku ke-4 adalah .... (A) 12 (B) 14 (C) 16 (D) 18 (E) 20

176

SBMPTN 2017 Matematika Dasar, Soal SBMPTN 2017 - Matematika Dasar - Kode Soal 264

07. Seseorang memelihara ikan di suatu kolam. Rata-rata bobot ikan per ekor pada saat panen dari kolam tersebut adalah (6 - 0,02x) kg, dengan x menyatakan banyak ikan yang dipelihara. Maksimum total bobot semua ikan pada saat panen yang mungkin adalah ... kg. (A) 400 (B) 420 (C) 435 (D) 450 (E) 465

08. Lima bilangan asli membentuk suatu barisan geometri dengan rasio positif. Jika jumlah 3 suku terbesar dan jumlah 3 suku terkecil barisan geometri tersebut berturut-turut adalah 171 dan 76 maka jumlah 5 bilangan tersebut adalah .... (A) 125 (B) 130 (C) 180 (D) 211 (E) 347

2

09. Jika f(x) = x dan g(x) = x + 1 maka daerah asal fungsi komposisi f ○ g adalah .... (A) {x | -∞ < x < ∞} (B) {x | x ≠ 1} (C) {x | x ≠ 0} (D) {x | x ≥ 0} (E) {x | x ≥ 1}

177

SBMPTN 2017 Matematika Dasar, Soal SBMPTN 2017 - Matematika Dasar - Kode Soal 264

10.

H

G

E

F P

D

C

Q A

B

Diketahui kubus ABCD.EFGH dengan P adalah titik tengah BF dan Q adalah titik potong AP dan BE seperti pada gambar. Jika panjang rusuk kubus tersebut 6 cm maka jarak Q ke H adalah ... cm. (A) 3 13 (B) 2 17 (C) 3 3 (D) 2 6 (E) 4 11. Luas daerah penyelesaian sistem pertidaksamaan x + y ≥ 2, x + 4y ≤ 3, y ≥ 0 adalah ... satuan luas. (A) (B) (C) (D) (E)

1 6 1 3 1 2 25 24 25 12

12. Gradien peta garis y = x + 1 di bawah pencerminan terhadap sumbu –x dan 1

kemudian translasi   adalah ....  2 (A) -2 (B) -1 (C) 0 (D) 1 (E) 2

178

SBMPTN 2017 Matematika Dasar, Soal SBMPTN 2017 - Matematika Dasar - Kode Soal 264

13.



x 1 x2  2 x

dx 

(A)

1 x2  2x  C 2

(B)

x2  2 x  C

(C) 2 x 2  2 x  C (D) ( x 2  x) x 2  2 x  C 1 2 x x 2 C (E) 2 2 2 ( x  2 x) x  2 x 3

14. Jika f(x) = ax + b dan lim x4

2 x 1  f ( x) 4

maka f(2) = …. (A) -6 (B) -2 (C) 2 (D) 4 (E) 6

15. Jika 4 laki-laki dan 2 perempuan duduk dalam satu baris sehingga tidak ada 2 perempuan yang duduk berdekatan maka banyak susunan duduk berbeda yang mungkin adalah .... (A) 384 (B) 408 (C) 432 (D) 456 (E) 480

179

SBMPTN 2017 Matematika Dasar Soal SBMPTN 2017 - Matematika Dasar - Kode Soal 233

01. Misalkan AT adalah transpos matriks A. Nilai T

ab yang memenuhi  a 4    1 2  1 2        b a   2 1  2 1  adalah … (A) (B) (C) (D) (E)

-20 -2 1 10 20

02. Jika himpunan penyelesaian |2x - a| < 5 adalah { x|-1<x<4} maka nilai a adalah .... (A) -4 (B) -3 (C) -1 (D) 3 (E) 4 03.

A

K

L

B

M

N

C

Pada segitiga siku-siku sama kaki ABC, sisi AB dan BC masing-masing terbagi menjadi tiga bagian yang sama, berturutturut oleh titik K, L, M, dan N. Jika luas ΔABC adalah x cm2 maka luas ΔKMN adalah ... cm2. x (A) 3 2x (B) 9 x (C) 9 x (D) 18 x (E) 36

180

SBMPTN 2017 Matematika Dasar, Soal SBMPTN 2017 - Matematika Dasar - Kode Soal 233

04. Jika f  x  

3 x 2  2x dan g  x   maka x 1 x 1

daerah asal fungsi f·g adalah .... (A) { x|-∞ < x < ∞} (B) { x|x ≠ -1} (C) { x|x ≠ -1 dan x ≠ 1} (D) { x|x < -1 atau x > 1} (E) { x|-1 < x < 1} 05. Diketahui median dan rata-rata berat badan 5 balita adalah sama. Setelah ditambahkan satu data berat badan balita, rata-ratanya meningkat 1 kg, sedangkan mediannya tetap. Jika 6 data berat badan tersebut diurutkan dari yang paling ringan ke yang paling berat, maka selisih berat badan antara balita terakhir yang ditambahkan dan balita di urutan ke-4 adalah ... kg. (A) 4 (B) 9

2

(C) 5

(D) 6 (E)

13 2

06. Hasil bagi suku pertama oleh suku ke-3 suatu barisan aritmetika adalah ½ . Jika suku ke-5 barisan tersebut adalah 12 maka suku ke-7 adalah …. (A) 12 (B) 14 (C) 16 (D) 22 (E) 28 07. Seseorang memelihara ikan di suatu kolam. Rata-rata bobot ikan per ekor pada saat panen dari kolam tersebut adalah (6 - 0,02x)kg dengan x menyatakan banyak ikan yang dipelihara. Maksimum total bobot semua ikan pada saat panen yang mungkin adalah … kg. (A) 400 (B) 420 (C) 435 (D) 450 (E) 465 181

SBMPTN 2017 Matematika Dasar, Soal SBMPTN 2017 - Matematika Dasar - Kode Soal 233

08. Lima bilangan asli membentuk suatu barisan geometri dengan rasio positif. Jika jumlah 3 suku terbesar dan jumlah 3 suku terkecil barisan geometri tersebut berturut-turut adalah 171 dan 76 maka jumlah 5 bilangan tersebut adalah …. (A) 125 (B) 130 (C) 180 (D) 211 (E) 347 09. D i ke t a h u i f un g s i f(x) = 2x - 4 da n g(x) = x2 + ax + b. Jika (g о f )(2) = 2 dan (g ο f )(3) = 8 maka nilai a + b adalah …. (A) 1 (B) 2 (C) 3 (D) 4 (E) 5 H

10.

G F

E

M

T D

C N

A

B

Diketahui kubus ABCD.EFGH dengan M dan N berturut-turut adalah titik tengah FG dan BC, serta T adalah titik pada AM sehingga NT tegak lurus AM, seperti pada gambar. Jika panjang rusuk kubus tersebut 8 cm maka panjang NT adalah … cm. (A) (B) (C) (D) (E)

182

8 3 8 5 7 5 5 3 3 2

5 5

5 2

2

SBMPTN 2017 Matematika Dasar, Soal SBMPTN 2017 - Matematika Dasar - Kode Soal 233

11. Luas daerah penyelesaian sistem pertidaksamaan x - y ≤ 4, 2x + y ≥ 2, y ≤ 0 adalah … satuan luas. (A) 1

1 2

(B) 2 (C) 2

1 2

(D) 3

(E) 4 12. Titik (2, 3) dicerminkan terhadap garis y = -x dan kemudian ditranslasikan dengan a   ke titik (3, 2). Peta titik (3, 2) di bawah b

transformasi yang sama adalah .... (A) (-3, -2) (B) (-2, -3) (C) (3, 2) (D) (4, 1) (E) (6, 4) 13. 

3 1  x  1

(A) (B) (C) (D) (E)

x

dx  ....

3x  2 x x  C 2 x  3x x  C 3x x  2 x  C 2 x x  3x  C 3x  2 x x  C

14. Jika f(x) = ax2 + b dan lim

x 1

maka a + b = ….

x 1  x  f  x

 4

1 2 1 (B)  4 1 (C)  8

(A) 

(D) 0 (E)

1 8

183

SBMPTN 2017 Matematika Dasar, Soal SBMPTN 2017 - Matematika Dasar - Kode Soal 233

15. Sebuah bilangan ganjil 5 angka memuat tepat 4 angka ganjil dan tidak memiliki angka berulang, serta tidak memuat angka 0. Banyak bilangan berbeda dengan ciri tersebut adalah …. (A) 1.920 (B) 1.940 (C) 1.960 (D) 2.100 (E) 2.400

184

SBMPTN 2016 Matematika Dasar Latihan Soal Matematika Dasar SBMPTN 01. Diketahui 7  7 adalah salah satu akar x2  ax  b  0 dengan b bilangan real negatif dan a suatu bilangan bulat. Nilai terkecil a adalah .... (A) -5 (B) -4 (C) 0 (D) 4 (E) 5

02. Jika A2 x  2, maka (A) (B) (C) (D) (E)

A5 x  A5 x  .... A3 x  A3 x

31 18 31 9 32 18 33 9 33 18

03. Suatu garis yang melalui titik (0,0) membagi persegi panjang dengan titik-titik sudut (1,0), (5,0), (1,12), dan (5,12) menjadi dua bagian yang sama luas. Gradien garis tersebut adalah .... (A)

1 2

(B) 1 (C) 2 (D)

12 5

(E) 3

185

SBMPTN 2016 Matematika Dasar, Latihan Soal Matematika Dasar SBMPTN

04. Semua bilangan real x yang memenuhi x x3 adalah ....  x 3 x  2 9 atau x  3 2 9 (B) x   atau  2  x  3 2 9 (C)   x  2  x  3 2 9 (D)   x  3 2

(A) x  

(E) x  3 atau  2  x  3

05. Jika grafik fungsi y  x2   9  a  x  9a diperoleh dari grafik fungsi y  x 2  2 x  3 melalui pencerminan terhadap garis x  4, maka a  .... (A) 7 (B) 5 (C) 3 (D) -5 (E) -7

06. Tujuan finalis menyanyi tingkat SMA di suatu kota berasal dari 6 SMA yang berbeda terdiri atas empat pria dan tiga wanita. Diketahui satu pria dan satu wanita berasal dari SMA “A”. Jika urutan tampil diatur bergantian antara pria dan wanita, serta finalis dari SMA “A” tidak tampil berurutan, maka susunan urutan tampil yang mungkin ada sebanyak .... (A) 144 (B) 108 (C) 72 (D) 36 (E) 35

186

SBMPTN 2016 Matematika Dasar, Latihan Soal Matematika Dasar SBMPTN

07. Diberikan fungsi f  x   ax  1 dan g  x   x  1. Jika

 f  g  x    g  f  x  , maka f  2  g 1 = …. (A) 2 (B) 1 (C) 0 (D) -1 (E) -2

08. Jika fungsi f dan g mempunyai invers dan memenuhi f  x  5  g  2 x  1 , maka 2 f 1  x   .... 1 (A) g  x   11 1 (B) g  x   9 1 (C) g  x   6 1  x  (D) g  2   6 1 (E) g  2 x   6

09. Diketahui matriks  1 0 a 1 1   A  dan B   a b  0 1 b  0 1   Jika a  0 dan AB tidak mempunyai invers, maka b = .... (A) 0 1 3 (C) 1 2

(B)

(D) 1 (E) 3

187

SBMPTN 2016 Matematika Dasar, Latihan Soal Matematika Dasar SBMPTN 4 3 7 6 9 10. Bilangan log  ab  ,log  a b  ,dan log  a b  merupakan tiga suku pertama barisan aritmatika. Jika suku ke-11 barisan tersebut p adalah log  a  , maka p  ....

(A) (B) (C) (D) (E)

29 55 66 95 121

11.

Titik X, Y, dan Z terletak pada segitiga ABC sehingga AZ=AY, BZ=BX, CX=CY seperti pada gambar. Jika BC, CA, dan AB berturut – turut adalah a cm, b cm, dan c cm, maka 2AY= ....cm (A) a + b + c (B) a - b + c (C) a + b - c (D) - a - b + c (E) b + c - a

12. Nilai ujian matematika di suatu kelas berupa bilangan bulat positif yang tidak lebih besar dari pada 10. Rata-rata nilai ujian matematika untuk 40 siswa di kelas tersebut adalah 7. Dua orang mengikuti ujian susulan dan memperoleh nilai yang berbeda dan ternyata merupakan nilai yang paling tinggi dan paling rendah di kelas tersebut. Jika rata-rata nilai 42 siswa tersebut tetap 7, maka jangkauan data nilai ujian 42 siswa di atas yang mungkin ada sebanyak .... (A) 1 (B) 2 (C) 3 (D) 4 (E) 5

188

SBMPTN 2016 Matematika Dasar, Latihan Soal Matematika Dasar SBMPTN

x2  4

 lim ax 2  b dan lim ax 2  b  2, 13. Jika xlim 2 x  2 x 1 x 2

maka a - b= .... (A) -4 (B) -2 (C) 0 (D) 2 (E) 8

14. Jika  x, y   1,1 dan  x, y    a, 2  merupakan penyelesaian 3x  y  b dan cx - dy  1, maka a +b +c -d = .... (A) 7 (B) 6 (C) 6 1 2

(D) -6 (E) -7

15. Semua bilangan real x yang memenuhi 1 1  adalah .... x  2 1 x

(A) x < 1 (B) x > 1 (C) x < 2 (D) x  1 atau

3 x2 2

3 2

(E) 1  x  atau x  2

189

SBMPTN 2016 Matematika Dasar Matematika Dasar kode 317 - Soal

01. Misalkan dua persamaan kuadrat mempunyai satu akar yang sama, yaitu 2 dan akar-akar lainnya berkebalikan. Jika salah satu persamaan itu adalah x 2  ax  6  0 , maka persamaan kuadrat lainnya adalah .... (A) x2 + x – 6 = 0 (B) 3x2 – 7x + 2 = 0 (C) 3x2 + 4x – 6 = 0 (D) x2 – x +

2 =0 3

(E) 2x2 – 3x – 4 = 0

2x 02. Jika A  2, maka

(A) (B) (C) (D) (E)

A5 x  A5 x  .... A3 x  A3 x

31 18 31 9 32 18 33 9 33 18

03. Suatu garis yang melalui titik (0,0) membagi persegi panjang dengan titik-titik sudut (1,0), (5,0), (1,12), dan (5,12) menjadi dua bagian yang sama luas. Gradien garis tersebut adalah .... (A)

1 2

(B) 1 (C) 2 (D)

12 5

(E) 3

190

SBMPTN 2016 Matematika Dasar, Matematika Dasar kode 317 - Soal

04. Semua bilangan real x yang memenuhi x 2 x adalah ....  2 x x (A) (B) (C) (D) (E)

 2  x  0 atau  2  x  2 x   2 atau 0  x  2 x   2 atau x  2 x   2 atau x  2 0 x2

05. Jika grafik fungsi y  x2   9  a  x  9a diperoleh dari grafik fungsi y  x 2  2 x  3 melalui pencerminan terhadap garis x  4, maka a  .... (A) 7 (B) 5 (C) 3 (D) -5 (E) -7 06. Tujuh finalis menyanyi tingkat SMA di suatu kota berasal dari 6 SMA yang berbeda terdiri atas empat pria dan tiga wanita. Diketahui satu pria dan satu wanita berasal dari SMA “A”. Jika urutan tampil diatur bergantian antara pria dan wanita, serta finalis dari SMA “A” tidak tampil berurutan, maka susunan urutan tampil yang mungkin ada sebanyak .... (A) 144 (B) 108 (C) 72 (D) 36 (E) 35 07. Jika tabel berikut menyatakan hasil fungsi f dan g, x

0 1 2 3

f(x) 1 3 1 -1 g(x) 2 0 1 2 maka  f  g  f 1   f  g  f  2  = …. (A) 2 (B) 1 (C) 0 (D) -1 (E) -2

191

SBMPTN 2016 Matematika Dasar, Matematika Dasar kode 317 - Soal

08. Jika fungsi f dan g mempunyai invers dan memenuhi f  2 x   g  x  3 , maka f-1(x) = …. (A) g 1  x  3  2 2 (B) (C) (D) (E)

g 1  2 x  6  2 g 1  x   3 2 g 1  x   6 2 g 1  x   4

09. Diketahui matriks 8 A a

a  1 1  dan B    , dan C 1 b b 

adalah matriks berukuran 2 × 2 yang mempunyai invers. Jika AC dan BC tidak memiliki invers, maka 3a2 + 4b3 = .... (A) 16 (B) 20 (C) 24 (D) 28 (E) 36

10. Misalkan Uk dan Sk berturut-turut menyatakan suku ke-k dan jumlah k suku pertama suatu barisan aritmatika. Jika U2 – U4 + U6 – U8 + U10 – U12 + U14 – U16 + U18 = 20, maka S19 = …. (A) 630 (B) 380 (C) 210 (D) 105 (E) 21

192

SBMPTN 2016 Matematika Dasar, Matematika Dasar kode 317 - Soal

11.

Diketahui semua titik sudut segienam beraturan ABCDEF terletak pada lingkaran yang berjari-jari 2 cm seperti pada gambar. Luas daerah yang tidak diarsir pada segienam tersebut adalah … cm2.

(A) 3 (B) 2 3 (C) 4 (D) 3 3 (E) 6 3

12. Nilai ujian matematika 30 siswa pad suatu kelas berupa bilangan pecahan cacah yang tidak lebih daripada 10. Rata-rata nilai mereka 8 dan hanya terdapat 5 siswa yang memperoleh nilai 7. Jika p menyatakan banyak siswa yang memperoleh nilai kurang dari 7, maka nilai p terbesar yang mungkin adalah .... (A) 5 (B) 7 (C) 9 (D) 11 (E) 14

13. Jika a dan b bilangan bulat, serta lim

x2  x  b  a , maka a-b .... 2 x

(A) (B) (C) (D) (E)

-5 -3 -1 2 5

x2

193

SBMPTN 2016 Matematika Dasar, Matematika Dasar kode 317 - Soal

14. Jika (x,y) = (1,3) dan (x,y) = (a,1) merupakan penyelesaian x - 2y = b dan cx + dy =10 maka a + b + c - d = .... (A) -9 (B) -6 (C) 0 (D) 3 (E) 9

15. Semua bilangan real x yang memenuhi |x - 2| > x2 - 4 adalah .... (A) x < -2 atau x > 3 (B) x < -2 atau x > 2 (C) x < -2 atau x > 0 (D) -2 < x < 3 (E) -3 < x < 2

194

SBMPTN 2015 Matematika Dasar

46. Jika a dan b adalah bilangan real positif, maka (A) -2 (B) -1 (C) 0



3

  2

a b 

3

ab



2 3 ab

2

 ....

(D) 1 (E) 2

47. Diketahui suatu barisan aritmetika dengan suku pertama dan suku ketiga berturut-turut adalah k - 1 dan 3k + 1. Jika suku kesepuluh adalah 98, maka suku kelima barisan tersebut adalah .... (A) 58 (B) 56 (C) 48 (D) 46 (E) 36

48. Diketahui persegi panjang ABCD. Jika panjang BE = EF = FC = 5cm, dan panjang DG = panjang GH = panjang HC = 3cm, maka luas daerah yang diarsir adalah .... cm2. (A) 22,5 (B) 45 (C) 60 (D) 67,5 (E) 90 49. Diketahui plog 2 = 9 dan qlog 4 = 8. Jika s = p3 dan t = q2, maka nilai tlog s adalah .... (A) 1/4 (B) 1/2 (C) ²/3 (D) ³/2 (E) 2

195

SBMPTN 2015 Matematika Dasar, Matematika Dasar Kode Soal 616

50. Diagram di bawah ini menyajikan data (dalam bilangan bulat) nilai sementara dari nilai ujian ulang mahasiswa peserta kuliah Matematika. Ujian ulang diikuti hanya oleh peserta kuliah tersebut dengan nilai sementara lebih kecil daripada 6. Jika yang dinyatakan lulus kuliah adalah mahasiswa yang memperoleh nilai sementara tidak lebih kecil daripada 6 atau nilai ujian ulangnya adalah 6 maka ratarata nilai mahasiswa yang lulus mata kuliah tersebut adalah ....

(A) 6,33 (B) 6,50 (C) 6,75 (D) 7,00 (E) 7,25 51. Himpunan penyelesaian pertidaksamaan 1 x  1 adalah .... 1 x

(A)  x  R | x  1

(B) x  R | x  1 (C)  x  R | x  1

(D) x  R | x  0 atau x  1

(E) x  R | x  1 atau x  1

196

SBMPTN 2015 Matematika Dasar, Matematika Dasar Kode Soal 616

52. Diketahui suatu fungsi f bersifat

f   x    f  x  untuk setiap bilangan real x. Jika f (3)  5 dan f (5)  1 , maka

f ( f (3))  ....

(A) -5 (B) -2 (C) -1 (D) 1 (E) 2

53. Diketahui sistem persamaan linier  2x 1 2  3 y  3  2  3,   4 x  y  2 x  2 y  3.  6 3

Nilai x-y adalah .... (A) -2 (B) -1 (C) 0 (D) 1 (E) 2

54. Empat orang siswa akan mengikuti suatu perlombaan karya inovatif. Untuk itu, diperlukan biaya Rp 900.000,00. Karena masing-masing memiliki kondisi keuangan yang berbeda, besar kontribusi masing-masing siswa tidak sama. Siswa A memberikan kontribusi setengah dari jumlah kontribusi tiga siswa lainnya. Siswa B memberikan kontribusi sepertiga dari jumlah kontribusi tiga siswa lainnya. Siswa C memberikan kontribusi seperempat dari jumlah kontribusi tiga siswa lainnya. Besar kontribusi siswa D adalah Rp .... (A) 150.000,00 (B) 180.000,00 (C) 195.000,00 (D) 225.000,00 (E) 300.000,00 197

SBMPTN 2015 Matematika Dasar, Matematika Dasar Kode Soal 616

55. Jika f  x  2   (A) (B) (C) (D) (E)

1 , maka f 1  x   .... 2  5x

1  12 x 5x 1  12 x 5x 1 2x 5x 1 2x 5x 1 12  5x

 2a a  56. Jika A    merupakan matriks yang  4 a mempunyai invers, maka hasil kali semua nilai a yang mungkin sehingga

det  A2   8det  A1  adalah ....

(A) (B) (C) (D) (E)

0 1 2 3 4

57. Jika a dan b adalah bilangan prima dan semua akar x2 – ax + b = 0 merupakan bilangan bulat positif, maka nilai ab2 adalah .... (A) 8 (B) 12 (C) 18 (D) 27 (E) 45

198

SBMPTN 2015 Matematika Dasar, Matematika Dasar Kode Soal 616

58. Jika grafik fungsi y = 8x – x2 memotong sumbu –x di titik A dan B, serta C adalah titik puncaknya, maka luas segitiga ABC adalah .... (A) 32 (B) 48 (C) 64 (D) 72 (E) 80

59. Ghiqo mengikuti lima kali test matematika dengan nilai empat test pertamanya berturut-turut adalah 8, 7, 5, dan 7. Jika median dan rata-rata lima nilai tes adalah sama, maka nilai tes terakhir Ghiqo adalah .... (A) 6 (B) 7 (C) 8 (D) 9 (E) 10

60. Empat buku berjudul Matematika dan dua buku berjudul Biologi akan disusun di lemari buku dalam satu baris. Misalkan A adalah kejadian susunan buku sehingga tidak ada tiga atau lebih buku dengan judul yang sama tersusun secara berurutan. Jika buku dengan judul yang sama tidak dibedakan, maka peluang kejadian A adalah .... (A) 1/5 (B) 2/5 (C) 1/2 (D) 3/5 (E) 4/5

199

Xpedia Matematika Kapita Selekta – USM Set 01

01. Jika 2 < x < 3, dan |x -1|-|x -2|+ |x -3| = ? (A) -2 (B) x (C) -x (D) 4 - x (E) 3x - 6 02. Jika 0 < a < b dan a2 + b2 = 4ab, maka b a ? b-a 03. Misalkan a dan b bilangan real yang berbeda. b a a 10 b 2 , maka =…. Jika a b b 10 a (A) 1 1 (B) 2 4 (C) 5 5 (D) 4 3 (E) 2 04. f(x) = ax2 + bx - 2, f (2009) = 18, g(x) = ax2 + bx + 5, maka nilai dari g( 2009 ) .... (A) (B) (C) (D) (E)

3 2 4 2 5 5 3 3

05. Jumlah semua bilangan bulat antara -20 dan 21 yang memenuhi pecahan

x 2 3x 10 x2 x 2 (A) (B) (C) (D) (E)

0 adalah ….

26 27 28 29 30 206

Xpedia Matematika, Kapita Selekta – USM Set 01

06. Diketahui f(2x - 1) = 3x + 7. Jika f(a) = 40, maka nilai a adalah…. (A) 11 (B) 2 (C) 6 (D) 21 (E) 5 07. Jumlah semua bilangan yang memenuhi 2

persamaan ( x - 25) x - 3 log (x 2) 0 adalah…. (A) -2 (B) -4 (C) 1 (D) 2 (E) 4 09. Apabila diketahui f(0) = 1, f ’(0) = -3, g(0) = 2, g’(0) = 4 dan , h( x ) h’(0) = …. 3 (A) 4 10 (B) 4

f(x) maka g( x )

(C) -2 (D) (E)

10 4 10 16

10. Nilai x yang memenuhi sistem persamaan |2log x + 3log y2 - 5|+|2log x - 3log y| = 0 adalah…. (A) (B) (C) (D) (E)

207

2

2 2 2 4 4 2

Xpedia Matematika, Kapita Selekta – USM Set 01

11. Panjang AB pada gambar adalah….

(A) 3 2 (B) 4 3 (C) 6 5

39 5 100 (E) 7 (D)

208

Xpedia Matematika, Kapita Selekta – USM Set 01

12. Pada gambar di bawah ini, ABC segitiga sama sisi dengan panjang sisi 2. Persamaan parabola yang berpuncak di A dan melalui titik B dan C adalah…. y

C

B

(A) y

1 (x - 3 ) 2 3

y

3(x - 1)2

(B)

(C) y

209

3 x

1 3

2

(D) y

3(x - 3 ) 2

(E) y

1 (x - 1) 2 3

x

A

O

Xpedia Matematika, Kapita Selekta – USM Set 01

13.Diketahui grafik fungsi f(x) seperti gambar di bawah, maka grafik fungsi f(3x) adalah…. y

p

x

a

(A)

(B)

(C)

(D)

(E)

210

Xpedia Matematika, Kapita Selekta – USM Set 01

16.log A = 0.222 …. dan log B = 0,333 …. maka log (A 2 .3 B ) .... (A) 0,111…. (B) 0,5252…. (C) 0,555…. (D) 0,333…. (E) 0,2323…. 17. Garis singgung pada kurva y = 2x3 - x2 di titik potongnya dengan sumbu -x yang absisnya positif mempunyai gradient ….

5 2 (B) 2 3 (C) 2 (D) 1 1 (E) 2 18. Jumlah semua bilangan yang memenuhi (A)

persamaan ( x 2 16 ) x 2 (A) -2 (B) 0 (C) 2 (D) 4 (E) 6

0 adalah….

19. Bila f(x) = x2 - 5, banyaknya jawaban persamaan f[f(x - 1)] - 4 adalah…. (A) 1 (B) 2 (C) 3 (D) 4 (E) 5 20. Kurva y = (x - 2)2 - 4 diperoleh dengan menggeser kurva y = x2 + 6 …. (A) Ke kiri 2 satuan dan ke atas 2 satuan (B) Ke kanan 2 satuan dan ke atas 10 satuan (C) Ke kiri 2 satuan dan ke atas 10 satuan (D) Ke kiri 2 satuan dan ke bawah 10 satuan (E) Ke kanan 2 satuan dan bawah 10 satuan

211

Xpedia Matematika Kapita Selekta - Set 01

01. Bentuk sederhana dari:

7

4 3

(A) (B) (C) (D) (E)

7

....

4 3

14 1 14 4

4 7 14

02. Grafik : y 3x 1 x 2 2 x 1 untuk x < 0, akan berimpit dengan .... (A) y = 2x + 2 (B) y = 2x-2 (C) y = 4x-2 (D) y = 4x +2 (E) y = 4x 2 03. Bentuk 1 tan x (A) 0 x 2

(B) 0 x (C) 0 (D) 0

2

x

2

x

(E)

3 2

x

04. Grafik y

x

2

(E)

y

–4

y

x

4

(C)

4 x adalah....

(D)

(A) y

(B)

sec x dipenuhi oleh....

x

4

4

x

2

x

y

y

–4

4

x

212

Xpedia Matematika, Kapita Selekta - Set 01

x 1 ax mempu2 3 nyai penyelesaian x > 5. Nilai a = .... (A) 2 (B) 3 (C) 4 (D) 5 (E) 6

05. Pertidaksamaan 2 x a

06. Jika x = 3 merupakan penyelesaian dari : ( x - k )(x - 2k) < 2 maka .... 1 (A) k 7 atau k 2 1 (B) k 3 atau k 1 3 2 (C) k 1 atau k 7 1 (D) k 2

7

(E) 1 k

3

1 2

07. Jika y x 2 8x 10 maka nilai y yang me menuhi x 2 - 7x + 10 < 0 adalah .... 2 (A) 6 y (B) -7 < y < -5 (C) -6 < y < -5 (D) -5 < y < -1 1 (E) 1 x 3 2 2x 1 3

81 dipenuhi 08. Pertidaksamaan 9 3 oleh.... (A) -4 < x < 3 (B) -2 < x < 2 (C) -3 < x < -2 atau 2 < x < 4 (D) -3 < x < -2 atau 2 < x < 4 (E) x < -4 atau –2 < x < 2 atau x > 3 09. Untuk 3 < x < 9 maka pertidaksamaan : x x 3 x 9 x 2 x 6 dipenuhi oleh .... (A) x ≤ 1 atau x ≥ 5 (B) 1 ≤ x ≤ 5 (C) 5 ≤ x < 9 (D) x ≤ 5 atau x > 9 (E) 3 < x ≤ 5

213

Xpedia Matematika, Kapita Selekta - Set 01

10. Batas-batas x yang memenuhi: 14 2 x x 3 adalah.... (A) x < 5 (B) -1 < x < 5 (C) x < -1 atau 5 < x ≤ 7 (D) 3 ≤ x < 5 (E) -1 < x ≤ 7 2 6 x 8 digeser ke kanan 12. Parabola y x sejauh 2 satuan searah dengan sumbu x dan digeser ke bawah sejauh 3 satuan. Jika parabol hasil penggeseran ini memotong sumbu x di x1 dan x2 maka x 1 x 2 .... (A) 8 (B) 9 (C) 10 (D) 11 (E) 12

13. Suatu populasi hewan mengikuti hukum pertumbuhan yang berbunyi N(t) = 100000 . 2 t -2 N(t) = Besar populasi pada saat t t = Waktu dalam satuan tahun Agar besar populasi menjadi 3 kali lipat populasi awal (saat t = 0), maka t = .... (A) 10log 3 (B) 10log 3 - 2 (C) 2log 3 - 4 (D) 2log 3 - 2 (E) 2log 3 14. Diketahui empat titik A, B, C, dan D yang berada pada lingkaran dengan panjang AB = 4 cm, BC = 3 cm, CD = 3 cm, dan AD = 6 cm. Cosinus sudut BAD adalah .... (A)

14 33

16 33 17 (C) 33

(B)

(D)

19 33

(E)

20 33

214

Xpedia Matematika, Kapita Selekta - Set 01

4

3

2

15. Jika P(x) x 5 x 9 x 13 x a dibagi dengan (x + 3) bersisa 2, maka P(x) dibagi (x + 1) akan bersisa .... (A) 2 (B) -3 (C) 4 (D) -5 (E) 6 16. Agar persamaan x3 + nx + 2 = 0 memiliki akar kembar, maka nilai n2 + n = .... (A) 0 (B) 2 (C) 6 (D) 12 (E) 20 17. Jika

maka bentuk sederhana dari | x - 5| | x 3|-|10 - x| adalah ....

(A) (B) (C) (D) (E)

18.

x

1

1

x+2 x-2 2-x 1-x x n

lim

n

(A) 0

n 1

2n

3n 6

n

...

1

(B) 2 (C) 1 3

(D) 2 (E) 2 2

p-3

3-p

)x 1 19. Jika grafik y x - (2 menyinggung sumbu x, maka nilai p yang memenuhi adalah (A) 3 (B) 4 (C) 5 (D) 6 (E) 7

215

Xpedia Matematika, Kapita Selekta - Set 01

20. Persamaan garis yang tegak lurus dengan 3x + 4y + 5 = 0 dan membagi lingkaran x2 + y2 + 4x + 6y - 11 = 0 atas dua bagian yang sama adalah (A) 4x - 3y + 2 = 0 (B) 4x - 3y - 2 = 0 (C) 4x + 3y - 2 = 0 (D) 4x - 3y - 1 = 0 (E) 3x + 4y - 1 = 0

216

Xpedia Matematika Kapita Selekta - USM Mat - Set 02

02. Jika ax3 + bx2 + cx + 13 = 0 dengan x = -1 maka untuk x = 1 nilai dari ax3 - bx2 + cx - 5 = .... (A) 0 (B) 1 (C) 5 (D) 8 (E) 10 04. Urutan bilangan real dari urutan terkecil sampai bilangan terbesar adalah .... (A) 750 , 2175 , 575 , 7-50 (B) 750 , 575 , 2175 , 7-50 (C) 7-50 , 575 , 750 , 2175 (D) 7-50 , 750 , 575 , 2175 (E) 7-50 , 750 , 2175 , 575 05. Persamaan sin5358 (3x) = 3p - 5 terpenuhi apabila .... (A)

1 2 1 3 5 3 5 3

p 1

p p (C) p (D) (E) 5 p (B)

217

2 1 2 6

Xpedia Matematika, Kapita Selekta - USM Mat - Set 02

08. Grafik y = f(x) seperti pada gambar ujian adalah ....

Grafik y = f ’(x) adalah ....

(A)

(D)

(B)

(E)

(C) 11. Salah satu akar ax3 - bx = 0 adalah -2 maka akar yang lain adalah .... (A) -4 (B) -1 (C) 1 (D) 2 (E) 4 12. a + b = 8p, maka nilai maksimum a . b = .... (A) 4p (B) 8p (C) 16p (D) 8p2 (E) 16p2 14. Segitiga dengan panjang sisi-sisinya 2 cm, 7 cm, 9 cm akan terbentuk .... (A) Segitiga lancip dengan sudut paling kecil di depan sisi 2 (B) Segitiga lancip dengan sudut paling kecil di depan sisi 9 (C) Segitiga tumpul (D) Segitiga siku-siku (E) tidak bisa dibuat segitiga 218

Xpedia Matematika, Kapita Selekta - USM Mat - Set 02

15. Grafik y 4 x 3 x 2 4 x 4 untuk x < 0 akan berhimpit dengan .... (A) (B) (C) (D) (E)

y = 3x + 5 y = 3x - 5 y = 5x - 1 y = 5x + 1 y = 5x + 5

16. Perbandingan luas yang diarsir dan yang tidak diarsir bila panjang AB = BC = CD adalah ....

(A) (B) (C) (D) (E) 17.

2:1 3:1 5:2 7:3 7:4

x 2 12x 36 6 - x dipenuhi oleh x = .... (A) 0 x 6 (B) - 6 x 0 (C) x 6 (D) x 6 (E) semua bilangan real

1 2x - 1 19. 2 - x - 2

1 2

(A) x < 2 atau 2 < x < 4 (B) 21 < x < 2 atau x > 2 (C) 21 < x < 2 atau 2 < x < 4 (D) x > 21 (E) x < 4

219

Xpedia Matematika, Kapita Selekta - USM Mat - Set 02

20. Diketahui parabola y = ax2 + bx - 12 seperti pada gambar berikut. y 4

x

-1 5

Yang menunjukkan grafik y = a(x - 1)2 + b(x - 1) - 5 adalah ....

(A)

(B)

(C)

(D)

(E)

220

Xpedia Matematika Kapita Selekta - Set 02

01. Agar grafik y 3 16

x2 tidak di atas

sumbu x, maka .... (A) - 4 x 4 (B) 0 x 4 (C) - 4 x 0 (D) x 4 (E) x - 4 03. px2 - 3px + 5(p - 3) = 0 akar-akarnya a dan b. 4 Jika a 4 b 641 maka p2 - p = .... (A) 4 (B) 3 (C) 2 (D) 1 (E) 0 04. Jika akar-akar persamaan kuadrat x2 - 2x - 4 = 0 adalah dan 2 maka 2 4 3 4 (A) (B) (C) (D) (E)

dan

4 -4 8 -8 16

05. Salah satu nilai t yang menyebabkan pecahan

x 2 - 8x 2t bisa disederhanakan menjadi x 2 - 5x t x x (A) (B) (C) (D) (E)

221

adalah .... 4 5 6 7 8

Xpedia Matematika, Kapita Selekta - Set 02

x

07. Persamaan a 4 -akar real jika .... (A) b 2 - 4ac (B) b 2 - 4ac (C) b 2 - 4ac (D) b 2 - 4ac (E) b 2 - 4ac

b 2

x

c 0 memiliki akar

0, ac 0, ab 0 0, ac 0, ab 0 0, ac 0, ab 0

0, ac 0, ab 0 0, ac 0, ab 0

08. Grafik y = |2log x| dengan grafik y = 2log |x| akan berimpit untuk harga .... (A) setiap x > 0 (B) x -1 (C) x 1 (D) 0 x 1 (E) x 1 09. Jika x1 dan x2 akar-akar persamaan:

3

2

log x 2

(A) (B) (C) (D) (E)

- 4.3

2

log 2x

27 0 maka x1 + x2 =....

4 5 6 7 8

10. Apabila

2 x 5 y 10 z dengan x 0 dan,

1 1 1 2 maka 4 X = .... x y z (A)

1 2

(B) 2 (C) 1 25 (D) 2 (E) 5 2

222

Xpedia Matematika, Kapita Selekta - Set 02

13. Kecepatan dan laju pertumbuhan penduduk suatu kota untuk t tahun yang akan datang dinyatakan sebagai:

N t

400t 600

t ,0 t 9

Jika banyak penduduk saat ini adalah 5.000 jiwa, maka banyak penduduk 9 tahun yang akan datang adalah .... (A) 37.000 Jiwa (B) 35.000 Jiwa (C) 33.000 Jiwa (D) 32.000 Jiwa (E) 30.000 Jiwa 14. Proyeksi titik (2, 3) pada garis y = x adalah .... (A) 5 5 , 2 2 (B) 7 , 7 3 3 (C)

9 9 , 4 4

(D) 11 , 11 5 5 (E)

3 3 , 2 2

16. Dari 8 pasangan suami isteri akan dibentuk tim beranggotakan 5 orang terdiri dari 3 pria dan 2 wanita dengan ketentuan tak boleh ada pasangan suami isteri. Banyaknya tim yang dapat dibentuk adalah .... (A) 56 (B) 112 (C) 336 (D) 560 (E) 672 17. Penyelesaian pertidaksamaan 2

10.2x -3x -8 (A) - 2 x (B) 2 x (C) - 3 x (D) - 3 x (E) - 3 x

223

8.5x 5 6 6 5 2

2

-3x -9

adalah ....

Xpedia Matematika, Kapita Selekta - Set 02

18. Persamaan garis singgung kurva y = x3 - 3x2 + 5x dengan gradien terkecil adalah .... (A) y = x + 1 (B) y = x + 2 (C) y = 2x + 1 (D) y = 2x + 3 (E) y = 3x - 1 19. x1 dan x2 akar-akar persamaan kuadrat 2x2 - 4x + 3k - 15 = 0. Jika x1 = 5 + 2x2 maka k = .... (A) 1 (B) 2 (C) 3 (D) 4 (E) 5

4x 2 - 7x - 2

20. xlim (A) (B) (C)

2

x - 9x - x - 1

....

4 9 3 2 5 8

(D) -1 (E)

224

Xpedia Matematika Kapita Selekta – Set 08

01. log b

log ab 2 log a 2 b 3

02. Jika tan

tan

cos cos sin( )

1 cos(x 2)

2

log p

p dan p 0 maka

?

03. xlim2 ( x 2 x 2) 2

04.

log a 9 b10 ?

?

2 ( 2 log 3 r )

4

log q ,

3 Maka p = ? ( dalam q dan r )

225

Xpedia Matematika, Kapita Selekta – Set 08,

f(x) y = 3

08.

garis y = 3

Jika garis y = 3 menyinggung f ( x) x3 5x2 ax b di titik A. Memotong di titik (1, 3), maka a - b = ?

09. Jika 2

log x

Maka

1 log y

1 log( xy)

1 .dan

1 log x

2 log y

8.

?

10. Jika ( x k ) adalah faktor dari x 2 3 x k . Maka k = ?

11. Jika persamaan kuadrat x 2 3x 1 0 akar akar x1 dan x 2 .dengan x1 > x 2 . Maka

x1 =? x2

226

Xpedia Matematika, Kapita Selekta – Set 08,

12. Garis g menyinggung kurva y sin x cos x 1

di x . Gradien garis yang tegak lurus 9 3 adalah ...

13. Jika x 2

10 x 2 ,maka

14. Jika 9 (2x 7)2

15.

227

x

lim

~

1

1 2

x ?

25 maka nilai x …

1 4

....

1 2x

?

Xpedia Matematika Kapita Selekta – Set 08 halaman 4

16. Jika

f ' ( x) 2x2 x 1,maka nilai maksimum

f (x) = ?

1 2

(A) (B) 1

1

(C) f ( 2 ) (D) f (1) (E) f ( 1)

17. Jika f ( x)

18. 1

5 2

1 cos x , Maka f ' ( x) sin x

4 5

5 4

16 25

....

?

?

19. Jika persamaan kuadrat

x2

2x

2

log ( 2 log n)

akar riil, maka nilai n ….

0 mempunyai

228

Xpedia Matematika Kapita Selekta – Set 08

20. Jika 3 log x Maka x

229

y

3

log y 2 . dan ?

2

log( x y) 4 ,.

Xpedia Matematika Kapita Selekta – Set 10

01. Persamaan kuadrat 3x2 - 6x + y - 1 = 0, dengan α dan β sebagai akar – akar persamaan kuadrat tersebut. Diketahui ( 2 2 )( 2 2 ) 1 , maka p2 - 2p = ?

02. Diketahui suatu deret aritmatika:

1 2

1 2

3

1 4

4

1 8

5

1 16

....

maka jumlah semua deret tersebut?

w w w w w

03. Jika cosec x p 3 sec 2 x , maka sin x = ….

04. Diketahui segitiga ABC dengan sisi a, b dan c. Jika (b + c) : (a + c) : (a + b) = 4 : 5 : 6, maka sin A : sin B : sin C = ....

230

Xpedia Matematika, Kapita Selekta – Set 10

07. Jika x1 dan x2 adalah akar - akar dari 5 x 49 x 4 5 log(2 6) 7 log16 2

0

Maka nilai x1 + x2 = ?

08. Penyelesaian dari x | x 4 |

4 adalah ….

10. Persamaan kuadrat 3x2 + bx + c = 0 memiliki akar-akar 2

1 2

dan

1

. Jika

7 maka b + c =

11. log a dan log b adalah akar-akar persamaan 2x2 - (2k - 4)x - 3 = 0. Jika a.b = 1000, maka k=?

231

Xpedia Matematika, Kapita Selekta – Set 10

12. Penyelesaian pertidaksamaan |x2 - 5x| < 0 adalah ….

13. Jika f(x) = 2x + 4, maka f(x + 3) - f(x - 1) = ....

14. Jika lim x

sin(ax a) 1

15. Jika tan x =

x2 1

2, a = ?

5 dan x beada di dikuadran

pertama, Maka

5

log cos x

5

log sin x = ...

232

Xpedia Matematika, Kapita Selekta – Set 10

16. Pada segitiga ABC diketahui bahwa perbandingan sisi-sisi a : b : c = 2 : 3 : 4, maka sin (A + B) = ?

18. Suatu deret dengan S4 = u1 + u2 + u3 + u4 merupakan deret aritmatika dan u1 > u2 Jika determinan matriks S4 = 2, maka

u1

u2

u3

u4

u1

u2

u3

u4

adalah - 2 dan

1

=?

19. Terdapat teh sukabumi dengan harga Rp 960 dan teh slawi Rp 1200. Untuk membuat teh dengan harga Rp 1000 maka perbandingannya?

20. Garis y = 3x + 5 memotong parabola y = x2 - 4x + 2 di titik A dan B, maka panjang ruas garis AB adalah ….

233

Xpedia Matematika Kode Soal

01. Bentuk sederhana dari:

7  4 3  7  4 3  .... (A) 14 1 (B) 4 14 (C) 4 (D) 7 (E) 14

02. Grafik: y  3x  1  x  2x  1 untuk x < 0, akan berimpit dengan .... (A) y = 2x + 2 (B) y = 2x - 2 (C) y = 4x - 2 (D) y = 4x + 2 (E) y = 4x 2

03. Bentuk: 1  tan 2 x =Sec x dipenuhi oleh .... (A) 0  x  2  (B) 0  x  2  (C) 0  x  2 (D) 0  x   3 (E) 0  x  2 04. Grafik: y = ||x|2 - 4|x|| adalah .... y

y

(A)

(D) 4

x

4

y

x

y

(B)

(D) 4

-4

x

2

x

y

(C) -4

4

x

234

Xpedia Matematika, Kode Soal

05. Pertidaksamaan 2x  a 

x  1 ax  2 3

mempunyai penyelesaian x > 5. Nilai a = .... (A) 2 (B) 3 (C) 4 (D) 5 (E) 6 06. Jika x = 3 merupakan penyelesaian dari: (x - k)(x - 2k) < 2, maka .... (A) k > 7 atau k < 21 (B) k > 3 21 atau k < 1 (C) 72 < k < 1 (D) 21 < k < 7 (E) 1 < k < 3 21 07. Jika y = x2 - 8x + 10, maka nilai y yang memenuhi x2 - 7x + 10 < 0 adalah .... (A) -6  y < -2 (B) -7 < y < -5 (C) -6 < y < -5 (D) -5 < y < -1 (E) 1 < x < 3 21 08. Untuk 3 < x < 9, maka pertidaksamaan: x|x - 3| + |x - 9| - |x - 2|  x + 6 dipenuhi oleh .... (A) x  1 atau x  5 (B) 1  x  5 (C) 5  x < 9 (D) x  5 atau x > 9 (E) 3 < x  5 09. Batas-batas x yang memenuhi:

14  2 x  x  3 adalah .... (A) (B) (C) (D) (E)

235

x<5 -1 < x < 5 x < -1 atau 5 < x  7 3 x<5 -1 < x  7

Xpedia Matematika Kode Soal

01. Jika 2 < x < 3, dan |x - 1| - |x - 2| + |x - 3| = .... (A) -2 (B) x (C) -x (D) 4 - x (E) 3x -6 02. Jika 0 < b < a dan a2 + b2 = 4ab maka

a b .... a b

(A) 2 (B) 3 (C) 2 (D) 3 (E) 4

03. Misalkan a dan b bilangan real yang berbeda.

Jika (A) (B) (C) (D) (E)

a b

a 10 b b 10a

2, maka

b .... a

1 4 1 2 4 5 5 4 3 2

04. f(x) = ax2 + bx - 2, f(2009) = 18, g(x) = ax2 + bx - 5, maka nilai dari

g( 2009 ) .... (A) 3 2 (B) (C) (D) (E)

4 2 5 5 3 3

236

Xpedia Matematika, Kode Soal

05. Jumlah semau bilangan bulat antara -20 dan 21 yang memenuhi pecahan

x 2 3x 10 x2 x 2 (A) (B) (C) (D) (E)

0 adalah ...

26 27 28 29 30

06. Diketahui f(2x - 1) = 3x + 7. Jika f(a) = 40, maka nilai a adalah .... (A) 11 (B) 2 (C) 6 (D) 21 (E) 5 07. Jumlah semua bilangan yang memenuhi persamaan ( x 2 25 ) x 3 log( x 2 ) 0 adalah .... (A) -2 (B) -4 (C) 1 (D) 2 (E) 4 08. Apabila diketahui f(0) = 1, f'(0) = -3, f(x) g(0) = 2, g'(0) = 4 dan h( x ) g ( x ) , maka h'(0) = .... (A) - 43 (B) 104 (C) -2

(D) - 104 (E) - 10 16

09. Nilai x yang memenuhi sistem persamaan |2 log x 3 log y 2 5| |2 log x 3 log y| 0 adalah .... (A) (B) (C) (D) (E)

237

2 2 2 2 4 4 2

Xpedia Matematika, Kode Soal

10. Panjang AB pada gambar adalah .... B

3

A

(A) 3 2

4

C

(B) 4 3 (C) 6 5 (D) (E)

39 5 100 7

238

Xpedia Matematika Kode Soal

01. Kedua akar persamaan: x2 - 2ax + a + 6 = 0 akan lebih besar dari 1 untuk .... (A) 3  a  7 (B) 1  a  3 (C) 1 < a < 7 (D) a  3 (E) 3  a  7 02. px2 - 3px + 5(p - 3) = 0 akar-akarnya adalah  dan . Jika 4 + 4 = 641 maka p2 - p = .... (A) 4 (B) 3 (C) 2 (D) 1 (E) 0 03. Jika akar-akar persamaan kuadrat x2 - 2x - 4 = 0 adalah  dan  maka (2 -  - 4)(2 - 3 - 4) = .... (A) 4 (B) -4 (C) 8 (D) -8 (E) 16 04. Persamaan a.4x + b.2x + c = 0 memiliki akarakar real jika .... 2 (A) b  4ac  0, ac  0, ab  0

2 (B) b  4ac  0, ac  0, ab  0 2 (C) b  4ac  0, ac  0, ab  0

2 (D) b  4ac  0, ac  0, ab  0 2 (E) b  4ac  0, ac  0, ab  0

05. Jika x1 dan x2 akar-akar persamaan

3

2

log x 2

 4.3

2

log 2 x

maka x1 + x2 = .... (A) 4 (B) 5 (C) 6 (D) 7 (E) 8

239

 27  0

Xpedia Matematika, Kode Soal

06. Apabila 2x = 5y = 10z dengan x  0 dan 1 1 1 x x  y  z  2, maka 4 = .... (A) 21 (B) 2 (C) 215 (D) 5 (E) 52 07. Proyeksi titik (2, 3) pada garis y = x adalah ....

( 52 , 52 ) ( 73 , 73 ) ( 49 , 49 ) ( 115 , 115 ) 3 3 (E) ( 2 , 2 ) (A) (B) (C) (D)

08. Dari 8 pasangan suami istri akan dibentuk tim beranggotakan 5 orang terdiri dari 3 pria dan 2 wanita dengan ketentuan tak boleh ada pasangan suami istri. Banyaknya tim yang dapat dibentuk adalah .... (A) 56 (D) 560 (B) 112 (E) 672 (C) 336 09. Penyelesaian pertidaksamaan 2

10.2x 3x8  8.5x (A)  2  x  5 (B) 2  x  6

2

3 x 9

adalah ....

(C)  3  x  6 (D) 3  x  5 (E) 3  x  2

10. Persamaan garis singgung kurva y = x3 - 3x2 + 5x dengan gradien terkecil adalah .... (A) (B) (C) (D) (E)

y=x+1 y=x+2 y = 2x + 1 y = 2x + 3 y = 3x - 1

240

Xpedia Matematika, Kode Soal

11.

lim

x

4 x2  7  x  2 x  9x2  x  1

(A)  (B)  (C)  (D) 1 (E) 

 ....

4 9 3 2 5 8

12. 212 - 1 habis dibagi oleh .... (A) 17 (B) 23 (C) 27 (D) 33 (E) 35 13. Jika x = 0,252525.... dan y = 0,125125125...., maka 33x + 333y = .... (A) 25 (B) 40 (C) 50 (D) 100 (E) 150 14. Parabola y = ax2 - bx + c tidak memotong sumbu-x dan menyinggung parabola y = 2ax2 + x + 5c, maka .... (A)  13  b 

2 3

(B)  23  b  1 (C)  13  b  1 (D) b   13 atau b  23 (E) b   13 atau b  1 15. Sebuah kapur barus berbentuk tabung dengan diameter alas lingkaran sama dengan tinggi tabung. Kapur barus tersebut menyublim sehingga bentuknya selalu berbentuk tabung yang diameter alasnya sama dengan tinggi tabung. Laju perubahan volume kapur tersebut terhadap tingginya pada saat tingginya 2 satuan adalah .... (A) 2 (B) 3 (C) 4

241

(D) 6 (E) 9

Xpedia Matematika, Kode Soal

16. Jika sin a  51 , maka nilai dari 16

1 cos2 a

(A) (B) (C) (D) (E)

 1sin1 2 a  1sin2 4 a  1sin4 8 a  ....

4 6 8 10 12

17. Diketahui 1 < x < 13 dan -11 < y < 5 maka pernyataan di bawah ini yang benar adalah .... (1) -10 < x + y < 18 (2) -4 < x - y < 8 (3) -11 < x . y < 65 10002 18. 1652  852  ....

(A) (B) (C) (D) (E)

5 50 10 1000 0,1

19. Jika jari-jari bola bertambah 25%, maka luas kulitnya bertambah .... (A) 12,5% (B) 25% (C) 47,5% (D) 56,25% (E) 75% 20. Lingkaran x2 + y2 - 4x + 6y - 45 = 0 memotong sumbu -x di titik A dan B, bila P adalah pusat lingkaran dan sudut APB = , maka pernyataan di bawah ini yang benar adalah .... 21 (1) tan   20 (2) AB = 14 (3) luas segitiga APB = 21

242

Xpedia Matematika DP SNMPTN Matematika 04

01. Bentuk sederhana dari : 7

4 3

7

4 3

....

(A) 14 (B) 14 14 (C) 4 (D) 7 (E) 14

02. Grafik : y = 3x + 1 + x 2 2 x 1 untuk x < 0 akan berimpit dengan …. (A) y = 2x + 2 (B) y = 2x - 2 (C) y = 4x - 2 (D) y = 4x + 2 (E) y = 4x

03. Bentuk : 1 tan 2 x (A) 0 ≤ x ≤ 2π (B) 0 < x <

π 2

(C) 0 ≤ x ≤

π 2

sec x dipenuhi oleh …

(D) 0 < x < π 3π

(E) π ≤ x ≤ 2

04. Jika y = x2 - 8x + 10 maka nilai y yang memenuhi x2 - 7x + 10 < 0 adalah …. (A) -6 ≤ y < -2 (B) -7 < y < -5 (C) -6 < y < -5 (D) -5 < y < -1 (E) 1 < x < 3

243

1 2

Xpedia Matematika

, DP SNMPTN Matematika 04

05. Pertidaksamaan 9 3|2x 1|-3 81 dipenuhi oleh …. (A) -4 < x < 3 (B) -2 < x < 2 (C) -4 < x < -3 atau 2 < x < 3 (D) -3 < x < -2 atau 2 < x < 4 (E) x < -4 atau –2 < x < 2 atau x > 3

06. Untuk 3 < x < 9, maka pertidaksamaan : x|x - 3|+|x - 9|-|x - 2|≥ x + 6 dipenuhi oleh …. (A) x ≤ 1 atau x ≥ 5 (B) 1 ≤ x ≤ 5 (C) 5 ≤ x < 9 (D) x ≤ 5 atau x > 9 (E) 3 < x ≤ 5

07. Para bola y = x2 - 6x + 8 digeser ke kanan sejauh 2 satuan searah dengan sumbu x dan digeser ke bawah sejauh 3 satuan. Jika parabol hasil penggeseran ini memotong sumbu x di x1 dan x2 maka x1 + x2 = …. (A) 8 (B) 9 (C) 10 (D) 11 (E) 12

08. Diketahui empat titik A, B, C, dan D yang berada pada lingkaran dengan panjang AB = 4cm, BC = 3cm, CD = 3cm, dan AD = 6cm. Kosinus sudut BAD adalah …. (A) 14

33

(B)

16 33

(C) 17 33 19

(D) 33 20

(E) 33 244

Xpedia Matematika DP SNMPTN Mat 05

01. Jarak dari kota A dan kota B 5 mil dan jarak dari kota B dan kota C 4 mil. Jarak kota A dan kota C TIDAK mungkin …. (A) 1 (B) 4 (C) 8 (D) 9 (E) 10 02. Himpunan I beranggotakan 6 bilangan bulat berurutan. Himpunan J beranggotakan semua bilangan bulat yang merupakan hasil pengurangan masing-masing anggota I dengan 3 dan hasil penjumlahan masing-masing anggota I dengan 3. Berapa perbedaan jumlah anggota J dan I ? (A) 0 (B) 2 (C) 3 (D) 6 (E) 9

03. Jika x < y apa yang pasti benar ? (A) x2 < y2 (B) -y < -x (C) x2 < xy (D) xy < y2 (E) 2x < y

04. Jika x bilangan bulat dan

x 7 adalah 2

bilangan bulat, mana yang pasti benar ? I. x ganjil II.x adalah kelipatan 7 III.

x 5 2

(A) (B) (C) (D) (E)

I II III I dan II I dan III 246

Xpedia Matematika, DP SNMPTN Mat 05

05. Pada sebuah klub, median dari umur anggota anggotanya adalah 11. Apa yang pasti benar ? I. Anggota tertua paling sedikit 1 tahun lebih tua dari anggota termuda II. Jika ada yang berumur 10 tahun, maka ada yang berumur 12 tahun III. Mpdus dari umur anggota adalah 11 (A) (B) (C) (D) (E)

Tidak ada I II III II dan III

06. Hasil kali dua bilangan bulat terletak antara 102 dan 115. Bilangan bulat yang tidak mungkin adalah …. (A) 5 (B) 10 (C) 12 (D) 15 (E) 20

07. Jika rata-rata (rataan aritmatik) dari 5 bilangan bulat genap aritmatik berurutan adalah n. Berapa median dari bilanganbilangan tersebut? (A) 0 (B) 2 (C) n (D) n - 2 (E) n - 4

247

Xpedia Matematika, DP SNMPTN Mat 05

08.

Pada gambar di atas lingkaran-lingkaran itu menyinggung satu sama lain dan menyinggung persegi panjang pada titik berhuruf. Jari-jari tiap lingkaran 1. Mana yang paling mendekati luas daerah yang diarsir …. (A) (B) (C) (D) (E)

6 4 3 2 1

09.

Pada gambar di atas, jari-jari lingkaran berpusat S adalah dua kali jari-jari lingkaran berpusat O dan < RST dua kali < POQ. Jika luas daerah arsir lingkaran O adalah 3 berapa luas daerah arsir lingkaran S? (A) 24 (B) 12 (C) 6 (D) 3 (E)

3

2

10. 1,2,1,-1,-2 …. 5 suku pertama dari suatu barisan ditunjukkan di atas setelah suku kedua, setiap suku diperoleh Dengan mengurangi suku sebelumnya dengan Suku sebelumnya lagi . Sebagai contoh, suku ketiga diperoleh dengan mengurangi suku kedua dengan suku pertama. Berapa jumlah 36 suku pertama barisan di atas ? (A) 0 (B) 4 (C) 12 (D) 24 (E) 30

248

Xpedia Matematika, DP SNMPTN Mat 05

11. Jika p adalah bilangan prima lebih dari 3. Mana yang bukan faktor dari 6p ? (A) P2 (B) 6p (C) 3p (D) 2p (E) 3

12.

Jika garis l, m, n paralel . Berapa nilai dari x + y? (A) 210 (B) 220 (C) 230 (D) 240 (E) 250 13. 1 0.0 N x 1 y

0.00P

1 Pecahan 1 dan dapat ditulis dalam y x Bilangan decimal seperti di atas, dimana N dan P mempersentasikan digit berbeda. Mana yang 1 jika S sama dengan N kali P xy dan S adalah sebuah digit ? (A) 0.0000 S (B) 0,000 S (C) 0,00 S (D) 0,0 S (E) 0, S sama dengan

249

Xpedia Matematika, DP SNMPTN Mat 05

14.

Pada gambar di atas A dan B adalah pusat dari lingkaran. Jika luas tiap lingkaran 10, berapa luas persegi panjang ? (A) 20 (B) 20 (C) (D) (E)

10 π

40 50

60

15. Titik-titik pada sebuah grafik fungsi linear kontinu f(x) dinyatakan dalam (a.b) sehingga –2 ≤ a ≤ 6 dan –1 ≤ b ≤ 4. Jika grafik f(x) dimodifikasi sehingga –4 ≤ a ≤ 6 tanpa mengubah skala x dan y, maka grafik baru akan menunjukkan …. (A) Sebuah garis dengan kemiringan lebih besar dan terlihat lebih curam (B) Sebuah garis dengan kemiringan sama dan terlihat lebih curam (C) Sebuah garis dengan kemiringan lebih kecil dan terlihat kurang curam (D) Sebuah garis dengan kemiringan sama tetapi terlihat kurang curam (E) Sebuah garis dengan kedalaman dan kemiringan yang sama 16. Jika f(x) = 2x2 + tx - 2. Berapa nilai t sehingga titik (-5,a) dan (5,a) terletak pada grafik f(x) ? (A) -5 (B) -1 (C) 0 (D) 1 (E) 5

250

Xpedia Matematika, DP SNMPTN Mat 05

17. Sebuah bukti tidak langsung (indirect proof) dari pernyataan “jika x adalah bilangan riil antara 0 dan 1, maka x2 kurang dari x” dapat dimulai dengan asumsi bahwa …. (A) x = -1 (B) x = 1 (C) x2 lebih dari x (D) x2 paling sedikit x (E) x bukan bilangan riil antara 0 dan 1

18. Apa daerah hasil dari f(x) = 3 sin kx - 4 cos. Kx + 2 dimana k adalah bilangan bulat lebih dari 50 ? (A) 3 ≤ y < 7 (B) -7 ≤ y ≤ 3 (C) -3 ≤ y ≤ 7 (D) -7 ≤ y ≤ -3 (E) Tidak dapat ditentukan dari informasi yang ada

251

Xpedia TPA Basic Mathematical Skills - Algebra and Arithmetics - Soal Aritmatika

(01) 1234 + 271 =

(25) 1024 / 16 =

(02) 372 - 241 = (03) 512 - 714 = (04) 25 x 72 = (05) 26 x 73 = (06) 125 x 6 = (07) 24 x 16 = (08) 27 x 30 = (09) 51 x 18 = (10) 23 x 400

=

(11) 72 x 15

=

(12) 4500 + 720 = (13) 217 x 30

=

(14) 1,23 x 3

=

(15) 0,27 x 4

=

(16) 0,12 x 21

=

(17) 0,03 x 5,4 = (18) 2700 x 320 = (19) 510 x 0,05 = (20) 27,3 x 0,02 = (21) 4800 x 2.300.000 = (22) 300 / 25 = (23) 420 / 3 = (24) 720 / 5 = 252

Xpedia TPA Basic Mathematical Skills - Algebra and Arithmetics - Soal

01. (3a + b)2 = .... 02. 4a2 - 9b2 = .... 03. x2 - 2x - 48 = .... 04. x2 + 32x + 240 = .... 05. 2x2 + 11x + 12 = .... 06. 2x2 - x -10 = .... 07. 4x2 + 10x + 6 = .... 08. 4x2 - 7x - 2 = .... 09. x2 - 3x - 10 = .... 10. (2x + 7y)2 = .... 11. (3a + 2b + c)2 = .... 12.

5  2 6 = ....

13.

7  2 12 = ....

14.

9  2 1 2 = ....

15. (2x + 5y)2 = .... 16. (2x - 3y)3 = ....

p

2

17.

253





p q q q q  1 p  1 = ....

Xpedia TPA Basic Mathematical Skills - Algebra and Arithmetics - Soal Aljabar 2

01. Untuk semua bilangan bulat x dan y. 4x(x) - 3xy(2x) = (A) 12x2 y(x - 2x) (B) 2x2 (2 - 3y) (C) 4x2 (x - 3y) (D) xy (- x) (E) 2x2 (2 + 3y)

02. Jika 3x2 + 2x = 40. maka 15x2 + 10x = (A) 120 (B) 200 (C) 280 (D) 570 (E) 578 03. Ekspresi -2(x+2) + x(x+2) ekivalen dengan .... (I) x2 - 4 (II) (x - 2) (x + 2) (III) x2 - 4 x - 4 04. Mana yang paling benar untuk semua nilai x,y dan z? (I) (x + y) + z = (z + y) + x (II) (x - y) + z = (z - y) - x (III) (x × y) × z = (z × y) × x 05. Simbol  melambangkan satu dari operator fundamental aritmatika : +.–.×.+. Jika (x  y) × (y  x) = 1 untuk semua nilai posistif dari x dan y, maka  melambangkan .... (A) hanya + (B) hanya × (C) hanya + atau × (D) hanya – (E) hanya 

07. Jika 2x2 -5x = 9, maka 12x2 - 30x = .... (A) -54 (B) -6 (C) 18 (D) 36 (E) 54 08. Persamaan di bawah ini benar untuk nilai p? (p + 2)2 = (p - 5)2 (A) -2 dan 5 (B) 2 dan -5 (C) 0 dan 1 (D) hanya 1,5 (E) hanya 3,5

3x

09. Untuk semua nilai positif m dan n, jika m - nx = 2. maka x =? (A)

2 m − 2n 3

(B) 3 + 2n 2m (C)

3 2 m - 2n

(D) 2 m - 3 2n (E)

2m 3 + 2n

06. Jika 5d + 12 = 24 maka 5d - 12 = (A) -24 (B) -12 (C) 0 (D) 12 (E) 24 254

Xpedia TPA, Basic Mathematical Skills - Algebra and Arithmetics - Soal Aljabar 2

11. Jika 4m- 7n = 10 dan 2m +2n = 4,berapa nilai 2m -9n? 12. Jika 9p = 3a +1 dan 7p = 2a - 3. maka p dinyatakan dalam a adalah .... (A) 3a + 1 7 (B) 2a − 3 9 (C) 2a 63 (D) 7a 9 (E) a + 4 2 13. Jika m6 =

2 3 dan m5 = y , maka m y 6

dinyatakan dalam y ? 18 (A) y 3

(B)

y 2

18 − y 3 (C) 6y

(D)

y3 18

2 (E) y

14. Jika 7x + 2y - 6z = 12, dan jika x,y, dan z 2+z positif maka berapa nilai dari 7x + 2y (A) 1/12 (B) (C) (D) (E)

255

1/4 7/12 1/6 5/12

15. Jika (A) (B) (C) (D) (E)

2 3 4b 1 a = dan = , maka = 2b 5 3c 7 c 2,35 9,70 8,35 17,70 9,35

16. (x+a) (x+1) = (x2 + 6x + a), maka a =.... 17. Jika a2 + b2 = 8 dan ab = -2 maka (a+b)2 = .... (A) 4 (B) 6 (C) 8 (D) 9 (E)16 18. Jika f2 - g2 = -10 dan f + g = 2, maka berapa nilai dari f - g? (A) -20 (B) -12 (C) -8 (D) -5 (E) 0 19. Jika x > 0, maka ( x + 2 )2 − 1 (x + 1)2 − 1 x2 − 1 + + ( x + 3 ) = .... ( x + 2) ( x + 1)

(A) (B) (C) (D) (E)

(x+1)2 (x-1)2 3x-1 3x 3(x+1)2 y 2 − 36

20. Jika y =3p dan p ≠ 2, maka = .... ( y − 6)2 (A) 1 p+2 (B) p − 2 3p + 2 (C) 3p − 2

(D)

3p + 2 3p 9p 2 + 36

(E) 9p 2 - 36

Xpedia TPA, Basic Mathematical Skills - Algebra and Arithmetics - Soal Aljabar 2

21. Jika n (A) (B) (C) (D) (E)

1 1 = x, maka n2 + 2 = n n

x2 - 2 x2 - 1 x2 x2 + 1 x2 + 2

dalam a dan x adalah?

22. (a + b + 2) (a + b + 2) = (A) (a + b)2 + 4 (B) (a + b)2 + 4(a + b) (C) (a + b)2 + 4(a + b) +4 (D) a2 + b2 + 4 (E) a2 + b2 + 4ab 23. Jika 12x2 = 7. maka 7(12x2)2 = .... (A) 49 (B) 84 (C) 98 (D) 144 (E) 343

24. Jika n.p. dan t tidak nol dan n4 p7 t9 (A) (B) (C) (D) (E)

=

4 n 3 p7 t -9

. maka n = ....

1/4 1/2 4 4p2t2 4p18t18

25. Jika a=

y + a2 26. Jika y ≠ 4a dan x = y − 4 a , y dinyatakan

b+x , maka x dinyatakan dalam a,b, c +x

(A)

4a − 4a 2 x x +1

(B)

a 2 − 4 ax x +1

(C)

a 2 + 4 ax x +1

(D)

a 2 + 4 ax x −1

(E)

a 2 − 4 ax x −1

3 27. Jika 2a + 3 = 6 maka = .... 4 a + 6 (A) 1/4 (B) 1/2 (C) 1 (D) 2 (E) 3

28. Jika a = 4b . c =8b2 . dan b ≠ 0. c −a maka = .... 4b (A) (B) (C) (D) (E)

-2b 1-2b 2b 2b - 1 2b + 3

dan c adalah .... a − bc (A) a −1 (B)

b − ac a −1

(C)

a + bc a +1

(D)

ac + b a +1

(E)

ac − b a

256

Xpedia TPA, Basic Mathematical Skills - Algebra and Arithmetics - Soal Aljabar 2

29. Jika (A) (B) (C) (D) (E)

2 1 (x2 + x) = . maka x + 1 = .... x 2

1/4 1/2 1 3/2 2

30. Jika akar - akar persamaan x2 + 2x - 5 = 0 1 1 adalah a dan b, maka 2 + 2 = .... a b (A) −

6 25

(B)

1 24

(C)

6 25

(D)

14 25

(E)

24 25

257

XPedia TPA Basic Mathematical Skills-Mathematical Modelling and Ratio –

01. Tuliskan tiga persamaan yang ekivalen 2 y dengan  ! x 3

02. Tiga orang membagi uang Rp24.000 dengan rasio 2:3:5. Tuliskan besar yang didapat tiap orang!

03. Ada 12 laki-laki dan g perempuan di kelas A dan ada 27 perempuan dan b laki-laki di kelas B. Dalam tiap kelas, rasio jumlah lakilaki terhadap perempuan sama. Jika b=g, berapa banyak siswa di kelas A? 04. Sebuah mesin yang bekerja dengan laju konstan. menghasilkan 25 botol tiap 6 menit. Berapa jam yang dihabiskan untuk menghabiskan 1.000 botol? 05. Jika m meteor memasuki atmosfer bumi setiap x hari (m > 0). maka berapa banyak meteor yang memasuki atmosfer bumi dalam m.x hari? 06. Jika x adalah hasil kali dari 0,3 dan 0,2, maka x ekivalen dengan rasio dari 6 dengan? 07. Tabung A berisi 6 bola merah. Tabung B berisi dua bola merah dan empat bola hijau yang harus dipindahkan dari tabung B ke tabung A sehingga perbandingan jumlah bola hijau terhadap bola merah sama untuk kedua tabung? 08.

90 orang siswa sedang berkumpul. Perbandingan jumlah wanita terhadap pria dalam kumpulan itu adalah 2 banding 3. Berapa wanita yang ada?

258

Xpedia TPA, Basic Mathematical Skills - Mathematical Modelling and Ratio – Soal

09. Jika =

x 3  ,maka 5x + 1 = .... y 1 5

(A) 3y + 1 (B) 3y + 2 (C) 3y + 3 (D) 3y + 4 (E) 3y + 5 10. Sebuah peta digambar dengan skala berikut dua kota yang berjarak x kilometer digambar dalam peta berjarak 4cm. Jika dua kota berjarak x + 2. berapa jarak mereka dalam peta? (A) 4(x + 2) (B) 6 (C)

4x x2

(D)

4(x  2) x

(E)

6 x

11. Jika 3.600 kaos bola didistribusikan ke 4 toko dengan rasio 1 : 2 : 3 : 4. berapa jumlah maksimum kaos bola yang dida pat oleh toko?

2 12. Sepeda motor Oktar menggunakan galon 5 bensin untuk menempuh 8 kilometer. Berapa jarak yang bisa ditempuh sepeda motor Oktar dengan 5 galon bensin?

15. 5 adalah berapa persen dari 26? 16. 35% dari 28 adalah ? 17. 60 adalah 15% dari?

259

Xpedia TPA, Basic Mathematical Skills - Mathematical Modelling and Ratio – Soal -

18. Berapa persenkah 35 dari 20? 19. Berapa persen lebih besarkah 1.200 dari 1.500? 20. Jika harga sebuah sweater diturunkan dari Rp80.000 menjadi Rp68.000, berapa persenkah penurunan harganya? 21. Jika populasi sebuah kota meningkat dari 3,2 juta menjadi 4,48 juta. Berapa persen peningkatannya? 22. Bilangan apa yang 30% lebih besar dari 20? 23. Meningkatkan sebuah bilangan dengan 20% nya lalu mengurangi bilangan yang terbentuk dengan 20%-nya sama saja dengan mengalikan bilangan mula-mula dengan? 24. Jika sisi sebuah segi empat berkurang 5%. berapa persenkah pengurangan luas segiempat itu? 25. 28% dari 50 sama saja dengan 50% dari ....

29. Jika x adalah

2 % dari 90, maka 1-x = .... 3

30. Harga sekotak baterai setelah pajak 5% adalah Rp8.400, Berapa harga baterai sebelum dikenai pajak? 32. Jika panjang sebuah persegi panjang diperbesar 20% dan lebarnya diperbesar 30%. maka berapa persenkah peningkatan luas persegi panjang itu?

260

Xpedia TPA, Basic Mathematical Skills - Mathematical Modelling and Ratio – Soal

33. Jika 12 ons larutan 30% garam dicampur dengan 24 ons larutan 60% garam. maka berapa persen konsentrasi garam di larutan itu? 35. a · b = konstan A. Jika a menjadi dua kali mula-mula, maka b menjadi ..... kali mula-mula B. Jika a menjadi setengah kali mula-mula maka b menjadi .... kali mula-mula C. Jika b menjadi 3 kali mula-mula maka a menjadi .... kali mula-mula D. Jika b menjadi 1/3 kali mula-mula maka a menjadi .... kali mula-mula 36.

a = konstan b

A. B. C. D.

37.

a2 = konstan b A. B. C. D.

261

Jika a menjadi empat kali mula-mula maka b menjadi .... kali mula-mula Jika a menjadi seperempat kali mulamula mereka b menjadi .... kali mulamula Jika b menjadi 8 kali mula-mula maka a menjadi .... kali mula-mula Jika b menjadi 1/6 kali mula-mula maka a mejadi .... kali mula-mula

Jika a menjadi dua kali mula mula maka b menjadi .... kali mula-mula Jika a menjadi setengah kali mula-mula maka b menjadi .... kali mula-mula Jika b menjadi 3 kali mula-mula maka a menjadi .... kali mula-mula Jika b menjadi 1/3 kali mula-mula maka a menjadi .... kali mula-mula

Xpedia TPA, Basic Mathematical Skills - Mathematical Modelling and Ratio – Soal

40. a · b = konstan A. Jika a menjadi dua kali mula-mula, maka b menjadi .... kali mula-mula B. Jika a menjadi setengah kali mula-mula maka b menjadi .... kali mula-mula C. Jika b menjadi 9 kali mula-mula maka a menjadi .... kali mula-mula D. Jika b menjadi 1/9 kali mula-mula, maka a menjadi .... kali mula-mula ab = c

44.

A. B. C. D. E. F. G. H.

konstan

Jika a menjadi 2 kali mula-mula, b menjadi 1/2 kali mula-mula, maka c menjadi .... kali mula-mula. Jika a menjadi 1/4 kali mula-mula, b menjadi 8 kali mula-mula maka c menjadi .... kali mula-mula Jika a menjadi 3 kali mula-mula, b menjadi 1/9 kali mula-mula maka c menjadi .... kali mula-mula Jika a menjadi 1/3 kali mula-mula, b menjadi 9 kali mula-mula, maka c menjadi …. Kali mula-mula Jika b menjadi 4 kali mula-mula dan c menjadi 1/8 kali mula-mula maka a menjadi .... kali mula-mula Jika b menjadi 1/2 kali mula-mula dan c menjadi 2 kali mula-mula maka a menjadi .... kali mula-mula Jika a menjadi 5 kali mula-mula dan c menjadi 1/5 kali mula-mula maka b menjadi .... kali mula-mula Jika a menjadi 3 kali mula-mula dan c menjadi 9 kali mula-mula maka b menjadi .... kali mula-mula

262

Xpedia TPA, Basic Mathematical Skills - Mathematical Modelling and Ratio – Soal

50. Kecepatan reaksi dari suatu reakasi gas dinya -takan sebagai v=k[A] [B]. Bila volume yang ditempati gas-gas tersebut tiba-tiba diperkecil 1/4 kali dari volume semula, maka kecepatan reaksinya dibandingkan dengan kecepatan reaksi semula akan menjadi .... (A) 1/8 (B) 1/16 (C) 4/1 (D) 8/3 (E) 16/1 51. Secara teoritis banyaknya cuplikan dengan kadar belerang 80% yang dapat menghasilkan 8 gram SO3 adalah (Ar O = 16 S = 32) [UMPTN 4995 Kimia Rayon A] (A) 3 g (B) 4 g (C) 5 g (D) 6 g (E) 8 g

263

Xpedia TPA Basic Mathematical Skills - Graphics - Soal

01. Buat sketsa grafik! (A) x = 1 x = -1 x = -1 x = -7 x=8 (B) y = 2 y = -7 y = -1 y=8

(C) y = x

(D) y = -x

(E) y = 2x y = 4x y = 16x

(F) y =

1 1 1 x, y = x, y = x 2 4 16

264

Xpedia TPA, Basic Mathematical Skills - Graphics - Soal

Buat sketsa dari persamaan berikut! (G) -2y = x -5y = x -10y = x

(H)

1 y = x, 3

02.(A) y = x+1 y=x+2 y=x+ 3

(B) y + 1 = x y+2=x y+3=x

(C) y=x-1 y=x-2 y=x-3

(D) y -1 = x y -2 = x y -3 = x

265

1 y = x, 6

1 y=x 9

Xpedia TPA, Basic Mathematical Skills - Graphics - Soal

(E) y + 1 = x + 3

(F) y -1 = x -3

(H) x + y = 3

(I) 2x + y = 6

(J) 2y + x = 8

(A) y = 2x + 1 y = 2x + 3 y = 2x + 5

266

Xpedia TPA, Basic Mathematical Skills - Graphics - Soal

(B) y = 2 (x + 1) y = 2(x + 3) y = 2(x + 5)

(C) y = 2x -1 y = 2x -3 y = 2x -5

(F) y -1 = 2 (x + 1)

(G) y - 1 = 2 (x - 1)

(H) y + 1 = 2(x - 1)

04. (A) y=x2

(B) y=-x2

267

Xpedia TPA, Basic Mathematical Skills - Graphics - Soal

(C) x = y2

(D) x = -y2

(E) y = x2 + 1 y = x2 + 2 y = x2

(F) y + 1 = x2 y + 2 = x2 y + 3 = x2

(G) y = x2 + 5

(H) y = x2 - 2

(I) y = (x - 3)2

268

Xpedia TPA, Basic Mathematical Skills - Graphics - Soal

(J) y = -x2 -2 y = -x2 + 3

(K) y = -(x - 2)2 y = -(x - 5)2

(L) y = (x - 2) (x - 1)

(M) y = (x + 2) (x - 1)

(N) y = -(x - 2) (x + 1)

05. (A) y = x3

269

Xpedia TPA Basic Thinking and Reasoning Skills - Set and Categories - Soal

01. Buatlah diagram Venn dari Entitas di bawah ini! S : Semua produk A : Merk Nokia B : Merk Sony-Ericsson C : Hand phone 02. Buatlah diagram Venn dari Entitas di bawah ini! Negara - Kota - Jakarta 03. Buatlah diagram Venn dari Entitas di bawah ini! Teh - Kopi - Gula 04. Buatlah diagram Venn dari Entitas di bawah ini! Kucing - Singa - Anjing 05. Buatlah diagram Venn dari Entitas di bawah ini! Paus - Lumba-lumba - Ikan 06. Buatlah diagram Venn dari Entitas di bawah ini! A : Objek Kajian Fisika B : Objek Kajian Matematika C : Angka 07. Buatlah diagram Venn dari Entitas di bawah ini! S : Semua fungsi y = f(x) A : Fungsi y=f(x) yang selalu naik pada x  R B : Fungsi y=f(x) yang selalu turun pada x  R C : Fungsi kuadrat untuk x  R

08. Buatlah diagram Venn dari Entitas di bawah ini! Bilangan Riil - Bilangan Rasional - Bilangan Negatif 09. Buatlah diagram Venn dari Entitas di bawah ini! A : Gaya Listrik B : Gaya Gravitasi C : Tegangan Listrik 10. Buatlah diagram Venn dari Entitas di bawah ini! A : Besaran Vektor B : Besaran Skalar C : Gaya 11. Buatlah diagram Venn dari Entitas di bawah ini! A : Gaya B : Percepatan C : Kecepatan 12. Buatlah diagram Venn dari Entitas di bawah ini! A : Gas Hidrogen B : Atom Hidrogen C : Air 13. Buatlah diagram Venn dari Entitas di bawah ini! A : Tubuh Manusia B : Jantung Manusia C : Organ Tubuh Manusia 14. Buatlah diagram Venn dari Entitas di bawah ini! A : Ikatan ionik B : Unsur logam C : Unsur magnesium 270

Xpedia TPA, Basic Thinking and Reasoning Skills - Set and Categories - Soal

15. Buatlah diagram Venn dari Entitas di bawah ini! S : Semua fungsi A : Semua fungsi y = f(x) y  1 untuk semua xR B : Semua fungsi y = f(x) y > 1 untuk semua xR C : y = f(x) = sin x, untuk semua x  R 16. Buatlah diagram Venn dari Entitas di bawah ini! Sel Tumbuhan - DNA - Inti Sel

17. Buatlah diagram Venn dari Entitas di bawah ini! A : Mobil produksi Jepang B : Mobil merk Toyota C : Mobil sedan 18. Buatlah diagram Venn dari Entitas di bawah ini! S : A: B: C:

Semua titik (x,y) pada diagram cartesius x2+y2 = 9 x2+y2 = 25 y=x

19. Buatlah diagram Venn dari Entitas di bawah ini! S : Semua orang A : Orang-orang atheis B : Orang-orang komunis C : Orang-orang religius 20. Buatlah diagram Venn dari Entitas di bawah ini! A : Aset (aktiva) B : Liabilitas (kewajiban) C : Ekuitas 271

Xpedia TPA Basic Thinking and Reasoning Skills - Inductive Reasoning - Soal

Bagian 1. 01. 1, 10, 19, 28 .... 02. 5, 1, 1 , 1 .... 5 25

06. 2, 6, 4, 12, 10, 30, 28 .... 07. 1, 1, 2, 5, 11, 21 .... 08.

2 -1 3

03. 1, 3, 5, 6, 9, 12 ....

4 3 1

04. 20, 10, 5 .... 05. 5, 12, 26, 54 ....

5

2 (2) 1 3 (9) 2 4 (1) 0 2 ( ...) 3

10.

7 (33) 2 (-21) 3(0) 3 ( ...)

11.

4( ) 3 3 (16) 1 4 (125) 1 6 ( ... ) 6

12.

21 (BAGI) 79 62 ( …) 45

13.

FA (40) BA GI (27) EB BA ( ... ) GI

14.

3 (3) 1 4 (20) 5 7 (14) 2 1 ( ...) 1

07. 3, 8, 15, 24 ....

09. 199, 196, 191, 184 .... 10. 4, 6, 10, 14, 22 .... 11. 4, 9, 25, 49 .... 12. 3, 7, 11, 15, 19 .... 13. 5, 25, 61, 113 .... 14. 4, 9, 14, 21, 30, 41 .... 15. 5, 34, 65, 98 16. 1000, 995, 990, 1000, 970, 1010 .... 17. 404, 208, 116, 82 .... Bagian 2. 01. A, E, I, M .... 02. A, J, S, B ....

11

09.

06. 10, 14, 22, 38 ....

08. 5, 17, 65, 257 ....

1 0 1

4 5 3 2

15. Hitunglah ada berapa banyak angka 1 yang muncul, Jika kamu menulis : a . 1-100 b. 1-1000

03. AA, CZ, EY, GX .... 04. M, Q, O, S, Q, U .... 05. 3, 8, 6, 11, 9, 14, 12 .... 272

Xpedia TPA Basic Thinking and Reasoning Skills - Deductive Reasoning and Logic - Soal

01. Jika risky nakal maka mama marah, jika mama marah maka bumi berguncang. Kesimpulan apa yang bias diambil dari faktafakta berikut. (A) risky nakal (B) mama marah (C) bumi berguncang (D) rizky tidak nakal (E) mama tidak marah (F) Bumi tidak berguncang 02. Jika meta atau cherina nakal maka mama marah. Jika mama marah maka mama makan. Kesimpulannya adalah ! (A) Meta nakal (B) Meta tidak nakal (C) Mama tidak marah (D) Mama tidak makan (E) Meta & cherina tidak nakal 03. Jika ikan mengaum maka gajah mengembik. Jika gajah mengembik maka viktor dan ina mengikik, ternyata Viktor mengikik tapi Ina tidak mengikik, Tentukan apakah kesimpulan di bawah int (b/s/m). (A) Ina mengikik (B) Gajah mengembik (C) Gajah tidak mengembik (D) Ikan mengaum (E) Ikan tidak mengaum (F) Viktor atau ina mengikik 04. Semua professor botak, ada orang botak yang bukan professor. Tentukan apakah (T/F/M) (A) Jika Adi adalah prof, maka Adi botak (B) Jika bugsi botak, maka bugsi profesor 05. Tidak gading yang tak retak (pl). Tentukan (T/F/M) (A) Semua gading retak (B) Ada yang bukan gading yg retak (C) Semua yang retak adalah gading

273

06. Ada pelajar yang bolos semua yang bolos masuk angin. (A) Semua pelajar masuk angin (B) Ada pelajar yang masuk angin 07. Ada ikan yang nganggur ada yg nganggur yang adalah motor . (A) Ada ikan yang adalah motor (B) Ada motor yang nganggur 08. Semua P adalah Q. Ada Q yang bukan R. Tentukan mana yang pasti benar … (A) Ada Q yang R (B) Tidak ada Q yang P (C) Ada P yang bukan R (D) Tidak ada P yang bukan Q

Xpedia TPA Basic Thinking and Reasoning Skills - Deductive Reasoning and Logic - Soal Analitik

Enam buku yang ditulis oleh enam pengarang - J, K, L, M, N, dan O disusun dalam suatu lemari buku yang terdiri dari 6 rak. Rak tersebut dinomori mulai dari yang tertinggi 1 ke yang terendah 6. Tepat satu buku akan diletakkan pada setiap rak. Buku oleh K harus diletakkan dua rak di bawah buku oleh J. Buku oleh O harus diletakkan di rak pertama atau rak keenam. Buku oleh L tidak boleh diletakkan pada rak tepat di atas atau pada rak tepat di bawah buku oleh M. 01. Mana yang merupakan urutan yang mungkin bagi buku, mulai dari rak teratas sampai rak terbawah. (A) J, N, K, M, L, O (B) J, N, K, L, M, O (C) J, M, K, L, N, O (D) O, J, K, M, N, L (E) N, J, M, K, O, L 02. Semua ini merupakan urutan penyusunan buku yang mungkin dari rak teratas sampai rak terbawah, KECUALI …. (A) J, L, K, N, M, O (B) L, J, M, N, K, O (C) L, N, J, M, K, O (D) M, J, N, K, L, O (E) O, M, J, L, K, N 03. Apa yang TIDAK MUNGKIN benar ? (A) Buku oleh J ada di rak kedua (B) Buku oleh J ada di rak ketiga (C) Buku oleh M ada di rak ketiga (D) Buku oleh L ada di rak kelima (E) Buku oleh J ada di rak kelima

04. Buku oleh pasangan mana yang TIDAK MUNGKIN diletakkan berurutan di rak tiga dan empat ? (A) J dan M (B) K dan L (C) L dan K (D) M dan N (E) N dan J 05. Jika buku oleh L dan M terpisah tepat satu rak, maka buku oleh pengarang mana yang dapat diletakkan di rak antara buku oleh L dan M ? I. N II. J III. K (A) Hanya I (B) Hanya II (C) Hanya I dan II (D) Hanya II dan III (E) I, II, dan III 06. Jika buku oleh J ada di rak ke empat, maka apa yang PASTI benar ? (A) O ada di rak pertama (B) L ada di rak kedua (C) N ada di rak ketiga (D) M ada di rak kelima (E) N ada di rak kelima 07. Jika buku oleh M diletakkan dua rak di atas buku oleh J, maka semua ini PASTI benar, KECUALI …. (A) Buku O diletakkan di atas buku M (B) Buku M diletakkan di atas buku N (C) Buku J diletakkan di atas buku L (D) Buku J diletakkan di atas buku K (E) Buku L diletakkan diatas buku K

274

Xpedia TPA, Basic Basic Thinking and Reasoning Skills - Deductive Reasoning and Logic - Soal Analitik

08. Jika buku yang ditulis oleh L dipisahkan dari buku yang ditulis oleh M sebanyak 3 rak, maka mana yang merupakan daftar lengkap dari rak yang boleh ditempati buku yang ditulis oleh N ? (A) Ketiga (B) Keempat (C) Kedua. Ketiga (D) Ketiga, keempat (E) Kedua, ketiga, keempat 09. Sebuah bangunan terdiri dari 6 lantai dan tiap lantai berisi satu bisnis, 6 bisnis itu adalah agen asuransi praktek dokter, firma hukum, percetakan, studio tari, dan toko baju. Agen asuransi terletak di bawah lantai 4. Praktek dokter terpisah dari agen asuransi sejauh jumlah lantai yang memisahkan agen asuransi dengan firma hukum serta jumlah lantai itu mungkin nol. Percetakan terletak di antara studio tari dan agen asuransi Firma hukum terletak tepat di atas atau tepat di bawah percetakan. Mana yang merupakan susunan yang mungkin, dari lantai satu sampai lantai enam ? (A) Praktek dokter, agen asuransi, firma hokum, percetakan, studio tari, took baju (B) Praktek dokter, agen asuransi, firma hokum, took baju, percetakan, studio tari (C) Agen asuransi, firma hukum, percetakan, praktek dokter, studio tari, toko baju (D) toko baju, firma hukum, percetakan, agen asuransi, praktek dokter, studio tari (E) Studio tari, percetakan, agen asuransi, firma hukum, praktek dokter, toko baju 10. Jika studio tari ada di lantai 5, maka toko baju terletak di lantai ? (A) Satu (B) Dua (C) Tiga (D) Empat (E) Enam

275

11. Jika percetakan terletak di lantai lima, di mana lantai letak toko baju ? (A) Satu (B) Dua (C) Tiga (D) Empat (E) Enam 12. Jika toko baju terletak di lantai lima, maka mana yang PASTI benar ? (A) Agen asuransi ada di lantai satu (B) Praktek dokter ada di lantai dua (C) Firma hukum ada di lantai tiga (D) Studio tari ada di lantai empat (E) Percetakan ada di lantai enam 13. Seorang guru music menjadwalkan resital untuk 5 orang. Resital itu terdiri dari 5 partitur dan tiap partitur akan dinyanyikan oleh satu orang dengan diiringi piano. Wilona, Menik, dan Shinta bias bernyanyi Victor, Bernie, Shinta bias memainkan piano Shinta harus bernyanyi pada partitur ketiga Bernie harus mengiringi partitur ke empat Tidak ada orang yang boleh tampil pada dua partitur berurutan. Mana yang pasti benar? (A) Bernie adalah pengiring untuk partitur pertama (B) Bernie adalah pengiring untuk partitur kedua (C) Shinta adalah vokalis untuk partitur keempat (D) Menik adalah vokalis untuk partitur keempat (E) Shinta adalah vokalis untuk partitur kelima

Xpedia TPA, Basic Basic Thinking and Reasoning Skills - Deductive Reasoning and Logic - Soal Analitik

14. Jika shinta menyanyikan partitur pertama, mana yang PASTI benar ? (A) Wilona menyanyikan partitur pertama (B) Menik menyanyikan partitur kedua (C) Shinta menyanyikan partitur kelima (D) Victor mengiringi partitur pertama (E) Victor mengiringi partitur kelima 15. Jika Wilona sakit dan tidak dapat tampil, maka semua ini benar KECUALI …. (A) Shinta vokalis pada partitur pertama (B) Victor mengiringi partitur pertama (C) Menik vokalis pada partitur kedua (D) Menik vokalis pada partitur keempat (E) Shinta pengiring pada partitur kelima

276

SBMPTN 2018 - TPA TPA 2018 01 Proposisi, Verbal, Sebab-Akibat

01. Andri gemar bermain basket. Setiap minggu ia berolahraga di lapangan. Rudi setiap minggu berolahraga, tetapi ia tidak gemar bermain basket. Simpulan yang paling tepat adalah …. (A) Andri dan Rudi gemar bermain basket setiap minggu di lapangan (B) Andri dan Rudi gemar bermain basket di lapangan (C) Andri dan Rudi berolahraga setiap minggu (D) Andri dan Rudi berolahraga setiap minggu di lapangan (E) Andri dan Rudi berolahraga di lapangan 02. Kegiatan ekstrakurikuler dilakukan di luar ruangan jika diadakan pada hari Minggu. Jika kegiatan ekstrakurikuler dilakukan di luar ruangan, siswa mengenakan pakaian dinas lapangan. Simpulan yang paling tepat adalah …. (A) jika kegiatan tidak diadakan pada hari Minggu, siswa tidak mengenakan pakaian dinas lapangan (B) jika siswa tidak mengenakan pakaian dinas lapangan, kegiatan ekstrakurikuler tidak diadakan pada hari Minggu (C) jika kegiatan ekstrakurikuler tidak dilakukan di luar ruangan, siswa tidak mengenakan pakaian dinas lapangan (D) jika siswa mengenakan pakaian dinas lapangan, kegiatan ekstrakurikuler diadakan pada hari Minggu (E) jika kegiatan ekstrakurikuler dilakukan di luar ruangan, kegiatan tersebut tidak diadakan pada hari Minggu

284

03. Deni berencana membeli komputer pada akhir tahun nanti. Jika Deni menabung dari uang saku hariannya, dia akan membeli komputer dengan memori besar. Jika Deni mendapatkan tambahan uang dari orang tuanya, ia akan membeli komputer layar datar dengan resolusi tinggi. Pada akhir tahun, ternyata Deni tidak membeli komputer dengan memori besar atau komputer layar datar dengan resolusi tinggi. Simpulan yang paling tepat adalah …. (A) Deni tidak menabung dari uang saku hariannya atau tidak mendapatkan tambahan uang dari orang tuanya (B) Deni menabung dari uang saku hariannya atau mendapatkan tambahan uang dari orang tuanya (C) Deni tidak menabung dari uang saku hariannya, tetapi mendapatkan tambahan uang dari orang tuanya (D) Deni menabung dari uang saku hariannya, tetapi tidak mendapatkan tambahan uang dari orang tuanya (E) Deni tidak jadi membeli komputer pada akhir tahun 04. Sebagian pegawai negeri bukan guru sekolah dasar. Semua peserta seminar adalah guru sekolah dasar. Simpulan yang paling tepat adalah …. (A) semua guru sekolah dasar adalah peserta seminar (B) semua pegawai negeri bukan peserta seminar (C) sebagian pegawai negeri adalah guru sekolah dasar (D) sebagian pegawai negeri bukan peserta seminar (E) sebagian guru sekolah dasar bukan peserta seminar

SBMPTN 2018 - TPA, TPA 2018 01 Proposisi, Verbal, Sebab-Akibat

05. Tidak ada hutan lindung yang mengalami kebakaran di Pulau A. Beberapa hutan lindung di Pulau A adalah kawasan wisata. Simpulan yang paling tepat adalah …. (A) beberapa kawasan wisata tidak mengalami kebakaran (B) beberapa hutan lindung yang mengalami kebakaran adalah kawasan wisata (C) beberapa hutan lindung yang mengalami kebakaran bukan kawasan wisata (D) tidak ada hutan lindung di Pulau A yang merupakan kawasan wisata (E) tidak ada kawasan wisata yang mengalami kebakaran 06. (1) pada musim kering tahun ini, petani di Desa Sukamaju gagal panen (2) warga Desa Sukamaju mencari air ke desa lain Manakah di bawah ini yang menggambarkan hubungan di antara kedua pernyataan? (A) pernyataan 1 adalah penyebab dan pernyataan 2 adalah akibat (B) pernyataan 2 adalah penyebab dan pernyataan 1 adalah akibat (C) pernyataan 1 dan 2 adalah penyebab, namun tidak saling berhubungan (D) pernyataan 1 dan 2 adalah akibat dari dua penyebab yang tidak saling berhubungan (E) pernyataan 1 dan 2 adalah akibat dari suatu penyebab yang sama 07. (1) siswa dilarang membawa telepon genggam ke sekolah (2) beberapa siswa mengalami penurunan prestasi Manakah di bawah ini yang menggambarkan hubungan di antara kedua pernyataan? (A) pernyataan 1 adalah penyebab dan pernyataan 2 adalah akibat (B) pernyataan 2 adalah penyebab dan pernyataan 1 adalah akibat (C) pernyataan 1 dan 2 adalah penyebab, namun tidak saling berhubungan (D) pernyataan 1 dan 2 adalah akibat dari dua penyebab yang tidak saling berhubungan (E) pernyataan 1 dan 2 adalah akibat dari suatu penyebab yang sama

08. (1) jumlah siswa TK yang suka menyanyikan lagu anak-anak semakin meningkat (2) banyak acara di TV menyelenggarakan lomba menyanyi lagu anak-anak Manakah di bawah ini yang menggambarkan hubungan di antara kedua pernyataan? (A) pernyataan 1 adalah penyebab dan pernyataan 2 adalah akibat (B) pernyataan 2 adalah penyebab dan pernyataan 1 adalah akibat (C) pernyataan 1 dan 2 adalah penyebab, namun tidak saling berhubungan (D) pernyataan 1 dan 2 adalah akibat dari dua penyebab yang tidak saling berhubungan (E) pernyataan 1 dan 2 adalah akibat dari suatu penyebab yang sama 09. (1) agen perumahan memberi penawaran khusus untuk pembelian rumah (2) pemerintah membuat kebijakan ganti rugi bagi warga yang terkena dampak pembangunan Manakah di bawah ini yang menggambarkan hubungan di antara kedua pernyataan? (A) pernyataan 1 adalah penyebab dan pernyataan 2 adalah akibat (B) pernyataan 2 adalah penyebab dan pernyataan 1 adalah akibat (C) pernyataan 1 dan 2 adalah penyebab, namun tidak saling berhubungan (D) pernyataan 1 dan 2 adalah akibat dari dua penyebab yang tidak saling berhubungan (E) pernyataan 1 dan 2 adalah akibat dari suatu penyebab yang sama

285

SBMPTN 2018 - TPA, TPA 2018 01 Proposisi, Verbal, Sebab-Akibat

10. (1) terjadi peningkatan jumlah impor gula dibandingkan dengan tahun sebelumnya (2) banyak penduduk Indonesia menderita penyakit kencing manis Manakah di bawah ini yang menggambarkan hubungan di antara kedua pernyataan? (A) pernyataan 1 adalah penyebab dan pernyataan 2 adalah akibat (B) pernyataan 2 adalah penyebab dan pernyataan 1 adalah akibat (C) pernyataan 1 dan 2 adalah penyebab, namun tidak saling berhubungan (D) pernyataan 1 dan 2 adalah akibat dari dua penyebab yang tidak saling berhubungan (E) pernyataan 1 dan 2 adalah akibat dari suatu penyebab yang sama

286

SBMPTN 2018 TPA TPA 2018 02 Logika Analitik

01. Lima orang sahabat, yaitu Ronny, Ricky, Deli, Susi, dan Hana hendak membeli jus. Hanya tersedia 3 gelas untuk setiap jenis jus, yaitu mangga, sirsak, stroberi, jambu, nanas, dan alpukat. Ronny menyukai jus mangga, stroberi, dan jambu. Ricky menyukai jus sirsak, jambu, dan alpukat. Sementara itu, Deli tidak menyukai mangga, stroberi, dan alpukat. Susi dan Hana sama-sama menyukai mangga. Selain mangga, Hana juga menyukai jambu dan nanas, sedangkan Susi menyukai stroberi dan alpukat. Salah satu dari lima orang sahabat itu harus mengalah dalam memilih jus …. (A) (B) (C) (D) (E)

alpukat jambu mangga sirsak stroberi

02. Toko X mengadakan undian berhadiah. Terdapat lima hadiah utama dengan nomor undian 1 sampai 5. Lima orang pelanggan, yaitu Ano, Bono, Lino, Seno, dan Tono mendapat kesempatan mengambil nomor undian. Seno mendapat nomor tepat sesudah Ano, tetapi sebelum Tono. Lino mendapat nomor terkecil. Sementara itu, Bono mendapat nomor ganjil. Pelanggan yang mendapat nomor paling besar adalah …. (A) (B) (C) (D) (E)

03. Delapan orang (A, B, C, D, E, F, G, dan H) se da ng be la ja r be rsa ma de ng a n menggunakan meja persegi panjang dan empat kursi yang saling berhadapan. E dan G duduk di kedua ujung meja pada sisi yang sama. C duduk berseberangan dengan A dan F. D duduk di ujung meja, tepat di hadapan E. H duduk tepat di sebelah B. Jika G dan H tepat berhadapan, yang duduk di sebelah D adalah …. (A) A (B) B (C) C (D) E (E) F 04. Ibu bermaksud menyiapkan empat hidangan untuk menu makan siang yang dipilih di antara delapan hidangan, yaitu A, B, C, D, E, F, G, dan H. berikut ketentuan dalam memilih menu. (1) antara hidangan A dan C harus dipilih salah satu. (2) Apabila dalam menu terdapat hidangan E, hidangan D tidak dapat dipilih. (3) F dan B harus dihidangkan bersama-sama. (4) H dan E tidak dapat dihidangkan bersama-sama. Jika pada menu terpilih hidangan H, susunan menu yang mungkin di bawah ini adalah …. (A) A, B, G, H (B) A, E, F, H (C) B, D, F, H (D) C, D, G, H (E) C, F, D, H

Ano Bono Lino Seno Tono 287

SBMPTN 2018 TPA, TPA 2018 02 Logika Analitik

05. Empat sesi terapi medis dijadwalkan secara beruntun pada hari Senin sampai Kamis. Diketahui terdapat sepuluh pasien yang mengikuti salah satu terapi ini, yaitu K, L, M, N, O, P yang merupakan pasien anakanak dan V, W, X, Y yang merupakan pasien dewasa. K, O, V melakukan terapi pada hari Senin, L dan X pada hari Selasa, M dan W pada hari Rabu, sedangkan yang lainnya pada hari Kamis. Seorang mahasiswa kedokteran akan memilih empat pasien untuk menjadi responden penelitiannya dengan kondisi sebagai berikut. (1) Hanya dua pasien dewasa yang terpilih menjadi responden. (2) Jika N terpilih, pasien anakanak yang mengikuti terapi pada hari senin tidak boleh dipilih. (3) Pasien dewasa pada hari terapi yang berurutan tidak boleh dipilih bersamaan. (4) Paling tidak terdapat satu pasien yang mengikuti terapi pada hari Kamis. Jika X terpilih, pasien yang terpilih menjadi responden penelitian adalah …. (A) (B) (C) (D) (E)

288

L, M, V, X K, P, W, X K, N, X, Y N, O, X, Y M, N, X, Y

SBMPTN 2018 - TPA TPA 2018 03 Barisan & Aritmetika

01. 1, 3, 2, 6, 5, 15, 14, .... (A) 24 (B) 28 (C) 32 (D) 42 (E) 52 02. 100, 95, ..., 91, 92, 87, 88, 83 (A) 90 (B) 92 (C) 94 (D) 96 (E) 97 03. 6, 2, 5, 10, 6, 9, 18, 14, 17, .... (A) 55 (B) 51 (C) 34 (D) 28 (E) 20 04. 0, 5, 10, 8, 13, 26, 24, 29, ..., 56, 61 (A) 27 (B) 48 (C) 56 (D) 58 (E) 60 05. 2, 4, 1, ..., 15, 11, 16, ..., 59, 65 (A) 3, 32 (B) 6, 64 (C) 10, 32 (D) 5, 63 (E) 5, 64

06. Urutan bilangan dari yang terbesar hingga terkecil yang paling tepat adalah .... (A) (B) (C) (D) (E)

9 , 1,33, 5 9 , 1,20, 5 7 , 1,20, 4 5 , 1,50, 4 5 , 1,60, 3

3 , 1,0 2 5 , 0,9 4 6 , 0,9 5 4 , 1,20 3 3 , 1,20 2

07. 24,99% dari 2.499 adalah .... (A) 625,5001 (B) 645,5001 (C) 634,5001 (D) 624,5001 (E) 620,5001 08. 32 × 23 = .... (A) 3 × 16 (B) 6 × 12 (C) 9 × 6 (D) 9 × 4 (E) 3 × 8 09. Diketahui x, y, dan z adalah bilangan positif. Jika x < y dan x > 3z, maka …. (A) y < z (B) x – z > y (C) y > z (D) x + 3z = 2y (E) y < 3z 10. Diketahui x - z ≥ 2y dan y - z ≤ 2x, maka .... (A) x ≤ 2y (B) x ≤ y (C) x ≥ 2y (D) x = 2y (E) x ≥ y

289

SBMPTN 2018 - TPA, TPA 2018 03 Barisan & Aritmetika

11. Lima bulan berturut-turut diperoleh keuntungan 142,5, 153, 164,5, 155, dan 161. Jika rata-rata keuntungan sampai bulan keenam sebesar 156, maka keuntungan pada bulan keenam adalah .... (A) 156 (B) 160 (C) 161 (D) 166 (E) 170

14. Bulan Januari harga barang Rp60.000.000,00. Bulan Februari harga naik 10%. Selama bulan Maret harga barang bulan Februari didiskon sebesar 10%. Harga barang bulan Maret adalah .... (A) Rp59.400.000,00 (B) Rp60.000.000,00 (C) Rp66.000.000,00 (D) Rp72.000.000,00 (E) Rp120.000.000,00

12. Empat mahasiswa membandingkan uang saku yang diperoleh tiap bulan. A mendapatkan uang saku 20% lebih banyak dari B. Uang saku C separuh dari uang saku D. Uang saku B sama dengan uang saku C. Apabila uang saku A Rp600.000,00 per bulan, uang saku D adalah ... per bulan. (A) Rp500.000,00 (B) Rp600.000,00 (C) Rp800.000,00 (D) Rp1.000.000,00 (E) Rp1.200.000,00

15. Berikut data nilai rata-rata kelas hasil ujian empat mata pelajaran: Matematika, Fisika, Kimia, dan Biologi pada lima kelas di suatu sekolah menengah atas.

13. Toko P menjual barang 10 buah dengan harga Rp5.000,00 per buah, mendapatkan keuntungan sebesar Rp2.000,00. Ketika barang dijual dengan diskon 5%, barang terjual sebanyak 50 buah. Manakah pernyataan berikut yang benar? (A) harga dasar barang adalah Rp4.600,00 (B) keuntungan diperoleh setelah diskon Rp15.000,00 (C) penjualan dengan diskon 5% merugikan (D) penjualan dengan diskon 5% menguntungkan 5 kali lipat (E) harga penjualan Rp4.700,00 per buah menguntungkan

290

Kelas

Jumlah Matematika Fisika Kimia Biologi Siswa

3A

50

8,5

7

8

6,5

3B

45

8

7,5

8

7

3C

50

8

9

6,5

7,5

3D

40

9

7

7

8

3E

45

7

9

7

9

Jika prestasi kelas ditentukan oleh nilai ratarata empat mata pelajaran tiap siswa, maka kelas terbaik adalah .... (A) kelas 3A (B) kelas 3B (C) kelas 3C (D) kelas 3D (E) kelas 3E

SBMPTN 2018 TPA TPA 2018 04 Pola Gambar

08.

01.

(A) (B) (C) (D) (E) (A)

(B)

(C)

(D)

(E)

09.

02.

(A) (B) (C) (D) (E) 10. (A)

(B)

(C)

(D)

(E) (A) (B) (C) (D) (E)

03. 11. (A)

(B)

(C)

(D) (E)

04.

(A)

(B)

(C)

(D)

(E)

(A)

(B)

(C)

(D)

(E)

(A)

(B)

(C)

(D)

(E)

(A)

(B)

(C)

(D)

(E)

12.

05.

(A)

(B)

(C)

(D)

(E) 13.

06. (A) (B) (C) (D) (E) 07. (A) (B) (C) (D) (E)

291

SBMPTN 2018 TPA, TPA 2018 04 Pola Gambar

14.

(A)

(B)

(C)

(D) (E)

(A)

(B)

(C)

(D)

15.

292

(E)

SBMPTN 2017 - TPA TPA 2017 01 Proposisi, Verbal, Sebab-Akibat

01. Jika saya ke kampus, saya akan bertemu Heru dan Joni. Hari ini saya ke kampus. Simpulan yang tepat adalah .... (A) saya dan Heru bertemu Joni (B) saya bertemu Heru karena Joni tidak di kampus (C) saya ke kampus untuk bertemu Heru dan Joni (D) saya akan bertemu Heru dan Joni (E) saya, Heru, dan Joni akan bertemu di kampus 02. Jika guru matematika menambah jam pelajaran di hari Rabu, nilai siswa banyak yang meningkat. Jika nilai siswa banyak yang meningkat, siswa dapat mengikuti ekstrakurikuler. Simpulan yang tepat adalah .... (A) jika guru matematika menambah jam pelajaran, nilai siswa banyak yang meningkat (B) guru matematika menambah jam pelajaran di hari Rabu jika siswa tidak mengikuti ekstrakurikuler (C) jika guru matematika menambah jam pelajaran di hari Rabu, ekstrakurikuler ditiadakan (D) siswa dapat mengikuti kegiatan ekstrakurikuler jika guru matematika menambah jam pelajaran di hari Rabu (E) nilai siswa banyak yang meningkat jika tidak mengikuti ekstrakurikuler

03. Jika kinerja perbankan syariah menurun, pertumbuhan ekonomi syariah juga akan menurun. Jika bank-bank syariah tidak melakukan promosi kepada masyarakat, pangsa pasar bank syariah akan menurun. Saat ini pertumbuhan ekonomi syariah meningkat atau pangsa pasar bank syariah mengalami peningkatan. Simpulan yang tepat adalah .... (A) kinerja perbankan syariah menurun atau bank-bank syariah tidak melakukan promosi perbankan syariah kepada masyarakat (B) kinerja perbankan syariah menurun karena bank-bank syariah tidak melakukan promosi kepada masyarakat (C) kinerja perbankan syariah meningkat atau bank-bank syariah melakukan promosi kepada masyarakat (D) kinerja perbankan syariah meningkat dan bank-bank syariah melakukan promosi kepada masyarakat (E) kinerja perbankan syariah meningkat walaupun bank-bank syariah tidak melakukan promosi kepada masyarakat 04. Semua pegawai teladan mendapat penghargaan pemerintah. Tidak ada koruptor yang mendapat penghargaan pemerintah. Berdasarkan dua pernyataan di atas, simpulan yang tepat adalah .... (A) semua yang mendapat penghargaan pemerintah adalah pegawai teladan yang tidak korupsi (B) tidak ada koruptor yang merupakan pegawai teladan (C) ada pegawai teladan yang tidak mendapat penghargaan pemerintah karena korupsi (D) semua pegawai teladan yang tidak mendapat penghargaan pemerintah adalah koruptor (E) tidak ada koruptor yang bukan merupakan pegawai teladan

293

SBMPTN 2017 - TPA, TPA 2017 01 Proposisi, Verbal, Sebab-Akibat

05. Tidak ada orang pintar yang mencontek saat ujian. Beberapa orang yang mencontek saat ujian adalah orang yang malas belajar. Berdasarkan dua pernyataan diatas, simpulan yang paling tepat adalah .... (A) semua orang malas mencontek pada saat ujian (B) semua orang pintar pernah malas belajar (C) beberapa orang pintar tidak perlu belajar untuk ujian (D) beberapa orang yang tidak mencontek adalah orang yang pintar (E) beberapa orang yang malas belajar bukan orang pintar

09.

06. Narapidana : Penjara = .... (A) Semut : Gula (B) Harimau : Hutan (C) Tentara : Perang (D) Kerbau : Kandang (E) Burung : Langit 07. Karbon : Pensil = .... (A) Resleting : Celana (B) Mesin : Pabrik (C) Cabe : Sambal (D) Jati : Kayu (E) Bus : Kendaraan 08. Riak : Ombak = .... (A) Lepas : Rontok (B) Dingin : Cair (C) Mendung : Hujan (D) Hangat : Panas (E) Gelap : Gulita

294

(1) Angka kematian ibu akibat kehamilan usia muda masih tergolong tinggi. (2) Pemerintah mengkaji ulang peraturan perundangan yang mengatur batasan minimal usia menikah. Manakah pilihan di bawah ini yang menggambarkan hubungan pernyataan (1) dan (2)? (A) pernyataan (1) adalah penyebab dan pernyataan (2) adalah akibat (B) pernyataan (2) adalah penyebab dan pernyataan (1) adalah akibat (C) Pernyataan (1) dan (2) adalah penyebab namun, tidak saling berhubungan (D) pernyataan (1) dan (2) adalah akibat dari dua penyebab yang tidak saling berhubungan (E) pernyataan (1) dan (2) adalah akibat dari suatu penyebab yang sama

10. (1) Banyak penyanyi baru yang memulai kariernya lewat ajang pencarian bakat. (2) Stasiun televisi sering menayangkan acara ajang pencarian bakat. Manakah pilihan di bawah ini yang menggambarkan hubungan pernyataan (1) dan (2)? (A) pernyataan (1) adalah penyebab dan per-nyataan (2) adalah akibat (B) pernyataan (2) adalah penyebab dan pernyataan (1) adalah akibat (C) pernyataan (1) dan (2) adalah penyebab namun, tidak saling berhubungan (D) pernyataan (1) dan (2) adalah akibat dari dua penyebab yang tidak saling berhubungan (E) pernyataan (1) dan (2) adalah akibat dari suatu penyebab yang sama

SBMPTN 2017 TPA TPA 2017 02 Analitik, Barisan

01. Diketahui ada 5 jenis ponsel dari merek X yang dijual di toko Y. Hanya ponsel GB, GD, dan GE yang dilengkapi kamera. Hanya ponsel GA, GC, dan GD yang dilengkapi koneksi internet. Ponsel yang dilengkapi memori eksternal hanya ponsel GA, GB, dan GE. Jika seseorang ingin membeli dua ponsel yang dilengkapi koneksi internet dengan kamera dan yang lainnya dilengkapi memori eksternal, ponsel manakah yang akan dipilih .... (A) GA dan GC (B) GA dan GD (C) GB dan GE (D) GC dan GD (E) GD dan GE 02. Enam orang sedang duduk berderet di bangku dengan nomor urut 1 sampai 6. Emi duduk di kursi paling pinggir. Jordi duduk tepat di antara Tina dan Mirna. Emi dan Mirna duduk mengapit Serli. Di samping Tina duduklah Dita. Jika Emi duduk di kursi nomor 6, siapakah yang berada di kursi urutan kedua dan keempat? (A) Mirna dan Jordi (B) Tina dan Mirna (C) Tina dan Serli (D) Serli dan Jordi (E) Dita dan Jordi 03. Sebuah toko menjual boneka dengan harga yang beragam. Harga boneka ayam hanya lebih murah dari salah satu boneka yang tersedia. Boneka macan lebih mahal dari beruang, ayam, dan koala. Boneka panda lebih mahal dari beruang tetapi lebih murah dari koala. Harga boneka ke-3 termahal adalah .... (A) macan (B) ayam (C) beruang (D) koala (E) panda

04. Bu Guru memberikan kebebasan kepada para siswa untuk membuat empat makalah dari topik yang berbeda. Di bawah ini persyaratan yang diberikan: (1) Siswa memilih empat topik diantara J, K, L, M, N, O, P, Q, dan R. (2) Antara topik L, M, dan O harus dipilih satu. (3) Topik N dan J harus dipilih bersamaan. (4) Topik P dan K harus bersama O. (5) Topik Q harus bersama L. Jika seorang siswa tidak ingin memilih topik K dan R, topik makalah yang dapat ia kerjakan adalah .... (A) J, L, N, Q (B) L, M, N, Q (C) J, M, P, Q (D) J, L, P, Q (E) J, M N, Q 05. Tujuh regu pramuka A, B, C, D, E, F, dan G akan mengikuti latihan rutin. Regu A, B, dan C akan mengikuti latihan baris-berbaris. Regu D dan E akan mengikuti latihan talitemali. Regu F dan G akan mengikuti latihan sandi. Di antara regu terdapat tiga regu putra, yaitu B, D, dan G, sedangkan sisanya regu putri. Pelatih menetapkan jadwal latihan sebagai berikut: (1) Regu putra yang akan latihan tali-temali akan berlatih setelah G. (2) Regu putri berlatih sandi tepat sebelum regu putra berlatih sandi. (3) Salah satu regu putri yang berlatih barisberbaris ada di urutan terakhir. (4) Sebelum regu F, ada 2 regu putri yang berlatih secara berturut-turut. Jika latihan dimulai oleh regu putra, empat regu pertama yang mungkin latihan adalah .... (A) B C E F (B) B A E G (C) G D C E (D) G D A F (E) D C E B 295

SBMPTN 2017 TPA, TPA 2017 02 Analitik, Barisan

06. 5, 8, 14, 26, 50, .... (A) 62 (B) 74 (C) 86 (D) 98 (E) 100 07. 4, 7, 14, 17, ..., 37, 74 (A) 31 (B) 34 (C) 38 (D) 61 (E) 64 08. 3, 9, 13, 8, 24, 28, 23, .... (A) 26 (B) 35 (C) 40 (D) 58 (E) 69 09. 4, 2, 8, 32, ..., 22, 88, 44 (A) 12 (B) 16 (C) 22 (D) 24 (E) 48 10. 4, 5, 2, 10, ..., 8, 48, 51, 46, 322 (A) 4 (B) 6 (C) 8 (D) 10 (E) 12

296

SBMPTN 2017 TPA TPA 2017 03 Aritmetika, Tabel

01. Jika (a - 2ab)2 = 4 maka (a - 2ab) = .... (A) 1 (B) 2 (C) 3 (D) 8 (E) 16 02. Jika a + 2b - c = 2 dan a + c = 2 maka a + b = .... (A) 1 (B) 2 (C) 3 (D) 4 (E) 5 03. Jika x ≥ 3 maka .... (A) 4x - 10 ≥ 2 (B) 4x - 10 ≤ 2 (C) 4x - 10 ≥ 3 (D) 4x - 10 ≤ 3 (E) 4x - 10 > 3

06. Berikut ini adalah data sebuah keluarga besar yang terdiri atas kakak Ibu, Ibu, dan 3 anak. Nama

Tanggal Lahir

A

30 Januari 1989

B

15 April 1965

C

30 Mei 1985

D

15 Oktober 1960

E

01 November 1990

Siapakah yang pasti anak ke-2 dalam keluarga tersebut? (A) A (B) B (C) C (D) D (E) E

04. Jika x + y = 5 dan x +2y = 7 maka .... (A) x = 2 (B) y = 3 (C) xy = 12 (D) x - y = 1 (E) x / y = 2 05. Jika a > 0 dan b < 0 maka .... a 0 (A) b (B)

a 0 b

(C)

a 0 b

(D)

a 0 b

(E)

a 0 b

297

SBMPTN 2017 TPA, TPA 2017 03 Aritmetika, Tabel

07. Kualitas keamanan sebuah kampung ditentukan oleh jumlah pos kamling. Tabel berikut menunjukkan kondisi tersebut pada lima kampung. Kondisi yang ideal adalah setiap dua puluh rumah di dalam kampung diharapkan memiliki satu buah pos kamling.

Ada berapa kampung yang memenuhi kondisi ideal? (A) 1 (B) 2 (C) 3 (D) 4 (E) 5 08. Lima orang membaca 2 jenis buku yang sama dengan waktu membaca seperti dalam tabel berikut.

Siapakah yang kecepatan membaca per bukunya paling rendah? (A) Andin (B) Betzy (C) Citra (D) Dina (E) Erika

298

09. Tiga kotak masing-masing berisi 25 buah kartu dengan lima jenis yang memiliki gambar berupa huruf A, C, dan F serta angka 1 dan 3.

Jika Banu secara acak mengambil satu kartu pada tiap kotak maka ia memiliki kemungkinan besar memiliki kartu .... (A) kartu C, 3, dan 1 (B) dua kartu angka satu kartu F (C) satu kartu angka, dua kartu huruf (D) satu kartu huruf, dua kartu angka (E) semuanya kartu angka

SBMPTN 2017 TPA, TPA 2017 03 Aritmetika, Tabel

10. Sebuah kedai kopi yang baru saja dibuka selama seminggu sedang melakukan pengecekan terhadap hasil penjualan berbagai jenis minumannya selama seminggu. Menu

Modal per Cangkir

Harga Jual per Cangkir

Jumlah Penjualan (dalam Cangkir)

Espresso

Rp15.000,00

Rp20.000,00

110

Americano

Rp17.000,00

Rp20.000,00

80

Moccachino

Rp14.000,00

Rp20.000,00

150

Cafe Latte

Rp18.000,00

Rp25.000,00

110

Machiato

Rp12.000,00

Rp15.000,00

100

Jika setiap pembelian Espresso dan Americano diberikan diskon 10% sementara Moccachino, Café Latte, dan Machiato diberikan diskon sebesar 20%, menu manakah yang memberikan keuntungan paling sedikit bagi kedai kopi tersebut? (A) Espresso (B) Americano (C) Moccachino (D) Café Latte (E) Machiato

299

SBMPTN 2016 TPA Soal Logika

01. Pimpinan meminta karyawan menyerahkan laporan kegiatan jika kegiatan telah dilaksanakan. Jika laporan kegiatan telah diserahkan, honor karyawan dibayarkan. Simpulan yang paling tepat adalah sebagai berikut ... (A) Jika honor kegiatan dibayarkan, pimpinan dapat menugaskan karyawan membuat laporan. (B) Laporan kegiatan belum diserahkan berarti honor pimpinan tidak dibayarkan. (C) Jika pimpinan meminta laporan, kegiatan segera dilaksanakan. (D) Honor karyawan tidak dibayarkan berarti kegiatan belum dilaksanakan (E) Jika honor tidak ada, kegiatan tidak dapat dilaksanakan 02. Pada hari minggu lalu, di Kampung Bambu terjadi tindakan kejahatan berupa perampokan atau pencurian meskipun keamanan kampung ditingkatkan. Ternyata, di Kampung Bambu tidak pernah terjadi perampokan. Simpulan yang paling tepat adalah sebagai berikut ... (A) Tidak ada warga Kampung Bambu yang mengalami tindakan kejahatan pada hari Minggu yang lalu. (B) Tidak ada warga Kampung Bambu yang mengalami pencurian pada hari Minggu yang lalu. (C) Pada hari Minggu yang lalu, di Kampung Bambu baru saja terjadi tindakan kejahatan berupa pencurian. (D) Di Kampung Bambu tidak terjadi tin-dakan kejahatan pada hari minggu lalu karena keamanan kampung ditingkatkan. (E) Sejak hari minggu yang lalu, di Kampung Bambu tidak terjadi tindakan kejahatan.

300

03. Jika seseorang rajin berolahraga, badannya menjadi sehat. Jika aliran darah lancar, organ tubuh dapat bekerja dengan baik. Saat ini, seseorang sedang tidak sakit atau organ tubuhnya tidak bekerja dengan baik. Simpulan yang paling tepat adalah sebagai berikut ... (A) Aliran darah orang tersebut sedang terganggu atau badannya tidak sehat. (B) Orang tersebut tidak rajin olahraga sehingga mengganggu aliran darah (C) Terjadi penyumbatan pada aliran darah dan orang tersebut tidak rajin berolahraga (D) Orang tersebut tidak rajin berolahraga atau aliran darahnya tidak lancar (E) Orang tersebut tidak rajin berolahraga dan organ tubuhnya tidak baik. 04. Semua pelajar memakai sepatu hitam. Beberapa yang hadir di sekolah tidak memakai sepatu hitam. Berdasarkan dua pernyataan diatas, simpulan yang paling tepat adalah sebagai berikut ... (A) Beberapa yang hadir di sekolah bukan pelajar (B) Beberapa yang hadir di sekolah memakai sepatu hitam. (C) Beberapa yang memakai sepatu hitam bukan pelajar. (D) Semua pelajar tidak memakai sepatu hitam. (E) Semua yang memakai sepatu hitam adalah pelajar.

SBMPTN 2016 TPA - Soal Logika |

05. Beberapa pekerja seni berperasaan halus. Semua orang yang berperasaan halus suka keindahan. Berdasarkan dua pernyataan diatas, simpulan yang paling tepat adalah sebagai berikut... (A) Beberapa orang yang suka keindahan adalah pekerja seni. (B) Beberapa pekerja seni tidak suka keindahan. (C) Beberapa orang berperasaan halus bukan pekerja seni. (D) Beberapa orang yang suka keindahan bukan pekerja seni. (E) Beberapa orang yang bukan pekerja seni suka keindahan. 06. (1) Siswa yang mengikuti bimbingan belajar semakin banyak. (2) Siswa menyediakan lebih banyak waktu untuk belajar. Manakah di bawah ini yang menggambarkan hubungan pernyataan (1) dan (2)? (A) Pernyataan (1) adalah penyebab dan pernyatan (2) adalah akibat (B) Pernyataan (2) adalah penyebab dan pernyataan (1) adalah akibat. (C) Pernyataan (1) dan (2) adalah penyebab, namun tidak saling berhubungan. (D) Pernyataan (1) dan (2) adalah akibat dari dua pernyataan yang saling berhubungan. (E) Pernyataan (1) dan (2) adalah akibat dari suatu penyebab yang sama. 07. (1) Polusi di kota sudah sangat memprihatinkan. (2) Banyak orang menderita infeksi saluran pernapasan akut. Manakah di bawah ini yang menggambarkan hubungan pernyataan (1) dan (2)? (A) Pernyataan (1) adalah penyebab dan pernyataan (2) adalah akibat (B) Pernyataan (2) adalah penyebab dan pernyataan (1) adalah akibat (C) Pernyataan (1) dan (2) adalah penyebab, namun tidak saling berhubungan

(D) Pernyataan (1) dan (2) adalah akibat dari dua penyebab yang tidak saling berhubungan. (E) Pernyataan (1) dan (2) adalah akibat dari suatu penyebab yang sama. 08. (1) Guru meminta murid mencuci tangan sebelum makan. (2) Orang tua membekali anaknya makan siang. Manakah di bawah ini yang menggambarkan hubungan pernyataan (1) dan (2)? (A) Pernyataan (1) adalah penyebab dan pernyataan (2) adalah akibat. (B) Pernyataan (2) adalah penyebab dan pernyataan (1) adalah akibat. (C) Pernyataan (1) dan (2) adalah penyebab, namun tidak saling berhubungan. (D) Pernyataan (1) dan (2) adalah akibat dari dua penyebab yang tidak saling berhubungan. (E) Pernyataan (1) dan (2) adalah akibat dari suatu penyebab yang sama. 09. (1) Petani tebu tidak bisa menjual hasil buminya. (2) Terjadi penambahan jenis tanaman pertanian. Manakah di bawah ini yang menggambarkan hubungan pernyataan (1) dan (2)? (A) Pernyataan (1) adalah penyebab dan pernyataan (2) adalah akibat. (B) Pernyataan (1) dan (2) adalah penyebab, namun tidak saling berhubungan. (C) Pernyataan (1) dan (2) adalah penyebab, namun tidak saling berhubungan. (D) Pernyataan (1) dan (2) adalah akibat dari dua penyebab yang tidak saling berhubungan. (E) Pernyataan (1) dan (2) adalah akibat dari suatu penyebab yang sama.

301

SBMPTN 2016 TPA - Soal Logika

10. (1) Terjadi peningkatan jumlah kata dari bahasa asing yang diserap ke dalam bahasa Indonesia. (2) Terjadi peningkatan kemampuan pelajar Indonesia dalam berbahasa Inggris. Manakah di bawah ini yang menggambarkan hubungan pernyataan (1) dan (2)? (A) Pernyataan (1) adalah penyebab dan pernyataan (2) adalah akibat. (B) Pernyataan (1) dan (2) adalah penyebab, namun tidak saling berhubungan. (C) Pernyataan (1) dan (2) adalah akibat dari dua penyebab yang tidak saling berhubungan (D) Pernyataan (1) dan (2) adalah akibat dari dua penyebab yang saling berhubungan (E) Pernyataan (1) dan (2) adalah akibat dari suatu penyebab yang sama. 11. Lima orang bernama P, Q, R, S, T mengikuti perlombaan renang. Q mendahului P. S didahului oleh T. R berada di depan T. P mendahului R. Perenang yang menjadi juara kedua adalah .... (A) P (B) Q (C) R (D) S (E) T 12. Sebuah stasiun televisi memuat ketentuan untuk memutar iklan dalam setiap jeda dalam program dengan ketentuan sebagai berikut. Dalam setiap jeda harus ditayangkan enam iklan. Iklan yang dapat ditayangkan adalah A, B, C, D, E, F, G dan H. Jika iklan A ditayangkan, iklan D tidak dapat ditayangkan. Iklan H hanya dapat ditayangkan jika iklan G ditayangkan. Jika dalam suatu jeda program tidak ditayangkan iklan C dan F, iklan yang harus ditayangkan selama jeda tersebut adalah .... (A) A, B atau D, E (B) A, B atau H, G (C) D, E atau H, G (D) D, E atau B, G (E) B, G atau A, H

302

13. Ibu guru membagikan buku untuk lima siswa berprestasi sesuai dengan buku favorit mereka. Hanya tersedia satu buah untuk setiap jenis buku. Berikut ini adalah jenis buku favorit tiap siswa. Tita menyukai kamus dan buku agama. Sinta menyukai buku sains, kamus, agama, dan sastra. Reni menyukai buku agama saja. Qila menyukai buku agama dan sastra. Putri menyukai buku sains, agama, sastra, dan sejarah. Buku yang harus diberikan kepada Tita adalah .... (A) sains (B) kamus (C) agama (D) sastra (E) sejarah 14. Satu keluarga bertamasya ke Dunia Fantasi. Ayah ingin melihat Istana Boneka dan Balada Kera, tetapi tidak mau naik Korakora. Ibu ingin melihat Balada Kera dan naik Bianglala, tetepi tidak mau pergi ke wahana lain, kecuali jika ia ditemani naik Bianglala. Hari mau naik Halilintar dan Kora-kora, tetapi tidak mau ikut ke Istana Boneka jika Ibu tidak ikut. Sari mau naik Halilintar dan Ontang-anting, tetapi tidak mau ikut naik Bianglala, kecuali Ayah mau naik Kora-kora. Yang harus berkorban agar keluarga itu mencoba sebanyak mungkin wahana adalah .... (A) Ibu (B) Sari (C) Ayah (D) Hari dan Sari (E) Ayah dan Ibu

SBMPTN 2016 TPA - Soal Logika |

15. A, B, C, D, E, F, G, dan H mengikuti pramuka di regu yang berbeda, yaitu Regu Rajawali, Elang, dan Garuda, tiap regu tidak lebih dari 3 orang. Setiap orang menyukai dua hobi di antara membaca, memasak, merajut, dan melukis. D anggota Regu Elang dan tidak suka membaca. Anggota Regu Rajawali hanya F dan A. A suka melukis, E dan H tidak berada di regu yang sama dengan D, C suka dan tidak menjadi anggota di Regu Garuda. G tidak menjadi anggota di Regu Elang dan tidak suka merajut. Anggota Regu Elang semuanya suka merajut, tetapi tidak suka melukis. Semua anggota Regu Rajawali suka merajut. Jika diketahui 6 anggota regu suka memasak, kombinasi yang benar adalah .... (A) F - Rajawali - memasak - melukis (B) A - Rajawali - memasak - merajut (C) D - Elang - memasak - melukis (D) C - Elang - membaca - memasak (E) B - Elang - memasak - merajut

303

01. Jika k adalah bilangan bulat positif genap yang habis dibagi 3, 4, dan 8 maka 2k – 8 adalah .... (A) > 186 (B) > 88 (C) ≥ 88 (D) > 40 (E) ≥ 40 02. Jika 10 – 2x > 2; x adalah bilangan bulat positif, maka nilai x – 1 adalah .... (A) ≤ 3 (B) < 3 (C) ≤ 4 (D) < 4 (E) = 4 03. Jika X, Y, dan Z adalah bilangan bulat positif kurang dari 25 yang TIDAK habis dibagi 3, tetapi habis dibagi 5, jika X > Y > Z. Maka nilai dari X + Y – Z adalah .... (A) 5 (B) 10 (C) 15 (D) 25 (E) 35 04. Jika ab – 2 = 0 dan b = 3 manakah pernyataan di bawah ini yang TIDAK tepat ? (A) (B) (C) (D) (E)

2 b ab < b 2 = 3a 3a – 2b = -4 1
05. Jika a = 4b, c = 8b, dan a adalah bilangan bulat genap positif antara 7 dan 10, maka nilai 75% c – b berada antara .... (A) (B) (C) (D) (E)

304

8 < 75%c – b < 10 8 ≤ 75%c – b ≤ 10 8 < 75%c – b ≤ 10 6 < 75%c – b < 8 6 ≤ 75%c – b ≤ 8

06. 3, 6, 4, 2, 4, 2, 1, …. (A) 1 (B) 2 (C) 4 (D) 6 (E) 8 07. 1, 2, 6, 12, 16, 32, 36, …. (A) 38 (B) 40 (C) 72 (D) 76 (E) 102 08. 2, 8, 32, 16, 64, 32, 72, …. (A) 80 (B) 86 (C) 102 (D) 128 (E) 288 09. 12, 8, 16, 12, 22, 18, 30, …. (A) 20 (B) 24 (C) 26 (D) 28 (E) 30 10. 3, 3, 5, 2, 8, 13, 7 …. (A) 49 (B) 35 (C) 21 (D) 12 (E) 6

11. Data hasil penjualan rata-rata setiap bulan sebuah warung serba ada sebagai berikut.

Jika promosi berhasil meningkatkan omzet penjualan dua kali lipat untuk minuman dan sayuran, berapa besar total keuntungan yang diperoleh warung serba ada tersebut ? (A) 40 juta (B) 48 juta (C) 65 juta (D) 73 juta (E) 75 juta 12. Berikut ini adalah penjualan kendaraan bermotor dari suatu distributor selama satu tahun terakhir

13. Pada seleksi beasiswa ada 5 siswa (A, B, C, D, dan E) yang telah memenuhi kriteria pemberian beasiswa. Dari 5 orang siswa yang diseleksi, hanya 2 orang yang diberi pembebasan biaya masuk perguruan tinggi X.

Beasiswa akan diberikan pada siswa yang memiliki rata-rata nilai tertinggi. Apabila ada siswa yang memiliki nilai yang sama maka urutan nilai yang diutamakan adalah nilai 3, nilai 2, kemudian nilai 1. Siapakah yang paling mungkin mendapat beasiswa karena menempati peringkat 1 dan 2 berdasarkan kriteria seleksi tersebut ? (A) A dan C (B) B dan D (C) C dan D (D) C dan E (E) D dan E 14. Supermarket Y bermaksud menganalisis keuntungan penjualan dari berbagai minuman dengan data sebagai berikut.

Berdasarkan data di atas, kapankah penjualan kendaraan bermotor warna hitam lebih tinggi dari rata-rata penjualan semua kendaraan bermotor pada periodenya? (A) Jan-Feb (B) Mar-Apr (C) Mei-Jun (D) Jul-Agst (E) Sept-Okt Dua merek minimum yang memberikan jumlah keuntungan penjualan paling kecil adalah .... (A) U dan Q (B) T dan R (C) Q dan S (D) R dan U (E) S dan T

305

15. Butik “Gaul” memiliki data penjualan pakaian bulan Januari sebagai berikut.

Produk apakah yang memberikan keuntungan lebih dari Rp500.000,00 jika pakaian mendapatkan keuntungan 10% dari harganya? (A) Atasan dan rok (B) Atasan dan celana (C) Rok dan celana (D) Rok dan baju panjang (E) Celana dan baju panjang

306

SBMPTN 2016 TPA Pola Gambar 8.

1.

(A) (B) (C) (D) (E) (A)

(B)

(C)

(D)

(E)

9. (A) (B) (C) (D) (E)

2. 10. (A)

(B)

(C)

(D)

(A) (B) (C) (D) (E)

(E) 11.

3.

(A)

(B)

(C)

(D)

(A) (B) (C) (D) (E)

(E) 12.

4.

(A)

(B)

(C)

(D)

(A) (B) (C) (D) (E)

(E) 13.

5.

(A)

(B)

(C)

(D)

(A) (B) (C) (D) (E)

(E) 14.

6. (A) (B) (C) (D) (E)

(A) (B) (C) (D) (E)

7. 15. (A) (B) (C) (D) (E)

(A) (B) (C) (D) (E) 307

SBMPTN 2017 - TPA TPA 2017 04 Pola Gambar

08.

01.

?

? (A)

(B) (C)

(D) (E)

02.

(A) (B) (C) (D) (E)

09.

?

? (A)

(B) (C)

(A) (B) (C) (D) (E)

(D) (E)

10.

03.

?

? (A) (B)

(C)

(D) (E)

04.

(A) (B) (C) (D) (E)

11.

? (A)

(B) (C)

(D) (E)

05.

?

? (A)

(B) (C)

(D) (E)

(A)

(B)

(C)

(D)

(E)

(A)

(B)

(C)

(D)

(E)

12.

06.

? (A) (B) (C) (D) (E)

07.

(A) (B) (C) (D) (E)

308

?

SBMPTN 2017 - TPA, TPA 2017 04 Pola Gambar

13.

? ? (A)

(B)

(C)

(D)

(E)

(A)

(B)

(C)

(D)

(E)

(A)

(B)

(C)

(D)

(E)

14.

? ? 15.

? ?

309

SBMPTN 2015 TPA 01 - Logika 01. Anto menyukai pelajaran bahasa lnggris. Ia ingin melanjutkan sekolah di Australia. Brian, teman sekelas Anto, menyukai pelajaran bahasa lnggris. Kesimpulan mana yang benar? (A) Anto dan Brian belajar bahasa lnggris bersama. (B) Anto dan Brian ingin melanjutkan sekolah di Australia. (C) Anto dan Brian menyukai sekolah di Australia. (D) Anto dan Brian belajar bahasa lnggris di Australia. (E) Anto dan Brian menyukai pelajaran bahasa Inggris 02. Sesuai dengan kesepakaan direktur dengan karyawan perusahaan “X”, karyawan mengundurkan diri dengan diberi pesangon atau perusahaan ditutup. Ternyata perusahaan ditutup. Kesimpulan mana yang benar? (A) Perusahaan tidak memberi pesangon kepada karyawan. (B) Karyawan memilih perusahaan ditutup. (C) Sebagian karyawan diberi pesangon. (D) Direktur memberhentikan sebagian karyawan. (E) Sebagian karyawan tidak mau mengundurkan diri. 03. Semua penghuni asrama Kartini berasal dari luar kota. Ada mahasiswa kampus-X yang merupakan penghuni asrama Kartini. Kesimpulan mana yang benar? (A) Beberapa mahasiswa berasal dari luar kota. (B) Ada penghuni asrama Kartini yang bukan mahasiswa kampus X. (C) Ada mahasiswa kampus X yang berasal dari luar kota. (D) Beberapa mahasiswa kampus X bukan merupakan penghuni asrama Kartini. (E) Ada mahasiswa di asrama Kantini yang tidak berasal dari luar kota. 310

04. Semua siswa yang pandai pasti lulus ujian. Sebagian siswa kelas 12 bukan siswa yang pandai. Kesimpulan mana yang benar? (A) Semua yang lulus ujian adalah siswa kelas 12. (B) Sebagian siswa yang pandai tidak lulus ujian. (C) Semua siswa yang pandai adalah siswa kelas 12. (D) Sebagian siswa kelas 12 tidak mengikuti ujian. (E) Sebagian siswa kelas 12 tidak lulus ujian. 05. Jika Kurniawan berminat pada sepakbola, maka ia akan rajin berlatih sepakbola. Kurniawan dapat berprestasi pada bidang sepakbola jka rajin berlatih sepakbola. Tahun ini Kurniawan rajin berlatih sepakbola. Kesimpulan mana yang benar? (A) Kurniawan berminat pada sepakbola. (B) Kurniawan dapat berprestasi pada bidang sepakbola. (C) Kurniawan tahun lalu belum berminat pada sepakbola. (D) Kurniawan ingin berprestasi pada bidang sepakbola. (E) Kurniawan tidak rajin berlatih sepakbola tahun lalu.

SBMPTN 2015 - TPA 02 - Verbal 06. Kaki : Badan = Roda : .... (A) Pedati (B) Rantai (C) Sadel (D) Rem (E) Ban 07. Garam : Hambar = Canda : .... (A) Tawa (B) Bingung (C) Lucu (D) Gembira (E) Serius 08. Payung : Basah = Jaket : .... (A) Dingin (B) Nyaman (C) Panas (D) Hangat (E) Berkeringat 09. Sapi : Susu = Baterai : .... (A) Arang (B) Listrik (C) Panas (D) Senter (E) Aki 10. Minggu : Bulan = Liter : .... (A) Galon (B) Air (C) Mililiter (D) Beras (E) Volume

311

SBMPTN 2015 - TPA. 03 - Analitik 11. Enam regu lomba gerak jalan berasal dari tiga SMA, yakni regu Sudirman dan Diponegoro dari SMA 1. Regu Hasanuddin dan Antasari berasal dari SMA 5, dan Regu Patimura dan Teuku Umar dari SMA 7. Satu regu dari SMA 7 datang terlambat sehingga tidak diperbolehkan mengikuti lomba. Jika masing -masing regu dari SMA 5 selalu berangkat setelah regu dari SMA yang berbeda, maka kemungkinan urutan regu adalah .... (A) Pattimura, Sudirman, Teuku Umar, Diponegoro, Hasanuddin (B) Sudirman, Hasanuddin, Antasari, Teuku Umar, Diponegoro (C) Antasari, Sudirman, Teuku Umar, Hasanuddin, Diponegoro (D) Teuku Umar, Hasanuddin, Diponegoro, Sudirman, Antasari (E) Hasanuddin, Diponegoro, Pattimura, Sudirman, Teuku Umar 12. Harga sewa di enam blok perumahan berbeda. Rumah di Blok B lebih mahal daripada di Blok A, tetapi lebih murah daripada di Blok E. Rumah di Blok F lebih murah daripada di Blok A dan di Blok C. Rumah di Blok D lebih mahal daripada di Blok F, tetapi lebih murah daripada di Blok C. Harga rumah di Blok C lebih murah daripada di Blok A. rumah yang harga sewanya termurah kedua adalah rumah di blok .... (A) A (B) B (C) C (D) D (E) E 13. Suatu perguruan tinggi memberikan kesempatan pendidikan kepada siswa berprestasi dengan ketentuan pilihan jurusan sebagai berikut. Setiap siswa boleh memilih empat jurusan. Jurusan yang dapat dipilih adalah Sastra, Farmasi, Seni Rupa, Kedokteran, Psikologi, Ilmu Politik, dan Ilmu Komunikasi. Siswa boleh mengambil Kedokteran atau Psikologi, tetapi tidak keduanya. Ilmu Politik dan Farmasi tidak boleh diambil dalam kesempatan yang sama. Farmasi dan Psikologi 312

harus dipilih bersamaan. Jika dalam suatu kesempatan seorang siswa mengambil Psikologi, jurusan lain yang harus diambil siswa tersebut adalah .... (A) Sastra, Farmasi, Kedokteran, Ilmu Komunikasi (B) Sastra, Farmasi, Seni Rupa, Ilmu Komunikasi (C) Sastra, Seni Rupa, Ilmu Politik, Ilmu Komunikasi (D) Farmasi, Seni Rupa, Ilmu Politik, Ilmu Komunikasi (E) Farmasi, Seni Rupa, Ilmu Komunikasi, Kedokteran 14. Seorang ibu membeli buah di pasar dengan kriteria murah dan segar. Dari lima macam buah, yaitu anggur, belimbing, ceri, duku, dan pisang, diperoleh harga dan kesegaran buah sebagai berikut.  Harga anggur lebih tinggi dari harga ceri Dibanding pisang, ceri lebih segar.  Harga ceri lebih tinggi dari harga belimbing. Dibanding duku, belimbing lebih segar.  Harga belimbing lebih tinggi dari harga duku. Dibanding anggur, duku lebih segar.  Harga duku lebih tinggi dari harga pisang. Dibanding ceri, belimbing lebih tidak segar. Jika ibu tersebut akan memilih dua buah yang paling murah dan yang paling segar, maka yang akan dipilih benturut-turut adalah .... (A) pisang dan ceri (B) pisang dan belimbing (C) pisang dan duku (D) anggur dan ceri (E) duku dan ceri 15. Agung menentukan prioritas dalam hidupnya sebagai berikut. Agung ingin tenteram sebelum menikah, melanjutkan pendidikan setelah punya jabatan, dan menikah sebelum punya jabatan, Ia juga akan menikah setelah punya pekerjaan. Setelah punya jabatan, ia

SBMPTN 2015 - TPA. 03 - Analitik

akan sukses. Jika saat ini Agung telah punya jabatan, maka keinginan yang telah tercapai adalah .... (A) tenteram, menikah, sukses (B) tenteram, punya pekerjaan, melanjutkan pendidikan (C) punya pekerjaan, menikah, sukses (D) tenteram, punya pekerjaan, menikah (E) tenteram, menikah, melanjutkan pendidikan

313

SBMPTN 2015 - TPA 04 - Barisan

1. 4, 3, 9, 5, 4, 10, 6, 5, .... (A) 7 (B) 8 (C) 10 (D) 11 (E) 12

7. 9, 9, 7, 4, 16, 11, 5, .... (A) 35 (B) 20 (C) 12 (D) 2 (E) -1

2. 5, 4, 6, 3, 7, 2, 8, .... (A) 0 (B) 1 (C) 3 (D) 9 (E) 11 3. 2, 8, 12, 3, 12, 16, 4, 16, .... (A) 5 (B) 8 (C) 10 (D) 18 (E) 20 4. 2, 8, 4, 16, 8, 32, 16, 64, .... (A) 16 (B) 24 (C) 32 (D) 72 (E) 128 5. 5, 7, 11, 19, 35, 67, 131, .... (A) 159 (B) 239 (C) 259 (D) 269 (E) 329 6. 2, 3, 4, 6, 6, 9, 8, 12, .... (A) 7 (B) 10 (C) 11 (D) 13 (E) 14

314

SBMPTN 2015 - TPA 05 - Aritmatika 1. Jika p adalah bilangan habis dibagi 7 dan nilainya lebih besar dari 14 dan kurang dari 28, sedangkan q adalah bilangan ganjil di antara 23 dan 27, maka pernyataan yang paling tepat adalah .... (A) p = q (B) p rel="nofollow"> q (C) p < q (D) 2p < q (E) 2q < p 2. Jika a ≥ 3, maka nila 5a+3 adalah .... (A) ≥ 8 (B) > 18 (C) ≥ 18 (D) > 23 (E) ≥ 23 3. Jika m = 2a + 3 dan n = 2a – 1, maka 4m – 2n = .... (A) 2a + 4 (B) 4a + 10 (C) 4a + 14 (D) 6a + 10 (E) 6a + 14

(D) 12 (E) 13 7. Jika p > 3 dan q > 4, maka nilai p x q adalah .... (A) < 12 (B) = 12 (C) > 12 (D) ≥ 20 (E) > 20 8. Jika p = 2a + 3 dan q = 2b – 3 dengan a > 0 dan b > 0, maka nilali (p +q ) +2 adalah ..... (A) < 0 (B) > 0 (C) > 1 (D) < 1 (E) < 2

4. Jika p = q + 2, q = r – 1, dan r = 5, maka yang benar adalah .... (A) p < q < r (B) p < r < q (C) q < r < p (D) q < p < r (E) r < p < q 5. Jika a = b, maka 4a + 3b = .... (A) 7 (B) 7b (C) 7ab (D) 2ab (E) 2a 6. Jika a x b = 12 dengan a dan b adalah bilangan bulat positif, maka nilai maksimum a + b – 1 adalah .... (A) 6 (B) 7 (C) 8 315

Pilihlah gambar (A), (B), (C), (D), atau (E) sebagai gambar (4) agar hubungan antara gambar (3) dan (4) seperti hubungan anatara gambar (1) dan (2)!

34.

31.

35.

32.

36. 33.

37.

316

Pilihlah gambar (A), (B), (C), (D), atau (E) untuk melanjutkan pola gambar yang tersedia!

42.

38.

39.

40.

41.

43.

44.

45.

317

SBMPTN 2015 Tes Potensi Akademik Kode Soal

01. SINONIM PUNAH (A) Usai (B) Rusak (C) Hilang (D) Langka (E) Binasa 02. ABSURD = ... (A) Gila (B) Asli (C) Serap (D) Bingung (E) Mustahil

03. PARIPURNA = ... (A) lengkap (B) perdana (C) panjang (D) parlemen (E) musyawarah

04. INTIMIDASI = ... (A) agitasi (B) tekanan (C) dorongan (D) provokasi (E) ancaman

05. ANEKSASI = ... (A) pengambilan (B) pembredelan (C) pembantaian (D) penyerobotan (E) penyanderaan

318

SBMPTN 2015 Tes Potansi Akademik, Kode Soal

06. ANTONIM RUNYAM >< ... (A) murah (B) ringan (C) mudah (D) terang (E) sederhana

07. LIHAI >< ... (A) licik (B) kaku (C) tolol (D) bodoh (E) lamban

08. MEMIKAT >< ... (A) menepis (B) merusak (C) menjauhi (D) menangkal (E) mengganggu

09. MEMBAUR >< ... (A) melebar (B) membagi (C) memisah (D) mengurai (E) menyebar

10. CURAM >< ... (A) terjal (B) dalam (C) landai (D) tumpul (E) pendek

319

SBMPTN 2015 Tes Potansi Akademik, Kode Soal

11. ANALOGI SUNGAI : selokan (A) AIR : hujan (B) GUNUNG : bukit (C) SELAT : tanjung (D) GURU : sekolah (E) GARIS : lingkaran

12. SENDOK : makan (A) API : panas (B) PALU : paku (C) BUKU : terbit (D) ANGIN : bertiup (E) SENAPAN : menembak

13. BENGKEL : mekanik = ... : ... (A) TRUK : sopir (B) RUMAH : arsitek (C) KAPAL : nelayan (D) TOKO : pramuniaga (E) RUMAH SAKIT : pasien

14. TENUN : benang = ... : ... (A) IKAT : tali (B) UKIR : kayu (C) LIPAT : kertas (D) ANYAM : rotan (E) CETAK : batu bata

15. MOBIL : bagasi = ... : ... (A) KAPAL : palka (B) KUDA : pelana (C) RUMAH : dapur (D) SEPEDA : sadel (E) PESAWAT : kargo

320

SBMPTN 2015 Tes Potensi Akademik Kode Soal

01. Barisan 120 30 10 5 ...

(A) (B) (C) (D) (E)

1 2 3 4 5

02. 2 3 5 9 17 ...

(A) (B) (C) (D) (E)

23 27 33 34 40

03. 100 6 90 18 80 ...

(A) (B) (C) (D) (E)

36 45 48 54 72

321

SBMPTN 2015 Tes Potensi Akademik, Kode Soal

04. 1 1 1 2 1 4 1 ...

(A) (B) (C) (D) (E)

1 2 4 8 12

05. 0,5 2,5 1 7,5 3 22,5 12 ...

(A) (B) (C) (D) (E)

322

34,5 60 60,5 67 67,5

SBMPTN Tes Potensi Akademik Kode Soal

01.

Dosen, Dokter, Pegawai

02.

Hitam, Hitam, Malam

03. Pensil, Alat Tulis, Tas

04. Artis, Pelawak, Sutradara

05. Bangunan, Rumah, Hotel

326

SBMPTN Tes Potansi Akademik, Kode Soal

06. Hewan, Ayam, Sapi

07.

Nutrisi, Mineral, Vitamin

327

Xpedia TPA Bab 2 –Soal Analogi

01. Palindrom :: Kata = (A) Batang :: Pohon (B) Panggung :: Aktor (C) Kamus :: Definisi (D) Marah :: Emosi (E) Gunung :: Bukit 02. Paradigma :: Masalah = (A) Binokular :: Pemandangan (B) Perspektif :: Fenomena (C) Telepon seluler :: Komunikasi (D) Linguistik :: lingual (E) Epik :: Cerita 03. Plasebo :: Obat = (A) Aktor :: Sutradara (B) Dukun :: Susuk (C) Benteng :: Prajurit (D) Manekin :: Peragawati (E) Narsis :: Cinta 04. Volatil :: Stabil = (A) Cermat :: Prinsip (B) Tumbuhan :: Pupuk (C) Nekat :: Berani (D) Cacat :: Sempurna (E) Gunung :: Bukit 05. Kaleng :: Timah = (A) Kaset :: Pita (B) Tenis :: bola (C) Kawat :: Tembaga (D) Nitrogen :: Udara

277

Xpedia TPA Basic Verbal Skills - Soal Sinonim

Carilah Sinonim dari kata-kata di bawah ini!

07.

01. Anatema = (A) Tubuh (B) Topik (C) Teman (D) Musuh (E) Lawan

Kognitif = (A) Afektif (B) Psikomotorik (C) Akal (D) Emosi (E) Jiwa

08.

02. Anomali = (A) Air (B) Normal (C) Cacat (D) Aneh (E) Deviasi

Konundrum = (A) Teka - teki (B) Hitam (C) Misteri (D) Jawaban (E) Final

09.

Enigma = (A) Misteri (B) Nyata (C) Biasa (D) Aneh

10

Ekstrapolasi = (A) Perpanjangan (B) Prediksi (C) Interpolasi (D) Penambahan (E) Pengenalan

11.

Nexus = (A) Saraf (B) Jaringan (C) Semesta (D) Masa depan (E) Kutub

12.

Periferal = (A) Sentral (B) Celah (C) Teritori (D) Rendahan (E) Pinggiran

13.

Plegmatik = (A) Malas (B) Lembek (C) Jinak (D) Agresif (E) Kokoh

03. a. Apokriphal (A) Valid (B) Salah (C) Mati (D) Hidup (E) Misteri 04. Aposteois = (A) posteriori (B) Aposteriori (C) Pemberhalaan (D) Pemujaan (E) Paska 05. Atenuasi = (A) Degradasi (B) Improvisasi (C) Normalisasi (D) Restrukturisasi (E) Organisasi 06. Kapitulasi = (A) Menang (B) Kalah (C) Modal (D) Simpulan (E) Eksekusi

278

Xpedia TPA, Basic Verbal Skills - Soal Sinonim

14.

Pletora = (A) Amuk (B) Berkelimpahan (C) Marah (D) Perluasan (E) Perbaikan

01.

STAGNASI = .... (A) Kelancaran (B) Berhenti (C) Kemacetan (D) Bergelombang (E) Naik-turun

15.

Proliferasi = (A) Penyebaran (B) Segresi (C) Ameliorasi (D) Informasi (E) Penghalusan

02.

SIKLUS = .... (A) Kelas (B) Proses (C) Derajat (D) Daur (E) Rangkaian

16.

Sanguin = (A) Murung (B) Dendam (C) Ceria (D) Tegar (E) Percaya diri

03.

ABRASI = .... (A) Pengeroposan (B) Peleburan (C) Pengurangan (D) Penimbunan (E) Pengkikisan

17.

Strata = (A) Status (B) Lapisan (C) Lipatan (D) Keadaan (E) Skala

04.

MODERAT = .... (A) mutakhir (B) kontemporer (C) berimbang (D) kekinian (E) miniatur

05.

HOLISTS = .... (A) partikular (B) spesial (C) bersifat suci (D) Lintas disiplin (E) menyeluruh

06.

AKUISISI = .... (A) menjual (B) menekan (C) memasukkan (D) mengambil (E) membeli

279

Xpedia TPA, Basic Verbal Skills - Soal Sinonim

09.

NARSIS = .... (A) kecewa (B) luwes (C) sombong (D) cinta diri (E) gamang

18.

Random = .... (A) mendasar (B) keseluruhan (C) acak (D) minimal (E) maksimal

10.

HEDONIS = .... (A) kepuasan (B) jor-joran (C) memburu materi (D) memburu cinta (E) memburu kesenangan

20.

INSINUASI = .... (A) keadaan (B) kesempatan (C) sindiran (D) lembaga (E) perasaan halus

11.

ABSOLUT = .... (A) tuhan (B) selalu (C) harus (D) mutlak (E) pasti

21.

EVOKASI = .... (A) menggugah rasa (B) pengungsian (C) penilaian (D) menyaingi (E) hasrat

13.

RATIFIKASI = .... (A) pengesahan (B) peringkat (C) penilaian (D) perjanjian (E) produk hukum

22.

DOGMA = .... (A) doktrin (B) normatif (C) agama (D) ideologi (E) filosofi

14.

EMBARKASI = .... (A) pendaratan (B) pemberangkatan (C) kepergian (D) bandara (E) teriminal

23.

SCAN = .... (A) cetak (B) pindai (C) faksimili (D) pindah (E) salin

15.

HERBAL = .... (A) obat (B) ramuan (C) berhubungan dengan tanaman (D) sayur-mayur (E) suplemen

24.

DOWNLOAD = .... (A) unduh (B) ambil (C) oper (D) transfer (E) turun

25.

RENDEZVOUS = .... (A) pertemuan (B) persetujuan (C) penyusupan (D) rahasia (E) perkumpulan

280

Xpedia TPA, Basic Verbal Skills - Soal Sinonim

26.

BAHTERA = …. (A) pernikahan (B) sejahtera (C) bahagia (D) perahu (E) kehidupan

28.

HOMOGEN = …. (A) udara (B) harmonis (C) sepadan (D) homoseks (E) sejenis

30.

KONTRADIKSI = .... (A) penandatanganan kontrak (B) pertentangan (C) perdebatan (D) perjanjian (E) perhelatan

281

Xpedia TPA Basic Verbal Skills - Soal Antonim

Carilah antonim dari kata di bawah ini ! 03. Anomali x …. (A) Labil (B) Normal (C) Searah (D) Kacau (E) Menyimpang 04. Rivalitas x …. (A) Persengketaan (B) Keributan (C) Caci maki (D) Penyesuaian (E) Persaingan 05. Valuabel x …. (A) Murah (B) Murahan (C) Berharga (D) Utama (E) Istimewa 06. Modern x …. (A) Mutakhir (B) Sederhana (C) Kuno (D) Kontemporer (E) Tradisional 07.

09. Prominen x …. (A) Terkemuka (B) Utama (C) Biasa (D) Tertinggi (E) Elit 12. Harmoni x …. (A) Selaras (B) Janggal (C) Sesuai (D) Merdu (E) Indah 13. Acuh x …. (A) Memperhatikan (B) Membiarkan (C) Tidak ikut campur (D) Sombong (E) Tidak peduli 14. Heterogen x …. (A) Homonim (B) Sejenis (C) Plural (D) Beragam (E) Campuran

Prolog x …. (A) Pembukaan (B) Epilog (C) Monoton (D) Akhiran (E) Epilepsi

15. Sporadis x …. (A) Jamur (B) Kerap (C) Jarang (D) Selalu (E) Seperti

08. Swadaya x …. (A) Mandiri (B) Bergantung (C) Tenggang rasa (D) Memihak (E) Antipati

16. Periferal x …. (A) Marginal (B) Pusat (C) Jurang (D) Puncak (E) Pinggir

282

Xpedia TPA, Basic Verbal Skills - Soal Antonim

17. Inklusif x …. (A) Membuka (B) Toleransi (C) Liberal (D) Dibatasi (E) Eksekutif

24. Nomadik x …. (A) Menetap (B) Mapan (C) Sesuai norma (D) Berpindah-pindah (E) Tak teratur

18. Universal x …. (A) Spesial (B) Kontekstual (C) Partikular (D) Spasial (E) General 19. Anomali x …. (A) Konformis (B) Alienasi (C) Aneh (D) Janggal (E) Ekstrimis 20. Antagonis x …. (A) Pemeran utama (B) Aktor (C) Pemeran pengganti (D) Protagonis (E) Rival 21. Fakta x …. (A) Imajinasi (B) Fiktif (C) Pendapat (D) Kenyataan (E) Pernyataan 22. Efisien x …. (A) Efektif (B) Mangkus (C) Sangkil (D) Boros (E) Hemat 23. Inkosisten x …. (A) Tepat (B) Ajek (C) Tidak tetap (D) Cepat (E) Lambat

283

Xpedia TPA Verbal - Set 1

Sinonim 01. Lanun = …. (A) Perompak (B) Khayal (C) Pencuri (D) Batuan (E) Emosi

07. Fortifikasi x = …. (A) Penguatan (B) Pelemahan (C) Penundaan (D) Penyegeraan (E) Pembukaan Antonim

02. Jenggala = …. (A) Lebat (B) Bukit (C) Gurun (D) Hutan (E) Semak

08. Oral x = …. (A) Baca (B) Simak (C) Imunisasi (D) Tulis (E) Virus

03. Istal = … (A) Singgasana (B) Rumah (C) Pemberhentian (D) Kandang (E) Sarang

09. de Facto x …. (A) Secara hukum (B) Secara fakta (C) Secara logis (D) Secara umum (E) Secara khusus

04. Demagogi = …. (A) Menghasut (B) Menghina (C) Membela (D) Mempelajari (E) Menentang 05. Ceruk = …. (A) Pusat (B) Simpang (C) Tanda (D) Lekuk (E) Celah 06. Superior = …. (A) Hebat (B) Ekstrovert (C) Introvert (D) Rendah (E) Beragam

416

Analogi 10. Pendar : Bohlam = … (A) Bising : Speaker (B) Buku : Kamus (C) Obat : Sakit (D) Lelah : Kerja (E) Kanan : Kiri 11. Peron : Penumpang = … (A) Baju : Penjahit (B) Tenda : Pengungsi (C) Auditorium : Ruang (D) Perusahaan : Buruh (E) Koran : Redaktur 12. Placebo : Pasien = …. (A) Dokter : Suster (B) Rumah sakit : Klinik (C) Mantri : Perawat (D) Resep : Apoteker (E) Vitamin : Apotik

Xpedia TPA, Verbal - Set 1

13. Rotan : Perabot = … (A) Kayu : Tukang (B) Rayap : Mebel (C) Silikon : Prosesor (D) Tembok : Bilik (E) Kusen : Bingkai

417

Xpedia TPA Verbal - Set 2

Sinonim 01. Mintakat = …. (A) Perjanjian (B) Rembug (C) Zona (D) Sedekah (E) Berkah 02. Tantrum = … (A) Urung (B) Amuk (C) Senang (D) Prihatin (E) Ikhlas

07. Gradual = … (A) Seluruh (B) Perlahan (C) Sebagian (D) Bertahap (E) Selangkah 08. Inkonstitusional = …. (A) Berontak (B) Undang-undang (C) Sah (D) Ilegal (E) Peraturan Analogi

03. Sintal = …. (A) Rapuh (B) Molek (C) Cekatan (D) Sehat (E) Kurus

09. Pesawat terbang : Kabin = …. (A) Laci : Meja (B) Gedung : Eskalator (C) Rumah : Ruangan (D) Roda : Kursi (E) Mobil : Ban

04. Demarkasi = …. (A) Pemberangkatan (B) Penyusupan (C) Penandaan (D) Perbatasan (E) Penundaan

10. Bugil : Pakaian = …. (A) Gundul : Hutan (B) Bulu : Rambut (C) Botak : Rambut (D) Mogok : Mobil (E) Lapar : Minum

05. Klandestin = …. (A) Rahasia (B) Penting (C) Persatuan (D) Kesukuan (E) Secara umum

11. Diameter : Lingkaran = … (A) Radius : Jari-jari (B) Sudut : Lancip (C) Diagonal : Segi Empat (D) Panjang : Lebar (E) Sudut : Busur

06. Pegat = …. (A) Cegat (B) Rujuk (C) Lelah (D) Segar (E) Lanjut

12. Insang : Ikan = …. (A) Kuda : Rumput (B) Manusia : Makanan (C) Kucing : Tikus (D) Harimau : Daging (E) Paru-Paru : Manusia

418

Xpedia TPA Verbal - Set 3

01. Getas = …. (A) Lunak (B) rapuh (C) Lembek (D) Lemas (E) Kuat

07. Mobil : Bensin = … (A) Sapi : Susu (B) Pesawat Terbang : Baling-Baling (C) Penyakit : Virus (D) Hewan : Makanan (E) Bakteri : DNA

02. Pilin = …. (A) Memutar (B) Mengikat (C) Menarik (D) Memangkas (E) memuntir

08. ….. berhubungan dengan BUMI, sebagai mana KUBUS berhubungan dengan …. (A) Indah - Kotak (B) Bulat - Bentuk (C) Bola - Dadu (D) Tanah - Gedung (E) Air - Rangka

03. Situ = … (A) Sungai (B) Rawa (C) Danau (D) Lembah (E) Hutan 04. Pejal = …. (A) Kosong (B) Isi (C) Berat (D) Ringan (E) Bosan Analogi 05. Kardiologi : Jantung = … (A) Sosiologi : Kebudayaan (B) Patologi : Peta (C) Farmakologi : Obat-obatan (D) Akuntasi : Perusahaan (E) Biologi : Ilmu 06. Biologi : Ilmu = …. (A) Beringin : Pohon (B) Astronomi : Galaksi (C) Teori : Praktek (D) Geologi : Lapangan (E) Percobaan : Laboratorium

09. …. berhubungan dengan BEBEK, sebagai mana MAMALIA berhubungan dengan … (A) Telur - Rumput (B) Pertanian - Peternakan (C) Burung - Susu (D) Unggas - Tupai (E) Ladang - Margasatwa 10. Berhubungan dengan PRESIDEN, sebagaimana GURU berhubungan dengan … (A) Negara - Murid (B) Rakyat - Buku (C) Keputusan - Disiplin (D) Negara - Sekolah (E) Menteri - Kepala Sekolah 11. …. berhubungan dengan TEPUNG, sebagai mana PAKAIAN berhubungan dengan …. (A) Timbangan - Penampilan (B) Mentega - Celana (C) Roti - Kain (D) Pasar - Took (E) Adonan - Ukuran 12. ….berhubungan dengan PANAS, sebagai mana ES berhubungan dengan … (A) Payung - Segar (B) Matahari - Dahaga (C) Gesekan - Dingin (D) Api - Kutub (E) Siang - Cair 419

Xpedia TPA, Verbal - Set 3

13. …berhubungan dengan JARUM, sebagai mana PETANI berhubungan dengan … (A) Dokter - Pupuk (B) Penjahit - Padi (C) Dokter - Nelayan (D) Penjahit - Cangkul (E) Perawat - Sawah 14. ...berhubungan dengan SENAPAN, sebagaimana PETENIS berhubungan dengan …. (A) Perang - Medali (B) Tentara - Raket (C) Peluru - Net (D) Tembakan - Latihan (E) Perang - Lapangan 15. ….berhubungan dengan UANG, sebagai mana GELAS berhubungan dengan … (A) Jabatan - Bening (B) Penghasilan - Kaca (C) Kekayaan - Haus (D) Investasi - Air (E) Dompet - Air

420

Xpedia TPA Verbal - Set 4

Sinonim 01. Hatta = …. (A) Atau (B) Jika (C) Karena (D) Seandainya (E) Maka

07. Altruis x …. (A) Peduli (B) Egois (C) Pemarah (D) Sedih (E) Dendam Analogi

02. Zenit = …. (A) Pertemuan (B) Puncak (C) Dasar (D) Tepi (E) Pusat

08. Doktor : disertasi = … (A) Kyai : Jemaah (B) Menteri : Keppres (C) Sarjana : Skripsi (D) Buruh : Upah (E) Guru : Murid

03. Warkat = …. (A) Kartu Pos (B) Amplop (C) Surat (D) Prangko (E) Wesel

09. Lampu : Gelap = … (A) Makanan : Kenyang (B) Makanan : Penuh (C) Makanan : Gizi (D) Makanan : Mulas (E) Makanan : Lapar

04. Lejar = …. (A) Kuat (B) Malas (C) Lemah (D) Penat (E) Terang

10. Kadal : Reptil = …. (A) Burung : Aves (B) Kuda : Omnivora (C) Ikan : Avertebrata (D) Anjing : Vertebrata (E) Lele : Amphibi

Antonim 05. Nokturnal x … (A) Diurnal (B) Krepuskular (C) Hibernasi (D) Insomnia (E) Tidur 06. Ekuilibrium x …. (A) Setara (B) Konstan (C) Dinamis (D) Kontra (E) Condong

11. Air : Haus = … (A) Angin : Panas (B) Minyak : Api (C) Makanan : Lapar (D) Gelap : Lampu (E) Rumput : Sapi 12. Kuda : Andong = … (A) Gajah : Belalai (B) Sapi : Pedati (C) Delman : kusir (D) Bajak : Kerbau (E) Pesawat : Avtur

421

Xpedia Test Potensi Akademik Set – 05 Latihan Soal Analitik

Text 1 Kolam renang “Asik Sehat” memberikan jadwal renang untuk sebuah sekolah pukul 15:00–17:00. Setiap anak hanya boleh berenang sekali seminggu. Hari kamis dan hari Minggu disediakan khusus untuk anak perempuan. Tiap hari selasa, kolam renang ditutup untuk pembersihan. Tiga anak perempuan (Bian, Cira, dan Emma) dan dua anak laki-laki (Ado dan Davin) dari sekolah tersebut mempunyai beberapa syarat: 1. Bian dan Cira tidak mau berenang berenang bersama anak laki-laki. 2. Karena alergi air kotor, Ado dan Emma tidak mau berenang di hari lain, selain hari pertama dan kedua setelah kolam dibersihkan. 3. Davin kurang pandai berenang sehingga tidak mau berenang bersama Ado. 4. Setiap hari Senin dan Jumat pukul 15:00 – 17:00, Davin harus les piano. 5. Setiap hari Sabtu dan Minggu pukul 15:00 –17:00, Cira dan Ado harus latihan bola.

1D. Cira berenang setiap hari …. (A) Rabu (B) Kamis (C) Jumat (D) Sabtu (E) Minggu

1A. Yang mungkin berenang pada hari Jumat atau Minggu adalah …. (A) Ado (B) Bian (C) Cira (D) Davin (E) Emma

2A. Karyawan yang statusnya menikah adalah …. (A) Sari, Joko, dan Ida (B) Endang, Joko, dan Putra (C) Adi, Endang, dan Putra (D) Adi, Putra, dan Joko (E) Sari, Joko, dan Adi

1B. Yang berenang pada hari Sabtu adalah …. (A) Ado (B) Bian (C) Cira (D) Davin (E) Emma

2B. Urutan karyawan dengan gaji terendah sampai dengan tertinggi adalah …. (A) Sari, Ida, Endang (B) Endang, Sari, Ida (C) Ida, Endang, Sari (D) Sari, Endang, Ida (E) Ida, Sari, Endang

1C. Yang mungkin berenang bersamaan adalah …. (A) Cira dan Davin (B) Ado dan Cira (C) Bian dan Davin (D) Emma dan Ado (E) Davin dan Emma

Text2 Diantara 6 karyawan berikut: Adi, Joko, Putra, Sari, Endang, dan Ida, ada 3 orang yang belum menikah. Berdasarkan lama bekerja, 2 orang bekerja 10 tahun dan sisanya kurang dari 10 tahun. Gaji masing-masing Rp2.000.000/bulan. Bagi yang sudah menikah ada tambahan gaji Rp200.000/bulan dan yang telah bekerja 10 tahun mendapat tambahan lagi Rp300.000/bulan. a) Joko dan Ida belum menikah dan masa kerjanya sama. b) Sari dan Joko memiliki status sama, tetapi masa kerjanya berbeda. c) Endang dan Sari masa kerjanya sama, tetapi statusnya berbeda. d) Adi dan Putra memiliki masa kerja yang sama dengan Joko.

331

Xpedia Test Potensi Akademik, Set – 05 Latihan Soal Analitik

2C. Karyawan yang mendapat gaji bulanan terbesar adalah …. (A) Joko (B) Endang (C) Sari (D) Adi (E) Putra 2D. Berikut ini pernyataan yang salah adalah …. (A) Sari sudah bekerja lebih dari 10 tahun. (B) Joko dan Ida tidak mendapat tambahan gaji. (C) Ida dan Sari sama-sama belum menikah. (D) Putra sudah menikah, tetapi masa kerjanya belum 10 tahun. (E) Adi mendapat tambahan gaji Rp300.000 setiap bulan.

332

Xpedia TPA Latihan Soal Analitik - Set 06

Text1 1A—1D Sepuluh siswa (A, B, C, D, E, F, G, H, I, dan J) duduk berurutan dalam satu baris. Posisi duduk siswa adalah sebagai berikut. (a) A duduk di kursi ketiga sebelah kanan E, dan E berada di salah satu ujung barisan. F berada di ujung barisan yang lain (b) D duduk di samping kiri F, dikursi kedua sebelah kanan C (c) D tepat berada di sebelah kanan H, di kursi keempat sebelah kanan I. (d) J duduk bersebelahan dengan A dan B, di kursi ketiga sebelah kiri G (e) G duduk di antara I dan C 1A. Yang duduk pada posisi kelima dari kiri adalah … (A) I (B) J (C) E (D) G (E) D 1B. Jika H dan A saling bertukar posisi, maka posisi sekarang A adalah bersebelahan dengan … (A) J dan D (B) I dan B (C) G dan F (D) C dan D (E) C dan F 1C. Yang duduk bersebelahan dengan D adalah … (A) F dan E (B) A dan E (C) E dan B (D) H dan F (E) G dan H 1D. Yang duduk pada posisi ujung barisan adalah ... (A) F dan G (B) F dan E (C) E dan G (D) B dan G (E) D dan A

Text 2 2A—2D Hasil tes tiga orang siswa Mita, Budi, dan Lulu pada mata pelajaran matematika, fisika, dan kimia adalah 90, 70, dan 50. Tidak ada nilai mata pelajaran yang sejenis memiliki nilai sama. Nilai fisika Lulu sama dengan nilai kimia Budi. Nilai kimia Mita sama dengan nilai fisika Budi. Nilai matematika Mita adalah 90 dan nilai fisika Budi adalah 50. 2A. Siapakah yang nilai kimianya paling tinggi? (A) Mita (B) Budi (C) Lulu (D) Mita dan Budi (E) Budi dan Lulu 2B. Siapakah yang nilai matematikanya paling rendah? (A) Mita (B) Budi (C) Lulu (D) Mita dan Budi (E) Budi dan Lulu 2C. Siapakah yang nilai rata-ratanya fisika dengan kimia paling rendah? (A) Mita (B) Budi (C) Lulu (D) Mita dan Budi (E) Budi dan Lulu 2D. Siapakah yang rata-ratanya nilai matematikanya dengan fisika paling tinggi? (A) Mita (B) Budi (C) Lulu (D) Mita dan Budi (E) Budi dan Lulu

333

Xpedia TPA Kelas 09 Latihan Analitik - Set 1

01. Berilah tanda T, F atau ? untuk pernyataan di bawah ini. T jika pernyataan BENAR F jika pernyataan SALAH ? jika pernyataan Mungkin Benar Bacaan: Deta berdiri di depan Fendi dalam sebuah barisan dan terletak di depan Noni dan Lulu. 1. 2. 3.

Noni berdiri di belakang Lulu Lulu berdiri tepat di depan Deta Fendi berdiri tepat di belakang Deta

02. Berilah tanda T, F atau ? untuk pernyataan di bawah ini. T jika pernyataan BENAR F jika pernyataan SALAH ? jika pernyataan Mungkin Benar Bacaan: Dalam sebuah group yang terdiri dari empat orang, W lahir sebelum X dan setelah Y. Z lahir sebelum W. 1. 2. 3. 4.

W lebih muda dari Y Y lebih tua dari X Y paling muda Y dan Z berumur sama

03. Saat piknik, Mika membeli minuman ringan untuk Amy, Ben, Lisa, John, dan juga untuk dirinya. Dia membeli es teh, jus mangga, Fanta merah, Pepsi, dan Sprite. Mika tidak suka minuman berkarbonasi. Amy memilih minuman antara Sprite atau Pepsi. Ben hanya suka minuman bersoda. Lisa memilih minuman yang bisa dicampur gula dan lemon. John hanya menyukai minuman yang berwarna bening. Minuman apakah yang diminum oleh amy? (A) Sprite (B) Pepsi (C) Es teh (D) Jus mangga (E) Fanta merah 334

04. Saat piknik, Mika membeli minuman ringan untuk Amy, Ben, Lisa, John, dan juga untuk dirinya. Dia membeli es teh, jus mangga, Fanta merah, Pepsi, dan Sprite. Mika tidak suka minuman berkarbonasi. Amy memilih minuman antara Sprite atau Pepsi. Ben hanya suka minuman bersoda. Lisa memilih minuman yang bisa dicampur gula dan lemon. John hanya menyukai minuman yang berwarna bening. Manakah di bawah ini yang BENAR? (A) John mendapat minuman es teh. (B) Ben mendapat minuman Sprite. (C) Ben mendapat minuman Pepsi (D) Ben mendapat minuman Fanta merah (E) Amy mendapat minuman Sprite 05. Saat piknik, Mika membeli minuman ringan untuk Amy, Ben, Lisa, John, dan juga untuk dirinya. Dia membeli es teh, jus mangga, Fanta merah, Pepsi, dan Sprite. Mika tidak suka minuman berkarbonasi. Amy memilih minuman antara Sprite atau Pepsi. Ben hanya suka minuman bersoda. Lisa memilih minuman yang bisa dicampur gula dan lemon. John hanya menyukai minuman yang berwarna bening. Siapakah yang mendapatkan jus mangga? (A) Mika (B) Amy (C) Ben (D) Lisa (E) John

Xpedia TPA Kelas 09, Latihan Analitik - Set 1

06. Tujuh orang - J, K, L, M, N, O, dan P berdiri dalam suatu antrian yang menghadap ke sebuah loket. N terletak di depan O Ada satu orang yang berdiri di antara L dan P M berada tepat di belakang J O berada di belakang K dan M Jika M dan L berturut-turut berada di urutan ke-4 dan ke-5 dalam antrian tersebut, manakah yang Pasti benar? (A) K berada di urutan ke-1 (B) N berada di urutan ke-1 (C) O berada di urutan ke-6 (D) O berada di urutan ke-7 (E) P berada di urutan ke-6 07. Tujuh orang - J, K, L, M, N, O, dan P berdiri dalam suatu antrian yang menghadap ke sebuah loket. N terletak di depan O Ada satu orang yang berdiri di antara L dan P M berada tepat di belakang J O berada di belakang K dan M Jika J berdiri tepat di belakang P, manakah yang PASTI BENAR? (A) J berada di urutan ke-5 (B) L berada di urutan ke-1 (C) L berada di urutan ke-2 (D) M berada di urutan ke-6 (E) O berada di urutan ke-7

09. Polisi baru saja mengetahui bahwa salah satu pernyataan yang diberikan oleh keempat tersangka pencurian (Susan, Jenny, Bob, dan Ron) berikut ini adalah SALAH dan ketiga lainnya BENAR. Susan : “Bob adalah pelakunya.” Bob : “Ron adalah pelakunya.” Jenny : “Ron bukan pelakunya.” Ron : “Pelakunya hanya ada satu orang.” Pelaku pencurian tersebut adalah ... (A) Jenny (B) Ron (C) Jenny dan Ron (D) Bob (E) Tidak cukup informasi untuk menjawab soal 10. Intan sekarang berusia 12 tahun. Sedangkan umur Anto dua kali lebih tua daripada umur Badu. Umur Badu tiga tahun lebih tua daripada umur Nida. Jika umur Intan lima tahun lebih tua daripada umur Sari yang setahun lebih muda daripada Anto, maka urutan mereka dari yang termuda ke yang tertua adalah ... (A) Nida, Badu, Anto, Sari, Intan (B) Badu, Nida, Sari, Intan, Anto (C) Intan, Badu, Sari, Anto, Nida (D) Nida, Badu, Sari, Anto, Intan (E) Intan, Nida, Sari, Badu, Anto

08. Ada 7 kotak peti, masing-masing diberi nomor 1 sampai 7. Buah jambu, melon, semangka, mangga, jeruk, dan duku akan dimasukkan ke dalam peti-peti tersebut dengan aturan sebagai berikut:  duku harus dimasukkan ke peti nomor 4  semangka tidak boleh diletakkan tepat disamping melon  jeruk harus diletakkan di samping mangga. Jika melon diletakkan di peti di nomor 2, maka mana yang tidak boleh dilakukan? (A) Semangka diletakkan di peti nomor 3 (B) Jeruk diletakkan di peti nomor 5 (C) Mangga diletakkan di peti nomor 7 (D) Semangka diletakkan di peti nomor 5 (E) Jambu diletakkan di peti nomor 1 335

x

Xpedia TPA Set 02 – Latihan Soal Analitik

Dalam tiap pertemuan di sebuah kelas, Aliya, Bella, Chu, Danti, Erly, Fenda, dan Haifa harus maju mengerjakan suatu soal di papan tulis. Masing-masing murid harus mengerjakan tepat satu soal secara bergantian,

01. Manakah yang MUNGKIN merupakan urutan dari yang maju pertama sampai terakhir? (A) Aliya, Haifa, Chu, Fenda, Danti, Erly, Bella (B) Chu, Fenda, Danti, Haifa, Bella, Erly Aliya (C) Aliya, Haifa, Bella, Chu, Danti, Fenda, Erly (D) Fenda, Chu, Danti, Aliya, Haifa, Bella, Erly (E) Bella, Aliya, Danti, Erly, Haifa, Fenda, Chu 02. Manakah yang TIDAK MUNGKIN yang merupakan urutan dari yang maju pertama sampai terakhir? (A) Bella, Aliya, Danti, Fenda, Chu, Haifa, Erly (B) Aliya, Haifa, Chu, Fenda, Bella, Erly, Danti (C) Danti, Haifa, Bella, Fenda, Chu, Aliya, Erly (D) Haifa, Aliya, Chu, Fenda, Danti, Erly, Bella (E) Danti, Chu, Fenda, Bella, Aliya, Haifa, Erly 03. Jika Chu dan Danti berturut-turut maju di urutan ke-4 dan ke -5, manakah yang PASTI BENAR? (A) Erly maju di urutan ke-6 (B) Erly maju di urutan ke-7 (C) Bella maju di urutan ke-6 (D) Haifa maju di urutan ke-1 336

Chu Erly Ada Haifa Rontidak Samantha Bob Ali Delilah duduk harus tepat ada harus dan duduk duduk mau diada Fenda satu maju berhadapan maju meja duduk didi orang setelah meja di harus tepat D kursi kursi yang diyang Csebelah setelah maju yang Aliya dengan letaknya kedua maju letaknya sebelun maju Ron atau Samantha disebelah 2antara sebemeja Erly dua

(E) Aliya maju di urutan ke-1 04. Jika Chu maju tepat setelah Danti, mana yang PASTI BENAR? (A) Aliya maju di urutan ke-1 (B) Bella maju di urutan ke-1 (C) Chu maju di urutan ke-4 (D) Erly maju di urutan ke-7 (E) Haifa maju di urutan ke-2 Delilah, Samantha, Jeany, Putri, Ali, Bob, Ron, dan Ed makan bersama. Mereka duduk di empat meja yang berderet menempel secara berurutan. Pada setiap meja, hanya ada 2 orang yang duduk berhadapan, sedangkan laki-laki dan perempuan tidak boleh duduk bersebelahan. Meja C terletak paling kiri. Meja B terletak di antara meja A dan D.

05. Yang duduk berhadapan di meja B adalah …. (A) Ali dan Jeany (B) Ed dan Delilah (C) Ed dan Jeany (D) Ed dan Putri (E) Ali dan Delilah 06. Anak perempuan yang duduk di meja A, B, C, dan D adalah …. (A) Putri, Jeany, Samantha, Delilah (B) Putri, Delilah, Samantha, Jeany (C) Delilah, Jeany, Samantha, Putri (D) Jeany, Putri, Samantha, Delilah (E) Delilah, Putri, Samantha, Jeany

Xpedia TPA, Set 02 – Latihan Soal Analitik

07. Pernyataan manakah yang PALING BENAR? (A) Ed duduk di sebelah Bob (B) Ron dan Jeany duduk berhadapan di meja D (C) Samantha duduk di sebelah Jeany (D) Putri duduk diantara Jeany dan Delilah (E) Ali dan putri duduk berhadapan di meja A 08. Jika Ali duduk di antara Bob dan Ed, yang duduk di hadapan Ali adalah…. (A) Samantha (B) Putri (C) Ron (D) Delilah (E) Jeany

337

Xpedia TPA Kelas 09 Set 03 – Latihan Soal Analitik

Namira, Ulul, Veda, dan Marisa duduk berdampingan menghadap ke utara. Mereka mengenakan baju berwarna putih, kuning, hijau, dan biru (tidak sesuai berurutan nama). Mereka duduk dengan ketentuan sebagai berikut :  Namira duduk tepat di sebelah barat yang berbaju kuning  Yang berbaju putih duduk tepat disebelah timur Ulul  Yang berbaju hijau dan biru selalu duduk berdekatan 01. Berikut ini manakah yang mungkin menjadi urutan duduk mereka dari barat ke timur? (A) Veda, Namira, Ulul, Marisa (B) Namira, Ulul, Veda, Marisa (C) Ulul, Namira, Veda, Marisa (D) Marisa, Veda, Namira, Ulul (E) Namira, Marisa, Ulul, Veda 02. Berikut ini manakah yang mungkin menjadi urutan warna baju mereka dari barat ke timur? (A) biru, putih, kuning, hijau (B) biru, kuning, hijau, putih (C) hijau, biru, kuning, putih (D) Hijau, putih, kuning, biru (E) Hijau, kuning, biru, putih 03. Jika Veda berdekatan dengan yang berbaju biru, maka pernyataan yang benar adalah …. (A) Namira berbaju putih (B) Ulul berbaju kuning (C) Veda berbaju hijau (D) Namira berbaju biru (E) Marisa berbaju putih 04. Jika Marisa berbaju hijau, maka pernyataan yang pasti SALAH adalah …. (A) Namira berbaju putih (B) Ulul berbaju kuning (C) Ulul berbaju biru (D) Veda berbaju kuning (E) Veda berbaju biru 338

05. Jika Ulul di sebelah timur Namira, maka pernyataan yang pasti BENAR adalah …. (A) Marisa berbaju putih (B) Marisa berbaju kuning (C) Namira berbaju biru (D) Ulul berbaju kuning (E) Namira berbaju putih Dalam pertandingan bulu tangkis Arman selalu kalah melawan Bambang, tetapi dalam cabang olah raga yang lainnya ia selalu menang bila bertanding melawan Bambang. Candra selalu menang dalam pertandingan tenis meja melawan Bambang, tetapi dalam cabang bulu tangkis ia akan kalah bila bertanding melawan Arman. Dudi adalah pemain bulu tangkis terbaik tetapi dalam cabang tenis meja dia tidak sebaik Bambang. Dalam cabang tenis meja , Edi lebih baik daripada Arman, sedangkan dalam cabang bulu tangkis ia menempati urutan dibawah Dudi. 06. Siapa pemain tenis meja terbaik diantara kelima atlet tersebut? (A) Edi (B) Dudi (C) Candra (D) Arman (E) Bambang 07. Untuk cabang olahraga tenis meja, ranking pemain terbaik yang manakah yang paling tepat dari urutan dibawah ini? (A) Bambang – Arman – Candra – Dudi – Edi (B) Arman – Bambang – Candra – Bambang – Dudi (C) Edi – Arman – Candra – Bambang – Dudi (D) Edi – Dudi – Candra – Bambang – Arman (E) Dudi – Edi – Candra – Bambang – Arman

Xpedia TPA Kelas 09, Set 03 – Latihan Soal Analitik

08. Untuk cabang olahraga bulu tangkis rangking pemain terbaik manakah yang paling tepat dari urutan dibawah ini? (A) Dudi – Edi – Arman – Bambang – Candra (B) Dudi – Edi – Bambang – Arman – Candra (C) Bambang – Arman – Dudi – Edi – Candra (D) Bambang – Dudi – Edi – Arman – Candra (E) Dudi – Edi – Candra – Bambang – Arman

339

Xpedia TPA TPA Latihan Soal Analitik - Set 04

Text 1 Pada tingkat 4 di Hogwards, ada 4 mata pelajaran yang harus diambil : Ramuan, Mantra, Transfigurasi, dan Astronomi. Harry, Hermione, Ron, dan Neville diharuskan untuk memilih mata pelajaran tersebut, Setiap murid, minimal mengambil 1 mata pelajaran dan maksimal mengambil 3 mata pelajaran.  Hermino mengambil mata pelajaran apapun yang diambil oleh Ron  Harry dan Neville memilih semua mata pelajaran yang berbeda dengan Ron  Ada 1 anak yang mengambil Astronomi  Ada 2 anak yang mengambil Ramuan  Ada 2 anak yang mengambil Transfigurasi  Ada 3 anak yang mengambil Mantra 01A. Jika HANYA mengambil 2 mata pelajaran saja, manakah dari pernyataan berikut ini yang PASTI BENAR? (A) Harry tidak mengambil Astronomi (B) Ron tidak mengambil Ramuan (C) Hermione tidak mengambil Astronomi (D) Neville tidak mengambil Ramuan (E) Neville tidak mengambil Astronomi 01B. Manakah pernyataan di antara pernyataan dibawah ini yang PASTI BENAR? (A) Harry mengambil mata pelajaran yang lebih banyak daripada Ron (B) Neville mengambil mata pelajaran yang lebih banyak daripada Hermione (C) Hermione mengambil mata pelajaran yang lebih banyak daripada Neville (D) Hermione mengambil mata pelajaran yang lebih banyak daripada Ron (E) Harry mengambil mata pelajaran yang lebih banyak daripada Neville

340

01C. Dibawah ini merupakan nama murid yang MUNGKIN mengambil mata pelajaran Ramuan dan juga Mantra, KECUALI …. (A) Hermione (B) Harry (C) Ron (D) Neville (E) Harry dan Neville 01D. Jika Ron HANYA mengambil 2 mata pelajaran saja, maka pernyataan di bawah ini pasti salah, KECUALI? … (A) Hermione mengambil Astronomi (B) Hermione mengambil tepat 2 mata pelajaran saja (C) Neville mengambil Ramuan (D) Neville mengambil Astronomi (E) Neville mengambil tepat 3 mata pelajaran. 01E. Jika Harry mengambil tepat 3 mata pelajaran, manakah di antara pernyataan di bawah ini yang PASTI BENAR? (A) Harry mengambil Ramuan. (B) Harry mengambil Transfigurasi. (C) Harry mengambil Astronomi. (D) Neville mengambil Ramuan. (E) Neville mengambil Astronomi. 01F. Manakah

pernyataan yang TIDAK MUNGKIN BENAR untuk murid yang

mengambil mata pelajaran Astronomi? (A) Dia juga mengambil Mantra, tetapi tidak mengambil Transfigurasi. (B) Dia pasti mengambil Mantra, dan juga Transfigurasi. (C) Dia juga mengambil Transfigurasi, tetapi tidak mengambil Mantra. (D) Dia juga mengambil Ramuan, tetapi tidak mengambil Transfigurasi. (E) Dia tidak mengambil Ramuan dan Transfigurasi.

Xpedia TPA, TPA Latihan Soal Analitik - Set 04

01G. Pernyataan di bawah ini pasti salah, KECUALI …. (A) Harry mengambil 3 mata pelajaran : Ramuan, Mantra, dan Transfigurasi. (B) Harry mengambil 3 mata pelajaran : Ramuan, Transfigurasi, dan Astronomi (C) Hermione mengambil 3 mata pelajaran : Ramuan, Mantra, dan Transfigurasi. (D) Hermione mengambil 3 mata pelajaran : Ramuan, Transfigurasi, dan Astronomi (E) Neville mengambil 3 mata pelajaran : Ramuan, Transfigurasi, dan Astronomi.

341

Xpedia TKPA Set 23 – Geometri Campuran SBMPTN 02

01. Banyaknya tabung pada gambar adalah … (A) (B) (C) (D) (E)

19 20 21 23 25

02. Bangun ruang dibentuk dari 3 kubus kosong, banyak prisma pejal B yang menempati seluruh ruang tersebut adalah … (A) (B) (C) (D) (E)

4 6 8 16 20

03. Banyak kubus yang diperlukan memenuhi bok tersebut adalah … (A) (B) (C) (D) (E)

untuk

9 12 21 27 81

04. Semua silinder berukuran sama. Berapakah banyak silinder yang tersusun pada gambar berikut? (A) (B) (C) (D) (E)

31 32 33 34 35

05. Jika panjang sisi persegi 2 cm, maka luas daerah yang berwarna gelap adalah … (A) (B) (C) (D) (E) 342

(16-4π) cm2 (16-2π) cm2 (16-π) cm2 (4-2π) cm2 (4-π) cm2

06. Perbandingan keliling bidang berwarna gelap dengan keliling bidang ABCD adalah … A B (A) 1 : 1 (B) 1 : 2 (C) 2 : 3 (D) 3 : 4 (E) 4 : 7 C D 07. Bangun ruang dibentuk dari 3 kubus kosong masing-masing dengan rusuk 2 cm. Jika ada bangun prisma B pejal menempati ruang tersebut, maka volume ruang yang tersisa sama dengan … (A) 12 cm3 (B) 14 cm3 (C) 16 cm3 B (D) 18 cm3 (E) 20 cm3 08. Jika panjang CD = 12 cm, AB = 2 CF, dan CD = 2 AB, maka panjang FD = …

(A) (B) (C) (D) (E)

6 cm 7 cm 8 cm 9 cm 10 cm

09. Jika potongan-potongan bola pada di gambar kubus berikut disusun, maka banyak bola yang terbentuk adalah … (A) (B) (C) (D) (E)

1 2 4 6 8

Xpedia TKPA, Set 23 – Geometri Campuran SBMPTN 02

10. Selisih luas daerah yang berwarna hitam dan abu-abu adalah …

(A) (B) (C) (D) (E)

100 115 120 130 140

11. Jika persegi panjang 7 cm × 10 cm dan segitiga ABC dengan alas 5 cm dan tingginya 8 cm saling bertindih, maka selisih luas kedua bidang yang diarsir adalah (A) 40 cm2 (B) 45 cm2 (C) 50 cm2 (D) 55 cm2 (E) 60 cm2

343

Xpedia TPA Set 22 – Geometri Campuran SBMPTN (01)

01. Jika setiap sel memiliki sisi 2 cm, maka keliling daerah yang berwarna gelap adalah …. (A) (B) (C) (D) (E)

46cm 48cm 52cm 54cm 56cm

02. Jika setiap sel memilliki sisi yang sama, maka perbandingan keliling bidang yang berwarna gelap dengan keliling bidang ABCD adalah …. (A) (B) (C) (D) (E)

5 : 10 10 : 5 9:5 5:9 9 : 10

03. Jika rusuk setiap kubus kecil adalah 1 cm, maka luas permukaan bangun di bawah ini adalah …. (A) 52cm2 (B) 54cm2 (C) 56cm2 (D) 58cm2 (E) 60cm2

04. Banyaknya kubus pada bangun di bawah ini adalah …. (A) (B) (C) (D) (E)

24 25 26 27 28

05. Jika jari-jari lingkaran pada gambar di bawah ini adalah 2cm, maka keliling persegi sama dengan ….

344

(A) (B) (C) (D) (E)

32cm 24cm 16cm 12cm 8cm

06. Perbandingan luas daerah yang berwarna gelap terhadap luas persegi dalam gambar ini adalah …. (A) (B) (C) (D) (E)

2 : 16 3 : 16 4 : 16 5 : 16 6 : 16

07. Jika CD= 5 2cm dan AC tegak lurus AB, maka panjang AB adalah ….

(A) (B) (C) (D) (E)

20cm 16cm 12cm 10cm 8cm

08. Jika luas persegi besar 196 cm2 maka keliling lingkaran adalah …. (A) (B) (C) (D) (E)

7π 8π 9π 14π 18π

09. Banyak segitiga pada gambar berikut adalah …. (A) (B) (C) (D) (E)

9 10 11 12 13

Xpedia TPA, Set 22 – Geometri Campuran SBMPTN (01)

10. Banyak persegi pada gambar di bawah ini adalah …. (A) (B) (C) (D) (E)

9 10 13 14 15

11. Luas daerah yang berwarna gelap pada gambar di bawah ini adalah …. (A) (B) (C) (D) (E)

66cm2 68cm2 70cm2 72cm2 74cm2

345

Xpedia TPA Set 21 - Sistem

01. Jika 3x  2 y  72 dan y  3x, maka x  ... (A) 6 (B) 7 (C) 8 (D) 9 (E) 10 02. Jika 4m  7n  10 dan 2m  2n  4, maka nilai dari 2m  9n  .... 03. Selisih dari dua buah bilangan adalah 4 dan hasil penjumlahannya adalah -7. Berapakah hasil perkaliaanya?.... (A) -33,0 (B) -28,0 (C) -10,25 (D) 8,25 (E) 10,5 04. Diketahui 9 p  3a  1 dan 7 p  2a  3 , manakah di antara pilihan berikut yang mengekspresikan p jika dinyatakan dalam a ?

3a  1 7 2a  3 (B) 9 (A)

(C)

7a 9 a4 (E) 2

(D)

2a 63

05. Harga satuan hotdog dan dua coke adalah Rp54.000,00. Harga tiga hotdog dan satu coke adalah Rp 87.000,00. Berapakah harga satu hotdog? (A) Rp 15.000,00. (B) Rp 19.500,00. (C) Rp 24.000,00. (D) Rp 25.000,00. (E) Rp 27.000,00.

346

3 y2 5 06. Diketahui m  dan m  , manakah di y 6 antara pilihan di bawah ini yang mengekspresikan m jika dinyatakan dalam y ? 6

(A)

18 y3

3 (B) y 18 y (C) 2

(D)

2 y

(E)

18  y 3 6y

07. Hasil penjumlahan dari dua buah angka adalah 5 dan selisihnya adalah 2. manakah di antara pilihan di bawah ini yang mungkin menjadi selisih dari bilangan kuadrat keduanya ? (A) -17 (B) -3 (C) 3 (D) 10 (E) 21 08. Jika 7 x  2 y  6 z  12 dan jika nilai x, y, dan z adalah positif, maka berapakah nilai dari 2 z ? 7x  2 y (A) (B) (C) (D) (E)

1/12 1/6 1/4 5/12 7/12

Xpedia TPA, Set 21 - Sistem

a 3 4b 1 a  dan  , maka  .... 2b 5 3c 7 c 2 17 (A) (D) 35 70

09. Jika

(B)

9 70

(C)

8 35

(E)

9 35

10. Jika a  b  s dan a  b  t , manakah di antara pilihan berikut yang mengekspresikan nilai ab dalam bentuk s dan t ?

s (D)

(A) st (B) (C)

s  t 2

s (E)

2

2

 t2 

2  t2  4

s  t 2

347

Xpedia TPA Set 20 – Soal Cerita

01. Jika perbandingan p dan q adalah 9 : 7 dan perbandingan q dan r adalah 14 : 3, maka berapakah perbandingan p dan r? (A) 1:6 (B) 27:98 (C) 2:5 (D) 5:2 (E) 6:1 02. Biaya hidup di Kota Tua naik 20% antara tahun 1970 dan 1980, dan naik 50% antara 1970 dan 1990. Berapa persenkah kenaikan biaya hidup antara tahun 1980 dan 1990? (A) 15% (B) 20% (C) 25% (D) 30% (E) 35% 03. Ketika x diambil dari 24 dan sisanya dibagi dengan x, hasilnya adalah 3. Berapakah x? (A) 4 (B) 5 (C) 6 (D) 8 (E) 12 04. Tiga tahun yang lalu, umur Nadine setengah dari umur Mary sekarang. Jika Mary sekarang umurnya empat tahun lebih tua dari Nadine, berapakah umur Mary sekarang? 05. Joanna awalnya mempunyai buku yang dua kali lebih banyak dibanding Emily. Setelah dia memberikan 5 bukunya kepada Emely, Joanna masih mempunyai 10 buku lebih banyak daripada Emily. Berapakah jumlah buku yang Joanna punya di awal? 06. Tim Basket Johnan mempunyai perbandingan menang-kalah 7:5. Jika tim tersebut bermain dengan total 48 pertandingan dan tidak ada satupun yang hasilnya seri, berapa lebihnya pertandingan yang telah dimenangkan tim Johnan dibanding kekalahannya?

348

07. Ketika Shibuya Pet Store pertama kali buka, perbandingan antara kucing dengan anjingnya adalah 4 banding 5. Semenjak itu, jumlah kucing di sana berlipat dua kalinya dan jumlah anjingnya bertambah 12 ekor. Jika perbandingan antara kucing dan anjing sekarang adalah 1:1, berapa ekor kucingkah yang dimiliki Shibuya Pet Store ketika pertama kali buka? 08. Sebuah stoples berisi kelereng yang berwarna merah, hijau, dan biru. Jika jumlah kelereng merah adalah tiga kali lebih banyak dari kelereng hijau, dan jumlah kelereng hijau adalah lima kali lebih banyak dari kelereng biru, maka manakah di antara pilihan di bawah ini yang MUNGKIN menjadi jumlah kelereng total di dalam stoples tersebut? (A) 38 (B) 39 (C) 40 (D) 41 (E) 42 09. SMA Kanakita mempunyai 504 orang murid. Seperempat muridnya adalah anak kelas 2, dan 3/7 dari anak kelas 2 tersebut adalah laki -laki. Jika sepertiga murid perempuan kelas 2 tersebut mengambil pelajaran bahasa Jerman, berapa banyak anak perempuan kelas 2 yang TIDAK mengambil pelajaran bahasa Jerman? (A) 24 (B) 36 (C) 48 (D) 72 (E) 126 10. Di dalam sebuah toples terdapat berbagai kelereng dengan motif polkadot, garis-garis, atau keduanya (polkadot-garis). Perbandingan antara kelereng motif garis-garis dengan yang tidak bergaris adalah 3:1, dan perbandingan antara kelereng polkadot dengan yang bukan polkadot adalah 2:3. Jika enam buah kelereng mempunyai motif polkadot-garis, ada berapa buah kelereng semuanya? (A) 16 (B) 18 (C) 20 (D) 36 (E) 40

Xpedia TPA Set 19 - Pertidaksamaan dan Mutlak

01. Jika 2 - 4x < 20, manakah di antara pilihan berikut yang TIDAK MUNGKIN menjadi nilai x? (A) -5 (B) -4 (C) -3 (D) -2 (E) -1 02. Jika x<0, xy>0, dan xyz>0, manakah di antara pilihan berikut yang PASTI bernilai positif ? (A) x2yz (B) xy2z (C) xyz2 (D) xy2 (E) xz2 03. Jika –1<x<0, maka manakah urutan yang BENAR untuk nilai x, x2, x3 dan x4? (A) x<x2<x3<x4 (B) x<x3<x4<x2 (C) x4<x3<x2<x (D) x3<x<x2<x4 (E) x<x2<x4<x3 04. Jika |m|>-2, maka manakah di antara pilihan berikut ini yang merepresentasikan semua kemungkinan nilai m? (A) m > -2 (B) m 2 (C) m 2 dan m 2 (D) -2<m<2 (E) Semua bilangan riil 05. Diketahui r=5w=7a dan r≠O, berapakah nilai r-w jika dinyatakan dalam a? (A) 28a

28a 5 (C) a 7a (D) 5 a (E) 7 (B)

06. Jika x adalah rata-rata dari k dan 10, dan y adalah rata-rata dari k dan 4. Berapakah rata-rata dari x dan y jika dinyatakan dalam k? (A) (B) (C) (D) (E)

k  14 4 k  14 2 k 7 2 7k 14k

07. Diketahui m=2x-5 dan n=x+7, manakah di antara pilihan berikut ini yang mengekspresikan x jika dinyatakan dalam m dan n? (A) m – n + 2 (B) m n2 (C) 2 (m  n + 12) mn2 (D) 2 m  n  12 (E) 2 08. Diketahui b=2a-4 dan c=a+2, manakah di antara pernyataan berikut ini yang mengekspresikan a jika dinyatakan dalam b dan c? I. b-c+6 bc2 II. 3 III. (A) (B) (C) (D) (E)

2c-b-8 I saja II saja I dan II I dan III I, II, dan III

09. manakah di antara pilihan berikut ini yang equivalen dengan pernyataan “Jarak dari 1 ke x lebih besar dibanding dengan jarak dari 3 ke x? I. |x-1|>|x-3| II. x>3 atau x<1 III. x>2 349

Xpedia TPA, Set 19 - Pertidaksamaan dan Mutlak

(A) (B) (C) (D) (E)

I saja II saja II dan II I dan III I, II, dan III

10. Berapakah satu-satunya bilangan bulat n yang memenuhi? 20-2n>5 dan

2n  4? 3

11. Manakah di antara pilihan berikut yang equivalen dengan pernyataan |x-2|<1? (A) x<3 (B) x<-1 (C) 1<x<3 (D) -1<x<3 (E) -3<x<-1 12. Tambahan Mutlak. (A) |x-2|≤3 (B) |x-2|≥ 3

350

Xpedia TPA Set 18 – Faktorisasi

01. Bel A dan bel B berbunyi bersamaan tepat pada jam 12:00 siang. Setelah itu, bel A berbunyi setiap 72 menit dan bel B berbunyi setiap 54 menit. Pada jam berapakah kedua bel tersebut berbunyi lagi? (A) 15:18 (B) 15:24 (C) 15:36 (D) 15:54 (E) 16:16 02. x dan y adalah semua bilangan riil. Jika xy = 7, maka  x  y    x  y   .... (A) y2 (B) 0 (C) -7 (D) -14 (E) -28 2

2

03. Perhatikan gambar, jika m ≠ n, berapakah gradien garis tersebut? (A) m ≠ n (B) m  n m2  m (C) 2 n n 1 (D) mn 1 (E) mn

05. Jika y = 3p dan p ≠ 2, maka  y  6 2 (A) 1 p2 (B) p2 (C)

3p  2 3p  2

(D)

3p  2 3p

 ....

9 p 2  36 (E) 9 p 2  36

1 1  x , maka berapakah n 2  2 jika n n dinyatakan dalam x ?

06. Jika n  (A) (B) (C) (D) (E)

x2 2 x2  1 x2 x2 + 1 x2 + 2

07. Untuk semua bilangan riil x, (x + a)(x + 1)= x2 + 6x + a, maka nilai a = ….

04. jika x > 0, maka

x 2  1  x  1  1  x  2   1   = …. x 1 x2 x3 2

(A) (B) (C) (D) (E)

y 2  36

2

08. Jika c2 + d2 = 8 dan cd = -2, maka (c + d) = …. (A) 4 (B) 6 (C) 8 (D) 9 (E) 16

(x + 1)2 (x  3x 1 3x (3x + 1)2

351

Xpedia TPA, Set 18 – Faktorisasi

09. Jika f 2  g   10 dan f  g  2 , maka berapakah nilai f  g ? (A) (B) (C) (D) (E)

-20 -12 -8 -5 0

10. Jika x > y, maka manakah pilihan di bawah ini yang menyatakan bahwa ketika selisih antara x dan y dikalikan dengan hasil penjumlahannya, hasil kalinya adalah 18? (A) (B) (C) (D) (E)

(x – y)2 = 18 (x + y)2 = 18 (x – y)(x + y) = 18 x2 – y2 = 18 x2 + y2 = 18

352

Xpedia TPA Set 17 - Bilangan

01. Jika m dan n keduanya adalah bilangan bulat ganjil manakah di antara pernyataan berikut ini yang PASTI BENAR ? I. m2  n2 adalah genap II. m2  n2 habis dibagi 4 2 III.  m  n  habis dibagi 4 (A) (B) (C) (D) (E)

Tidak ada I saja I dan II I dan III I, II, dan III

6AA 02. 8  50B4 Jika A dan B merepresentasikan digit angka yang berbeda yang akan memnuhi perkalian bersusun di atas, berapakah nilai B? (A) 2 (B) 3 (C) 5 (D) 6 (E) 8 03. Jika j adalah bilangan bulat antara 1 dan 500 yang habis dibagi 9 dan k adalah bilangan bulat antara 1 dan 500 yang habis dibagi 7, berapakah j + k ? (A) (B) (C) (D) (E)

126 127 128 129 130

04. Jika 60 adalah hasil kali antara empat buah bilangan bulat yang tiap bilangannya lebih besar dari 1, maka berapakah hasil penjumlahan dari keempat bilangan bulat tersebut? 05. Jika n adalah bilangan bulat dan 2n adalah faktor dari 1 2  3  4  5  6  7  8  9, berapakah nolai n yang terbesar yang mungkin ? (A) 5 (B) 6 (C) 7 (D) 8 (E) 9

06. J i k a x  xy a d a l a h 4 kali dari x  xy, dan xy  0, manakah di antara pilihan berikut ini yang mempunyai nilai tepat satu buah ? (A) x (D) x  xy (B) y

(E) x  xy

(C) xy 07. Jika a, b, c, d, dan e adalah bilangan cacah, dan a ( b ( c + d )+ e ) ganjil, manakah diantara pilihan berikut ini yang TIDAK BOLEH genap? (A) a (B) b (C) c (D) d (E) e 08. a  b  c  7 cd e 9 Jika setiap huruf diatas merepresentasikan bilangan bulat positif yang berbeda, maka nilai c =…. (A) 1 (B) 2 (C) 3 (D) 4 (E) 5 ABB

09. 9B7  AA7C Jika A, B, dan C adalah digit yang berbeda yang memenuhi penjumlahan bersusun di atas, maka berapakah nilai A + B + C ? (A) 4 (B) 9 (C) 14 (D) 16 (E) 17

353

Xpedia TPA, Set 17 - Bilangan

10. Jika r dan s adalah bilangan bulat positif dan s  1  2r manakah di antara pernyataan berikut ini yang PASTI BENAR? I. s ganjil II. r genap s 1 III.  Adalah bilangan bulat r r (A) I saja (B) III saja (C) I dan II (D) I dan III (E) I, II, dan III

354

Xpedia TPA Set 16 - Akar

01. Angka pangkat dua dari sebuah bilangan positif adalah dua kali dari angka itu sendiri. Angka berapakah itu? 1 1 (A) (D) 2 8 1 (B) 4

(C)

(E) 1

02. Jika a2 1  10 dan b2  1 15, berapakah nilai terbesar yang mungkin untuk a  b ? (A) -3 (B) -1 (C) 3 (D) 5 (E) 7

(A)

2 9

(B)

4 9

(C)

2 3

(A) 3m n

(D) 6 n

(B) 6m n

(E) 8 n

(C) 4 n

3 8

03. Jika 3 y 

05. Jika m dan n keduanya bernilai positif, maka manakah di antara pilihan di bawah ini yang 2m 18n ? equivalen dengan m 2

2 , maka nilai y 3  .... y 4 (D) 3

06. Sebuah persegi panjang mempunyai sisi dengan panjang a cm dan b cm. berapakah panjang diagonalnya ? (A)

a  b cm (D)

(B) a  b cm (C)

(E)

a 2  b2 cm

ab cm

a  b cm

07. Luas persegi A adalah sepuluh kali dari luas persegi B. Berapakah perbandingan antara kelilinga persegi A dengan keliling persegi B? (A) 10 : 4 (D) 4 10 :1

(E) 18

(B)

10 : 2

(C)

10 :1

08.

(E) 40 : 1

n

1 04. Jika x2  4, y 2  9, dan  x  2  y  3  0 , maka x3  y3  .... (A) -35 (B) -19 (C) 0 (D) 19 (E) 35

x Dalam gambar di atas, jika n adalah bilangan riil yang lebih besar dari 1, berapakah nilai x jika dinyatakan dalam n ? (A)

n2  1

(D) n - 1

(B)

n 1

(E) n + 1

(C)

n 1

355

Xpedia TPA, Set 16 - Akar

09. Jika

1 x  x  x, maka berapakah nilai x 2

yang mungkin ?

10.

1%  25% 100%  ?

(A) 2500 (B) 5% (C) Kita tidak bisa mengakarkan bentuk persen (D) 5 (E) 50%

356

Xpedia TPA Set 15 – Eksponen

01. Jika g = - 4,1 , maka nilai

3g 2

 3g 

2

 ....

(A) - 1 1 (B)  31 (C)  9 1 (D) 3 (E) 1 02. Jika (200)(4000)=8 × 10m,maka nilai m=…. (A) 2 (B) 3 (C) 4 (D) 5 (E) 6 03. Jika 2a2  3a  5a 2  9 , maka a  a 2  .... (A) 1 (B) 3 (C) 6 (D) 9 (E) 12 04. Jika 2  10 ,maka 2  .... (A) 20 (B) 40 (C) 80 (D) 100 (E) 200 x

2x

05. Jika 5x  y dan x bernilai positif, manakah di antara pilihan berikut yang nilainya sama dengan 5y 2 jika dinyatakan dalam x ? (A) (B) (C) (D) (E)

52 x 52 x1 252 x 1252 x 1252 x1

06. Jika p 

3n , maka bagaimana pengaruh m3

nilai p ketika n nilainya menjadi 4 kali semula dan m menjadi dua kali semula? (A) Nilai p tidak berubah (B) Nilai p menjadi setengah (C) Nilai p menjadi dua kali semula (D) Nilai p menjadi 4 kali semula (E) Nilai p menjadi 8 kali semula 07. n untuk semua bilangan rill. Nilai dari 2n  2n  .... 2n  2 (A) 2 (B) 2n (C) 2n1 n2 (D) n 1 2n (E) n 1 08. Jika m adalah bilangan bulat positif, maka manakah di antara pilihan berikut m1 ini yang equivalen 3 dengan (A) (B) (C) (D) (E)

3m  3m  3m ? 33m 33m1 9m 93 m

09. Jika 9x  25 ,maka 3x1  .... 10. Jika y adalah sebuah angka yang kurang dari 0 dan lebih dari-1, manakah di anatara pilihan di bawah ini yang mempunyai nilai bersama? (A) 100 y (B) y2 (C) y3 (D) y4 (E) y5

357

Xpedia TPA Set 14 – Persamaan

01. Jika 5d + 12 =24, maka 5d - 12 = … (A) -24 (B) -12 (C) 0 (D) 12 (E) 24 02. Angka berapakah yang jika dikurangi 7, akan bernilai 5 kali dari bilangan tersebut? 7 4 7  5 5  7 4  7 4 7

(A)  (B) (C) (D) (E)

03. Jika 2x2 - 5x = 9, maka 12x2 - 30x = … (A) -54 (B) -6 (C) 18 (D) 36 (E) 45 04. (p+2)2 = (p-5)2 Persamaan tersebut akan benar untuk nilai p= ... (A) -2 dan 5 (B) 2 dan –5 (C) 0 dan 1 (D) 1,5 saja (E) 3,5 saja 05. Hasil kali dari x dan y adalah 36. Jika nilai keduanya adalah bilangan bulat, maka berapakah nilai terkecil yang mungkin untuk x-y? (A) -37 (B) -36 (C) -35 (D) -9 -6 358

06. Grafik y = f(x) mempunyai titik (-1, 7) dan (1, 3). Manakah pernyataan di bawah ini yang mungkin menjadi f(x)? I. II. III. (A) (B) (C) (D) (E)

f(x) = |5x-2| f(x) = x2 - 2x + 4 f(x) = -2x + 5 I saja I dan II I dan III II dan III I, II, dan III

07. Jika 20  x  11, maka manakah di antara pilihan di bawah ini yang mungkin menjadi nilai x? (A) 9 saja (B) -9 dan 9 (C) 81 saja (D) 81 dan –81 (E) 961 08. m dan n untuk semua nilai positif. Jika , maka x = … 3x 2 m  nx (A)

2m  2n (B) 2 m3 3 2n (C) 3  2n 2m (D) 2m (E) 3  2 n 3 3m  2n

09. Jika 5  7  1, berapakah nilai x? x 5

2 10. Jika 2 y  y 2 maka y+5= …

5

Xpedia TPA Set 13 – Berhitung

01. Sebuah toko pizza menawarkan tiga jenis ukuran pizza. Dua jenis crust, dan delapan pilihan topping. Ada berapa pilihan pizza dengan satu topping yang dapat dipilih? (A) 13 (B) 16 (C) 24 (D) 48 (E) 60 02. 0, 2, 4, 6, 8 Ada berapa bilangan bulat berbeda antara 30 dan 70 yang hanya mengandung angka yang ada di atas? (A) 7 (B) 10 (C) 15 (D) 20 (E) 25 03. Ada berapa cara yang dapat dilakukan untuk mengurutkan empat buah lukisan berbeda dalam satu baris di dinding? (A) 12 (B) 24 (C) 36 (D) 48 (E) 64 04. Tim kasti Joni terdiri dari enam orang pemain. Ada berapa urutan pemukul yang dapat dibuat jika Joni HARUS memukul di urutan kedua, dan HARUS Zeke ATAU Ben yang memukul di urutan pertama? (A) 48 (B) 96 (C) 192 (D) 256 (E) 720 05. Di SMA Borjuis ada 36 anak yang mengambil matematika ATAU fisika ATAU keduanya, dan ada 10 anak yang mengambil matematika DAN fisika. Jika totalnya ada 31 anak di kelas matematika, ada berapa anak yang ada di kelas fisika? (A) 5

(B) (C) (D) (E)

8 11 15 21

06. Megan memberikan kartu kepada Dave sebanyak x, kepada Tina dua lebihnya dari yang diberikan kepada Dave, dan kepada Sam diberikan ke Tina. Jika dinyatakan dalam x, ada berapa kartu yang Megan berikan ke Dave, Tina dan Sam? (A) (B) (C) (D) (E)

3x + 9 3x – 1 3x – 3 x–1 x–3

07. Dari pilihan enam buah buku yang ada, empat di antaranya harus diletakkan di rak. Ada berapa kemungkinan susunan urutan berbeda yang dapat dilakukan? (A) 24 (B) 120 (C) 360 (D) 720 (E) 1296 08. Jika hari pertama pada suatu bulan adalah Hari Selasa. Hari ke-31 di bulan itu akan jatuh pada Hari …. (A) Rabu (B) Kamis (C) Jumat (D) Sabtu (E) Minggu 09. Seekor semut harus berjalan dari satu titik sudut sebuah kubus ke titik sudut lain yang posisinya bersebrangan (yang jaraknya terjauh dari posisi awal) dan kembali lagi ke posisi semula. Dia hanya boleh berjalan melalui rusuk kubus tersebut, tidak boleh menyeberangi sisinya, dia harus berjalan melalui tepat 6 rusuk dan boleh melalui rusuk yang sama sebanyak 2 kali. Ada berapa jalur berbeda yang dapat dipilih oleh semut itu? 359

Xpedia TPA, Set 13 – Berhitung

10. Ada enam bus yang digunakan untuk membawa 200 orang murid dalam sebuah acara piknik. Jika tiap bus harus memuat tidak lebih dari 40 orang dan tidak kurang dari 30 orang, maka ada berapa bus maksimum yang dapat memuat 40 orang murid? (A) 6 (B) 5 (C) 4 (D) 3 (E) 2

360

Xpedia TPA Set 12 – Mean, Median, Modus

01. Diketahui y=2x + 1 , berapakah rata-rata dari 2x , 2x , y , dan 3y , jika dinyatakan dalam x ? (A) (B) (C) (D) (E)

2x 2x+1 3x 3x+1 3x+2

02. Rata-rata dari tujuh buah bilangan bulat adalah 11. Jika tiap bilangan bulat tersebut kurang dari 20, maka berapakah nilai TERKECIL yang mungkin? (A) - 113 (B) - 77 (C) - 37 (D) - 22 (E) 0 03. Nilai tengah dari 8, 6, 1, dan p adalah 5. Berapakah nilai p ?

04. Rata-rata dari dua buah angka adalah z . Jika salah satu angka tersebut adalah x , berapakah nilai dari angka yang lain jika dinyatakan dalam x dan z ? (A) z - x (B) x - z (C) 2 z - x (D) x - 2 z xz (E) 2

05. Sebuah himpunan n buah angka mempunyai rata-rata 3k dan hasil penjumlahannya adalah 12m , di mana k dan m adalah bilangan positif. Bilangan nilai n jika dinyatakan dalam k dan m ? (A)

4m k

(D)

(B)

4k m

(E) 36 km

(C)

k 4m

m 4k

06. Rata-rata dari 5, 8, 2, dan s adalah 0. Berapakah mediannya? (A) 0 (B) 3, 5 (C) 3, 75 (D) 5 (E) 5, 5 07. Sebuah dadu dilempar sebanyak 20 kali dan hasilnya terlihat seperti table di atas. Jika rata -rata dari semua hasil lemparannya adalah a , mediannya adalah b , dan modusnya adalah c , manakah pernyataan yang PASTI BENAR ? I. a  b II. b > c III. c = 5 (A) I saja (B) II saja (C) I dan II (D) II dan III (E) I, II, dan III

361

Xpedia TPA, Set 12 – Mean, Median, Modus

09. Jika 30% larutan garam ditambahkan ke dalam 50% larutan garam, manakah di antara pilihan berikut yang mungkin menjadi konsentrasi campuran kedua larutan tersebut ? I. 40% II. 45% III. 50% (A) I saja (B) I danII (C) I dan III (D) II dan III (E) I, II, dan III

10. Terdapat sebuah himpunan A yang terdiri dari lima angka dengan median m . Jika terdapat himpunan B yang terdiri dari lima angka yang nilainya dua lebih besar dari tiap angka pada himpunan A, manakah di antara pernyataan berikut yang PASTI BENAR ? I. Median dari himpunan B lebih besar dari m II. Rata-rata dari himpunan B lebih besar dari m III. Selisih terbesar yang mungkin antara dua angka di himpunan B, lebih besar daripada selisih terbesar yang mungkin antara dua angka pada himpunan A. (A) I saja (B) I danII (C) I dan III (D) II dan III (E) I, II, dan III

362

Xpedia TPA Set 11 – Simbol

01. Untuk semua bilangan riil d, e, dan f, didefinisikan d*e*f = de+ef+df. Jika 2*3*x = 12, maka nilai x = … (A) (B) (C) (D) (E)

5 6 6 5 8 5 2 6

05. Untuk semua x≠0, didefinisikan $x

1 , berapakah nilai dari $ $5? x

06. Untuk semua bilangan riil tidak negatif x, didefinisikan x 

x . 4

Berapakah nilai

x jika x  1,5? a2 2

02. Jika b≠0, didefinisikan a○b =. b .Jika a○b =1, maka manakah pernyataan dibawah ini yang pasti BENAR? (A) a=b (B) a=|b| (C) a=-b (D) a2-b2=0 (E) a dan b keduanya positif 03. Pada jam digital, waktu yang menunjukkan pukul 07:07 disebut “double time” karena mempunyai angka jam dan menit yang sama. Contoh lainnya adalah 08:08 dan 10:10. Berapakah jarak waktu terdekat antara dua buah “double time”? (A) 11 menit (B) 49 menit (C) 60 menit (D) 61 menit (E) 101 menit 04. Dua angka disebut “angka komplementer” jika angka hasil penjumlahan dari angka 5 kebalikannya hasilnya 1. Contohnya, 5 dan 4

adalah “angka komplementer” karena 1  4 1. 5 5

Jika x dan y adalah angka “angka 2

komplementer”, dan jika x  , maka bera3 pakah y? 1 (A) -2 (D) 3 1 (B)  (E) 3 2 1 (C)  3

(A) (B) (C) (D) (E)

0,3 6 12 14 36

07. Untuk semua bilangan bulat n, didefinisikan bahwa [n] adalah hasil penjumlahan dari tiap digit n. Contohnya, [342] = 3+4+2 = 9. Jika a adalah bilangan bulat yang lebih besar dari 0, tetapi kurang dari 1000, manakah pernyataan yang PASTI BENAR? I. II. III. (A) (B) (C) (D) (E)

[10a]<[a]+1 [[a]]<20 Jika a genap, maka [a] juga genap Tidak ada jawaban yang tepat II saja I dan II II dan III I, II, dan III

08. Untuk semua bilangan pusat n. Jika didefinisikan n$=2n (untuk n genap) dan n$=n-3 (untuk n ganjil), maka nilai 13$$ =… (A) 10 (B) 13 (C) 20 (D) 23 (E) 26

363

Xpedia TPA, Set 11 – Simbol

09. Untuk semua bilangan riil t, didefinisikan 1 !t  1  . Manakah di antara pilihan di t bawah ini yang equivalen dengan !3-!2? 1 6 5 (B) ! 6 (C) !1

(A) !

(D) !

6 5

(E) !2 10. Untuk semua bilangan riil t, didefinisikan 1 !t  1  . Selain 0, berapakah nilai t yang t mungkin, untuk membuat !!t menjadi tidak terdenifisi? (A) 0 (B) 1 (C) 2 (D) 3 (E) 4

364

Xpedia TPA Set 10 – Fungsi

01. Jika f(x) = 3x2 – 6, berapakah nilai dari f(f(1))? (A) -33 (B) -17 (C) 21 (D) 27 (E) 75 02. Manakah di antara pilihan berikut yang merepresentasikan domain dari fungsi

h  x 

2x  3 ?  x  5

3 2

x2 + 8 x2 + 2x + 8 x2 + 4x + 8 x2 + 6 x2 – 4x + 8

5 < y < 10 5


54 x2 + 31 9 x2 – 4 18 x2 – 31 36 x2 – 14x +5 54 x2 – 17

3x  4 dan g  x   3  x , mana2x kah di antara pilihan berikut yang Bukan do-

04. Berapakah range dari fungsi f(x) = x2 + 1 terhadap domain –2 < x < 3? (A) (B) (C) (D) (E)



mempresentasikan f g  h  x   ?

07. Jika f  x  

03. Jika f(x) = x2 + 4, manakah yang equivalen dengan f(x + 2) ? (A) (B) (C) (D) (E)

12 36 51 140

06. f(x) = 6x – 7 g(x) = x2 –4 h(x) = 3x Manakah di antara pilihan berikut yang (A) (B) (C) (D) (E)

(A) x ≠ -5 (B) x ≠ 5 3 (C) x  2 3 (D) x  dan x  5 2 (E) Semua bilangan riil kecuali x 

(B) (C) (D) (E)

main dari f  g  x   ? (A) (B) (C) (D) (E)

-3 -1 0 1 3

08. Jika f  x   2 x  5 dan f  g  4   21 , manakah dia antara pilihan berikut yang equivalen dengan g  x  ? I. 3x – 4 II. x + 7 III. 2x (A) I saja (B) II saja (C) I dan dua (D) I dan III (E) I, II, dan III

05. f(x) = 6x – 7 g(x) = x2 – 4 h(x) = 3x Berapakah nilai dari h(g(4))? (A) 4 365

Xpedia TPA, Set 10 – Fungsi

09. JIka f  x   3x  n dimana n bernilai konstan dan f(2) = 0, maka f(0) = …. (A) -6 (B) -2 (C) 0 (D) 2 (E) 6 10. Untuk semua x > 0, fungsi g  x  didefinisi1

kan dengan persamaan g  x   x  x 2 . Berapakah nilai g 16  (A) (B) (C) (D) (E)

16 20 24 64 272

366

Xpedia TPA Set 09 – Barisan

01. Suku pertama dalam sebuah barisan adalah x. Tiap suku berikutnya adalah tiga kurangnya dari dua kali suku sebelumnya. Berapakah suku ke-5? (A) 8x - 21 (B) 8x - 15 (C) 16x - 39 (D) 16x - 45 (E) 32x - 93 02.

1 1 1 , , ,.... 8 4 2

Jika barisan tersebut adalah barisan geometri berapakah suku ke-13? (A) 27 (B) 28 (C) 29 (D) 210 (E) 211 03. Diketahui bilangan desimal 0,146285 dengan angka 146285 berulang sampai tak hingga. Dalam 500 digit pertama setelah koma, ada berapa banyak angka yang ganjil? (A) 83 (B) 166 (C) 167 (D) 168 (E) 332 04. 5, 6,5, 6, 6,5, 6, 6,6, 5, 6, 6, 6, 6, 5, …. Dalam barisan tersebut, angka 5 pertama diikuti dengan sebuah angka 6,angka 5 kedua diikuti dengan dua buah angka 6 dst. Jika barisan tersebut terus berlanjut ada berapa buah angka 6 yang terdapat antara angka 5 ke-44 dengan angka 5 ke-47? (A) 91 (B) 135 (C) 138 (D) 182 (E) 230 05. Suku pertama dalam sebuah barisan adalah5, suku berikutnya adalah 6 lebihnya dari satu suku sebelumnya. Berapakah suku ke-104?

(A) (B) (C) (D) (E)

607 613 618 619 625

06. Berapakah digit terakhir dari 337? (A) 1 (B) 3 (C) 5 (D) 7 (E) 9 07. Suku pertama sebuah barisan adalah 400. Tiap suku berikutnya adalah 20 kurangnya dari setengah suku sebelumnya. Berapakah suku keempat? 08. Jika terdapat barisan geometri yang dimulai dengan 640 dan 160 secara berurutan, maka suku ke-6 adalah ... 09. Terdapat dua suku pertama sebuah barisan. Tiap suku ganjil berikutnya bisa diperoleh dengan membagi suku sebelumnya dengan 2, contoh, suku ke-3 adalah 4 : 2 = 2 Tiap suku genap berikutnya bisa diperoleh dengan menambahkan 8 pada suku sebelumnya. Contoh, suku ke-4 adalah 2 + 8 = 10. Ada berapa sukukah yang ada sebelum suku BUKAN bilangan bulat pertama kali muncul? (A) 3 (B) 4 (C) 5 (D) 6 (E) 7 10. -2, 4, -8, …. Di atas adalah tiga suku pertama sebuah barisan. Jika tiap suku berikutnya adalah hasil kali dari dua buah suku sebelumnya, maka ada berapa bilangan negatif sampai di suku ke-90? (A) 16 (B) 30 (C) 45 (D) 60 (E) 66 367

Xpedia TPA 08 - Latihan Soal

05. (A)

3p  q  r pqr

(B)

3p  q  r 3

(C)

3p  q  r pqr

(D)

3 pqr 3p  q  r

igTa buah engkal cat yang berdiametr 10 cm dikat oleh ital esperti gambr di ah.wb

erB apakh pagnja tali nimal yang diperlukan? (B) (C) (D) (E)

(E) prq 02.

03.

Sarah mepunyai 7 apel, 5 mang , 3 jeruk. anyB aknya ahntmb jerku yang harus Sarah beli ehings a peringbad banyaknya eslurk uhan buah dan nyba aknya erj uk yang dia milk adimenj 3:1 ahld . (A) 6 (B) 5 (C) 4 (D) 3 (E) 2 Pe r h a t i k a n

gambar

04.

x- y)2= 12 dan

xy = 1, mak

x2+ y2= . (B) (C) (D) (E) 07. bawah

Kami ebr main tenis di lapang yang am,s dan ahri in kami beru.mte erB apa ming u lagi akmi abis ber main enist berams lagi? (B) (C) (D) (E)

15° 20° 25° 30° 08.

36 cm2 42 cm2 6 cm2 72 cm2

(A01. ) e B 4813,06° cm2 rapakah-rata rata dari p, q, r?

mak rata

- a ms

12 24 36 64

Jika nila rata

(B) (C) (D) (E)

3

13 12 1 10

Aku pergi erb main isten iapt 2 ingm u ekasli, om T pergi bermain enist tiap 4 ming u ekasli, and yathC pergi ber main tenis etiaps 6 ming u sekali.

!

entuka T luas daerah gunba di ahwb in! (B) (C) (D) (E)

Jika (

BCA.

entuka T besar (B) (C) (D) (E)

di

06.

41, cm 51,4 cm 61,4 cm 71,4 cm

- rata dari 4, ,5 6, 5,0 5,6 6,0 6,4

- rata dari 2, 3,

xdan

xdan yadlh 5, y DGOK«

Xpedia TPA 08 - Latihan Soal

09. Untuk mengisi 3/7 bagian sebuah kolam renang diperlukan waktu 6 jam. Berapakah waktu yang diperlukan untuk mengisi sisanya, jika pengisian kolam renang dilakukan dengan laju yang sama? (A) 2,5 jam (B) 5 jam (C) 7 jam (D) 8 jam (E) 14 jam 10. Total berat badan Jin dan Jun adalah 290 pounds. Total berat badan Jun dan Jen adalah 230 pounds. Jika mereka bertiga ditimbang, berat totalnya adalah 400 pounds. Berapa pounds. Berapa pounds -kah berat Jun? (A) 110 lbs (B) 120 lbs (C) 130 lbs (D) 140 lbs (E) 170 lbs

368

Xpedia TPA Set 07 – Berhitung Cepat

1 01. 0, 01  100  ....

(A) (B) (C) (D) (E)

0,02 0,001 0,001 0.01 0,2

02. 225,6 x 4,51 = …. (A) 956,40 (B) 1.017,456 (C) 1.228,66 (D) 1.356,78 (E) 1.458,97 03. 8 adalah 5 persen dari …. (A) 760 (B) 160 (C) 40 (D) 16 (E) 1,6 04. Jika

(A) (B) (C) (D) (E)

5 4 a a  b, sehingg nilai adalah …. 6 9 b

10 7 7 10 8 15 15 8 7 15

06. Dua puluh lima persen dari 60 adalah …. (A) 15 (B) 25 (C) 35 (D) 45 (E) 55 07. 105 + 105 + 105 + 105 + 105 = n · 55 Berapakah nilai n? (A) 2 (B) 5 (C) 24 (D) 100 (E) 160 08. Jika 2 < x < 5, dan 4 < y < 7, dan w = x + y, maka nilai w berada di antara nilai …. (A) 2 dan 7 (B) 4 dan 7 (C) 6 dan 12 (D) 5 dan 7 (E) 6 dan 9 09. Bentuk sederhana dari (2-3 + 5-1)-2 adalah …. (A) 169/1600 (B) 1600/169 (C) 12/13 (D) 1/169 (E) 169 10. Nilai 180 merupakan 45% dari bilangan …. (A) 18 (B) 24 (C) 81 (D) 300 (E) 400

05. Jika 213xy + 4 = 421, maka 639xy = …. (A) 1275 (B) 1263 (C) 1251 (D) 1242 (E) 1236 369

Xpedia TPA Set 06 – Soal Campuran 04

01. Working alone, pump A can empty a pool in 3 hours. Working alone, pump B can empty the same pool in 2 hours. Working together, how many minutes will it take pump A and pump B to empty the pool? (A) 72 (B) 75 (C) 84 (D) 96 (E) 108 02. If the area of a circle is 64π, then the circumference of the circle is .... (A) 8π (B) 16π (C) 32π (D) 64π (E) 128π 03. What is the sum of the five consecutive EVEN numbers whose everage (arithmetic mean) is 12? (A) 48 (B) 60 (C) 72 (D) 84 (E) 96 04. Gheri is n years old.. Carl is 6 years younger than Gheri and 2 years older than Jean. What is the sum of the ages of all three? (A) 3n + 16 (B) 3n + 4 (C) 3n - 4 (D) 3n - 8 (E) 3n - 14

05. The integers A, B, and C are consecutive and A < B < C. If A2 = C, which of the following could be the value of A? I. -1 II. 0 III. 2 (A) I only (B) III only (C) I & II only (D) I & III only (E) I, II, III 06. A subway car passes an average of 3 stations every 10 minutes. At this rate, how many stations will it pass in one hour? (A) 7 (B) 9 (C) 17 (D) 18 (E) 19 07. It takes 6 people 6 day to do a job. How many days would it take 2 people working at the same rate to do the job? (A) 3 (B) 6 (C) 12 (D) 16 (E) 18 08. During October, a store had sales of $30,000. If this was a 20 percent increase over the September sales, what were the September sales? (A) (B) (C) (D) (E)

22,500 24,000 25,000 27,000 28,000

Xpedia TPA, Set 06 – Soal Campuran 04

09. Which of the following is not a factor of 168? (A) 21 (B) 24 (C) 28 (D) 32 (E) 42 10. If the diameter of a circle increases by 50 percent, by what percent will the area of the circle increase? (A) (B) (C) (D) (E)

25 50 100 125 225

370

Xpedia TPA Set 05 – Soal Campuran (03)

01. Jika 2x = 10 dan 3y = 12, maka 4x + 6y =

02. Jika 5 kurangnya dari 4 kali sebuah bilangan menghasilkan angka 25, maka berapakah bilangan awal? 03. Rata-rata dari 3 buah bilangan adalah 5. Jika salah satu angka adalah 4. Berapakah hasil penjumlahan dari dua angka lainnya? 04. Untuk semua bilangan bulat m yang lebih dari 1. Didefinisikan bahwa ((m)) adalah hasil penjumlahan bilangan bulat dari 1 sampai m. Misal, ((4)) = 1 + 2 + 3 + 4 = 10. Berapakah nilai dari ((8)) - ((5))? 05.

08. Jika x = -1 adalah solusi dari persamaan x2 = 4x + c, dimana nilai c konstan. Berapakah nilai x lainnya yang memenuhi persamaan tersebut? 09. Dalam sebuah pertandingan bola yang diikuti oleh 5 tim, masing-masing tim bertanding sebanyak 3 kali melawan setiap tim lainnya dalam 1 season. Jadi, ada berapa pertandingankah totalnya di 1 season tersebut? 10. Keran air A jika dinyalakan sendirian, akan mengisi penuh bak mandi selama 3 jam. Keran air B jika dinyalakan sendirian akan mengisi penuh bak mandi selama 2 jam. Jika pada awalnya bak mandi tersebut kosong dan keran air A dinyalakan duluan selama 1 jam, baru kemudian keran B dinyalakan sampai bak mandi penuh, maka berapa menitkah keran air B menyala sampai akhirnya bak mandi tersebut penuh?

A, B, C, D Ada berapa kemungkinan yang berbeda jika kita ingin membuat 3 kombinasi huruf, tetapi tidak boleh ada huruf yang berulang. (kombinasi huruf ABC berbeda dengan (BCA)

06. Setiap suku dalam sebuah barisan berikut merupakan 1 kurangnya dari 3 kali suku sebelumnya. Jika suku ke-4 adalah 95, maka suku pertama barisan ini adalah … 07. Rata-rata dari 4 buah bilangan bulat genap berbeda adalah 50. Jika tidak ada satu bilanganpun yang lebih besar dari 60, maka angka terkecil yang paling mungkin adalah …

371

Xpedia TPA Set 04 – Konsep Pembagian

01. Ketika 8 dibagi dengan 12, sisanya adalah …. (A) (B) (C) (D) (E)

1 3 2 3 4 8 12

0 a b sisa 3 02. Jika a dan b positif, manakah pernyataan berikut ini yang PASTI BENAR? (A) (B) (C) (D) (E)

a=3 a = 3b b=0 b=3 b = 3a

03. 1, 3, 5, 7, 9, 1, 3, 5, 7, 9, 1, …. Jika barisan tersebut berlanjut terus dengan pola berulang yang sama, suku ke 103 adalah …. (A) 1 (B) 3 (C) 5 (D) 7 (E) 9 04. Jika p ganjil dan q genap, manakah di antara pernyataan berikut ini yang PASTI bernilai ganjil? I.

p2  q2

II.

p2 q2

III. (A) (B) (C) (D) (E)

p2q2

Tidak ada I saja I dan II I dan II I, II, dan III

05. Manakah yang merupakan Counterexample dari pernyataan: Semua bilangan prima adalah ganjil. (A) 2 (B) 3 (C) 9 (D) 11 (E) 12 06. Ketika sebuah bilangan bulat n dibagi dengan 10, sisanya adalah 7. Berapakah sisa dari bilangan n tersebut ketika dibagi 5? 07. m dan n adalah bilangan bulat positif. Jika m dibagi dengan n hasilnya 7 sisa 4. Manakah di di antara pilihan berikut yang mengekspresikan m dalam n? (A) 4n – 7 (B) 7n – 4 (C) 4n + 7 n (D)  4 7 (E) 7n + 4

k k 08. Jika 7 dan 12 keduanya adalah bilangan bulat positif, maka manakah di antara pilihan berikut ini yang juga merupakan bilangan bulat? k (A) 42 k (B) 24 k (C) 19 k (D) 15 k (E) 10

Xpedia TPA, Set 04

Konsep Pembagian

09. Jika nilai dan adalah bilangan bulat posia tif dan  2,5 , manakah pernyataan yang b PASTI BENAR? I. ( + ) bernilai ganjil II. ( + ) adalah kelipatan 7 5b III. adalah bilangan bulat a (A) (B) (C) (D) (E)

II saja I dan III I dan II II dan III I, II, dan III

10. Jika himpunan dan adalah bilangan bulat yang berurutan, manakah di antara pernyataan di bawah ini yang PASTI BENAR? I. Himpunan ini mempunyai 3 bilangan ganjil II. Himpunan ini mempunyai angka yang habis dibagi 5 III. +1 bernilai ganjil (A) I saja (B) II saja (C) I dan II (D) II dan III (E) I, II, dan III

372

Xpedia TPA Set 03 - Bilangan Negatif

01. b untuk semua bilangan rill, nilai dari - ( - b b -b - b )=…. (A) - 4b (B) 4b (C) b4 (D) - b4 (E) 4b4 02. Jika k=(m-1)(m-2)(m-3) , maka manakah diantara nilai m di bawah ini yang membuat nilai k menjadi lebih besar dari nol? (A) -2,47 (B) -1,47 (C) 0,47 (D) 1,47 (E) 2,47 03. w untuk sebuah bilangan rill, maka nilai dari – w2 - (- w )2 = …. (A) -w4 (B) -2w2 (C) 0 (D) 2w2 (E) w4 m  1 , manakah pernyataan di bawah n ini yang pasti benar ? I. m > n II. mn > 0 III. m > 1 (A) I saja (B) II saja (C) III saja (D) I dan II (E) I dan III

04. Jika

05. Jika x=-y dan x≠0 , maka manakah pernyataan di bawah ini yang pasti benar? I. x2 y3 < 0 II. (x + y)2 = 0 x III. <0 y (A) (B) (C) (D) (E)

III saja I dan II II dan III I dan III I,II, dan III

06. Jika mn4 p5>0 dan m < 0, maka manakah dari nilai berikut ini yang harus negatif ? (A) mn (B) m2 (C) p (D) mp (E) np 07. Jika 0 < a < b < c < d < e < f dan (a-b)(c-d) (e-f)(x)=(b-a)(d-c)(f-e) maka nilai x adalah ... (A) -4 (B) -3 (C) -2 (D) -1 (E) 0 08. Jika hasilnya penjumlahan bilangan bulat dari 15 sampai 50, inklusif, mempunyai hasil yang sama dengan penjumlahan bilangan bulat dari n sampai 50, inklusif, dan n<15 , maka n=…. (A) (B) (C) (D) (E)

-50 -49 -35 -15 -14

Xpedia TPA, Set 03 - Bilangan Negatif

09. Jika –2x < - 7 dan y < 2,5 , maka manakah di antara pernyataan berikut ini yang PASTI BENAR? I. xy > 0 II. x - y >0 III.x > 3 (A) II saja (B) I dan II (C) II, dan III (D) I dan III (E) I, II, dan III 10. Sebuah barisan diawali dengan angka-angka –1, 1, 1, … dst., dan tiap suku kelanjutannya adalah hasil kali dari tiga buah bilangan sebelumnya. ada berapa banyak dari 57 suku pertama pada barisan ini, yang bernilai negatif ? (A) 19 (B) 20 (C) 28 (D) 29 (E) 30

373

Xpedia TPA Set 02 - Latihan Soal

01. Jika 4 botol jus apel dicampur dengan 5 botol jus tomat dan 3 botol jus anggur, maka ada berapa bagian jus apel pada campuran tersebut? 1 (A) 6 1 (B) 4 1 (C) 3 4 (D) 9 2 (E) 3

04. Dalam sebuah peta berskala, jarak antara dua kota yang terpisah sejauh x km digambarkan terpisah sejauh 4 cm. Jika ada dua kota lainnya yang berjarak x + 2 km, berapa cm jarak mereka di dalam peta yang sama?

02. Lima perdelapan dari kelas A adalah anak laki-laki dan dua pertiga dari anak perempuannya berambut tidak gelap. Berapakah nilai pecahan untuk anak perempuan berambut gelap di kelas A tersebut? 1 (A) 24 1 (B) 10 1 (C) 8 1 (D) 4 1 (E) 3

05. Jika 3600 buah topi baseball di distribusikan ke 4 toko dengan perbandingan 1:2:3:4, berapa buah topikah maksimum yang dapat oleh salah satu toko tersebut?

03. Di dalam kotak A terdapat 6 buah kelereng berwarna merah dan tidak ada kelereng berwarna hijau. Di dalam kotak B terdapat dua buah kelereng berwarna merah dan empat buah kelereng berwarna hijau. Berapa banyak kelereng hijau yang harus dipindahkan dari kotak B ke kotak A supaya perbandingan antara kelereng hijau dengan merah di kotak A sama dengan kotak B? (A) 0 (B) 1 (C) 2 (D) 3 (E) 4 374

(A) 4(x + 2) (B) 6 4x (C) x2 4( x  2) (D) x 6 (E) x

(A) (B) (C) (D) (E)

360 720 1080 1440 14400

06. Ketika 14 diambil dari 6 kali sebuah bilangan, sisanya adalah 40. Berapakah setengah dari bilangan tersebut? 07. Jika 2  6  1  2  1  1 , maka nilai 3 7 6 3 6 x adalah ... 08. Jika x adalah hasil kali dari 0,03 dan 0,2, maka nilai x akan equivalen dengan 6 berbanding … 2 galon bensin 5 untuk berkendara sejauh 8 km. Dengan perbandingan yang sama, berapa km jarak yang bisa di tempuh jika menggunakan 5 galon bensin?

09. Motor Jack menggunakan

x

Xpedia TPA , Set 02 - Latihan Soal

10. Jika n>1 dan

nx  1, maka nilai x m x

adalah ... m n 1 m (B) n 1 m 1 (C) n m 1 (D) n 1 m 1 (E) n 1

(A)

11. Berapakah nilai y untuk

1  3? y

(A) -3 (B)  (C)

1 3

1 3

(D) 3 (E) 6 12. Manakah nilai yang paling besar? 2 1 (A) 3 2 (B) 1  3 2 (C) 1: 3 2 2 (D) : 3 3 (E) 2 :1 3

375

Xpedia TPA Set 01 ² Soal Campuran (01)

01. Manakah di antara pilihan di bawah ini yang 1 nilainya BUKAN merupakan 3 dari sebuah bilangan bulat? 1 (A) 3 (B) 1 5 (C) 2 16 (D) 2 (E) 10

02. Manakah di antara pilihan di bawah ini yang merupakan hasil kali dari dua buah bilangan bulat genap yang berurutan? (A) 22 (B) 36 (C) 48 (D) 60 (E) 72 03.

k  3  8 , berapakah nilai k? (A) 11 (B) 64 (C) 67 (D) 121 (E) 132

04. x untuk semua bilangan riil. Diketahui {x adalah bilangan bulat terkecil yang lebih dari x. Maka nilai {-5, 6 = …. (A) -6 (B) -5, 7 (C) -5, 5 (D) -5 (E) 1 05. Jika empat digit bilangan bulat positif terkecil tanpa angka yang berulang di ambil dari empat digit bilangan bulat positif terbesar tanpa angka yang berulang, maka hasilnya adalah …. 376

(A) (B) (C) (D) (E)

8642 1111 8853 2468 8888

06. Jika x > 1, manakah pernyataan di bawah ini yang nilainya akan naik, jika nilai x naik?

1 I. x 2 II.

x

1 III. 10  x (A) II saja (B) I dan II (C) I, II, dan III (D) III saja (E) II dan III

07. Selisih dari dua buah bilangan bulat adalah 4 dan hasil penjumlahannya adalah 14. Berapakah hasil kalinya? (A) 18 (B) 24 (C) 36 (D) 45 (E) 56 08. x dan y untuk semua bilangan riil, maka 4 x  x   3xy  2 x   .... 2 (A) 12 x y  x  2 x 

2 (B) 2 x  2  3 y 

(C) xy   x  2 (D) 2 x  2  3 y  2 (E) 4 x  x  3 y 

Xpedia TPA, Set 01 ² Soal Campuran (01)

09. Jika 3x2  2 x  40 , maka 15x2  10 x  .... (A) 120 (B) 200 (C) 280 (D) 570 (E) 578 10. Bentuk 2  x  2   x  x  2  akan equivalen dengan bentuk yang mana saja? I. x 2  4 II.  x  2  x  2  III. x2  4 x  4 (A) Tidak ada yang benar (B) I dan II (C) I, II, dan III (D) II saja (E) II dan III 11. Untuk semua x bilangan riil, 1 – (1 – (1 – x) – 1= …. (A) (B) (C) (D) (E)

x x–1 x–2 1–x 2–x

12. (1 – ( – (1– 3))) – (1– (1– (1– 2))) = …. (A) -3 (B) -2 (C) -1 (D) 2 (E) 3

377

Xpedia TPA Logika - Set 1

01. Jika hujan deras, maka jalanan basah. Jika jalanan basah, anak-anak tidak bisa bermain sepeda. Simpulan yang tepat adalah … (A) Jika hujan deras, anak-anak tidak bisa bermain sepeda (B) Jika hujan deras, anak-anak bisa bermain sepeda (C) Jika hujan tidak deras, jalanan tidak basah (D) Jika hujan tidak deras, anak-anak bisa bermain sepeda (E) Jika hujan tidak deras, jalanan tidak basah, dan anak-anak pasti bisa bermain sepeda 02. Jika kita berada di Solo, maka kita berada di Pulau Jawa. Pernyataan yang mempunyai arti sama dengan kalimat di atas adalah … (A) Jika kita berada di Pulau Jawa, maka kita berada di Solo (B) Jika kita tidak berada di Pulau Jawa, maka kita tidak berada di Solo (C) Jika kita tidak berada di Solo, maka kita tidak berada di Pulau Jawa (D) Jika kita berada di Pulau Jawa kita tidak mungkin berada di Solo (E) Jika kita berada di kota selain Solo, maka kita juga berada di Pulau Jawa 03. Jika ◊ adalah Δ, maka ◊ adalah o. ‘ Jika ◊ adalah * maka ◊ adalah o. ◊ adalah Δ atau ◊ adalah * Simpulan yang tepat adalah … (A) ◊ adalah * (B) Tidak ada ◊ yang (C) ◊ bukan Δ dan bukan * (D) ◊ adalah o (E) ◊ tidak dapat ditentukan

378

04. Manakah di antara pernyataan berikut yang TIDAK VALID? (A) Jika Pussy adalah kucing, maka Pussy adalah mamalia. Pussy adalah kucing. Jadi, pasti Pussy adalah mamalia (B) Jika kita berada di Perth, maka kita berada di Australia. Jadi, kita tidak berada di Perth jika kita tidak berada di Australia (C) Jika sesuatu itu TV, maka sesuatu itu adalah ikan. Mobil bukan ikan. Jadi, mobil bukan TV. (D) Marina tidak akan bisa menjadi pemain sumo. Sekarang ini, hanya ada laki-laki saya yang berprofesi sebagai pemain sumo dan Marina jelas bukan laki-laki (E) X adalah Z karena semua X adalah Y semua Y adalah Z. 05. Semua semut adalah serangga. Semua sehingga adalah hewan. Kesimpulan yang tepat dari premis di atas adalah … (A) Semua hewan adalah serangga (B) Semua semut adalah hewan (C) Semua serangga adalah semut (D) Ada hewan yang semut (E) Ada semut yang hewan 06. Ada sayur yang hijau Semua yang hijau itu bergizi Kesimpulan yang tepat adalah … (A) Sayur dan sesuatu yang hijau itu bergizi (B) Ada sayur yang tidak bergizi (C) Semua sayur itu bergizi (D) Ada sesuatu yang hijau yang bergizi (E) Ada sesuatu yang hijau yang bukan sayur

Xpedia TPA, Logika - Set 1

07. Semua artis itu sexy. Ada orang sexy yang jarang mandi. Manakah pernyataan yang paling tepat berdasarkan premis di atas? (A) Ada orang yang jarang mandi dan sexy (B) Semua yang jarang mandi, sexy (C) Ada orang yang sexy yang tidak jarang mandi (D) Ada artis yang jarang mandi (E) Ada orang sexy yang adalah artis 08. Ada penari yang kaya. Semua yang kaya, suka naik bajaj. Manakah pernyataan yang PASTI BENAR? (A) Semua yang kaya adalah penari (B) Semua penari, kaya (C) Ada penari yang suka naik bajaj (D) Semua penari suka naik bajaj (E) Ada penari yang kaya yang tidak suka naik bajaj 09. Semua A adalah B. Ada B yang bukan C. Tentukan mana yang PASTI BENAR? (A) Ada B yang C (B) Tidak ada B yang A (C) Ada A yang bukan C (D) Tidak ada A yang bukan C (E) Tidak ada a yang bukan B

11. Tak ada cinta yang tak buta Manakah pernyataan yang PASTI BENAR? (A) Semua yang buta, bukan cinta (B) Semua cinta itu buta (C) Ada yang bukan cinta yang tak buta (D) Ada yang bukan cinta yang buta (E) Semua yang buta adalah cinta 12. a) Semua bebek berkaki dua b) Donal adalah bebek Kesimpulan Dengan demikian, Donal berkaki dua (A) Valid (B) Invalid 13. a) Semua ahli logika adalah pemikir handal b) Semua pengacara adalah pemikir handal Kesimpulan Dengan demikian, semua ahli logika adalah pengacara (A) Valid (B) Invalid

10. Semua bapak-bapak berkumis. Ada orang berkumis yang suka makan telur. Manakah yang PASTI BENAR? (A) Semua yang suka makan telur berkumis (B) Ada orang berkumis yang adalah bapakbapak (C) Ada bapak-bapak yang suka makan telur (D) Ada orang yang berkumis yang tidak suka makan telur (E) Ada yang suka makan telur dan dia adalah orang yang berkumis

379

Xpedia TPA Logika - Set 2

01. a) Di mana ada kemiskinan, di sana ada kriminal b) Terdapat kemiskinan di Jakarta Kesimpulan Dengan demikian, pasti terdapat kriminal di Jakarta. (A) Valid (B) Invalid 02. a) Di mana ada kemiskinan, di sana ada kriminal. b) Terdapat kriminal di Jakarta. Kesimpulan Dengan demikian, pasti terdapat kemiskinan di Jakarta (A) Valid (B) Invalid 03. a) Pria hanya berpikir tentang sex b) Seorang pria bernama Yoga sedang berpikir Kesimpulan Dengan demikian, Yoga sedang berpikir tentang sex (A) Valid (B) Invalid 04. a) Semua bebek menggonggong b) Donal adalah bebek Kesimpulan Dengan demikian, Donal menggonggong (A) Valid (B) Invalid 05. a) Ketika saya lapar, saya dapat memakan seekor kuda b) Saya lapar sekarang Kesimpulan (A) Valid (B) Invalid

380

06. a) Biru dan kuning membuat hijau b) Biru dan merah membuat cokelat Kesimpulan Dengan demikian, biru dan putih membuat biru langit (A) Valid (B) Invalid 07. a) Obama tinggal di Beijing b) Beijing terdapat di Thailand Kesimpulan Dengan demikian, Obama tinggal di Thailand (A) Valid (B) Invalid 08. a) Pria berasal dari Mars B) Wanita berasal dari Venus Kesimpulan Dengan demikian, pria dan wanita tidak akan bisa mengerti satu sama lain (A) Valid (B) Invalid 09. Carilah pernyataan yang memiliki arti sama dengan pernyataan di bawah ini! “Semua kadal adalah reptil” (A) Ada kadal yang reptil (B) Jika ada sesuatu yang berupa kadal, dia pasti adalah reptil (C) Ada reptil yang kadal (D) Tidak semua reptil adalah kadal (E) Jika ada sekelompok reptil, mungkin di dalamnya ada kadal 10. Jika adik jajan, maka Ibu marah. Manakah pernyataan berikut yang PASTI BENAR? (A) Adik tidak jajan, jadi Ibu tidak marah (B) Adik tidak jajan, tetapi Ibu tetap marah (C) Jika Ibu tidak marah, maka berarti adik tidak jajan (D) Ibu tidak marah walaupun adik jajan (E) Jika Ibu marah, maka adik jajan

Xpedia TPA, Logika - Set 2

11. Jika Ucil makan donat, maka Ucil akan gendut. Jika Ucil gendut, maka Ucil akan sesak napas. Manakah kesimpulan dari premis di atas yang benar? (A) Kalau Ucil tidak makan donat, Ucil akan sesak napas (B) Jika Ucil gendut, berarti Ucil makan donat (C) Jika Ucil tidak gendut, maka Ucil tidak akan sesak napas (D) Ucil akan sesak napas jika Ucil makan donat (E) Ucil tidak akan gendut dan sesak napas jika Ucil makan donat

14. Jika ada orang yang suka pergi ke puncak, maka orang itu pasti suka udara dingin. Manakah pernyataan di bawah ini yang BENAR sesuai dengan premis di atas? (A) Alberto tidak suka udara dingin, jadi dia tidak suka pergi ke puncak (B) Alberto tidak suka pergi ke puncak, maka dia tidak suka udara dingin (C) Alberto tidak suka pergi ke puncak, tetapi dia suka udara dingin (D) Alberto tidak suka udara dingin, tetapi dia mau-mau saja kalau diajak pergi ke puncak (E) Alberto tidak suka ke puncak karena dia lebih suka pantai yang udaranya panas

12. Di dinding sebuah bioskop, terdapat tulisan “TIDAK BOLEH MEMBAWA KAMERA DAN ANJING” Joni yang melihat tulisan tersebut berkata, “Wah, berarti kalo bawa kamera doang boleh dong, kan itu tulisannya gak boleh bawa kamera dan anjing” Bagaimana pernyataan Si Joni? (A) Valid (B) Tidak Valid (C) Mungkin Valid (D) Mungkin tidak valid (E) Tidak dapat ditentukan ke-valid-annya 13. Jika kita makan rumput, maka kita adalah sapi. Jadi, kita adalah sapi kalau kita makan rumput. Bagaimanakah pernyataan di atas? (A) Valid (B) Tidak valid (C) Mungkin valid (D) Mungkin tidak valid (E) Tidak dapat ditemukan ke-valid-annya

381

Xpedia TPA Set 05 - Barisan Angka

01. 1/16, 1/8, 1/4, 1/2, …., …. (A) 1, 2 (D) 1/2, 1 (B) 1/2, 0 (E) 1, 1/2 (C) 1/4, 1 02. 8, 6, 19, 4, 30, 2, …, 0 (A) 41 (B) 13 (C) 30 (D) 34 (E) 14 03. 41, 43, 35, 46, 29, 49, …, ... (A) 35,46 (D) 23, 52 (B) 35, 52 (E) 52, 35 (C) 23, 36 04. 5, 10, 15, 12, 24, 29, 26, ... (A) 23 (B) 31 (C) 52 (D) 33 (E) 42 05. 302, 305, 309, …, 320, 327, 335 (A) 313 (D) 317 (B) 314 (E) 318 (C) 316 06. 5, 20, 11, 18, 17, 16, 23, 14, …, … (A) 16, 25 (D) 33, 17 (B) 29, 12 (E) 28, 22 (C) 24, 19 07. 2, 5, 11, 23, 47, … (A) 67 (D) 95 (B) 88 (E) 77 (C) 109 08. 1, 2, 3, 4, 6, 0, 7, 18, -3, …, …, ... (A) 10, 54, 0 (D) 54, -6, 10 (B) 10, -6, 54 (E) 54, 10, -6 (C) 10, 54, -6 382

09. 11, 14, 21, 24, ... (A) 31 (B) 34 (C) 37 (D) 40 (E) 39 10. 5, 9, 17, 33, 65, ... (A) 117 (B) 123 (C) 128 (D) 129 (E) 164

Xpedia TKPA Set 04 - Barisan Angka

01. 7, 8, 12, 13, 17, 18, …., …. (A) 19, 23 (B) 20, 24 (C) 22, 23 (D) 23, 27 (E) 22, 26

07. 1, 11, 121 ,1331, …. (A) 12451 (B) 14461 (C) 14451 (D) 14641 (E) 12621

02. 2, 5, 11, 20, 32, …. (A) 46 (B) 47 (C) 48 (D) 49 (E) 51

08. 7800, 1950, 390, …. (A) 78 (B) 65 (C) 45 (D) 87 (E) 77

03. 361, 19, 324, 18, 289, 17, …., …. (A) 256, 16 (B) 225, 15 (C) 265, 15 (D) 15, 265 (E) 16, 256

09. 1, -2, -6, 12, 36, …., …. (A) -48, -124 (B) 48, 144 (C) 72, 216 (D) -72, -216 (E) -58, 124

04. 6, 4, 2, 0, -2, -4, …. (A) -6 (B) -5 (C) 6 (D) 4 (E) 5

10. 4, 40, 6, 38, 10, 34, 16, 28, …., …. (A) 24, 20 (B) 20, 24 (C) 26, 22 (D) 22, 26 (E) 22, 24

05. 10, 10 , 20, 25, 10 , …., 20, 55 (A) 30 (B) 40 (C) 50 (D) 60 (E) 70 06. 4, 11, 30, 8, 19, 29, 16, 27, 28, …., …., …. (A) 32, 34, 37 (B) 33, 34, 35 (C) 32, 34, 35 (D) 32, 35, 27 (E) 33, 27, 35

383

Xpedia TPA Set 03 - Barisan Huruf

01. A, B, C, E, F, G, I, J, K, …, …, ... (A) L, M, N (D) M, N, O (B) M, N, P (E) N, P, Q (C) N, P, O 02. K, A, M, C, O, E, Q, G, …, ... (A) S, H (B) S, I (C) S, R (D) R, H (E) R, I 03. E, F, H, I, K, L, N, …, ... (A) P, Q (B) N, O (C) O, P

(D) O, Q (E) N, P

04. D, G, J, M, P, …, ... (A) S, T (B) S, V (C) R, S (D) V, T (E) T, V 05. Z, C, S, G, L, …, ... (A) K, H (D) K, D (B) E, I (E) K, E (C) K, M 06. A, B, C, C, D, E, E, F, G, G, H, I, …, …, ... (A) I, J, K (D) I, J, J (B) I, J, L (E) I, I, J (C) J, J, K 07. AA, YY, BB, WW, CC, … (A) ZZ (D) VV (B) XX (E) UU (C) TT 08. A, H, I, B, J, K, C, L, M, …, …, ... (A) D, L, N (D) E, L, N (B) L, D, N (E) E, O, P (C) D, N, O 384

09. B, C, E, F, H, I, K, …, ... (A) L, M (B) L, N (C) N, O (D) N, P (E) M, N 10. S, O, E, T, K, I, U, G, M, …, …, ... (A) (B) (C) (D) (E)

V, D, Q C, D, R U, E, R V, D, P V, C, Q

Xpedia TPA Barisan & Aritmatika - Set 4 1

01. Jika 213xy + 4 = 421, maka 639xy =… (A) 1275 (B) 1263 (C) 1251 (D) 1242 (E) 1236 02. Aku pergi bermain tenis tiap 2 minggu sekali, Tom pergi bermain tenis tiap 4 minggu sekali, dan Cathy pergi bermain tenis setiap 6 minggu sekali. Kami bermain tenis dilapangan yang sama, dan hari ini kami bertemu. Berapa minggu kami bisa bermain tenis bersama-sama lagi? (A) 8 (B) 12 (C) 24 (D) 36 (E) 64 03. Dua puluh lima persen dari 60 adalah … (A) 15 (B) 25 (C) 35 (D) 45 (E) 55 04. Jika nilai rata-rata dari 2, 3, x, dan y adalah 5, maka rata-rata dari 4, 5, 6, x dan y adalah ... (A) 4,6 (B) 5,0 (C) 5,6 (D) 6,0 (E) 6,4

06. 105 + 105 + 105 + 105 + 105 = n . 55 Berapakah nilai n ? (A) 2 (B) 5 (C) 24 (D) 100 (E) 160 07. Jika 2 < x < 5, dan 4 < y < 7, dan w= x+y, maka nilai w berada di antara nilai ... (A) 2 dan 7 (B) 4 dan 7 (C) 6 dan 12 (D) 5 dan 7 (E) 6 dan 9 08. Bentuk sederhana dari (2-3 + 5-1)-2 adalah ... (A) 169/1600 (B) 1600/169 (C) 12/13 (D) 1/169 (E) 169 09. Nilai 180 merupakan 45% dari bilangan … (A) 18 (B) 24 (C) 81 (D) 300 (E) 400

05. Untuk mengisi 3/7 bagian sebuah kolam renang diperlukan waktu 6 jam. Berapakah waktu yang diperlukan untuk mengisi sisanya, jika pengisian kolam renang dilakukan dengan laju yang sama? (A) 2,5 jam (B) 5 jam (C) 7 jam (D) 8 jam (E) 14 jam 385

Xpedia TPA Barisan & Aritmatika - Set 3

01. Berapakah rata-rata dari 3p, q, dan r? (A)

3p  q  r pqr

(B)

3p  q  r 3

3p  q  r (C) pqr 3 pqr (D) 3 p  q  r (E) pqr 02. 0,01 + (A) (B) (C) (D) (E)

07. Perhatikan gambar di bawah! Tentukan besar BCA.

1 = …. 100

0,02 0,001 0,001 0,01 0,2

(A) (B) (C) (D) (E)

03. 225,6 x 4,51=…. (A) 956,40 (B) 1.017,456 (C) 1.228,66 (D) 1.356,78 (E) 1.458,97

5 4 a a  b , sehingga nilai adalah …. 6 9 b

10 7 7 (B) 10 8 (C) 15

(A)

386

10o 15o 20o 25o 30o

08. Tentukan luas dearah bangun di bawah ini!

04. 8 adalah 5 persen dari …. (A) 760 (B) 160 (C) 40 (D) 16 (E) 1,6 05. Jika

06. Sarah mempunyai 7 apel, 5 mangga, 3 jeruk. Banyaknya tambahan jeruk yang harus sarah beli sehingga perbandingan banyaknya keseluruhan buah dan banyaknya jeruk yang dia miliki menjadi 3 : 1 adalah …. (A) 6 (B) 5 (C) 4 (D) 3 (E) 2

15 8 7 (E) 15

(D)

(A) (B) (C) (D) (E)

18 cm2 36 cm2 42 cm2 66 cm2 72 cm2

Xpedia TPA, Barisan & Aritmatika - Set 3

09. Tiga buah kaleng cat yang berdiameter 10 cm diikat oleh tali seperti gambar di bawah.

Berpakah panjang tali minimal yang diperlukan? (A) 31,4 cm (B) 41,4 cm (C) 51,4 cm (D) 61,4 cm (E) 71,4 cm 10. Jika …. (A) (B) (C) (D) (E)

( x  y ) 2  12 dan xy  1 , maka x2 + y2 = 14 13 12 11 10

387

Xpedia TPA Set 02 - Barisan Angka

01. 1, 5, 4, 3, 7, 1, 10, -1, 13, -3, …., …. (A) 16, -8 (B) 10, -5 (C) 15, 8 (D) 16, -5 (E) 4, 27

08. a, m, n, b, o, p, c, …., …., …., …. (A) d, e, f, g (B) q, r, d, s (C) d, q, r, e (D) q, r, d, e (E) d, e, g, h

02. 0, 10, 28, 54, 88, 130, …. (A) 150 (B) 164 (C) 172 (D) 180 (E) 196

09. 0, 6, 6, 20, 20, …. (A) 34 (B) 35 (C) 38 (D) 42 (E) 46

03. 3, 12, 48, 192, …. (A) 48 (B) 96 (C) 292 (D) 384 (E) 768 04. 7, 18, 29, 40, 51, …. (A) 62 (B) 63 (C) 61 (D) 64 (E) 65 05. 8, 8, 11, 16, 14, 24, …. (A) 24, 17 (B) 32, 22 (C) 24, 27 (D) 17, 32 (E) 17, 35 06. 1, 2, 4, 8, 16, …. (A) 32, 36 (B) 32, 40 (C) 32, 48 (D) 32, 64 (E) 64, 32 07. C, F, I, L, O, …., …. (A) Q, T (B) R, U (C) P, S (D) R, T (E) R, S 388

10. 10, 15, 12, 17, 14, 19, …. (A) 15 (B) 16 (C) 17 (D) 21 (E) 24

Set 01 - Barisan Angka dan Huruf

01. 420, 413, 416, 208, 201, 204, 102, .... (A) 51 (B) 95 (C) 105 (D) 109 (E) 204

07. 5, 10, 15, 20, 45, 40, 135, 80, .... (A) 115 (B) 160 (C) 240 (D) 270 (E) 405

02. 1,50; 4; 9; 19; 39; 79; ... ; 319; 639. (A) 120 (B) 159 (C) 160 (D) 205 (E) 220

08. 4, 4, 4, 7, 5, 4, 5, 8, 6, 4, ... , .... (A) 5, 9 (B) 6, 9 (C) 6, 8 (D) 6, 6 (E) 6, 5

03. 0,80; 1,20; 1,60; 2,00; 2,40; ... ; 3,20; 3,60; 4,00. (A) 2,60 (B) 2,80 (C) 3,10 (D) 3,20 (E) 3,50

09. 8, 12, 18, 26, .... (A) 32 (B) 34 (C) 36 (D) 40 (E) 42

04. 22, 18, 26, 22, 30, 26, 34, .... (A) 0 (B) 22 (C) 30 (D) 38 (E) 42

10. 8, 26, 48, 82, 120, 170, .... (A) 195 (B) 197 (C) 200 (D) 224 (E) 226

05. ... , ... , 16, 18, 32, 36, 64, 72, 128. (A) 7 dan 8 (B) 8 dan 7 (C) 8 dan 9 (D) 9 dan 8 (E) 10 dan 9 06. 3, 5, 9, 17, .... (A) 29 (B) 31 (C) 33 (D) 35 (E) 37

389

Xpedia TPA Pola Gambar - Set 1

Tentukan kelanjutan dari gambar sebelumnya!

05.

01.

06. 02.

03.

04.

390

07.

08.

Xpedia TPA, Pola Gambar - Set 1

09.

10.

391

Xpedia TPA Pola Gambar - Set 2

05.

01.

Jika I jadi II, maka III akan menjadi IV ? 02.

03.

06.

07.

04. 08.

392

Xpedia TPA, Pola Gambar - Set 2

09.

393

Xpedia TPA Pola Gambar - Set3

01. Perhatikan baik-baik gambar-gambar berikut di bawah ini, kemudian carilah gambar mana yang tidak sesuai dengan gambar-gambar lainnya. (A)

04. Perhatikan gambar berikut dengan baik, kemudian pilihlah satu gambar yang tidak sesuai dengan yang lainnya.

A

 D

 B

(B)

E

C (C)

05. Pada gambar di bawah ini satu gambar yang tidak sesuai. Gambar manakah itu?

(D)

(E) (A) 06.

02. Carilah gambar yang tidak sesuai!

(A)

(B)

(C)

(D)

(E)

03. Dari gambar dibawah ini, manakah yang berbeda? A  B C

E

(D)

seperti

ke ?

A

 D

 B

E

C

terhadap

seperti

A  B C 394

(C) ke

07.

 D

(B)

terhadap ...

 D E

(E)

Xpedia TPA, Pola Gambar - Set3

08.

terhadap

seperti

09.

terhadap ...

A

 D

 B C

E

Kelereng biru :

Bola merah

:

Kelereng merah

:

...

10. Pilihlah gambar yang merupakan kelanjutan dari gambar sebelumnya!

A

 B

C

 D

E

395

Xpedia TPA Kelas 7

01. Pilihlah salah satu gambar yang tidak sama atau tidak sekelompok dengan gambar lainnya !

02. Pilihlah salah satu gambar yang tidak sama atau tidak sekelompok dengan gambar lainnya!

03. Pilihlah salah satu gambar yang tidak sama atau tidak sekelompok dengan gambar lainnya!

04. Pilihlah salah satu gambar yang tidak sama atau tidak sekelompok dengan gambar lainnya!

06. Pilihlah salah satu gambar yang tidak sama atau tidak sekelompok dengan gambar lainnya!

07. Pilihlah salah satu gambar yang tidak sama atau tidak sekelompok dengan gambar lainnya!

08. Pilihlah salah satu gambar yang tidak sama atau tidak sekelompok dengan gambar lainnya!

09. Pilihlah salah satu gambar yang tidak sama atau tidak sekelompok dengan gambar lainnya!

05. Pilihlah salah satu gambar yang tidak sama atau tidak sekelompok dengan gambar lainnya! 10. Pilihlah salah satu gambar yang tidak sama atau tidak sekelompok dengan gambar lainnya!

396

Xpedia TPA Pola Gambar - Set5

01. Pilihlah gambar yang merupakan bangun yang sesuai dengan jaring-jaringnya!

A

 B

 D

E

C

02. Pilihlah gambar yang merupakan bangun yang sesuai dengan jaring-jaringnya!

A

 B

 D

E

C

03. Pilihlah gambar yang merupakan bangun yang sesuai dengan jaring-jaringnya!

A

 D

 B E

04. Pilihlah gambar yang merupakan bangun yang sesuai dengan jaring-jaringnya!

A

 B

 D

E

C

05. Pilihlah gambar yang merupakan bangun yang sesuai dengan jaring-jaringnya!

A

 B

 D

E

C

C 06. Banyak segitiga pada gambar berikut adalah …

(A) (B) (C) (D) (E)

6 10 11 12 13 397

Xpedia TPA, Pola Gambar - Set5

07. Pilihlah gambar yang merupakan kelanjutan dari gambar sebelumnya!

(A)

(B)

(C)

(D)

(E)

08. Pilihlah gambar yang merupakan kelanjutan dari gambar sebelumnya!

(A)

(B)

(C)

(D)

(E)

09. Pilihlah gambar yang merupakan kelanjutan dari gambar sebelumnya!

(A)

398

(B)

(C)

(D)

(E)

10. Pilihlah gambar yang merupakan kelanjutan dari gambar sebelumnya!

(A)

(B)

(C)

(D)

(E)

Xpedia TPA Pola Gambar - Set 6

01.

05.

02. 06.

03. 07.

04.

08.

399

Xpedia TPA, Pola Gambar - Set 6

09.

10.

400

Xpedia TPA Pola Gambar - Set 7

01. Manakah gambar yang tepat untuk mengisi kotak bertanda tanya?

04. Manakah gambar yang tepat untuk mengisi kotak bertanda tanya?

02. Manakah gambar yang tepat untuk mengisi kotak bertanda tanya? 05. Manakah gambar yang tepat untuk mengisi kotak bertanda tanya?

03. Manakah gambar yang tepat untuk mengisi kotak bertanda tanya?

06. Manakah gambar yang tepat untuk mengisi kotak bertanda tanya?

401

Xpedia TPA, Pola Gambar - Set 7

07. Manakah bangun jaringnya?

yang

sesuai

jaring-

(D) hanya 1, 2, dan 4 (E) hanya 1, 3 dan 4 10. Manakah yang merupakan kelanjutannya?

08. Manakah kubus yang sesuai jaring-jaringnya?

(A) (B) (C) (D) (E)

hanya 1, 3, dan 5 hanya 1, 2, dan 4 hanya 2, 4, dan 5 hanya 1, 2, 3, dan 5 hanya 1 dan 5

09. Manakah kubus yang sesuai jaring-jaringnya?

(A) hanya 2 dan 3 (B) hanya 2, 3, dan 5 (C) hanya 1 dan 4 402

Xpedia TPA Kelas 07 Set 8 - Analogi Dan Kelanjutan

01. Carilah gambar untuk mengisi kotak bertanda tanya, yang mempunyai analogi yang sama dengan pasangan gambar di sebelah kiri!

02. Carilah gambar untuk mengisi kotak bertanda tanya, yang mempunyai analogi yang sama dengan pasangan gambar di sebelah kiri!

03. Carilah gambar untuk mengisi kotak bertanda tanya, yang mempunyai analogi yang sama dengan pasangan gambar di sebelah kiri!

04. Carilah gambar untuk mengisi kotak bertanda tanya, yang mempunyai analogi yang sama dengan pasangan gambar di sebelah kiri!

403

05. Carilah gambar untuk mengisi kotak bertanda tanya, yang mempunyai analogi yang sama dengan pasangan gambar di sebelah kiri!

06. Carilah gambar untuk mengisi kotak bertanda tanya, yang mempunyai analogi yang sama dengan pasangan gambar di sebelah kiri!

Xpedia TPA, Set 8 - Analogi Dan Kelanjutan

07. Carilah gambar untuk mengisi kotak bertanda tanya, yang mempunyai analogi yang sama dengan pasangan gambar di sebelah kiri!

10. Carilah gambar untuk mengisi kotak bertanda tanya, yang merupakan kelanjutan dari gambar sebelumnya!

08. Carilah gambar untuk mengisi kotak bertanda tanya, yang merupakan kelanjutan dari gambar sebelumnya!

09. Carilah gambar untuk mengisi kotak bertanda tanya, yang merupakan kelanjutan dari gambar sebelumnya!

404

Xpedia TPA Analogi - Set 09

06.

01.

(A)

(B)

(C)

(D)

(E)

(A)

(B)

(C)

(D)

(E)

(A)

(B)

(C)

(D)

(E)

(A)

(B)

(C)

(D)

(E)

07.

02.

(A)

(B)

(C)

(D)

(E) 08.

03.

(A)

(B)

(C)

(D)

(E) 09.

04.

(A)

(B)

(C)

(D)

(E)

05.

(A)

(B)

(C)

(D)

(E)

(A)

(B)

(C)

(D)

(E)

10.

(A) 405

(B)

(C)

(D)

(E)

Xpedia TPA Set 10 - Analogi

01.

04.

02. 05.

03. 06.

Xpedia TPA, Set 10 - Analogi

10. 07.

08.

09.

406

Xpedia TPA Set 11 - Pilih Yang Berbeda

09.

01. (A)

(B)

(C)

(D)

(A)

(B)

(A)

(B)

(C)

(D)

(E)

(D)

(E)

(E) 10.

02. (A)

(B)

(C)

(D)

(E)

(A)

(B)

(D)

(E)

(A)

(B)

(C)

(D)

(E)

(A)

(B)

(C)

(D)

(E)

(A)

(B)

(D)

(E)

03. (C)

04.

05.

06. (C)

07. (A)

(B)

(C)

(D)

(E)

(A)

(B)

(C)

(D)

(E)

08.

407

(C)

Xpedia TPA Set 12 - Pilih Yang Berbeda

01.

06.

02.

07.

03.

04. 08.

05.

09.

10.

408

Xpedia TPA Set 13 – Pola Kelanjutan

01.

05.

02.

06.

03.

07.

04.`

08.

409

Xpedia TPA, Set 13 – Pola Kelanjutan

09.

10.

410

Xpedia TPA

07.

01.

(A)

(B)

(C)

(D)

(E)

(A)

(B)

(C)

(D)

(E)

(A)

(B)

(C)

(D)

(E)

(A)

(B)

(C)

(D)

(E)

(A)

(B)

(C)

(D)

(E)

08.

02.

(A)

(B)

(C)

(D)

(E) 09.

03.

(A)

(B)

(C)

(D)

(E)

04.

10.

(A)

(B)

(C)

(D)

(E)

05.

(A)

(B)

(C)

(D)

(E)

(A)

(B)

(C)

(D)

(E)

06.

411

Xpedia TPA Set 15 - Sembilan Kotak

04.

01.

(A)

(B)

(C)

(D)

(E)

(A)

(B)

(C)

(D)

(E)

05.

02.

?

? (A)

(B)

(C)

(D)

(E)

03.

(A)

(B)

(C)

(D)

(E)

(A)

(B)

(C)

(D)

(E)

06.

(A)

(B)

(C)

(D)

(E)

412

Xpedia TPA, Set 15 - Sembilan Kotak

07.

10.

(A)

(B)

(C)

(D)

(E)

(A)

(B)

(C)

(D)

(E)

(A)

(B)

(C)

(D)

(E)

08.

09.

413

(A)

(B)

(C)

(D)

(E)

Xpedia TPA Set 16 - Sembilan Kotak

01.

04.

(A)

(B)

(C)

(D)

(E)

02.

(A)

(B)

(C)

(D)

(E)

(C)

(D)

(E)

(C)

(D)

(E)

05.

(A)

(B)

(C)

(D)

(A)

(E)

(B)

06.

03.

(A) 414

(B)

(C)

(D)

(E)

(A)

(B)

Xpedia TPA, Set 16 - Sembilan Kotak

07.

09.

(A) (A)

(B)

(C)

(D)

(E)

(B)

(C)

(D)

(E)

10.

08.

(A) (A)

(B)

(C)

(D)

(B)

(C)

(D)

(E)

(E)

415

Xpedia TPA Basic Thinking and Reasoning Skills - Set and Categories - Soal

01. Buatlah diagram Venn dari Entitas di bawah ini! S : Semua produk A : Merk Nokia B : Merk Sony-Ericsson C : Hand phone 02. Buatlah diagram Venn dari Entitas di bawah ini! Negara - Kota - Jakarta 03. Buatlah diagram Venn dari Entitas di bawah ini! Teh - Kopi - Gula 04. Buatlah diagram Venn dari Entitas di bawah ini! Kucing - Singa - Anjing 05. Buatlah diagram Venn dari Entitas di bawah ini! Paus - Lumba-lumba - Ikan 06. Buatlah diagram Venn dari Entitas di bawah ini! A : Objek Kajian Fisika B : Objek Kajian Matematika C : Angka 07. Buatlah diagram Venn dari Entitas di bawah ini! S : Semua fungsi y = f(x) A : Fungsi y=f(x) yang selalu naik pada x  R B : Fungsi y=f(x) yang selalu turun pada x  R C : Fungsi kuadrat untuk x  R

08. Buatlah diagram Venn dari Entitas di bawah ini! Bilangan Riil - Bilangan Rasional - Bilangan Negatif 09. Buatlah diagram Venn dari Entitas di bawah ini! A : Gaya Listrik B : Gaya Gravitasi C : Tegangan Listrik 10. Buatlah diagram Venn dari Entitas di bawah ini! A : Besaran Vektor B : Besaran Skalar C : Gaya 11. Buatlah diagram Venn dari Entitas di bawah ini! A : Gaya B : Percepatan C : Kecepatan 12. Buatlah diagram Venn dari Entitas di bawah ini! A : Gas Hidrogen B : Atom Hidrogen C : Air 13. Buatlah diagram Venn dari Entitas di bawah ini! A : Tubuh Manusia B : Jantung Manusia C : Organ Tubuh Manusia 14. Buatlah diagram Venn dari Entitas di bawah ini! A : Ikatan ionik B : Unsur logam C : Unsur magnesium

Xpedia TPA, Basic Thinking and Reasoning Skills - Set and Categories - Soal doc. name: XPTPA0101

doc. version: 2010-10 |

15. Buatlah diagram Venn dari Entitas di bawah ini! S : Semua fungsi A : Semua fungsi y = f(x) y  1 untuk semua xR B : Semua fungsi y = f(x) y > 1 untuk semua xR C : y = f(x) = sin x, untuk semua x  R 16. Buatlah diagram Venn dari Entitas di bawah ini! Sel Tumbuhan - DNA - Inti Sel

17. Buatlah diagram Venn dari Entitas di bawah ini! A : Mobil produksi Jepang B : Mobil merk Toyota C : Mobil sedan 18. Buatlah diagram Venn dari Entitas di bawah ini! S : A: B: C:

Semua titik (x,y) pada diagram cartesius x2+y2 = 9 x2+y2 = 25 y=x

19. Buatlah diagram Venn dari Entitas di bawah ini! S : Semua orang A : Orang-orang atheis B : Orang-orang komunis C : Orang-orang religius 20. Buatlah diagram Venn dari Entitas di bawah ini! A : Aset (aktiva) B : Liabilitas (kewajiban) C : Ekuitas

Xpedia TPA Basic Thinking and Reasoning Skills - Inductive Reasoning - Soal

Bagian 1. 01. 1, 10, 19, 28 .... 02. 5, 1, 1 , 1 .... 5 25

06. 2, 6, 4, 12, 10, 30, 28 .... 07. 1, 1, 2, 5, 11, 21 .... 08.

2 -1 3

03. 1, 3, 5, 6, 9, 12 ....

4 3 1

04. 20, 10, 5 .... 05. 5, 12, 26, 54 ....

5

2 (2) 1 3 (9) 2 4 (1) 0 2 ( ...) 3

10.

7 (33) 2 (-21) 3(0) 3 ( ...)

11.

4( ) 3 3 (16) 1 4 (125) 1 6 ( ... ) 6

12.

21 (BAGI) 79 62 ( …) 45

13.

FA (40) BA GI (27) EB BA ( ... ) GI

14.

3 (3) 1 4 (20) 5 7 (14) 2 1 ( ...) 1

07. 3, 8, 15, 24 ....

09. 199, 196, 191, 184 .... 10. 4, 6, 10, 14, 22 .... 11. 4, 9, 25, 49 .... 12. 3, 7, 11, 15, 19 .... 13. 5, 25, 61, 113 .... 14. 4, 9, 14, 21, 30, 41 .... 15. 5, 34, 65, 98 16. 1000, 995, 990, 1000, 970, 1010 .... 17. 404, 208, 116, 82 .... Bagian 2. 01. A, E, I, M .... 02. A, J, S, B .... 03. AA, CZ, EY, GX .... 04. M, Q, O, S, Q, U .... 05. 3, 8, 6, 11, 9, 14, 12 ....

11

09.

06. 10, 14, 22, 38 ....

08. 5, 17, 65, 257 ....

1 0 1

4 5 3 2

15. Hitunglah ada berapa banyak angka 1 yang muncul, Jika kamu menulis : a . 1-100 b. 1-1000

Xpedia TPA Basic Thinking and Reasoning Skills - Deductive Reasoning and Logic - Soal

01. Jika risky nakal maka mama marah, jika mama marah maka bumi berguncang. Kesimpulan apa yang bias diambil dari faktafakta berikut. (A) risky nakal (B) mama marah (C) bumi berguncang (D) rizky tidak nakal (E) mama tidak marah (F) Bumi tidak berguncang 02. Jika meta atau cherina nakal maka mama marah. Jika mama marah maka mama makan. Kesimpulannya adalah ! (A) Meta nakal (B) Meta tidak nakal (C) Mama tidak marah (D) Mama tidak makan (E) Meta & cherina tidak nakal 03. Jika ikan mengaum maka gajah mengembik. Jika gajah mengembik maka viktor dan ina mengikik, ternyata Viktor mengikik tapi Ina tidak mengikik, Tentukan apakah kesimpulan di bawah int (b/s/m). (A) Ina mengikik (B) Gajah mengembik (C) Gajah tidak mengembik (D) Ikan mengaum (E) Ikan tidak mengaum (F) Viktor atau ina mengikik 04. Semua professor botak, ada orang botak yang bukan professor. Tentukan apakah (T/F/M) (A) Jika Adi adalah prof, maka Adi botak (B) Jika bugsi botak, maka bugsi profesor 05. Tidak gading yang tak retak (pl). Tentukan (T/F/M) (A) Semua gading retak (B) Ada yang bukan gading yg retak (C) Semua yang retak adalah gading

06. Ada pelajar yang bolos semua yang bolos masuk angin. (A) Semua pelajar masuk angin (B) Ada pelajar yang masuk angin 07. Ada ikan yang nganggur ada yg nganggur yang adalah motor . (A) Ada ikan yang adalah motor (B) Ada motor yang nganggur 08. Semua P adalah Q. Ada Q yang bukan R. Tentukan mana yang pasti benar … (A) Ada Q yang R (B) Tidak ada Q yang P (C) Ada P yang bukan R (D) Tidak ada P yang bukan Q

Xpedia TPA BAB 5 - Deductive Reasoning and Logic - Soal Analitik

Enam buku yang ditulis oleh enam pengarang - J, K, L, M, N, dan O - disusun dalam suatu lemari buku yang terdiri dari 6 rak. Rak tersebut dinomori mulai dari yang tertinggi 1 ke yang terendah 6. Tepat satu buku akan diletakkan pada setiap rak. Buku oleh K harus diletakkan dua rak di bawah buku oleh J. Buku oleh O harus diletakkan di rak pertama atau rak keenam. Buku oleh L tidak boleh diletakkan pada rak tepat di atas atau pada rak tepat di bawah buku oleh M.

05. Jika buku oleh L dan M terpisah tepat satu rak, maka buku oleh pengarang mana yang dapat diletakkan di rak antara buku oleh L dan M? I. N II. J III. K (A) hanya I (B) hanya II (C) hanya I dan II (D) hanya II dan III (E) I, II, dan III

01. Mana yang merupakan urutan yang mungkin bagi buku, mulai dari rak teratas sampai rak terbawah. (A) J, N, K, M, L, O (B) J, N, K, L, M, O (C) J, M, K, L, N, O (D) O, J, K, M, N, L (E) N, J, M, K, O, L

06. Jika buku oleh Jada di rak ke empat, maka apa yang pasti benar? (A) O ada di rak pertama (B) L ada di rak kedua (C) N ada di rak ketiga (D) M ada di rak kelima (E) N ada di rak kelima

02. Semua ini merupakan urutan penyusunan buku yang mungkin dari rak teratas sampai rak terbawah, kecuali …. (A) J, L, K, N, M, O (B) L, J, M, N, K, O (C) L, N, J, M, K, O (D) M, J, N, K, L, O (E) O, M, J, L, K, N 03. Apa yang tidak mungkin benar? (A) buku oleh J ada di rak kedua (B) buku oleh J ada di rak ketiga (C) buku oleh M ada di rak ketiga (D) buku oleh L ada di rak kelima (E) buku oleh J ada di rak kelima 04. Buku oleh pasangan mana yang tidak mungkin diletakkan berurutan di rak ke-tiga dan ke-empat? (A) J dan M (B) K dan L (C) L dan K (D) M dan N (E) N dan J

07. Jika buku oleh M diletakkan dua rak di atas buku oleh J, maka semua ini pasti benar, kecuali …. (A) buku O diletakkan di atas buku M (B) buku M diletakkan di atas buku N (C) buku J diletakkan di atas buku L (D) buku J diletakkan di atas buku K (E) buku L diletakkan diatas buku K 08. Jika buku yang ditulis oleh L dipisahkan dari buku yang ditulis oleh M sebanyak 3 rak, maka mana yang merupakan daftar lengkap dari rak yang boleh ditempati buku yang ditulis oleh N? (A) ketiga (B) ke-empat (C) kedua dan ketiga (D) ketiga dan ke-empat (E) kedua, ketiga, dan ke-empat

Xpedia TPA, BAB 5 - Deductive Reasoning and Logic - Soal Analitik

Sebuah bangunan terdiri dari 6 lantai dan tiap lantai berisi satu bisnis. 6 bisnis itu adalah agen asuransi, praktek dokter, firma hukum, percetakan, studio tari, dan toko baju. Agen asuransi terletak di bawah lantai 4. Praktek dokter terpisah dari agen asuransi sejauh jumlah lantai yang memisahkan agen asuransi dengan firma hukum serta jumlah lantai itu mungkin nol. Percetakan terletak di antara studio tari dan agen asuransi Firma hukum terletak tepat di atas atau tepat di bawah percetakan. 10. Mana yang merupakan susunan yang mungkin, dari lantai satu sampai lantai enam? (A) praktek dokter, agen asuransi, firma hukum, percetakan, studio tari, toko baju (B) praktek dokter, agen asuransi, firma hukum, toko baju, percetakan, studio tari (C) agen asuransi, firma hukum, percetakan, praktek dokter, studio tari, toko baju (D) toko baju, firma hukum, percetakan, agen asuransi, praktek dokter, studio tari (E) studio tari, percetakan, agen asuransi, firma hukum, praktek dokter, toko baju 11. Jika studio tari ada di lantai 5, maka toko baju terletak di lantai? (A) satu (B) dua (C) tiga (D) empat (E) enam 12. Jika percetakan terletak di lantai lima, di mana lantai letak toko baju? (A) satu (B) dua (C) tiga (D) empat (E) enam

13. Jika toko baju terletak di lantai lima, maka mana yang pasti benar? (A) agen asuransi ada di lantai satu (B) praktek dokter ada di lantai dua (C) firma hukum ada di lantai tiga (D) studio tari ada di lantai empat (E) percetakan ada di lantai enam Seorang guru musik menjadwalkan resital untuk 5 orang. Resital itu terdiri dari 5 partitur dan tiap partitur akan dinyanyikan oleh satu orang dengan diiringi piano. Wilona, Menik, dan Shinta bisa bernyanyi Victor, Bernie, dan Shinta bisa memainkan piano Shinta harus bernyanyi pada partitur ketiga Bernie harus mengiringi partitur ke empat Tidak ada orang yang boleh tampil pada dua partitur berurutan. 14. Mana yang pasti benar? (A) Bernie adalah pengiring untuk pertama (B) Bernie adalah pengiring untuk kedua (C) Shinta adalah vokalis untuk keempat (D) Menik adalah vokalis untuk keempat (E) Shinta adalah vokalis untuk kelima

partitur partitur partitur partitur partitur

15. Jika Shinta menyanyikan partitur pertama, mana yang pasti benar? (A) Wilona menyanyikan partitur pertama (B) Menik menyanyikan partitur kedua (C) Shinta menyanyikan partitur kelima (D) Victor mengiringi partitur pertama (E) Victor mengiringi partitur kelima

Related Documents

Tps - Tpa 2020
February 2021 2
Tpa
February 2021 8
Tpa-103
February 2021 3
Soal Tps Mk Bandung.pdf
January 2021 0
Cipete Selatan Tps 002
February 2021 0

More Documents from "Mohamad Amin"

Tps - Tpa 2020
February 2021 2